Oswaal CBSE Question Bank Class 9 Social Science, Chapterwise and Topicwise Solved Papers For 2025 Exams 9789359589978, 9359589977

Oswaal CBSE Question Bank Class 9 Social Science, Chapterwise and Topicwise Solved Papers For 2025 Exams248

123 47 40MB

English Pages 249 Year 2024

Report DMCA / Copyright

DOWNLOAD PDF FILE

Table of contents :
Cover
Contents
Latest CBSE Syllabus
Unit 1 : India and the Contemporary
World-I
1. The French Revolution
2. Socialism in Europe and the Russian Revolution
3. Nazism and the Rise of Hiter
Unit 2 : Contemporary India-I 4. India: Size and Location
5. Physical Features of India
6. Drainage
7. Climate
8. Population
Unit 3 : Democratic Politics-I 9. What is Democracy ? Why
Democracy ?
10. Constitutional Design
11. Electoral Politics
12. Working of Institutions
13. Democratic Rights
Unit 4 : Economics 14. People as Resource
15. Poverty as a Challenge
16. Food Security in India
Recommend Papers

Oswaal CBSE Question Bank Class 9 Social Science, Chapterwise and Topicwise Solved Papers For 2025 Exams
 9789359589978, 9359589977

  • 0 0 0
  • Like this paper and download? You can publish your own PDF file online for free in a few minutes! Sign Up
File loading please wait...
Citation preview

For 2025 Exam

BEST SELLER

CBSE

CBSE QUESTION

SAMPLE QUESTION PAPERS QUESTION BANK CLASS 9

BANK

Chapterwise & Topicwise

CLASS 9

SOCIAL SCIENCE • India and the Contemporary World-I • Contemporary India-I • Democratic Politics-I • Economics

Get the #OswaalEdge

SOCIAL SCIENCE

1

100% Updated

2

Timed Revision

3 4 5

with Latest Syllabus & Questions Typologies

with Topic wise Revision Notes & Smart Mind Maps

Extensive Practice with 1000+ Questions for Practice

Concept Clarity with 500+ Concepts & Concept Videos

100% Exam Readiness with Artificial Intelligence & Competency Based Questions

(1)

20th EDITION

I SB N SYLLABUS COVERED

YEAR 2024-25 “9789359589978”

CENTRAL BOARD OF SECONDARY EDUCATION DELHI

PUBLISHED BY OSWAAL BOOKS & LEARNING PVT. LTD.

C OPY RIGHT

RESERVED

1/11, Sahitya Kunj, M.G. Road, Agra - 282002, (UP) India

BY THE PUBLISHERS

All rights reserved. No part of this book may be reproduced, stored in a retrieval system, or transmitted, in any form or by any means, without written permission from the publishers. The author and publisher will gladly receive information enabling them to rectify any error or omission in subsequent editions.

1010, Cambourne Business Centre Cambridge, Cambridgeshire CB 236DP, United kingdom

0562-2857671

[email protected]

www.OswaalBooks.com

DISC L AIM ER This book is published by Oswaal Books and Learning Pvt Ltd (“Publisher”) and is intended solely for educational use, to enable students to practice for examinations/tests and reference. The contents of this book primarily comprise a collection of questions that have been sourced from previous examination papers. Any practice questions and/or notes included by the Publisher are formulated by placing reliance on previous question papers and are in keeping with the format/pattern/guidelines applicable to such papers. The Publisher expressly disclaims any liability for the use of, or references to, any terms or terminology in the book, which may not be considered appropriate or may be considered offensive, in light of societal changes. Further, the contents of this book, including references to any persons, corporations, brands, political parties, incidents, historical events and/or terminology within the book, if any, are not intended to be offensive, and/or to hurt, insult or defame any person (whether living or dead), entity, gender, caste, religion, race, etc. and any interpretation to this effect is unintended and purely incidental. While we try to keep our publications as updated and accurate as possible, human error may creep in. We expressly disclaim liability for errors and/or omissions in the content, if any, and further disclaim any liability for any loss or damages in connection with the use of the book and reference to its contents”. The Maps shown in this book are for reference only. They are Artistic Maps.

Kindle Edition (2)

Contents l

Latest CBSE Syllabus

6 - 14

In each chapter, for better comprehension, questions have been categorized according to the typology issued by CBSE as follows : R - Remembering,

U - Understanding,

A - Application,

C - Creating,

Unit 1 : India and the Contemporary

Unit 3 : Democratic Politics-I

World-I





1. The French Revolution

1 - 22

u Artificial Intelligence



2. Socialism in Europe and the



Russian Revolution

3. Nazism and the Rise of Hiter

23 - 43 44 - 63

Unit 2 : Contemporary India-I

4. India: Size and Location

64 - 74

u Artificial Intelligence



5. Physical Features of India

75 - 89



6. Drainage

90 - 102



7. Climate

103 - 116

u Artificial Intelligence

E - Evaluating.

9. What is Democracy ? Why



Democracy ?

128 - 142



10. Constitutional Design

143 - 153



11. Electoral Politics

154 - 169

u Artificial Intelligence



12. Working of Institutions

170 - 184



13. Democratic Rights

185 - 196

Unit 4 : Economics

14. People as Resource

197 - 210

u Artificial Intelligence



15. Poverty as a Challenge

211 - 222

u Artificial Intelligence



16. Food Security in India

8. Population 117 - 127

223 - 231

qq

These chapters will be assessed as part of Periodic assessments & interdisciplinary project work 1. 4 Forest, Society and Colonialism Interdisciplinary project as part of multiple assessments (Internally assessed for 5 marks) 5 Pastoralists in the Modern World (To be assessed in the Periodic Assessment only) 2. 5 Natural Vegetation and Wildlife (Only map pointing to be evalualed in the annual examination) Interdisciplinary project as part of multiple assessments (Internally assessed for 5 marks 3. 1 The Story of Village Palampur (To be assessed in periodic assessment only)

LOOK OUT FOR 'HIGHLY LIKELY QUESTIONS'

To access these chapters scan the code below

These questions are selected by Oswaal Editorial Board. These are highly likely to be asked in the upcoming examinations.

(3)

How to use this Book Chapter Navigation Tools

Syllabus

Topicwise & Conceptwise Segregation

Revision Notes

Prescribed by CBSE

For Focused & Systematic Study

Chapter Summary Developed by Oswaal Experts

Latest Typologies of Questions

Mind Maps

QR Codes

MCQs SA/LA/ Questions

For better retention of concepts

For Concept Videos

Artificial Intelligence In Compliance with NEP

Commonly Made Errors & Answering Tips

Academically Important Questions To Look out for Highly Expected Questions for the upcoming exams

To write perfect Answers

Self Assessment

Competency based Questions

For Unitwise Self-Assessment

To demonstrate application based knowledge of Concepts

(4)

Preface Elevate Your Performance, Surpassing the Past Get ready for another epic journey through the academic wonders of the academic year 2024-2025 with your trustworthy companion—Oswaal Books! Remember last year’s triumphs? Well, buckle up because we are about to make this year even more awesome! As the legendary dancer Martha Graham once said, “Practice means to perform, repeatedly in the face of all obstacles, some act of vision, of faith, of desire.” We have taken this wisdom to heart and packed it into our brandnew Question Banks for 2024-2025. They are a magical mix of CBSE Board Updates and specially crafted Questions tailored to the Latest Typologies. Oh, and did we mention the fantastic Learning Resources that come with them?

What makes these Question Banks truly exceptional? • 100% Updated Syllabus & With Latest Questions Typologies: We have got you covered with the latest and 100% updated curriculum • Timed Revision with Topic-wise Revision Notes, Smart Mind Maps & Mnemonics: Study smart, not hard! • Extensive Practice with 1000+ Questions & Fully Solved NCERT+Exemplar Questions: To give you 1500+ chances to become a champ!

• Concept Clarity with 500+ Concepts & Concept Videos: For you to learn the cool way—with videos and mind-blowing concepts



• NEP 2020 Compliance with Competency-Based Questions: For you to be on the cutting edge of the coolest educational trends.

If you are looking to conquer every study challenge, these Question Banks are your secret weapon. It is like having a superhero ally for your exams! So, let’s kick off this exciting journey, fill those learning gaps, and rock the year with ease and confidence. Big shoutout to our superhero team—the Oswaal Editorial Board! They’re the brains behind this incredible resource, working day and night just for you. And a massive thank you to you, our fellow Students, Parents & Teachers for your awesome inputs that make this book one-of-a-kind. Wishing you all the best, superheroes-in-the-making! Strive for greatness! Team Oswaal Books

(5)

Syllabus

SOCIAL SCIENCE

Class - IX (Code No. 087) Theory Paper

Time: 3 Hrs.

Max. Marks: 80

History (India and Contemporary World I) Section I Events and Process II

Chapter No

20 inclusive of Map pointing

No. of Periods

Marks Allocated

I

The French Revolution

15

II

Socialism in Europe and the Russian Revolution

15

III

Nazism and the Rise of Hitler

15

IV

Forest, Society and Colonialism Interdisciplinary project as part of multiple assessments (Internally assessed for 5 marks)

Livelihoods, Economies and Societies

Chapter Name

Suggestive no. of periods = 60

V

Pastoralists in the Modern World (To be assessed in the Periodic Assessment only)

5 10

Geography (Contemporary India-I)

Suggestive no. of periods = 55

20 inclusive of Map pointing

Chapter Name

No. of periods

Marks allocated

Chapter no 1

India – Size and Location

2

Physical Features of India

3

Drainage

10

4

Climate

12

5

Natural Vegetation and Wildlife (Only map pointing to be evalualed in the annual examination) Interdisciplinary project as part of multiple assessments (Internally assessed for 5 marks Population

6

18 + 2 Map point

17

Political Science (Democratic Politics - I) Chapter No. Chapter name 1 What is Democracy? 2

Why Democracy? Constitutional Design

3

Electoral Politics

17 + 3 Map point

3 5 8 Suggestive no. of periods = 50 No. of Periods

Marks as mentioned above 20 inclusive of Map pointing No. of marks allocated

10 10 8

4

Working of Institutions

12

5

Democratic Rights

10 Suggestive no. of periods = 50

Economics

(6)

20

20 inclusive of Map pointing

Syllabus Chapter No. Name of the Chapter 1 The Story of Village Palampur (To be assessed in periodic assessment only) People as Resource 2 Poverty as a Challenge 3 4

No. of Periods 10

Marks allocated -

10 15

20

15

Food Security in India

CLASS IX COURSE CONTENT History: India and Contemporary World-I Chapter No. and

Suggestive Teaching Learn-

Learning Outcome With

Name

ing Process

Specific Competencies

I

l Compare & contrast the l Class room discussions to l Compare and contrast the

The French Revolution:

II

Socialism in Europe and the Russian Revolution

Specific Learning Objectives

compare and contrast the conditions prevailed in conditions prevailed in conditions prevailed in France with the situations France that led to revoluFrance that led to revoluprevailed in India pre 1857 tion with the conditions tion with the conditions war. that led to the first war that led to the first war l Critically examine the need of Indian Independence. of Indian Independence. of voting rights of Common (1857). (1857). people at France which laid l Graphic Organisers to crit- l Critically Examine the situations that made the the foundation of future ically examine the situations that made the raise in Democracies. raise in demand of Voting demand of Voting Rights ● Examine various solutions Rights by passive citizens by passive citizens as well to address imbalances that as well as women as women l Propose solutions to admay lead to revolutions. l Debates to propose sodress such imbalances and lutions to address such discriminations that lead imbalances and discrimito revolutions nations that lead to revol Appraise the impact of the lutions l Inquiry based learning to French revolution on the appraise the impact of the world. French revolution on the world. Conclude with group Presentations l Interactive Textual interpretations to compare led to the rise of Russian and contrast the situaand French revolutions. tions that led to the rise l Evaluate the reasons that of Russian& French Revled to the rise of Lenin’s olutions communism and Marxist l Student led seminar to cite the impact of ColSocialism laboration and communication rendered by the philosophers and leaders in creating awareness amongst the common people l World café’ strategy to evaluate the situations that enabled Lenin’s Communism. l Socratic Discussions to Interpret the different ideas of philosophers and leaders that shaped the revolution l Analyse the situations that

(7)

l To compare and contrast

the situations that led to the rise of Russian& French Revolutions. l Cite the impact of Collaboration and communication rendered by the philosophers and leaders in creating awareness amongst the common people l Evaluate the situations that enabled Lenin’s Communism. l Interpret the different ideas of philosophers and leaders that shaped the revolution.

Syllabus III

Nazism and the Rise of Hitler

IV Forest Society and Colonialism V Pastoralists in the Modern World

l Analyse the manipulated

control of situations led by an individual. l Analyse the role of the “Treaty of Versailles in that led to the rise of Hitler. l Examine the circumstances that led to the rise and fall of Hitler l Discuss the critical significance of Nazism in shaping the politics of the modern world. l Appraise the war compensation compelled on Germany in the name of the “treaty of Versailles led to the rise of Hitler l Compare and contrast the Nazi ideology with fascism of Mussolini

l Watch

Video clipping from the last days of Adolf Hitler and discuss the reasons for rise and fall of Hitler l Dramatize the Nazi Propaganda /racial discrimination against Jews l Cartoon interpretation/ Image interpretation l Read passages from “Dairy of Anne frank “and other related literature and discuss the impact of Nazism l Jig saw strategy to critique the genocidal war waged against Jews by the Nazis

l Citing the events that

helped Hitler’s rise to power l Evaluate various character traits of Hitler l Compare and contrast the characteristics of Bismarck and Hitler l Analyse the role of “Treaty of Versailles in the rise of Nazism and Hitler l Critique the genocidal war waged against Jews by the Nazis. l Discuss the critical significance of Nazism in shaping the politics of modern world.

l Inter Disciplinary Project l Refer Annexure II

l Refer Annexure II

l Analyse the situations that

l Compare

with Chapter 5 of Geography “Natural Vegetation and Wild Life

l T charts and similar have created Nomadic sographic organizers to ciety compare and contrast the l Draw comparisons of Aflives of Pastoralists pre & rican pastoralists and Inpost colonialism. dian Pastoralists during l Art integration to depict colonial rule. the evolution of nomadic l Examine the how the colosociety. nial laws impacted liveli- l Research based presehood in pastoral commutations using resources nities provided to compare and l Appraise the contribution contrast the lives and the of Pastoralists in the modreasons for poverty of ern economy. pastoral nomads of India with African Pastoral Nomadic tribes. l Think- Pair- Share and summarize the reading of resources to analyse and infer varying patterns of developments within pastoral societies in different places in India.

and contrast the lives of Pastoralists pre & post colonialism l Analyse the situations that have created Nomadic society l Compare and contrast the lives and the reasons for poverty of pastoral nomads of India with African Pastoral Nomadic tribes. l Analyse and infer varying patterns of developments within pastoral societies in different places in India. l Analyse the impact of colonialism on forest societies leading to scientific forestry. l Enumerate the different processes through which transformation of livelihood occur in the modern world.

Political Science: Democratic politics - I Chapter No. and Name 1 What is Democracy? Why Democracy?

Specific Learning Objectives

Suggested Teaching Learning Process

Learning Outcome With Specific Competencies

● Examine the concept / ● World café and Café ● Compare and Contrast structural components of Democracy and its forms/ features in different countries

conversations strategies for introduction of concepts of ” Democracy, & features of Democracy

(8)

working of democracies of India and North Korea and infer on their differences and significance in each country.

Syllabus ● Examine and analyse the ● 4 corners strategy to dis- ● Define Democracy and

enumerate its features. working structure of the cuss “What & why of de● Evaluate the authenticity governments of India and mocracy? of the voting rights of the North Korea ● students create demoIndian population versus ● Analyse and infer on the cratic governance model the population of Iran. different historical proin the class. ● Interpret the statement cesses and forces that ● Cartoon interpretation to “Democracy provides a have contributed for the summarize the benefits method to deal with difpromotion of democracy. of democracy. ferences and conflicts” with reference to India. ● Summarize the features and benefits of democracy 2





Constitutional Design

Comprehend the purpose of constitution.



Enumerate the essential features that need to be kept in mind while drafting any constitution.





Examine the guiding values that created the Indian constitution.





Comprehend the roles and responsibilities as citizens of India.

3

● Comprehend the concept and system of elections.

Electoral Politics

● Evaluate the conditions that make Elections in India democratic.

Group Discussion to comprehend the purpose of constitution Poster making/ wall magazine for Comparing and contrasting between Preamble of South African constitution with the preamble of Indian constitution Role play strategy for creation of Indian constitution Declamation strategy for roles and responsibilities of citizens





Role play /have school council elections.





Design and present election manifesto





Create multiple parties and create symbols for elections





Use street play to create awareness about the right to vote.





● Analyse the implications of power of vote and power of recall.

● Appraise the role of election commission for the conduct of free and fair elections.

• •





(9)

Analyse the difference between written or unwritten constitutions with reference to India and USA. Describe the situation that led to creation of Indian Constitution. Compare and contrast between Preamble of South African constitution with the preamble of Indian constitution. Enumerate the roles and responsibilities as citizens of India Evaluate the role of political parties to adhere to electoral promises. Create a solution for eradication of malpractices in elections Differentiate between representative democracy and competitive party politics. Summarize the essential features of the Indian Electoral system. Examine the rationale for adopting the present Indian Electoral System.

Syllabus 4



Working of Institutions

• •

Examine the Roles , responsibilities and interdependency of all the 3 organs of the Government. Examine the rule of law in India and its relevance Understand the power and working of Indian Judiciary system and comprehend the hierarchy system of the judiciary in India.



Watch videos of Parliament and discuss the importance of question hour Present Moot court to evaluate the rule of Law Examine the relevant case studies to evaluate the rule of law Present Mock Parliament session to convert a bill into law

• • • •

Democratic Rights

the three organs are interdependent and independent to execute their roles l Summarize and evaluate the rule of law in India. l Represent the role of Parliament and its procedures. l Distinguish between political and permanent executive authorities and functions.

Conduct a mock inter- l Understand the parliaview with a parliamenmentary system of executarian tive’s accountability to Role play on features the legislature. of the political and per- l Understand the working manent executive of Indian Judiciary. Declamation on need • Analyse the need of to have rights and the having rights and catimportance of peregorize the rights. forming duties.

• 5

l Analyse and infer how

● Comprehend what it is







Debate the need to have rights in the light of study of Saudi Arabia.



Evaluate the statement “Democracy is meaningless without rights”



Case study to analyse the role of citizens when the rights are exercised or otherwise.



Analyse their role as responsible citizens.



6 thinking hats to discuss the current issues.



Summarize the flipped coexistence of rights versus duties



Organize a moot court to discuss the violation of individual rights.



Apply the process available to citizens for safeguarding rights.



Graphic organizer to summarize the coexistence of rights vs duties

.

to be a responsible citizen while performing their prescribed duties versus claiming rights Evaluate the role of rights in Democracy.

.

Geography: Contemporary India - I Chapter No. and Name 1

India - Size and Location

Specific Learning Objectives

Suggested Teaching Learning Process

l Examine how the location l Use GeoGebra, Google

of an area impacts its climate and time with reference to longitude and latitude. l Explore and analyze the trading and cultural relationships of India with its neighboring countries.

earth to represent and justify the reasons for the differences in climatic conditions, local and standard time.

( 10 )

Learning Outcome With Specific Competencies



Justify the reasons for the differences in climatic conditions, local and standard time.

Syllabus l Evaluate the situation &

reasons that made 82.5E* longitude as Time meridian of India. l Examine how location of India enables its position as a strategic partner in the subcontinent.

brainstorming strategy for inferring conditions and relationships of the people living in states that are sharing border with the neighbouring countries impact trade and culture. (Link for Carousel Brain storming Strategy https:// w w w. y o u t u b e . c o m / w atch?v=zZxaS7v1-jo)



l On map of India hypo-



l Carousel

thetically design two to four alternate longitudes on either side of 82.5*E and conclude on the selection l Draw out the rationale/ reasons behind fixing (82.5E) as a time meridian for India l PPT presentation to present alternate solutions. 2 Physical Features of India

● Justify why India is a ●

● ●

3 Drainage

sub- continent Examine the geological process that played a crucial role in the formation of diverse physical features in India Analyse the conditions and relationships of the people living in different physiographic areas. Examine various environmental issues.



• •

l Justify the statement that

Use Art integrated strategies like gallery walk/Model making to demonstrate how physical features make India a sub- continent. Role play to depict the lives and relationships amongst physiographic areas. Collaborative brainstorming and presentation using different modes such as flipped books, Journals, Collage and other suitable presentations.





• •



l Choice Board strategy l the rivers are lifeline of where each group to economy with reference take up one river and to India. focus on the areas they l Examine the information serve and the impact on l about different lakes and Economy of that area. infer on their contribution l Students will prepare a to Indian ecology PPT on lakes. l Distinguish between the l Street play strategy/ l rivers of north and south poster making/save India River songs/ to present l Analyse the flow of differawareness on water ent rivers of India to infer pollution and suggest on their impact on livelisolutions. hood. l

( 11 )

To Infer how the conditions and relationships of the people living in states that are sharing border with the neighbouring countries impact trade and culture. Justify the selection of 82.5E* longitude as Time meridian of India. (IST) Critically analyse the role of opening of Suez Canal in improvement of foreign trade. Propose alternative solution for the problems that arise due to the size & location.

Conclude why India is a subcontinent based on study of different physical features. Analyse the conditions and relationships of the people living in different physiographic areas. Enumerate the different environmental issues in India and propose solutions for these issues.

Enlist the different rivers, the areas they serve and their impact on the economy of That area. Enumerate the different lakes and describe their contribution to the Indian ecology. Present creative solutions to overcome the water pollution also to increase the contribution of water bodies to Indian economy Identify the river systems of the country and explain the role of rivers in human society.

Syllabus 4



Climate ●





5 Natural Vegetation and Wild life 6

Examine and analyse l Collect and Read the the factors that deterWeather reports and mine the climate of Inhave a class room disdia cussion to conclude about the factors controlling climate Discuss the mechanisms of monsoons in Indian l Watch videos and summarize the findings subcontinent. l Use Mind map/graphic organizers to EnumerAnalyse and infer ate and summarize the the reasons behind reasons for the wide the wide differdifference between the ence between day and day and night temperanight temperatures at tures at different geodifferent geographical graphical locations of locations of India. India To interpret how mon- l Read Newspapers, prepare and present mock soon acts as a Unifying drills on protocols as bond preventive action for various disasters

• • •





Infer how the factors determine the climate of India. Analyse and infer the effect of monsoon winds on rainfall of the Indian subcontinent. Analyse the temperatures between plateau region, Himalayan region, desert region and coastal region. Enumerate and summarize the reasons for the wide difference between temperatures at different geographical locations of India Propose protocols as preventive action for various disasters

Refer annexure II

Inter disciplinary project with History chapter no IV “Forest Society and Colonialism “

Refer annexure II

● Examine the reasons be-





hind the uneven distribution of population in India with specification to UP & Rajasthan and Mizoram and Karnataka.

Population

.

Research based learning/ art integration strategy (4 grid analysis) to analyse and infer the reasons behind the uneven distribution of population in India with specification to UP & Rajasthan and Mizoram and Karnataka



Analyse and infer the reasons behind the uneven distribution of population in India with specification to UP & Rajasthan and Mizoram and Karnataka. Enlist the factors that affect the population density.

Economics Chapter No. and

Specific Learning Objectives

Suggested Teaching Learn-

Learning Outcome with

ing Process

Specific Competencies

Name 1 The Story of Village Palampur

● Evaluate the prevail● ●

l Visit a nearest village, interview a farmer/ ing farming conditions visit local markets and in different states with interview farmers and reasons present it in the class. Examine the factors of (Experiential learning production and interdestrategy) pendence of the requirel Poster making/ Concept ments. map and gallery walk Examine the contributo Enlist the requiretion of non-farm acments of production tivities to the economic and summarize the ingrowth of the village. terdependence of these . requirements. l Present a business plan for non-farm activities by using the four factors of production.

( 12 )







Analyse and infer how the prevailing farming conditions impact economic development of different states Enlist the requirements of production and summarize the interdependence of these requirements. Enlist non-farm activities and depict the link with economic growth.

Syllabus •

2 People as Resource

Examine the various factors that constitute the quality of population Analyse the role of government in improving the quality of population. Examine the factors that contribute to unemployment.



● Comprehend that pov-







• •

3 Poverty as a Challenge

4



Food Security in India

• •

erty is a multifaceted concept inherent in the rural and urban conditions. Examine the measures taken by the government to eradicate poverty.

Examine the critical role of food security for its masses. Justify the rationale for the system of food security in India. Appraise the contributory role of Public Distribution system to address FSI











Substantiate the role of green revolution in strengthening the PDS.

Case study on quality of population. (Class room discussion) Collect sources from Newspaper/ Media and present the findings in the form of a collage or an album Neighbourhood survey on employment / employability in neighbourhood, analyse the quality of neighbourhood and present in PPT format.



PPT presentation using case study given in NCERT text on the reasons of rural and urban poverty. Declamation with data to Evaluate the efficacy of government to eradicate poverty Debate whether education can remove poverty



Case study and group discussion to substantiate the link between a wellstructured food security system and continuity of supply to masses. Invite relevant Govt. officials to speak on FSI &PDS.



Panel discussion/seminar on the impact of the green revolution and PDS.





• •



Analyse and infer the reasons that contribute to the quality of population Enumerate the different schemes of Government in some states and infer on the quality of people there by. Propose solutions to resolve unemployment problem

Analyse and infer the reasons of poverty in the rural and urban areas. Evaluate the efficacy of government to eradicate poverty. Compare how poverty estimates have transformed from 1993-94 to 2011-12 Examine the link between education and poverty. Enumerate various aspects of food security that will ensure continuity of supply to the masses. Examine, analyse and infer various sources of data that point to the rationale of FSI.



Enumerate different features of PDS that directly address FSI.



Analyse and infer the impact of Green revolution in strengthening the PDS.

CLASS IX LIST OF MAPS S. No. I

Subject History

Name of the

List of Areas to Be Map Pointed

Chapter French Revolution

Outline political map of France Locate/label/identify; • Bordeaux, Nantes, Paris, Marseille

Socialism in Europe

Outline political map of world locate/label/identify major countries of World War (central powers- Germany, Austria-Hungary, Turkey (Ottoman Empire) Allied Powers-France, England, Russia, USA)

( 13 )

Syllabus II

Geography

India: size &location

● ● ●

India-States with Capitals, Tropic of Cancer, Standard Meridian (Location and Labelling) Neighbouring countries

India physical features



Mountain Ranges: The Karakoram, The Zasker, The Shivalik, The Aravali, The Vindhya, The Satpura, Western & Eastern Ghats Mountain Peaks – K2, Kanchan Junga, Anai Mudi Plateau - Deccan Plateau, Chota Nagpur Plateau, Malwa Plateau Coastal Plains - Konkan, Malabar, Coromandel & Northern Circar (Location and Labelling)

● ● ● Drainage system

Rivers: (Identification only) ● The Himalayan River Systems-The Indus, The Ganges, and The Sutlej ● The Peninsular Rivers-The Narmada, The Tapi, The Kaveri, The Krishna, The Godavari, The Mahanadi ● Lakes: Wular, Pulicat, Sambhar, Chilika

Climate

● Percentage of rainfall in India, Monsoon wind directions

Population

● ●

Population density of all states The state having highest and lowest density of population

CLASS IX INTERNAL ASSESSMENT: 20 MARKS Type of Assessment

Description

Marks Allocated

Periodic assessment

Pen Paper Test.

5

Multiple assessment

Inter disciplinary project

5

(Quiz, debate, role play, viva, group discussion, visual expression, interactive bulletin boards, gallery walks, exit cards, concept maps, peer assessment, Self-assessment etc.) Subject enrichment activity

Project

5

Portfolio

Classwork, work done (activities / assignments) reflections, narrations, journals, etc. Achievements of the student in the subject throughout the year Participation of the student in different activities like heritage India quiz

5

CLASS IX PRSECRIBED TEXT BOOKS S.No

Subject

Name of the Book

Publisher

1

History

India and contemporary world-I

NCERT

2

Political Science

Democratic Politics

NCERT

3

Geography

Contemporary India-I

NCERT

4

Economics

Economics

NCERT

5

Disaster management

Together, towards a safer India - part II

CBSE

( 14 )

Exclusive School Books Suppliers VIJAYAWADA WEST KAMENG BANGLORE RAJKOT

MAHARASHTRA

ANDHRA PRADESH

Sri Vikas Book Centre, 9848571114, 9440715700,

PUNE

ASSAM

JALNA

Dutta Book Stall, 8729948473

KARNATAKA

CHENNAI

GUJRAT

HYDERABAD

Satish Agencies, 8861630123 Royal Stationers, 9824207514

ANDHRA PRADESH

KOLKATA

PORTBLAIR GUWAHATI

PATNA

Anil Paper Mart, 9422722522, (02482) 230733

TAMIL NADU

Bookmark-IT, 7305151653

TELANGANA

Sri Balaji Book Depot , 9676996199, (040) 27613300

WEST BENGAL

United Book House, 9831344622

Our Distributors

VISAKHAPATHAM JBD Educational, 9246632691, 9246633693, Sri Rajeshwari Book Link, 9848036014 VIJAYAWADA

Madhusheela Books & Stationery, 7875899892

INDORE

Akshaya Books Corner, 9666155555

Bhaiya Industries, 9893326853, Sushil Prakashan,(0731) 2503333, 2535892, 9425322330, Bhaiya Store, 9425318103, Arun Prakashan, 9424890785, Bhaiya Book Centre, 9424081874, Seva Suppliers, 9826451052

ANDAMAN & NICOBAR ISLAND

Krishna Book Centre, 9474205570, Kumar Book Depot, 9932082455, Kumar Book Depot, 9932082455, Sree aditya Book Centre, 8332972720, 7013300914

ASSAM

Book Emporium, 9675972993, 6000763186, Ashok Publication, 7896141127, Kayaan Enterprises, (0361) 2630443, Orchid Book house, 9864624209, Newco, 9864178188

JABALPUR

Vinay Pustak Sadan, 8962362667, Anand Books and Stationers, 9425323508

SAGAR

Princi Book Depot, Sagar, 9977277011

KATNI

Shri Mahavir Agency, 9425363412

BHOPAL

Gupta Brother, 9644482444

UJJAIN

BIHAR

Nova Publisher & Distributors, (0612) 2666404, Shri Durga Pustak Mandir, 9334477386, Sharda Pustak Bhandar, 9334259293, Vikas Book Depot, 9504780402, Alka Book Agency, 9835655005, Metro Book(E&C), Ishu Pustak Bhandar, 8294576789, Gyan Ganga Limited, 6203900312, Ishu Pustak Bhandar, ( E & C ), 9334186300/8294576789

PUNE

Shreenath Book Depot, 9827544045

MAHARASHTRA

Natraj Book Depot, (020) 24485054, 9890054092, Vikas Book House, 9921331187, Pravin Sales, 9890683475, New Saraswati Granth Bhandar, 9422323859, Akshar Books & Stationary, 7385089789, Vardhaman Educational, 9860574354, Yash Book Centre, 9890156763, Pragati Book Centre, (ISC), 9850039311, Praveen Sales, Pragati Book Centre, Pune ( E & C ), 9850039311 Shree Sainath Agencies, 7350294089, Maya Book Centre, (ISC), 9372360150 Vidyarthi Sales Agencies, 9819776110, New Student Agencies, 7045065799, Shivam Books & Stationery, 8619805332

Pustak Bhandar, 7870834225

CHATTISGARH

AURANGABAD MUMBAI

AMBIKAPUR

Saini Brothers, 9425582561, M.P Department Stores, 9425254264

JALGAON

BOKARO BHILAI

Bokaro Student Friends Pvt. Ltd, Bokaro, 7277931285 Anil Book Depot, 9425234260

LATUR KOLHAPUR

Yash Book House, 9637936999, Shri Ganesh Pustakalay, 9730172188 Granth the Book World, 9922295522

KORBA

Kitab Ghar, Korba ( E & C ), 9425226528

NANDED

MUZAFFARPUR

DURG

RAIPUR RAIGARH DELHI

Bhagwati Bhawani Book Depot, 0788-2327620, 9827473100

NAGPUR

Sharma Book Depot & Stat. (ISC), 9421393040

Laxmi Pustakalay and Stationers, (0712) 2727354, Vijay Book Depot, 9860122094

Shri Ramdev Traders, 9981761797, Gupta Pustak Mandir, 7974220323, Anil Publication, 9691618258/7999078802

NASHIK

Renuka Book distributor, 9765406133, Novelty Book Depot, 9657690220, Karamveer Book Depot, 9923966466, Arun Book & Stationers, 9423110953 Abhang Pustakalaya, 9823470756/9175940756 Rahul Book Centre, 9970849681, New India Book House, 9623123458

DELHI

YAVATMAL

Shri Ganesh Pustkalaya, 9423131275

Sindhu Book Deopt, 9981935763

DHULE

Mittal Books, (011) 23288887, 9899037390, Singhania Book & Stationer, 9212028238, AoneBooks, New Delhi, 8800497047, Radhey Book Depot, 9818314141, Batheja Super Store, 9871833924, Lov Dev & Sons, Delhi ( E & C ), 9811182352, Zombozone, 9871274082, LDS Marketing, 9811182352/9999353491

VASAI

Navjeevan Book Stall, 7020525561

Prime Book Centre, Vasai, 9890293662

ODISHA

GUJARAT

CUTTACK

Mahavir Stationers, 9429474177 College Store, (ISC) NO CALL 02637-258642, 9825099121

AMBALA PATIALA

VADODARA

Umakant Book Sellers & Stationer, 9624920709

HARYANA

FEROZPUR LUDHIANA

ROHTAK

Manish Traders, 9812556687, Swami Kitab Ghar, 9355611088,

CHANDIGARH

REWARI

Sanjay book depot, 9255447231

A. K. Mishra Agencies, 9437025991, 9437081319

BHAVNAGAR DAHOD VAPI

Patel Book, 9898184248, 9824386112, 9825900335, Zaveri Agency, 9979897312, 9979890330, Hardik Book Agency, (ISC) 079-24110043, 9904659821 Samir Book Stall, Bhavnagar (ISC) 9586305305 Collegian Book Corner, 9925501981 Goutam Book Sellers, 9081790813

BHUBANESHWAR M/s Pragnya, 8847888616, 9437943777, Padmalaya, 9437026922, Bidyashree, 9937017070, Books Godown, 7894281110 BARIPADA Trimurti Book World, 9437034735 KEONJHAR Students corner, 7008435418

SURAT

Shopping Point, 9824108663

JALANDHAR

VALSAD NAVSARI

BALLABGARH HISAR BHUNA

BOKARO RANCHI DUMKA HUBLI BANGLORE

BELLERI ERNAKULAM KOTTAYAM TRIVANDRUM CALICUT

CHHINDWARA GWALIOR

Babu Ram Pradeep Kumar, 9813214692

PUNJAB

Bharat Book Depot, 7988455354 Goel Sons, 9463619978, Adarsh Enterprises, 9814347613

Cheap Book Store, 9872223458, 9878258592, City Book Shop, 9417440753, Subhash Book Depot, 9876453625, Paramvir Enterprises, 9878626248 Sita Ram book Depot, 9463039199, 7696141911 Amit Book, 9815807871, Gupta Brothers, 9888200206, Bhatia Book Centre, 9815277131 Mohindra Book Depot, 9814920226

RAJASTHAN

Laxmi General Store, Ajmer, 0145- 2428942 9460652197 Vardhman Book Depot, 9571365020, 8003221190 Raj Traders, 9309232829

Kashi Ram Kishan lal, 9289504004, 8920567245 Natraj Book Distributors, 7988917452

AJMER KOTA

JAMMU

JAIPUR

Sahitya Sangam, 9419190177

UDAIPUR

Nakoda Book Depot, (01482) 243653, 9214983594, Alankar Book Depot, 9414707462 Ravi Enterprises, 9829060694, Saraswati Book House, (0141) 2610823, 9829811155, Goyal Book Distt., 9460983939, 9414782130 Sunil Book Store, 9828682260

AGARTALA

Book Corner, 8794894165, 8984657146, Book Emporium, 9089230412

Khurana Book Store, 9896572520

BHILWARA

JHARKHAND

JODHPUR

Bokaro Student Friends, (0654) 2233094, 7360021503, Bharati Bhawan Agencies, 9431740797

Crown Book Distributor & Publishers, (0651) 2213735, 9431173904, Pustak Mandir, 9431115138, Vidyarthi Pustak Bhandar, 9431310228

KARNATAKA

COIMBATORE

Renuka Book Distributor, (0836) 2244124, Vidyamandir Book Distributors, 9980773976 CHENNAI Krishna book house, 9739847334, Hema Book Stores, 9986767000, Sapna Book House Pvt. Ltd., 9980513242, Hema Book World, (Chamrajpet) (ISC) 080-40905110, 9945731121 Chatinya book centre, 9886064731

PUDUCHERRY

Academic Book House, (0484) 2376613, H & C Store, 9864196344, Surya Book House, 9847124217, 9847238314 Book Centre, (0481) 2566992 Academic Book House, (0471) 2333349, 9447063349, Ponni Book Stall, 9037591721

TRICHY

KERALA

Aman Book Stall, (0495) 2721282,

MADHYA PRADESH

Pustak Bhawan, ( E & C ), 8982150100

SALEM

THENI MADURAI VELLORE

HYDERABAD

Second Hand Book Stall, 9460004745

TRIPURA

TAMIL NADU

Majestic Book House, (0422) 2384333, CBSC Book Shop, 9585979752

Arraba Book Traders, (044) 25387868, 9841459105, M.R. Book Store (044) 25364596, Kalaimagal Store, (044) 5544072, 9940619404, Vijaya Stores, 9381037417, Bookmark It-Books & Stat. Store, 7305151653, M.K. Store, 9840030099, Tiger Books Pvt. Ltd., 9710447000, New Mylai Stationers, 9841313062, Prince Book House, Chennai, 0444-2053926, 9952068491, S K Publishers & Distributors, 9789865544, Dharma Book Shop, 8667227171 Sri Lakshmi Book Seller, 7871555145 Pattu book centre, 9894816280

P.R.Sons Book Seller, 9443370597, Rasi Publication, 9894816280 Maya Book Centre, 9443929274 Selvi Book Shoppe, 9843057435, Jayam Book Centre, 9894658036 G.K book centre and collections, 9894517994

TELANGANA

Sri Balaji Book Depot, (040) 27613300, 9866355473, Shah Book House, 9849564564 Vishal Book Distributors, 9246333166, Himalaya Book World, 7032578527

Agarwal Book Depot, 9425116210

0808

AHMEDABAD

( 15 )

Exclusive School Books Suppliers UTTARAKHAND

GORAKHPUR

Central Book House, 9935454590, Friends & Co., 9450277154, Dinesh book depot, 9125818274, Friends & Co., 9450277154

DEHRADUN

Inder Book Agencies, 9634045280, Amar Book Depot , 8130491477, Goyal Book Store, 9897318047, New National Book House, 9897830283/9720590054

JHANSI

Bhanu Book Depot, 9415031340

MUSSORIE

Ram Saran Dass Chanda kiran, 0135-2632785, 9761344588

KANPUR

Radha News Agency, 8957247427, Raj Book Dist., 9235616506, H K Book Distributors, 9935146730, H K Book Distributors, 9506033137/9935146730

UTTAR PRADESH

LUCKNOW

Vyapar Sadan, 7607102462, Om Book Depot, 7705871398, Azad Book Depot Pvt. Ltd., 7317000250, Book Sadan, 9839487327, Rama Book Depot(Retail), 7355078254,

Ashirwad Book Depot, 9235501197, Book.com, 7458922755, Universal Books,

9450302161, Sheetla Book Agency, 9235832418, Vidyarthi Kendra Publisher & Distributor Pvt Ltd, (Gold), 9554967415, Tripathi Book House, 9415425943

AGRA

Sparsh Book Agency, 9412257817, Om Pustak Mandir, (0562) 2464014, 9319117771,

MEERUT

ALLAHABAD

Mehrotra Book Agency, (0532) 2266865, 9415636890

NOIDA

Prozo (Global Edu4 Share Pvt. Ltd), 9318395520, Goyal Books Overseas Pvt.Ltd., 1204655555 9873387003

AZAMGARH

Sasta Sahitya Bhandar, 9450029674

PRAYAGRAJ

Kanhaiya Pustak Bhawan, 9415317109

ALIGARH

K.B.C.L. Agarwal, 9897124960, Shaligram Agencies, 9412317800, New Vimal Books, 9997398868, T.I.C Book centre, 9808039570

MAWANA

Subhash Book Depot, 9760262264

BULANDSHAHAR

Rastogi Book Depot, 9837053462/9368978202

BALRAMPUR

Universal Book Center, 8933826726

KOLKATA

BAREILLY

Siksha Prakashan, 9837829284

RENUKOOT

HARDOI

Mittal Pustak Kendra, 9838201466

DEORIA

Kanodia Book Depot, 9415277835

COOCH BEHAR

S.B. Book Distributor, Cooch behar, 9002670771

VARANASI

Gupta Books, 8707225564, Bookman & Company, 9935194495/7668899901

KHARAGPUR

Subhani Book Store, 9046891334

MATHURA

Sapra Traders, 9410076716, Vijay Book House , 9897254292

SILIGURI

Agarwal Book House, 9832038727, Modern Book Agency, 8145578772

FARRUKHABAD

Anurag Book Agencies, 8844007575

DINAJPUR

Krishna Book House, 7031748945

NAJIBABAD

Gupta News Agency, 8868932500, Gupta News Agency, ( E & C ), 8868932500

MURSHIDABAD

New Book House, 8944876176

DHAMPUR

Ramkumar Mahaveer Prasad, 9411942550

Sanjay Publication, 8126699922 Arti book centre, 8630128856, Panchsheel Books, 9412257962, Bhagwati Book Store, (E & C), 9149081912

Ideal Book Depot, (0121) 4059252, 9837066307

WEST BENGAL Oriental Publishers & Distributor (033) 40628367, Katha 'O' Kahini, (033) 22196313, 22419071, Saha Book House, (033), 22193671, 9333416484, United Book House, 9831344622, Bijay Pustak Bhandar, 8961260603, Shawan Books Distributors, 8336820363, Krishna Book House, 9123083874

Om Stationers, 7007326732

Entrance & Competition Distributors PATNA

BIHAR

CUTTAK

A.K.Mishra Agencies, 9437025991

Metro Books Corner, 9431647013, Alka Book Agency, 9835655005, Vikas Book Depot, 9504780402

BHUBANESHWAR

M/s Pragnya, 9437943777

CHATTISGARH KORBA

Kitab Ghar, 9425226528, Shri Ramdev Traders, 9981761797

PUNJAB JALANDHAR

DELHI

Cheap Book Store, 9872223458, 9878258592

RAJASTHAN

DELHI

Singhania Book & Stationer, 9212028238, Radhey Book depot, 9818314141, The KOTA Book Shop, 9310262701, Mittal Books, 9899037390, Lov Dev & Sons, 9999353491

Vardhman Book Depot, 9571365020, Raj Traders, 9309232829

NEW DELHI

Anupam Sales, 9560504617, A ONE BOOKS, 8800497047

Goyal Book Distributors, 9414782130

JAIPUR

HARYANA AMBALA

BOKARO

UTTAR PRADESH

Bharat Book Depot, 7988455354

AGRA

BHAGWATI BOOK STORE, 9149081912, Sparsh Book Agency, 9412257817, Sanjay Publication, 8126699922

JHARKHAND

ALIGARH

New Vimal Books, 9997398868

Bokaro Student Friends Pvt. Ltd, 7360021503

ALLAHABAD

Mehrotra Book Agency, (532) 2266865, 9415636890 Central Book House, 9935454590

MADHYA PRADESH

GORAKHPUR

INDORE

Bhaiya Industries, 9109120101

KANPUR

Raj Book Dist, 9235616506

CHHINDWARA

Pustak Bhawan, 9827255997

LUCKNOW

Azad Book Depot PVT LTD, 7317000250, Rama Book Depot(Retail), 7355078254 Ashirwad Book Depot , 9235501197, Book Sadan, 8318643277, Book.com , 7458922755, Sheetla Book Agency, 9235832418

MAHARASHTRA

PRAYAGRAJ

Format Center, 9335115561, Garg Brothers Trading & Services Pvt. Ltd., 7388100499

NAGPUR

Laxmi Pustakalay and Stationers, (0712) 2727354

PUNE

Pragati Book Centre, 9850039311

MUMBAI

New Student Agencies LLP, 7045065799

ODISHA

DEHRADUN

Inder Book Agancies, 9634045280

WEST BENGAL KOLKATA

Bijay Pustak Bhandar Pvt. Ltd., 8961260603, Saha Book House, 9674827254 United Book House, 9831344622, Techno World, 9830168159

Trimurti Book World, 9437034735

0808

BARIPADA

UTTAR PRADESH

( 16 )

UNIT-I

India and the Contemporary World-I

(Section-I : Events and Processes)

CHAPTER

1

Learning Objectives

THE FRENCH REVOLUTION

l Compare & contrast the conditions prevailed in France with the situations prevailed in India pre 1857 war. l Critically examine the need of voting rights of Common people at France which laid the foundation of future Democracies. l Examine various solutions to address imbalances that may lead to revolutions.

French Society During the Late

Topic-1 Eighteenth Century Revision Notes th

Study Time Max Time: 3:30 hr Max Questions: 81

TOPIC - 1 French Society During the Late Eighteenth Century .... P. 01 TOPIC - 2 The Outbreak of the Revolution and France Abolishes Monarchy and Becomes a Republic .... P. 06 TOPIC - 3 Women Revolution and the abolition of of slavery .... P. 13

 On the morning of 14 July 1789, the city of Paris was in a state of alarm. The King had commanded the troops to move into the city. Rumours spread that the King would soon order the army to open fire upon the citizens. Some 7,000 men and women gathered in front of the town hall and decided to form a peoples’ militia. They broke into a number of government buildings in search of arms.  Finally, a group of several hundred people marched towards the eastern part of the city and stormed the fortressprison, the Bastille, where they hoped to find hoarded ammunition. In the armed fight that followed, the commander of the Bastille was killed and the prisoners were released.  The days that followed saw more rioting both in Paris and the countryside. Actually, it was the beginning of a chain of events that ultimately led to the execution of the king in France. Scan to know  In 1774, Louis XVI of the Bourbon family of Kings ascended the throne of France. more about  Causes for an empty treasury of France under Louis XVI: this topic (i) Long years of war had drained the financial resources of France. (ii) High cost of maintaining an extravagant court at the Palace of Versailles. (iii) Under Louis XVI, France helped the thirteen American colonies to gain their independence from the common enemy, Britain. (iv) The war added more than a billion Livres to a debt that had already risen to more than 2 The French billion Livres. Revolution (v) Lenders, who gave the state credit, now began to charge 10 per cent interest on loans.  The French Society in the eighteenth century was divided into three Estates, and only members of the Third Estate paid taxes. (i) The members of the First Estate, that was the clergy, enjoyed certain privileges by birth. The most important of these privileges was exemption from paying taxes to the State. (ii) The members of the Second Estate were the nobility. They enjoyed feudal privileges by birth. These included feudal dues, which they extracted from the peasants. They were also exempted from paying taxes. (iii) The Third Estate comprised peasants, artisans, landless labour, servants, lawyers, doctors, administrative officials, traders, etc., they had to pay all taxes to the State.

2

Oswaal CBSE Question Bank Chapterwise & Topicwise, SOCIAL SCIENCE, Class-IX

THE FRENCH REVOLUTION

3

 Peasants made up about 90 per cent of the population.  The Church too extracted its share of taxes called 'tithes' from the peasants.  All members of the Third Estate had to pay taxes to the state including a direct tax, called ‘taille’ and a number of indirect taxes which were levied on articles of everyday consumption like salt or tobacco.

The Struggle to Survive  Increasing population of France led to a rapid increase in the demand for food grains. But the production of grains could not keep pace with the demand. So, the price of bread rose rapidly.  Wages of the workers did not keep pace with the rise in prices. So, the gap between the poor and the rich widened.  Things became worse whenever drought or hail reduced the harvest. This led to a subsistence crisis, something that occurred frequently in France during the Old Regime.

A Growing Middle Class Envisages an End to Privileges  The eighteenth century witnessed the emergence of social groups, termed the middle class. They earned their wealth through an expanding overseas trade and from the manufacture of goods such as woollen and silk textiles that were either exported or bought by the richer members of the society.  Members of the middle class were educated and believed that no group in the society should be privileged by birth. Rather, a person’s social position must depend on his merit.  In his Two Treatises of Government, Locke sought to refute the doctrine of the divine and absolute right of the monarch. Rousseau carried the idea forward, proposing a form of government based on a social contract between people and their representatives.  In The Spirit of the Laws, Montesquieu proposed a division of power within the government between the legislative, the executive and the judiciary.

Key Terms  Bastille: The Bastille was a fortress in Paris that was used as a state prison by the Kings of France.  Bourbon family: It is the name of the Royal family of French origin, members of which became rulers of several European countries.  Taille: A tax levied on the common people by the kings in France before 1789.  Tithes: A tax levied by the Church, comprising one-tenth of the agricultural produce.  Constitutional Monarchy: Form of government in which a king or queen acts as Head of State.  Livre: A unit of currency formerly used in France.

Key Fact  The storming of the Bastille is still honoured in France as a national holiday.

Key Dates  10th May, 1774: Louis XVI ascended the throne after the death of Louis XV.  1789: The year of subsistence crisis.

OBJECTIVE TYPE QUESTIONS A Multiple Choice Questions Q. 1. The city of Paris is located in: (A) Italy (B) England (C) France (D) Portugal Ans. Option (C) is correct.

U

(1 mark each)

Q. 2. In this book, Montesquieu proposed a division of power within the Government between the Legislative, the Executive and the Judiciary. R Identify the book. (A) The Social Contract (B) The Spirit of the Laws (C) Two Treatises of Government

4

Oswaal CBSE Question Bank Chapterwise & Topicwise, SOCIAL SCIENCE, Class-IX

(D) Das Capital R Ans. Option (B) is correct. Explanation: In this treatise, Montesquieu pleads for a constitutional system of government with separation of powers, the preservation of legality and civil liberties, and the end of slavery. Q. 3. Who among the following sought to refute the Doctrine of the Divine and Absolute right of the R Monarch?  (A) Locke (B) Rousseau (C) Aristotle (D) Plato Ans. Option (A) is correct. Explanation: Locke was from the reign of King Charles I. He believed that all humans are equal. So, through his book 'Two Treatises of Government', he sought to refute the Doctrine of the Divine and Absolute right of the Monarch. Q. 4. Choose the correct option from column I and II. R  Column I )a( Livre )b( Tithe )c( Clergy )d( Taille

)i( )ii( )iii( )iv(

Column II Unit of currency in France Tax to be paid directly to the state A tax levied by the Church Group of persons invested with special functions in the Church

(A) a-i, b-ii, c-iii, d-iv (B) a-ii, b-i , c-iv, d-iii (C) a-i, b-iii, c-iv, d-ii (D) a-iv, b-i, c-iii, iv-ii Ans. Option (C) is correct Q. 5. Study the given image carefully and answer the R question that follows:



Which of the following options best signifies this cartoon? (A) Demolition of Bastille (B) Democracy of People (C) People revolt against the king (D) None of the above

Ans. Option (A) is correct Explanation: This is a picture of the demolition or storming of the Bastille which took place on 14th July 1789.

B Assertion and Reason MCQs Directions: In the following questions, a statement of Assertion (A) is followed by a statement of Reason (R). Mark the correct choice as: (A) Both Assertion (A) and Reason (R) are true, and Reason (R) is the correct explanation of Assertion (A). (B) Both Assertion (A) and Reason (R) are true, but Reason (R) is not the correct explanation of Assertion (A). (C) Assertion (A) is true, but Reason (R) is false. (D) Assertion (A) is false, but Reason (R) is true. Q. 1. Assertion (A): So the price of bread which was the staple diet of the majority rose rapidly. Reason (R): Production of grains could not keep U pace with the demand. Ans. Option (A) is correct. Explanation: The population of France rose from about 23 million in 1715 to 28 million in 1789. This led to a rapid increase in demand for food grains. Production of grains could not pace with the demand. So the price of bread which was the staple diet of the majority rose rapidly. Q.2. Assertion (A): In the French Society of Estates, peasants made up to about 40% of the population. Reason (R): Only a small number of them owned E the land they cultivated. Ans. Option (D) is correct. Explanation: In the French Society of Estates, peasants made up to 90% of the population. However, only a small number of them owned the land they cultivated. About 60% of the land was owned by richer members of the Third Estate, Clergy and Nobles. Q. 3. Assertion (A): Tithe was a tax levied by the Church. Reason (R): It comprised one-tenth of the E agricultural produce.  Ans. Option (B) is correct

SUBJECTIVE TYPE QUESTIONS Very Short Answer Type Questions Q. 1. Name any two ‘Estates’ into which the French R society was divided before the Revolution.   Ans. The French society was divided into:

(2 marks each)

(i) The First Estate: Clergy (ii) The Second Estate: Nobility (iii) The Third Estate: Common people 

(Any two points) (1 × 2 = 2)

THE FRENCH REVOLUTION

Q. 2. Who were included in the Third Estate?

Q. 4. What led to the subsistence crisis?

grains could not keep pace with the demand. So, the price of bread which was the staple diet of the majority rose rapidly. Most workers received fixed wages which did not keep pace with the rise in prices. Things became worse whenever drought or hail reduced the harvest. This led to the subsistence crisis. (2)

R

Ans. The Third Estate was made up of peasants, artisans, landless labour, servants, lawyers, doctors, administrative officials, traders, etc. (Any four) (2) Q. 3. Describe the status of the nobles in France before E the revolution. Ans. The clergy and the nobles led a life of luxury and enjoyed many privileges. Whereas, the peasants and workers lived a miserable life. They had to pay heavy taxes and were forced into labour. The middle-class comprising lawyers, doctors, teachers, etc. also suffered humiliation at the hands of the clergy and the nobles. (2) A

Ans. The population of France rose from about 23 million in 1715 to 28 million in 1789. Production of

Q. 5. What did The Spirit of the Laws by Montesquieu propose? U Ans. In the Spirit of the Laws, Montesquieu proposed a division of power within the government between the legislative, the executive and the judiciary. This model of government was put into force in the USA, after the thirteen colonies declared their independence from Britain. (2)

Short Answer Type Questions Q. 1. How was the taxation policy responsible for the U French Revolution? Explain.  [Board Term-I, 2016] Ans. Refer to Long Ans 1.

Commonly Made Error

Very few students mentioned about Tithes,

Tailles and other indirect taxes peasants had to pay.

Answering Tip

Students should learn & understand about the taxation policy of France in 18th century.

Q. 2. Explain the composition of the French society during the late 18th century.R [Board Term-I, 2015] Ans. Composition of the French Society was: (i) Peasants, artisans and landless made up about 90 per cent of the population. (ii) 60 per cent of the land was owned by nobles and clergy. The clergy and nobility enjoyed certain privileges. (iii) The members of first two estates enjoyed privileges and paid no taxes. (iv) Peasants were obliged to render services to the Lord. The Third Estate had to pay various taxes. This led to an unrest among the Third Estate.  (Any three) (1 × 3 = 3)  [CBSE Marking Scheme, 2016]

5

(3 marks each)

Q. 3. Describe the status of the nobles in France before A [DDE Term-I, 2014] the Revolution. Ans. (i) The clergy and the nobles led a life of luxury and enjoyed numerous privileges. (ii) The middle class comprising lawyers, doctors, teachers, etc., also suffered humiliation at the hands of the clergy and the nobles. (iii) The nobles further enjoyed feudal privileges. (iv) Nobles were exempted from paying taxes.  (Any three) (1 × 3 = 3)  [CBSE Marking Scheme, 2014] Q. 4. Describe any three situations which compelled E Louis XVI to raise taxes in France. Ans. (i) Wars and Economic Crisis: Upon his accession, Louis XVI found an empty treasury. Long years of war had drained the financial resources of France. The nation had gone into deep debt because of the fighting in the Seven Years War (1756–1763) and the Revolutionary War in America under Louis XVI. In this war, France helped the 13 American colonies to gain their independence from Britain. The war added more than a billion Livres to a deb that had already risen to more than 2 billion Livres. (ii) Debt Trap: Lenders who gave the state credit, now began to charge 10 per cent interest on loans. So the French Government was obliged to spend an increasing percentage of its budget on interest payments alone. To meet its regular expenses, such as the cost of maintaining an army, the court, running government offices or universities the state was forced to increase taxes. (iii) Extravagant Court: Added to this was the cost of maintaining an extravagant court at the immense palace of Versailles. (Any three) (1 × 3 = 3)

6

Oswaal CBSE Question Bank Chapterwise & Topicwise, SOCIAL SCIENCE, Class-IX

Q. 5. What was the place of the monarch in France of the U Old Regime? Ans. In France of the Old Regime : (i) The monarch did not have the power to impose taxes according to his will alone. (ii) Rather he had to call a meeting of the Estates General which would then pass his proposals for

new taxes. (iii) The Estates General was a political body to which the three estates sent their representatives. However, the monarch alone could decide when to call a meeting of this body.  (Any three) (1 × 3 = 3)

Long Answer Type Questions Q. 1. Describe the social conditions that led to the outbreak of the French Revolution. R [Board Term-I, 2016] Ans. The following social conditions led to the outbreak of French Revolution: (i) The French society was divided into three Estates. (ii) The clergy and nobility enjoyed special privileges such as exemption from paying taxes. (iii) The Church too extracted its share of taxes called ‘Tithes’ from all members of the Third Estate. They had to pay taxes to the state as well. These included a direct tax, called ‘Taille’, and a number of indirect taxes which were levied on articles of everyday consumption like salt or tobacco. (iv) The nobility enjoyed feudal privileges like feudal dues and services by peasants. (v) All tax burden was on the Third Estate. So common people were very unhappy. (1×5=5)  [CBSE Marking Scheme, 2016] Q. 2. “A growing middle class in France during the 18th century envisages an end to privileges.” Analyse the statement. A [Board Term I, 2016] Ans. (i) The 18th century witnessed emergence of Middle Class. (ii) It earned wealth from manufacturing and trade. (iii) It included professionals like lawyers and administrative officers.

(5 marks each)

(iv) People were educated and believed that no group of society should be privileged. (v) All these ideas were put forward by philosophers like John Locke, Rousseau and Montesquieu. [CBSE Marking Scheme, 2016] (1×5=5) Q. 3. Describe the incident which took place in the morning of 14th July 1789 in France. R [Board Term I, 2015] Ans. The incident that took place on the morning of 14th July 1789 in France was: (i) Some 700 men and women gathered in front of the town hall and decided to form a people’s militia. (ii) They broke into a number of government buildings in search of arms. (iii) Finally, a group of several hundred people marched towards the eastern part of the city and stormed the fortress-prison, the Bastille. (iv) The commander of the Bastille was killed and the prisoners released, though there were only seven of them. (v) The fortress was demolished and its stone-fragments sold in the markets to all those who wished to keep a souvenir of its destruction. [CBSE Marking Scheme, 2015] (1 × 5 = 5)

Topic-2 The Outbreak of the Revolution Revision Notes  Louis XVI planned to impose further taxes to be able to meet the expenses of the state.  On 5th May 1789, Louis XVI called together an assembly of the Estates General to pass proposals for new taxes. Peasants, artisans and women were denied entry to the assembly and from participating in the meeting.  Voting in the Estates General in the past had been conducted according to the principle that each estate had one vote. But this time, members of the third estate demanded that voting should be conducted by the assembly as a whole, where each member would have one vote. When the King rejected this proposal, members of the third estate walked out of the assembly in protest.

Scan to know more about this topic

The Outbreak of the French Revolution

THE FRENCH REVOLUTION

7

 On 20th June 1789, the representatives of the third estate assembled in the hall of an indoor tennis court in the grounds of Versailles under the leadership of Mirabeau and Abbé Sieyès. They declared themselves a National Assembly and swore not to disperse till they had drafted a constitution for France that would limit the powers of the monarch.  Because of bad harvest, there was an increase in demand for food grains and which led to high price of bread. Due to this, people of France were becoming angry day by day. At the same time, the King ordered the troops to move into Paris. Finally, on 14th July, the agitated crowd stormed and destroyed the Bastille.  On the night of 4th August 1789, the Assembly passed a decree abolishing the feudal system of obligations and taxes. Members of the clergy too were forced to give up their privileges. Tithes were abolished and lands owned by the Church were confiscated. As a result, the government acquired assets worth at least 2 billion Livres. rance Becomes a Constitutional Monarchy F  The National Assembly completed the draft of the constitution in 1791. Its main objective was to limit the powers of the monarch.  These powers were now separated and assigned to different institutions – The Legislature, Executive and Judiciary. This made France a constitutional monarchy.  The constitution began with a Declaration of the Rights of Man and Citizen. Rights such as the Right to Life, Freedom of Speech, Freedom of Opinion, Equality before the Law, were established as ‘Natural and Inalienable Rights’. These rights belonged to each human being by birth and could not be taken away.

France Abolishes Monarchy and Becomes a Republic  Although Louis XVI had signed the Constitution, he entered into secret negotiations with the King of Prussia.  The National Assembly voted in April 1792 to declare war against Prussia and Austria. Thousands of volunteers thronged from the provinces to join the army.  Political clubs became an important rallying point for people who wished to discuss government policies and plan their own forms of action. The most successful of these clubs was that of the Jacobins, which got its name from the former convent of St. Jacob in Paris.  The members of the Jacobin club belonged mainly to the less prosperous sections of society. They included small shopkeepers, artisans such as shoemakers, pastry cooks, watch-makers, printers, as well as servants and dailywage workers. Their leader was Maximilian Robespierre.  On the morning of August 10th, 1792, the Jacobins stormed the Palace of the Tuileries and held the King as a hostage for several hours. Later, the assembly voted to imprison the royal family. Elections were held.  From then onwards, all men of 21 years and above, regardless of wealth got the right to vote.  The newly elected assembly was called the Convention. On 21st September 1792, it abolished the monarchy and declared France a Republic.  Louis XVI was sentenced to death by a Court on the charge of treason. On 21st January 1793, he was executed publicly at the Place de la Concorde.

The Reign of Terror  Robespierre’ rule in France (1793 to 1794) was known as the ‘Reign of Terror’  Robespierre followed a policy of severe control and punishment. All those whom he saw as being enemies of the Republic were arrested, imprisoned and then tried by a revolutionary tribunal.  Robespierre’ government issued laws placing a maximum ceiling on wages and prices.  Meat and bread were rationed.  Peasants were forced to transport their grain to the cities and sell it at prices fixed by the government.  The use of more expensive white flour was forbidden; all citizens were required to eat the pain d'egalite (equality bread) a loaf made of whole wheat.  Equality was also sought to be practiced through forms of speech and address.  The Churches were shut down and their buildings were converted into barracks or offices.  Finally, he was convicted by a court in July 1794, he was arrested, and on the next day, sent to the guillotine.

A Directory Rules France  The fall of the Jacobin Government allowed the wealthier middle classes to seize power.  A new Constitution was introduced which denied the vote to non-propertied sections of society.  The political instability of the Directory paved the way for the rise of a military dictator, Napoleon Bonaparte.

The French Revolution Let’s compare and contrast the conditions that prevailed in France that led to revolution with the conditions that led to the first war of Indian Independence. The comparison given below will help in giving a better understanding of the pre-revolt circumstances that led the general public to bring in the irreversible changes in France as well as India.

8

Oswaal CBSE Question Bank Chapterwise & Topicwise, SOCIAL SCIENCE, Class-IX

Conditions

Conditions that prevailed in France

Conditions that prevailed in India

i. Political condition

The Bourbon king of France, Louis XVI was an extremely autocratic and weak-willed king who led a life of absolute luxury. This led to a lot of disappointment among the general public who then were leading life of extreme poverty and widespread hunger.

The Policy of Annexation pursued by the British led to the annexation of various native states, which made the people feel that their rights were being usurped by the British Raj.

The social conditions in France in the late 18th century were highly unequal and exploitative. The first two Estates were the most privileged classes in French society. They were excused from the payment of taxes to the State. On the other hand, the Third Estate which formed the majority of the population were charged with excessive taxes with no political and social rights. As a result, they were extremely discontent.

The British Policy of Discrimination against Indians also led to the revolt. The British favoured Europeans over Indians in all aspects of life, which led to a feeling of bitterness among the people.

iii. Economic There was an increase in the French population, condition which resulted into more demand of food grains. So the price of bread rose rapidly, which the poor could not afford to buy. So the gap between the rich and poor widened.

The economic policies of the British also contributed to the outbreak of the revolt. The tax policies and other measures adopted by the British led to economic hardship for the people, which made them resentful of British rule.

ii. Social condition

The Doctrine of Lapse also allowed the British to annex any native state whose ruler died without a male heir. This led to great unrest among the people, as they felt that their rights were being snatched away.

The policy of introducing western education and culture led to a feeling of alienation and anger among the people.

iv. Religious The revolution initially began with attacks on The British introduced the greased cartridges. conditions Church corruption and the wealth of the higher These cartridges had to be greased with tallow, clergy. which is a substance derived from animal fat. This led to great resentment among the Hindu and Muslim soldiers, as they felt that this was against their religious beliefs.

Key Terms

   

Guillotine: A device consisting of two poles and a blade with which a person was beheaded. Assembly of the Estates General: It was composed of three estates – the clergy, nobility and commoners. Treason: The action of betraying one’s country or a government. Chateaux: A large French country house or stately residence belonging to a king or a nobleman.

Key Dates  1793 to 1794: This period is known as the 'Reign of Terror'.  5th May 1789: Louis XVI called together an assembly of the Estates General to pass proposals for the new taxes.  17th June 1789: Representatives of the Third Estate declared themselves as the National Assembly of the people of France, not of the Estates.  20th June 1789: The representatives of the third estate assembled in the hall of an indoor tennis court in the grounds of Versailles. They swore not to disperse till they had drafted a constitution for France.  14th July 1789: Storming of the Bastille; Beginning of the French Revolution.  4th August 1789: The National Assembly passed a decree abolishing feudal privileges and system of obligations and taxes.  1791: A Constitution is framed to limit the powers of the king and guarantee basic rights to all human beings.  21st September 1792: Abolition of the monarchy and declaration of France as a Republic.  21st January 1793: King Louis XVI was executed at the guillotine.

THE FRENCH REVOLUTION

9

Example

“Robespierre’s rule in France was known as the Reign of terror.” Justify the statement.

Answer:





Step 1: The period from 1793 to 1794 is referred to as the ‘Reign of Terror’. Robespierre followed a policy of severe control and punishment. Step 2: All those whom he saw as being ‘Enemies’ of the Republic—ex-nobles and clergy, members of other political parties, and even the members of his own party who did not agree with his methods,





were arrested, imprisoned and then tried by a revolutionary tribunal. Step 3: If the Court found them ‘guilty’, they were guillotined. Step 4: Robespierre’s Government issued laws, placing a maximum ceiling on wages and prices. Meat and bread were rationed. Peasants were forced to transport their grain to the cities and sell it at prices fixed by the Government. Step 5: Churches were shut down and their buildings converted into barracks or offices.

OBJECTIVE TYPE QUESTIONS A Multiple Choice Questions Q. 1. On which of the following date France declared R itself as a Republic? th (A) On 5 November 1791 (B) On 31st December 1789 (c) On 21st September 1792 (D) On 1st November 1792 Ans. Option (C) is correct. Explanation: On 21st September 1792 the newly elected assembly abolished the monarchy and declared France a republic. Q. 2. ____________ period is referred to as the Reign of R Terror? (A) 1789 to 1790 (B) 1791 to 1800 (C) 1793 to 1794 (D) 1794 to 1795 Ans. Option (C) is correct. Explanation: Robespierre followed a policy of control and punishment. All those whom he saw as being 'enemies' of the republic who did not agree with his methods – were arrested, imprisoned and then tried by a revolutionary tribunal. If the court found them 'guilty' they were guillotined. Q. 3. Which among the following depicts the E personification of the law? (A) The Law Tablet (B) The Red Phrygian cap (C) Blue-White-Red (D) The Winged Woman Ans. Option (D) is correct. Explanation: The majority of men and women in the eighteenth century could not read or write. So images and symbols were frequently used instead of printed words to communicate important ideas.

(1 mark each)

Q. 4. Choose the correct option from column I and II. C  Column I )a( The Broken Chain )b( The bundle of rods or fasces )c( Sceptre )d( Snake biting its tail to form a ring

)i(

Column II Symbol of Royal Power

)ii(

Act of becoming free

)iii( )iv(

Strength lies in unity Symbol of eternity

(A) a-ii, b-iii, c -i, d-iv (C) a-iv, b-i, c-iii, d-ii

(B) a-i, b-ii, c-iii, d-iv (D) a-iii, b-iv, c-i, d-ii

Ans. Option (A) is correct. Explanation: (a) A broken chain: Stands for the act of becoming free. (b) The bundle of rods or fasces: Strength lies in unity. (c) Sceptre: Symbol of royal power. (d) Snake biting its tail to form a ring: Symbol of Eternity. Q. 5. The term ........... is usually used to describe the Society and Institutions of France before 1789.  U (A) Clergy (B) Estate (C) Old Regime (D) Regime Ans. Option (C) is correct. Explanation: The political and social system of France until 1789 is known as Old Regime. Q. 6. Which country had an absolute monarchy before R 1857?  (A) France (B) India (C) Both (D) Neither Ans. Option (A) is correct. Explanation: France had an absolute monarchy before 1857. The Bourbon king of France, Louis XVI, was an extremely autocratic and weak-willed king who led a life of absolute luxury.

10

Oswaal CBSE Question Bank Chapterwise & Topicwise, SOCIAL SCIENCE, Class-IX

Q. 7. Which country experienced a revolution prior to 1857? (A) India (B) France (C) Both experienced a revolution (D) Neither experienced a revolution Ans. Option (B) is correct. Explanation: France experienced a revolution prior to 1857, the conditions in France led to a revolution that brought irreversible changes to the country.



Ans. Q. 2. Ans.

B Assertion and Reason MCQs Directions: In the following questions, a statement of Assertion (A) is followed by a statement of Reason (R). Mark the correct choice as: (A) Both Assertion (A) and Reason (R) are true, and Reason (R) is the correct explanation of Assertion (A). (B) Both Assertion (A) and Reason (R) are true, but Reason (R) is not the correct explanation of Assertion (A). (C) Assertion (A) is true, but Reason (R) is false. (D) Assertion (A) is false, but Reason (R) is true. Q. 1. Assertion (A): The news spread that Louis XVI planned to impose further taxes.

Q. 3.



Reason (R): Increases in taxes to meet the expenses of the state generated anger and protest against the E system of privileges.  Option (A) is correct. Assertion (A): A broken chain stands for the act of becoming free. Reason (R): Chains were used to fetter slaves. R Option (A) is correct. Explanation: In 18th century the majority of men and women could not read or write. So they used symbols where a broken chain signifies that chains were used to fetter slaves, and stands for the act of becoming free. Assertion (A): In France of Old Regime the Monarch had to call a meeting of the Estates General which would then pass his proposals for new taxes. Reason (R): However, the Monarch alone could decide when to call a meeting of this body.

A  Ans. Option (B) is correct. Explanation: In France of the Old Regime the Monarch did not have the power to impose taxes according to his will alone. Rather he had to call a meeting of the Estates general which would then pass his proposals for new taxes. However, the Monarch alone could decide when to call a meeting of this body.

SUBJECTIVE TYPE QUESTIONS Very Short Answer Type Questions Q. 1. What was the importance of ‘The Tennis Court U Oath’ in the French Revolution? Ans. ‘The Tennis Court Oath’ in the French Revolution was important because it signified for the first time that French citizens formally stood in opposition to Louis XVI. The oath was the reflection of the growing unrest against Louis XVI and laid the foundation for later events, including the Declaration of the Rights of Man and of the Citizen, the storming of the Bastille and the drafting of the Constitution of France. (2) Q. 2. What was the Convention? Describe its role in R France. Ans. The newly elected assembly formed in France in 1792 was called Convention. Role in France: (i) It abolished the monarchy and declared France as a republic. (ii) Hereditary monarch Louis XVI and his wife were sentenced to death.  (1+1/2 +1/2 =2)

(2 marks each)

Q. 3. What was Mirabeau and Abbe Sieye’s role in the U French Revolution. Ans. Mirabeau and Abbe Sieye’s role in the French Revolution: (i) Mirabeau realised the need of abolishing the feudal privileges that were given to the clergy and nobles. He also brought out a journal and delivered powerful speeches to the crowds that assembled at Versailles. (ii) Abbe Sieye originally a priest, wrote an influential pamphlet called, ‘What is the Third Estate?’. His influential writings during the French Revolution helped people to raise their voices for basic rights.  (1+1 =2) Q. 4. Why did the members of the Jacobin Club decide to wear long striped trousers similar to those worn E by dock workers? Ans. The members of the Jacobin Club decided to wear long striped trousers similar to those worn by dock workers because they wanted to set a different

THE FRENCH REVOLUTION

identity from the fashionable sections of the society, especially nobles, who wore knee breeches. It was a way of portraying the end of the power wielded by the wearers of knee breeches.  (2) Q. 5. What happened on the night of 4 August 1789 in U France?

Ans. On the night of 4 August 1789, the Assembly passed a decree abolishing the feudal system of obligations and taxes. Members of the clergy too were forced to give up their privileges. Tithes were abolished and lands owned by the Church were confiscated. As a result, the government acquired assets worth at least 2 billion livres.  (2)

Short Answer Type Questions Q. 1. Explain the events that led to the formation of the National Assembly. U [Board Term-I, 2016] OR Why was the National Assembly formed by the people of the Third Estate? Explain.  [Board Term-I, 2015] OR Give reasons as to why was the National Assembly formed by the people of the Third Estate? [Board Term-I, 2013] Ans. Reasons: (i) On 5th May 1789, Louis XVI called an assembly of the Estates General to pass proposal for new taxes. (ii) The First and Second Estates sent 300 representatives each to the assembly while 600 members of the Third Estate were present there. (iii) Voting in the Estates General in the past had been conducted according to the principle that each Estate had one vote. (iv) But members of the Third Estate demanded that voting now be conducted by the assembly as a whole, where each member would have one vote. (v) As the king rejected the proposal, members of the Third Estate walked out of the assembly in protest. (vi) On 20th June, in the tennis court of Versailles, they declared themselves a National Assembly and swore to draft the constitution and limit the powers of the Estate. (Any three) (1×3=3) [CBSE Marking Scheme, 2016] Q. 2. How did the political system work in France under the Constitution of 1791 ? Explain. U  [Board Term-I, 2016] Ans. The political system in France worked in the following manner: (i) It limited the powers of the monarch. Powers were distributed between Legislature, Executive and Judiciary. (ii) Constitutional monarchy was established. The National Assembly was supposed to make laws and it was to be indirectly elected, i.e., (citizens voted for a group of electors who in turn chose the assembly).

11

(3 marks each)

(iii) All the citizens did not enjoy the Right to Vote, only men above 25 years who paid taxes equal to at least 3 days of a labourer’s wage were given the status of active citizens, i.e., Right to Vote. Remaining men and all the women were passive citizens. (1×3=3) [CBSE Marking Scheme, 2016] Q. 3. What measures were taken by Robespierre to R bring equality in the French society?  [Board Term-I, 2014] Ans. Measures taken by Robespierre were: (i) Maximum ceilings on wages and prices. (ii) Meat and bread were rationed. (iii) Peasants were forced to transport grains to the cities and sell at a fixed price. (iv) Expensive white flour was forbidden. (v) Equality was sought. (vi) Churches were shut down and their buildings converted into barracks or offices.  (Any three) (1×3=3) Q. 4. What were the economic conditions in France and India during the pre-1857 period, and how did they compare? Ans. In pre-revolution France, the economic conditions were marked by an increase in population and a rapid rise in the price of bread, which led to a widening gap between the rich and poor. In India, British economic policies led to economic hardship for many Indians, which contributed to resentment of British rule. While there are some similarities between these economic conditions, they are not directly comparable due to differences in historical context and specific circumstances. (3)

Commonly Made Error

Some students give only bullet points of political system under the Constitution of 1791.

Answering Tip

Students should give brief explanation about the political set up in France under the Constitution of 1791.

12

Oswaal CBSE Question Bank Chapterwise & Topicwise, SOCIAL SCIENCE, Class-IX

Long Answer Type Questions Q. 1. Describe the main features of the Jacobin club in France during the French Revolution. R  [Board Term I, 2014] OR What was Jacobin club? Who were its Members? How did Jacobins contribute to carrying the French Revolution further? Ans. Jacobin club: It was a political club formed to discuss government policies and plan their own forms of action. ½ Its members mainly belonged to the less prosperous section of society. ½ Their contribution: (i) In 1792, they planned an insurrection of people of Paris. (ii) They stormed the Palace of the Tuileries, massacred the king’s guards and held the king hostage for several hours. (iii) Elections were held, men of 21 years or above, regardless of wealth, got the Right to Vote. (iv) Convention was formed. (½+½+4=5) [CBSE Marking Scheme, 2014] Q. 2. Explain any five main features of the Constitution E drafted in 1791. Ans. The five main features of the Constitution drafted in 1791 were as follows:

(i) The Constitution of 1791 vested the power to make laws in the National Assembly, which was indirectly elected. (ii) With the drafting of the new Constitution, the powers to govern the country were assigned to different institutions, i.e., the Legislature, Executive and the Judiciary. (iii) The Judiciary and the Legislature were elected by the people. Only men above 25 years of age who paid taxes equal to at least 3 days of a labourer’s wage were given the status of active citizens, that is, they were entitled to vote. (iv) Under the new Constitution, the powers of the monarch were limited. Most of the powers were in the hands of legislatures. (v) The ministers were also answerable to the legislature. The king enjoyed the veto power. (5) Q. 3. How did the political systems in France and India differ prior to 1857? R [Board Term-I, 2016] Ans. The political conditions in France prior to the French Revolution were characterized by an extremely autocratic and weak-willed Bourbon



(5 marks each) king, Louis XVI, who led a life of absolute luxury. This led to a lot of disappointment among the general public who then were leading life of extreme poverty and widespread hunger. In contrast, the political system in India prior to 1857 was under British rule, which favored Europeans over Indians in all aspects of life. The British policy of discrimination against Indians also led to a feeling of bitterness among the people. The policy of introducing western education and culture led to a feeling of alienation and anger among the people. However, it is important to note that these are just some examples from the given pages and may not fully answer your question about all differences between the political systems in France and India prior to 1857. (5)

Q. 4. ‘‘One of the most revolutionary social reforms of the Jacobins regime was the abolition of slavery in the French colonies.’’ Justify the statement. 

A [Board Term I, 2016]

Ans. Indeed, one of the most revolutionary social reforms of the Jacobins regime was the abolition of slavery in the French colonies; this is how it happened: (i) Colonies in the Caribbean – Martinique, Guadeloupe and San-Domingo were important suppliers of tobacco, indigo, sugar and coffee. (ii) Reluctance of Europeans to go to and work in distant lands meant shortage of labour on the plantations. (iii) Triangular slave trade started between Europe, Africa and America in 17th century. (iv) French merchants sailed from the Ports of Bordeaux or Nantes to African coast – slaves were brought, branded and shackled and packed into ships. After a 3 month long voyage across the Atlantic to the Caribbean – slaves were sold to plantation owners. (v) Exploitation of slave labour made it possible to meet the growing demand in European markets for sugar, coffee and indigo. (vi) The National Assembly held long debates about rights of man to be extended to all subjects including those in the colonies. (vii) Finally, the 1794 Convention passed a law to free all slaves in the French overseas possessions. (viii) Napoleon reintroduced slavery. (ix) Finally, in 1848, slavery was completed abolished.  (Any five) [CBSE Marking Scheme, 2016] (1×5=5)

THE FRENCH REVOLUTION

13

Topic-3 Women Revolution and the Abolition of Slavery Revision Notes Did woman have a Revolution?  From the very beginning, women were active participants in the events which brought about so many important changes in the French society.

Condition of Women During the French Revolution    

Scan to know more about this topic

l

The women hoped that their involvement would pressurise the revolutionary government to introduce measures to improve their lives. l Most women of the Third Estate had to work for a living. l Most women did not have access to education or job training. Only daughters of nobles or What Caused the French wealthier members of the Third Estate could study at a convent. Revolution? l Their wages were lower than those of men. In order to discuss and voice their interests, women started their own political clubs and newspapers. The Society of Revolutionary and the Republican Women was the most famous club. Women of France demanded the Right to Vote, to be elected to the assembly and to hold political offices. Laws introduced by the revolutionary government to improve the lives of women in France: l Together with the creation of state schools, schooling was made compulsory for all the girls. l Their fathers could no longer force them into marriage against their will. l Marriage was made into a contract entered into freely and registered under civil law. l Divorce was made legal and could be applied for by both women and men. l Women could now be trained for jobs, become artists, or run small businesses. It was finally in 1946 that women in France won the right to vote.

The Abolition of Slavery  One of the most revolutionary social reforms of the Jacobin regime was the abolition of slavery in the French Colonies.  A triangular slave trade began in the seventeenth century between Europe, Africa and America.  The exploitation of slave labourers made it possible to meet the growing demand in European markets for sugar, coffee and indigo.  Slavery was finally abolished in French colonies in 1848.

The Revolution and Everyday Life  The years following 1789 in France saw many changes in the daily lives of men, women and children.  One important law that came into effect soon after the storming of the Bastille in the summer of 1789 was the abolition of censorship.  Effects of abolition of censorship: l Now, the Declaration of the Rights of Man and Citizen proclaimed Freedom of Speech and Expression to be a natural right. l Newspapers, pamphlets, books and printed pictures flooded the towns of France from where they travelled rapidly into the countryside. l They all described and discussed the events and changes taking place in France. l Freedom of the Press also meant that opposing views of events could be expressed. l Plays, songs and festive processions attracted large numbers of people.  In 1804, Napoleon Bonaparte crowned himself as the Emperor of France.  He introduced many laws, such as : l The protection of private property and l A uniform system of weights and measures provided by the decimal system.  Napoleon was finally defeated at Waterloo in 1815.  The ideas of liberty and democratic rights were the most important legacy of the French Revolution. These ideas spread from France to the rest of Europe during the nineteenth century, where feudal systems were abolished.

14

Oswaal CBSE Question Bank Chapterwise & Topicwise, SOCIAL SCIENCE, Class-IX

Key Fact  The Jacobins attempted to eradicate Christianity in France. (Source: https://www.britannica.com/facts/French-Revolution)

Key Terms  Feudalism: A social system, in which the king would give a piece of land to the nobles who fought for him. While the peasants were obliged to live on their landlord’s land and serve him.  Jacobin club: A political club that existed during the French Revolution.

Key Dates  1789: Abolition of censorship came into effect.  1804: Napolean crowned himself as the Emperor of France.  1815: Napoleon was finally defeated in the Battle of Waterloo.  1848: Abolition of slavery in France and its colonies.  1946: Women in France won the Right to Vote.

OBJECTIVE TYPE QUESTIONS A Multiple Choice Questions Q. 1. Which of the following port city/cities owed it/ their economic prosperity to the flourishing slave R trade? (A) Bordeaux (B) Paris

(C) Nantes

(D) Both (A) and (C)

Ans. Option (D) is correct. Explanation: French merchants sailed from the ports of Bordeaux or Nantes to the African coast, where they bought slaves from local chieftains. Q. 2. In which year Napoleon Bonaparte crowned R himself as the Emperor of France? (A) 1791 (B) 1805

(C) 1804

(D) 1795

Ans. Option (C) is correct. Explanation: In 1804 Napoleon crowned himself as the Emperor of France and annexed large parts of Europe to increase his power so that he would not be dependent on anyone. Q. 3. Study the figure carefully and answer the question that follows:  A

(1 mark each)

Which of the following options best signifies this figure? (A) The women demand their rights. (B) The women marched to Versailles and brought the king back with them to Paris. (C) People participation in Government. (D) None of the above.

Ans. Option (B) is correct.

B Assertion and Reason MCQs Directions: In the following questions, a statement of Assertion (A) is followed by a statement of Reason (R). Mark the correct choice as: (A) Both Assertion (A) and Reason (R) are true, and Reason (R) is the correct explanation of Assertion (A). (B) Both Assertion (A) and Reason (R) are true, but Reason (R) is not the correct explanation of Assertion (A). (C) Assertion (A) is true, but Reason (R) is false. (D) Assertion (A) is false, but Reason (R) is true. Q. 1. Assertion (A): It was finally in 1946 that women in France won the Right to Vote. Reason (R): The example of the political activities of French women during the revolutionary years was E kept alive as an inspiring memory. Ans. Option (B) is correct. Explanation: Women suffrage got definitively adopted in the Constitution of France, on 27th October 1946. It stated that the French law

THE FRENCH REVOLUTION

guaranteed that 'all women have rights equal to that of men in all spheres' without exception. Q. 2. Assertion (A): The reluctance of Europeans to go and work in distant and unfamiliar lands meant a shortage of labour on the plantations. Reason (R): This was met by a triangular slave trade U between Europe, Africa and the Americas.  Ans. Option (A) is correct.

15

Q. 3. Assertion (A): The National Assembly held long debates about whether the rights of man should be extended to all French subjects including those in the colonies. Reason (R): Throughout the eighteenth century E there was criticism of slavery in France. Ans. Option (B) is correct.

SUBJECTIVE TYPE QUESTIONS Very Short Answer Type Questions R Q. 1. Who was Olympe de Gouges? Ans. (i) Olympe de Gouges was a French playwright and political activist. She was one of the most important women in France. (ii) Her writings on women's rights and abolitionism reached a large audience in various countries. (iii) She protested against the Constitution and the Declaration of the Rights of Man and Citizens as they excluded women from the basic rights that each human was entitled to.  (Any Two Points) (1 x 2=2)

Q. 2. Explain any two rights set forth in Olympe de U Gouges’ Declaration. Ans. (i) Woman is born free and remains equal to man in rights. (ii) The source of the sovereign resides in nation, which is nothing but the union of woman and man. (iii) No woman is an exception; she is accused, arrested and detained in cases determined by law. Women like men need to obey this rigorous law. 

[Any Two Points] (1×2=2)

Q. 3. What positive changes came in France with the A abolition of censorship? Ans. With the abolition of censorship: (i) The citizens proclaimed the freedom of speech and expression to be a natural right. (ii) Newspapers, pamphlets, books and printed pictures flooded the towns of France from where they travelled rapidly into the countryside.They all

described and discussed the events and changes taking place in France. Freedom of the press also meant that opposing views of events could be expressed. (iii) Plays, songs and festive processions attracted large numbers of people.  [Any Two Points]  (1×2=2) Q. 4. Describe the triangular slave trade between E Europe, Africa and America. Ans. An acute shortage of labour in the French Caribbean Colonies, Martinique, Guadeloupe and San Domingo, led to the triangular slave trade between Europe, Africa and America in the 17th century. French merchants sailed from the Ports of Bordeaux or Nantes to the African coast, where they bought slaves from local chieftains. Arms, textiles and wine were shipped from Europe to Africa, enslaved people from Africa to the Americas, and sugar, tobacco and other products from the Americas to Europe.  (2) Q. 5. Name any two important legacies of the French R Revolution. Ans. The ideas of liberty and democratic rights were the two most important legacies of the French Revolution. These spread from France to the rest of Europe during the nineteenth century, where feudal systems were in existence. Colonised people reworked the idea of freedom from bondage into their movements to create a sovereign nation state.  (2)

Short Answer Type Questions Q. 1. How was the French Revolution responsible for the societal changes ? A [Board Term-I, 2016] Ans. The French Revolution opened up the possibility of creating a dramatic change in the way society was structured. Before the 18th century, the French society was broadly divided into Estates and Orders.

(2 marks each)

(3 marks each)

Aristocracy of the Church ended. Economic and social power was controlled. New ideas about individual rights and who controlled social powers were discussed. (3)  [CBSE Marking Scheme, 2016]

16

Oswaal CBSE Question Bank Chapterwise & Topicwise, SOCIAL SCIENCE, Class-IX

Q. 2. Highlight the three laws enforced by Napoleon to modernise France.  R [Board Term-I, 2015] Ans. Laws enforced by Napoleon to Modernise France were: (i) He codified French laws known as Napoleon Code. (ii) He established uniform systems of weights and measures provided by the decimal system. (iii) He passed laws for protection of private property.  [CBSE Marking Scheme, 2015] (1×3=3) Q. 3. How was a revolution brought about in everyday life of French people? Explain.



R [Board Term I, 2015]

OR Explain how did the Freedom of Speech and Expression under the revolutionary government in France promote the ideas of Liberty and Equality into every day practice ?



Ans. A revolution was brought about in everyday life of the French people in the following ways: (i) One important law—The abolition of censorship came into effect soon after the storming of the Bastille in 1789. (ii) Freedom of Speech and Expression of man were considered as the natural rights. (iii) Newspapers, pamphlets, books and printed pictures flooded the towns of France from where it reached to the countryside. (iv) They all described and discussed the events and changes taking place in France. (v) Freedom of Press also meant that opposing views could be expressed. (vi) Plays, songs and festive processions attracted common people through which they could identify with ideas such as Liberty and Justice. (Any three) [CBSE Marking Scheme, 2015] (1×3=3)

Commonly Made Error

The students do not know whether the French

Answering Tip

Students should learn about the main features and changes brought by the French revolution.

Q. 4. How did philosophers influence the thinking of the people of France ? U [DDE Term-I, 2014] Ans. Philosophers influenced the thinking of the people of France as: (i) They believed that man’s destiny was in his own hands. (ii) They criticised the divine and absolute rights of the rulers. (iii) They propounded the idea of formation of government based on Social Contract between people and their representatives. (iv) They believed that all the powers should not be concentrated at one level. (Any three)(1×3=3) Q. 5. Give the reason for the walk out from the assembly of Estate General by the members of Third Estate on 5th May 1789. What were the two steps taken by the Members of Third Estate after the walk out? 

A [Board Term-I, 2013]

Ans. In the assembly of the Estate General, the Members of the Third Estate demanded that voting be conducted by the assembly where each member will have one vote, and this demand was rejected by the king. Representatives of the Third Estate walked out in protest, as they considered themselves to be spokesmen for all the people. They assembled in the hall of an indoor tennis court in the Grounds of Versailles. The two steps that were taken by the Members of Third Estate after the walk out: (a) They declared themselves a National Assembly. (b) They took an oath not to disperse till they had drafted a constitution for France limiting the powers of the monarch. They were led by Abbe Sieyes and Mirabeau. (1+2=3) [CBSE Marking Scheme, 2013]

Revolution was successful or not.

Long Answer Type Questions Q. 1. Evaluate the importance of the following years in concern with the French Revolution, 1774, 1789, 1791, 1804 and 1815.  A [Board Term-I, 2013] Ans. The importance of the following years in concern with the French Revolution: (i) 1774: Louis XVI ascended the throne of France. He believed in the Divine Right Theory of Kings and had no respect for freedom of liberty. Because of

(5 marks each) his empty treasure, he began to impose heavy taxes which was disliked by his own people also.

(ii) 1789: On 5th May 1789, Louis XVI called together an assembly of the Estates General to pass proposals for new taxes.

(a) The French Revolution started.



(b) On 14th July, the agitated crowd stormed and destroyed the Bastille.

THE FRENCH REVOLUTION



(c) Louis XVI accepted the principle that his powers would, from now on, be checked by the Constitution.



(d) On the night of 4th August 1789, the assembly passed a decree abolishing the feudal system of obligations and taxes. Members of the clergy too were forced to give up their privileges.

(iii) 1791: The French Legislative elections were held in September 1971 to elect the Legislative Assembly.

(a) The first written French Constitution was drafted by the National Assembly after the fall of the absolute monarchy.



(b) The rights of the privileged classes were abolished and Declaration of the Rights of Man and the Citizen was passed by France’s National Constituent Assembly.

(iv) 1804: Napoleon became the Emperor of France. He set out to conquer neighbouring European countries. He conquered Europe and saw his role as a 'Moderniser of Europe'. (v) 1815: Napoleon was defeated in the Battle of Waterloo. Many of his measures, that carried the revolutionary ideas of liberty and modern laws to other parts of Europe, had an impact on people, long after Napoleon had left. (1×5=5) Q. 2. Describe any five major events that led to the outbreak of the French Revolution. R [Board Term-I, 2016] Ans. Five major events that triggered the French Revolution were: (i) Meeting of the Estates General: Members of the Third Estate comprising peasants, artisans and women were denied entry to the Assembly which was called by Louis XVI on 5th May 1789 to propose new taxes. (ii) Demand for Universal Adult Franchise: Members of the Third Estate demanded the power for each member to vote. When the king rejected the demand, members of the Third Estate walked out of the Assembly in protest. (iii) Meeting of the newly formed National Assembly: The Third Estate assembled in an indoor tennis court of Versailles and declared themselves as the ‘National Assembly’. They believed in ending the feudal privileges of the nobles and clergy. (iv) Revolt by the Peasants: When the countryside rumours spread that the ripe corns would be destroyed by hired Bands of Brigands, peasants in several districts seized hoes and attacked the residences of their manors. They looted, hoarded grains and burnt down documents containing records of manorial dues. (v) Fall of Bastille Prison  : The severe winters increased the price of bread which was hoarded in the market. Angry women attacked the shops. At

17

the same time, the king ordered troops to move into Paris. On 14th July 1789, the agitated crowd stormed and destroyed the Bastille. (1×5=5) Q. 3. Analyse the impact of the French Revolution over Europe and other parts of the world. 

R [Board Term I, 2015]

Ans. Impact of the French Revolution: (i) Spread of ideas of Liberty and Democratic Rights. (ii) The people in colonies were inspired during their movement for independence. (iii) Individuals and leaders responded to the ideas coming from France, e.g., Tipu Sultan and Raja Ram Mohan Roy in India. (iv) Women’s movement for voting rights and equal wages continued through the next two hundred years. (v) Their fight for Right to Vote was carried out through International Suffrage Movement.  [CBSE Marking Scheme, 2015] (1×5=5) Q. 4. How did the laws introduced by the Revolutionary Government helped to improve the lives of women in France? Explain. R [Board Term I, 2015] Ans. Laws introduced for women in France were: (i) Schooling was made compulsory for girls. (ii) No forced marriages. Their fathers could no longer force them to marriage against their will. (iii) Divorce was made legal and it applied to both. (iv) Training for jobs and therefore women could become artists or run small businesses. (v) Marriage was made into a contract entered into and registered freely under civil Law. [CBSE Marking Scheme, 2015] (1×5=5) Q. 5. Explain the role played by women in France during the revolutionary years?  R [Board Term I, 2015] OR Evaluate the role of women in France before the revolution. U [Board Term I, 2012] Ans. The role of women in France before the revolution was: (i) Women started their own political clubs and newspapers. (ii) They demanded the Right to Vote. (iii) They demanded to be elected in the Assembly and to hold political office. (iv) The Revolutionary Government did introduce some laws for women. (v) During the ‘Reign of Terror’, the new government ordered to close women’s clubs and ban political activities. (vi) Women’s movement for voting right continued through the next two hundred years.

18

Oswaal CBSE Question Bank Chapterwise & Topicwise, SOCIAL SCIENCE, Class-IX

(vii) Women’s struggle for equal political rights continued during the ‘Reign of Terror’. The government banned their clubs and political activities. Many women got arrested. It was in 1946 that women got right to vote. (Any five)

Tipu Sultan and Raja Ram Mohan Roy were the two Indian leaders, who responded to the ideas of the revolution. (Any five) [CBSE Marking Scheme, 2015, 12] (1×5=5)

[CBSE Marking Scheme, 2015] (1×5=5) Q. 6. Describe the legacy of the French Revolution for the people of the world during the 19th and 20th centuries. A [NCERT] 

Commonly Made Error

[Board Term I, 2015] OR



What is the legacy of the French Revolution? Name two Indian leaders, who responded to the ideas of this Revolution. [Board Term I, 2014] OR

Explain the legacy of the ‘French Revolution’ to the world society. Ans. The Legacy of the French Revolution for the people of the World: (i) The French Revolution is the milestone in the history of mankind. It inspired revolutionary movement in almost every country of Europe and South America. (ii) It inspired social and economic changes in Europe. It inspired the Germans, Italians and Austrians to overthrow their oppressive regimes. (iii) It spread new ideas among the people. The revolution inspired struggling nations of Asia and Africa. (iv) It initiated mass movements and instilled the spirit of Nationalism among the people. (v) The ideas of Liberty and Democratic Rights, were the most important legacies of the French Revolution. (vi) The French Revolution gave the term ‘nation’ its modern meaning. (vii) The ideas of sovereignty were also given by the French Revolution. It means that a nation recognises no law or authority above its own. (viii) The French Revolution motivated and inspired the Revolutionary Movement of the former French colony of Haiti.

The students do not pay much focus on the significance of the French Revolution.

Answering Tip

The significance of the French Revolution can be seen in the unity the French experienced subsequent to the Revolution and Napoleon.

Q. 7. Who was Napoleon? Describe the rise of Napoleon in France. U [NCT Term I, 2014] OR Describe the rise of Napoleon. [Board Term I, 2013] OR How would you explain the rise of Napoleon ? [NCERT] Ans. Napoleon was the Emperor of France from 1804 to 1815. Rise of Napoleon: (i) After the formation of France as Republic in 1792, the ruler, Robespierre, gave more privileges to the wealthier section of the society. (ii) Robespierre was an autocrat. His act led to ‘Reign of Terrors’ for many years. (iii) After his rule came to an end, a Directory was formed so as to avoid concentration of power in one individual’s hand. But the members of the Directory fought among themselves, which led to political instability. (iv) In this period, Napoleon Bonaparte gained the control of the reign as a dictator. In 1804, he crowned himself as the Emperor of France. (v) He set out to conquer the neighbouring European countries. Soon, the Napoleonic armies came to be viewed everywhere as an invading force.  (5)

COMPETENCY BASED QUESTIONS Case based MCQs I Read the source given below and answer the E following questions: (1+1+1+1=4) French Society during the late eighteenth century The Church too extracted its share of taxes called

(4 marks each)

tithes from the peasants, and finally, all members of the Third Estate had to pay taxes to the state. These included a direct tax, called taille, and a number of indirect taxes which were levied on articles of everyday consumption like salt or tobacco. The burden of financing activities of the state through taxes was borne by the Third Estate alone.

THE FRENCH REVOLUTION

1. A group of persons invested with special function in the Church are called ______ . (A) Priest (B) Clergy (C) Noble (D) Feudal Lords Ans. Option (B) is correct. 2. Who enjoyed feudal privileges by birth? (A) The First Estate (B) The Second Estate (C) The Third Estate (D) Both (A) and (B) Ans. Option (D) is correct. Explanation: The members of the first and the second estates enjoyed certain privileges by birth. The most important of these privileges was exemption from paying taxes to the state. The nobles further enjoyed feudal privileges. These included feudal dues, which they extracted from the peasants. 3. A tax levied by the Church and was paid by Third Estate? (A) Francs (B) Dollars (C) Pounds (D) Tithes Ans. Option (D) is correct. Explanation: Tithe was a tax imposed by the Church on the peasants in the 18th century, comprising one-tenth of the agricultural produce. 4. The _____ that existed in the French society in the Old Regime became the cause of the French Revolution. (A) inequality (B) equality (C) partiality (D) discrimination Ans. Option (A) is correct. II. Read the source given below and answer the U following questions: (1+1+1+1=4) The French Revolution On the morning of 14 July 1789, the city of Paris was in a state of alarm. The king had commanded troops to move into the city. Rumours spread that he would soon order the army to open fire upon the citizens. Some 7,000 men and women gathered in front of the town hall and decided to form a peoples militia. They broke into a number of government buildings in search of arms. Finally, a group of several hundred people marched towards the eastern part of the city and stormed the fortress-prison, the Bastille, where they hoped to find hoarded ammunition. In the armed fight that followed, the commander of the Bastille was killed and the prisoners released − though there were only seven of them. Yet the Bastille was hated by all, because it stood for the despotic power of the king. The fortress was demolished and its stone fragments were sold in the markets to all those who wished to keep a souvenir of its destruction. 1. How many men and women gathered in front of the town hall? (A) 5,000 (B) 8,000 (C) 7,000 (D) 6,000

19

Ans. Option (C) is correct. 2. On 17th July 1789 the city of Paris was in a state of alarm because: (A) the king had ordered the army to open fire upon the citizens in reality. (B) rumours spread that the king would soon order the army to open fire upon the citizens. (C) there was a foreign attack on the citizens. (D) the earthquake struck Paris at that moment. Ans. Option (B) is correct. 3. In armed fight who was killed in the 'Fortress of Bastille? (A) The King (B) The Minister (C) The Commander (D) The Prisoner Ans. Option (C) is correct. Explanation: The Commander of the Bastille was killed in the armed fight and the prisoners were released. 4. Why did people hate 'Bastille'? (A) Because it stood for the despotic power. (B) Because it stood for the armed power. (C) Because it stood for the man power. (D) Because it stood for the money power. Ans. Option (A) is correct.

Subjective based Questions

I. Read the source given below and answer the following questions:(1+1+2=4)



A

The revolutionary wars brought losses and economic difficulties to the people. While the men were away fighting at the front, women were left to cope with the tasks of earning a living and looking after their families. Large sections of the population were convinced that the revolution had to be carried further, as the Constitution of 1791 gave political rights only to the richer sections of society. Political clubs became an important rallying point for people who wished to discuss government policies and plan their own forms of action. The most successful of these clubs was that of the Jacobins, which got its name from the former convent of St Jacob in Paris. Women too, who had been active throughout this period, formed their own clubs.



The members of the Jacobin club belonged mainly to the less prosperous sections of society. They included small shopkeepers, artisans such as shoemakers, pastry cooks, watch-makers, printers, as well as servants and daily-wage workers. Their leader was Maximilian Robespierre. A large group

20

Oswaal CBSE Question Bank Chapterwise & Topicwise, SOCIAL SCIENCE, Class-IX

among the Jacobins decided to start wearing long

demand were listed in some 40,000 letters which

striped trousers similar to those worn by dock

the representatives had brought with them.

workers. This was to set themselves apart from the



Voting in the Estates General in the past had been

fashionable sections of society, especially nobles,

conducted according to the principle that each

who wore knee breeches.

estate had one vote. This time too Louis XVI was determined to continue the same practice. But

1. When the men were away fighting at the front,

member of the Third estate demanded that voting

who became the bread earner?

now be conducted by the assembly as a whole,

Ans. When the men were away fighting at the front, the

where each member would have one vote. This was

women of the family became the bread earner.

one of the democratic principles put forward by

2. Which constitution gave political right only to the

philosophers like Rousseau in his book "The Social

richer section of the society?

Contract". When the king rejected this proposal,

Ans. The Constitution of 1791 gave political rights only

members of the Third estate walked out of the

to the richer sections of society. 3. Why did the Jacobins decide to start wearing long striped trousers and knee breeches? Ans. The Jacobins decided to start wearing long striped trousers and knee breeches as a symbol of equality. II. Read the source given below and answer the C following questions: (1+1+2=4)

On 5 May 1789, Louis XVI called together an

assembly in protest.

1. Why was an assembly of the Estates General called by Louis XVI?

Ans. The political and financial situation in France had grown very weak, forcing Louis XVI to summon the Estates General to pass proposals for new taxes. 2. What was the proportion of the representatives of First, Second and Third Estates in the resplendent

assembly of the Estates General to pass proposals for new taxes. A resplendent hall in Versailles was prepared to host the delegates. The First and

hall in Versailles? Ans. The first and second estates sent 300 representatives each, who were seated in rows facing each other on

Second Estates sent 300 representatives each, who

two sides, while the 600 members of the third estate

were seated in rows facing each other on two sides, while the 600 members of the Third estate had to stand at the back. The Third estate was represented by its more prosperous and educated members. Peasants, artisans and women were denied entry

had to stand at the back. 3. On what principle was the voting in the estates done in the past? Ans. The principle on which voting was conducted

to the assembly. However, their grievances and

in the Estates General was based on each estate having one vote.

Map Work Q

Syllabus

l l

Outline Political Map of France (For locating and labeling / Identification); Bordeaux; l Nantes; l Paris; l Marseilles

Locate and label the following regions on the given outline map of France:

(i) Bordeaux: Part of France related to slave trade. (ii) Nantes: Port related to slave trade. (iii) Paris: The place where the French Revolution occurred. (iv) Marseilles: Region not affected by the Great Fear. Ans.

(1 Mark each)

THE FRENCH REVOLUTION

21

(iii) Paris

(ii) Nantes

(i) Bordeaux

Map not to scale

(iv) Marseilles

Artificial Intelligence AI Project Cycle Integrated with Subsistence Crisis and cause and effect of Subsistence Crisis. Objectives:  Students will be able to identify the outcome of the French revolution.  Students will be able connect emotions (such as fear) and life events (such as poverty) to a tendency for citizens to want to rebel.  Students will be able to describe how revolutions are not uniformly supported in a country. Material Required: Pen, paper, Black Board chalk, Laptops and Internet connections, Power Point, French Revolution, Smart Board. PARAMETERS

DESCRIPTION

Chapter Covered

Chapter 1: The French Revolution

Name of the Book

India and the Contemporary World, History Class 9, NCERT

AI CONCEPTS INTEGRATED

Subject and Artificial AI Project Cycle Integrated with Subsistence crisis and Cause and Intelligence effect of Subsistence crisis Integrated Learning Objectives

l l l l

l

 tudents will be able to identify the causes of the S French Revolution with the help of loopy Students will be able to identify the main events of the French Revolution Students will be able to identify the outcomes of the French Revolutions Students will be able connect emotions (such as fear) and life events (such as poverty) to a tendency for citizens to want to rebel. Students will be able to describe how revolutions are not uniformly supported in a country.

Time Required

One period of 40 minutes class to clear Subsistence crisis topic 10 periods of 40 minutes each period to complete the lesson.

Classroom Arrangement

Seating arrangement - Theory Sessions Flexible arrangement - for activity

http://ncase.me/loopy/

22

Oswaal CBSE Question Bank Chapterwise & Topicwise, SOCIAL SCIENCE, Class-IX

Material Required

Pen, paper, Black Board chalk, Laptops and Internet connections, Power Point French Revolution, Smart Board.

Pre – Preparation Activities

watching the video The French Revolution: Timeline & Major Events, pausing to discuss the following questions at these times: 3:48--Had the Revolution ended here, before war broke out between France and a coalition of other countries, how might France be a different country than it is today? How might the history of Europe had been different? 6:48--Many of the Enlightenment thinkers were themselves quite wealthy and powerful. How might they have fared under the French Revolution?

Previous Knowledge

Students will be asked to think about the following questions How would you define an unjust government? What, if anything, would lead you to take part in a violent revolution?

Methodology

Who - The students are asked to collect data on different vulnerable groups. What - How does high prices and taxes influence a tendency for people/citizens to want to rebel? Is there anything about the three estates that seems unfair to you? Why did the nobles and clergy want each estate to only have one vote? Where – where is the problem in French society prior to revolution? Why do you think it is a problem? Data acquired- students are asked to make time line of the event from crowning of Louis XVI till fall of Directorial rule.

Learning Outcomes

The students will be able l To analyze the situation giving rise to various factors of Revolution through problem scooping method. l To understand the ideals of the French Revolution l To comprehend the legacy of the revolution represent the same through loop diagram.

Follow up Activities

Problem solving worksheet will be given to students.

Reflections

Discussion with Students on the role of AI application Any other AI application that can be used as an alternative. – www.inklewriter.com https://datavizcatalogue.com

www.inklewriter.com https:// datavizcatalogue.com 

Study Time Max Time: 3:30 hr Max Questions: 81

CHAPTER

SOCIALISM IN EUROPE AND THE RUSSIAN REVOLUTION

2

Learning Objectives

Analyse the situations that led to the rise of Russian and French revolutions. l Evaluate the reasons that led to the rise of Lenin’s communism and Marxist Socialism. l

Topic-1 The Age of Social Change

TOPIC - 1 The Age of Social Change .... P. 23 TOPIC - 2 The Russian Revolution .... P. 28

Revision Notes

 The French Revolution opened up the possibility of creating a dramatic change in the way in which society was structured.  Through the Revolution in Russia, socialism became one of the most significant and powerful ideas to shape society in the twentieth century.

TOPIC - 3 The February Revolution in Petrograd .... P. 33 TOPIC - 4 Changes after October and the Global influence .... P. 37

View of Liberals, Radicals and Conservatives

 One of the groups, which looked to change society were the liberals.  Liberals wanted a nation which tolerated all religions.  Liberals also opposed the uncontrolled power of dynastic rulers.  They wanted to safeguard the rights of individuals against governments.  They argued for a representative, elected Parliamentary Government, subject to laws interpreted by a well-trained Judiciary that was independent of Rulers and Officials.  However, they were not 'democrats'. They did not believe in Universal Adult Franchise.





Views of Radicals:





 The Radicals wanted a nation in which government was based on the majority of a





 Unlike Liberals, they opposed the privileges of big landowners and wealthy factory owners.





of a few. Views of Conservatives:





 Conservatives were opposed to Radicals and Liberals. After the nineteenth century, they accepted changes



The Age of Social ChangeSocialism in Europe and the Russian Revolution

country’s population.

 They were not against the existence of private property, but disliked concentration of property in the hands

but also believed that the past needed to be respected and change should begin slowly.

Industrial Society and Social Change

Scan to know more about this topic

 This was a time when new cities came up and new industrialised regions developed, railways expanded and the Industrial Revolution took place.  Unemployment was common, particularly during times of low demand for industrial goods.  Housing and sanitation were problems since towns were growing rapidly.

24

Oswaal CBSE Question Bank Chapterwise & Topicwise, SOCIAL SCIENCE, Class-IX

SOCIALISM IN EUROPE AND THE RUSSIAN REVOLUTION

25

he Coming of Socialism to Europe T  By the mid-nineteenth century in Europe, Socialism was a well-known body of ideas that attracted widespread attention.  Socialists were against private property and saw it as the root of all social ills of that time.  Some socialists believed in the idea of ‘cooperatives’. Robert Owen (1771–1858), a leading English manufacturer, sought to build a cooperative community called New Harmony in Indiana (USA).  In France, for instance, Louis Blanc (1813–1882) wanted the government to encourage cooperatives and replace capitalist enterprises.  Karl Marx (1818–1883) and Friedrich Engels (1820–1895) added other ideas to this body of arguments. Marx argued that industrial society was capitalist.  Idea of Communist Society:  Industrial society was capitalist. Capitalists owned the capital invested in factories and the profit of capitalists was produced by workers.  The conditions of workers could not improve as long as this profit was accumulated by private capitalists.  Marx believed that to free themselves from capitalist exploitation, workers had to construct a radically socialist society where all properties were socially controlled. This would be a ‘Communist Society.’  He was convinced that workers would triumph in their conflict with Capitalists. A communist society was the natural society of the future.

Support for Socialism

 By the 1870s, socialist ideas spread through Europe. To coordinate their efforts, socialists formed an international body–namely, the Second International. Workers in England and Germany began forming associations to fight for better living and working conditions.  By 1905, socialists and trade unionists formed a Labour Party in Britain and a Socialist Party in France. However, till 1914, socialists never succeeded in forming a government in Europe.

Key Terms  Second International: It was an organisation of socialist and labour parties.  Suffragette movement: A movement to give women the right to vote.

Key Dates  1870: Spread of socialist ideas through Europe.  1905: Socialists and trade unionists formed a Labour Party in Britain and a Socialist Party in France.

OBJECTIVE TYPE QUESTIONS A Multiple Choice Questions Q. 1. Who was Robert Owen? (A) A Russian leader (B) An English priest (C) An English manufacturer (D) An American politician Ans. Option (C) is correct.

R

Q. 2. What was meant by New Harmony? U (A) A bank (B) A co-operative community (C) A capitalist (D) Common man Ans. Option (B) is correct. Explanation: New Harmony was the name given to the cooperative community established by the

(1 mark each)

British socialist thinker Robert Owen in Indiana, USA. Q. 3. Look at the picture given below and identify the scene: R



26

Oswaal CBSE Question Bank Chapterwise & Topicwise, SOCIAL SCIENCE, Class-IX

(A) Painting of the Paris Commune of 1871 (B) Fighting on the streets of St. Petersburg between liberals and communists. (C) Revolt by common people against Lenin (D) None of the above Ans. Option (A) is correct. Explanation: This is a painting of the Paris Commune of 1871. It portrays a scene from the popular uprising in Paris between March and May 1871. This was a period when the town council (commune) of Paris was taken over by a ‘peoples’ government’ consisting of workers, ordinary people, professionals, political activists and others. Q. 4. When was the Second International for socialist ideas formed? R (A) In 1872 (B) In 1873 (C) In 1870 (D) In 1875 Ans. Option (C) is correct. Q. 5. What was Marseillaise? R (A) A war song (B) A house owner (C) A poor labourer (D) Medium land owner Ans. Option (A) is correct. Explanation: ‘Marseillaise’, originally written as a war song in 1792, it became a symbol of the commune and of the struggle for liberty.

B Assertion and Reason MCQs

(A)

In the questions given below, there are two statements marked as Assertion (A) and Reason (R). Read the statements and chose the correct option: Both (A) and (R) are true and (R) is the correct explanation of (A).

(B) (C) (D)

Both (A) and (R) are true, but (R) is not the correct explanation of (A). (A) is correct, but (R) is wrong. (A) is wrong, but (R) is correct.

Q. 1. Assertion (A): Socialists were against private property, and saw it as the root of all social ills of the time. Reason (R): The propertied were concerned only with personal gain and not with the welfare of E those who made the property productive.  Ans. Option (A) is correct. Explanation: Socialists generally argue that private property relations limit the potential of the productive forces in the economy. Q. 2. Assertion (A): Workers in England and Germany began forming associations. Reason (R): They wanted new machinery to be R installed in factories.  Ans. Option (C) is correct. Explanation: Workers in England and Germany began forming associations to fight for better living and working conditions. They set up funds to help members in times of distress and demanded a reduction of working hours and the right to vote. Q. 3. Assertion (A): Some socialists believed in the idea of cooperatives. Reason (R): Other socialists felt that cooperatives could not be built on a wide scale only through individual initiative: they demanded that E governments encourage cooperatives.  Ans. Option (B) is correct. Explanation: Socialists had different visions of the future.

SUBJECTIVE TYPE QUESTIONS Very Short Answer Type Questions Q. 1. How were liberals not democratic? U Ans. Liberals wanted a nation which tolerated all religions. However, they were not ‘democrats’: (i) They did not believe in the universal adult franchise, that is, the right of every citizen to vote. (ii) They felt men of the property mainly should have the right to vote. (iii) They also did not want the vote for women. (Any Two Points) (1 × 2 = 2) Q. 2. How were the ideas of the liberals different from E that of the radicals in Europe? Ans. (i) Liberals argued for a representative, elected parliamentary government, subject to laws interpreted by a well-trained judiciary that was independent of rulers and officials, whereas radicals were in favour of having a government based on

(2 marks each)

the majority of the country’s population. (ii) Liberals did not believe in the universal adult franchise, that is, the right of every citizen to vote. They felt men of property mainly should have the vote. They also did not want the vote for women. The radicals on the other hand supported women’s suffragette movements.  (2) R Q. 3. What do you know about Karl Marx? Ans. (i) Karl Marx was a philosopher who favoured socialism. (ii) He believed that to free themselves from capitalist exploitation, workers had to construct a radically socialist society, where all the properties were socially controlled.  (1 × 2 = 2) Q. 4. A communist society was the natural society of the A future. Explain.

SOCIALISM IN EUROPE AND THE RUSSIAN REVOLUTION

Ans. Marx believed that to free themselves from capitalist exploitation, workers had to construct a radically socialist society where all property was socially controlled. Workers had to overthrow capitalism and the rule of private property. This would be a communist society. He was convinced that workers would triumph in their conflict with capitalists. Hence, a communist society was the natural society of the future. (2)

Q. 5. How did the workers in England and Germany fight for better living and working conditions? U Ans. Workers in England and Germany: (i) Began forming associations to fight for better living and working conditions. (ii) They set up funds to help members in times of distress. (iii) They demanded a reduction of working hours and the right to vote. (Any Two) (1 × 2 = 2)

Short Answer Type Questions Q. 1. What was the vision of Robert Owen and Louis Blanc related to co-operatives? Explain. 

U [Board Term-I, 2016]

Ans. Views of Robert Owen: (i) He sought to build a cooperative community called New Harmony in Indiana (USA). Views of Louis Blanc: (ii) In France, Blanc wanted the government to encourage co-operatives and replace capitalist enterprises. (iii) These co-operatives were the associations of people who produced goods together and divided the profits according to the work done by members. (1 × 3 = 3) [CBSE Marking Scheme, 2016] Q. 2. Describe the ideas of Karl Marx and Friedrich Engels about the capitalists.  R [Board Term-I, 2016] Ans. (i) Marx argued that industrialist society was ‘capitalist’. (ii) The condition of workers could not improve as long as the profit was accumulated by private capitalists. (iii) Marx believed that to free themselves from capitalist exploitation, the workers had to construct a radical society. (iv) He believed that the communist society was the natural society of the future. (Any three) [CBSE Marking Scheme, 2016] (1×3=3

(3 marks each)

Commonly Made Error

Many students tend to take Liberals and Capitalists as same.

Answering Tips

Liberals advocates the freedom of the

individual; whereas Capitalist believes in private ownership of the means of production and their operation for profit. Students should learn more about them before attempting the question.

Q. 3. Describe the views of Radicals. R  [Board DDE Term-I, 2014] Ans. The views of Radicals were as follows : (i) The Radicals wanted to bring about immediate social change in Russia. (ii) They wanted to form a government based on the majority of country’s population. (iii) They were against private properties. [CBSE Marking Scheme, 2014] (1 × 3 = 3)

Long Answer Type Questions Q. 1. What is Socialism? Mention its four features.

27

(5 marks each)

R

Commonly Made Error

[Board Term-I, 2016] Ans. Socialism is a body of ideas to restructure society on equal status. Its four features are: (i) It is against ownership of private property. (ii) Society as a whole should control the property, collectively. (iii) It stands for collective social interests rather than personal gains. (iv) Socialism is in favour of cooperatives and that wages should be given according to work done.  [CBSE Marking Scheme, 2016] (1 + 4= 5)



Some students mention the causes of Socialism instead of its features.

Answering Tip

The students should read the question carefully and understand that the meaning of features/ characteristics is totally different from that of the causes.

28

Oswaal CBSE Question Bank Chapterwise & Topicwise, SOCIAL SCIENCE, Class-IX

Q. 2. Explain the thoughts and beliefs of Karl Marx which convinced the workers to enter into conflict with the Capitalists. U [Board Term-I, 2015]

Q. 3. What were the main objectives of Liberals in Russia? U [Board Term-I, 2013]

Ans. The thoughts and beliefs of Karl Marx: (i) Industrial society was capitalist. Profit of capitalists was produced by workers. (ii) Conditions of workers could not improve till capitalists accumulated wealth. (iii) Workers should overthrow capitalism and private property. (iv) Workers should construct a radically socialist society where all property would be socially controlled. This would be a communist state. (v) Karl Marx believed that all of this would help the workers win over the capitalists. [CBSE marking Scheme, 2015] (1 × 5 = 5)

(i) They wanted a nation which tolerated all the religions.

Commonly Made Error

(ii) They opposed the uncontrolled power of dynastic rulers. (iii) They wanted to safeguard the rights of the individual against the government. (iv) They did not believe in Universal Adult Franchise as they were not democrats. (v) Liberals argued for a representative Government. They were subjected to laws interpreted by a welltrained judiciary that was independent of rulers and officials. (1 × 5 = 5)

Commonly Made Error

The concept of 'Industrial Society' is not clear to

The students get confused between the ideas of Liberals, Radicals and Conservatives.

most of the students.

Answering Tip

Ans. The main objectives of Liberals were as follows:

An Industrial Society is a society driven by the

Answering Tip

Students should focus majorly on the ideas of Liberals and the reasons behind that.

use of technology to enable mass production, supporting a large population. A thorough reading of the topic will help in answering the question better.

Topic-2 The Russian Revolution Revision Notes  Socialists took over the Government in Russia through the October Revolution of 1917. The fall of monarchy in February 1917 and the events of October are normally called the Russian Revolution.

The Russian Empire in 1914  In 1914, Tsar Nicholas II ruled over Russia and its empire. Besides the territory around Moscow, the Russian empire included present-day Finland, Latvia, Lithuania, Estonia, parts of Poland, Ukraine and Belarus. It stretched to the Pacific and comprised today’s Central Asian states, as well as Georgia, Armenia and Azerbaijan.  The majority religion was Russian Orthodox Christianity.

Economy and Society

 In the beginning of the twentieth century, the vast majority of Russia’s people were agriculturists.  Russia was a major exporter of grain.  Industry was found in pockets. Prominent industrial areas were St. Petersburg and Moscow.  Most industries were the private properties of industrialists. Government supervised large factories to ensure minimum wages and limited hours of work.  Peasants cultivated most of the land but the nobility, the Crown and the Orthodox Church owned large properties.

Scan to know more about this topic

The Russian Revolution- 1917

SOCIALISM IN EUROPE AND THE RUSSIAN REVOLUTION

29

 Nobles got power and position through their services to the Tsar.  In Russia, peasants wanted the land of the nobles. ocialism in Russia S  The Russian Social Democratic Labour Party was founded in 1898 by the socialists who respected Marx’s ideas.  Socialists formed the Socialist Revolutionary Party in 1900. This party struggled for peasants’ rights and demanded that land belonging to nobles be transferred to peasants.  Vladimir Lenin (who led the Bolshevik group) thought that in a repressive society like Tsarist Russia, the party should be disciplined and should control the number and quality of its members.

A Turbulent Time : The 1905 Revolution  Russia was an Autocracy.  The year 1904 was a particularly bad one for Russian workers. Prices of essential goods rose so rapidly that real wages declined by 20 per cent. The membership of workers' associations increased dramatically.  When four members of the 'Assembly of Russian Workers' which had been formed in 1904, were dismissed at the Putilov Iron Works, there was a call for industrial action.  Over the next few days, more than 1,10,000 workers in St. Petersburg went on strike demanding a reduction in the working day to eight hours, an increase in wages and improvement in working conditions.  When the procession of workers led by Father Gapon reached the Winter Palace, it was attacked by the police and the Cossacks. Over 100 workers were killed and about 300 wounded. The incident, known as Bloody Sunday, started a series of events that became known as the 1905 Revolution.  During the 1905 Revolution, the Tsar allowed the creation of an elected consultative Parliament Scan to know or Duma. more about

The First World War and the Russian Empire

this topic

1. In 1914, war broke out between two European alliances – Germany, Austria and Turkey (the Central Powers) and France, Britain and Russia (later Italy and Romania). This was the First World War. The Russian Revolution- 1905 2. In Russia, the war was initially popular and people rallied around Tsar Nicholas II. 3. Defeats were shocking and demoralising. Russia's armies lost badly in Germany and Austria between 1914 and 1916. 4. The war also had a severe impact on industry. Russia's own industries were few in number and the country was cut off from other suppliers of industrial goods by German control of the Baltic Sea. 5. By 1916, Railway lines began to break down. 6. Able-bodied men were called up to the war. As a result, there was labour shortage and small workshops producing essentials were shut down.

Key Terms  Bolsheviks: A fraction of the Russian Social Democratic Labour Party led by Lenin based on the ideology of Marx and Engels. It seized power in the October Revolution of 1917.  Bloody Sunday: A mass of peaceful workers were fired upon by the Russian troops when they went to the Winter Palace to present a petition to the Tsar. This incident occurred on Sunday 22nd January 1905.  Duma: Russian Parliament of Legislature.  Divine Right Theory: The theory that believed that the King was the representative of the God on Earth and no one has the right to deny him.  Jadidists: Muslim reformers in the Empire of Russia.

Key Dates  1904: A strike began at the Putilov Iron Works in St. Petersburg.  22nd January, 1905: (Bloody Sunday) peaceful demonstrators arrived at the Winter Palace in St. Petersburg to present a petition to the Tsar.

30

Oswaal CBSE Question Bank Chapterwise & Topicwise, SOCIAL SCIENCE, Class-IX

OBJECTIVE TYPE QUESTIONS A Multiple Choice Questions Q. 1. When was the Socialist Revolutionary Party formed in Russia? R (A) In 1908 (B) In 1918 (C) In 1900 (D) In 1950 Ans. Option (C) is correct. Q. 2. What was the elected Consultative Parliament in Russia called? R (A) The Duma (B) The Lok Sabha (C) The Congress (D) The Senate Ans. Option (A) is correct.

Explanation: Duma is an elected consultative Parliament which was created as a result of the 1905 Revolution. Q. 3. Identify the Russian leader from the picture given R below:

Q. 4. Who was the ruler of Russia in 1914? (A) Catherine (B) Joseph Mazzini (C) Czar Nicholas II (D) Czar Nicholas I Ans. Option (C) is correct.



(A) (B) (C) (D) Q. 1.

Ans.

Q. 2.

Ans.

(B) Tsar Nicholas II (D) Alexander Kerensky

R

B Assertion and Reason MCQs



(A) Lenin (C) Joseph Stalin Ans. Option (B) is correct.

(1 mark each)

Q. 3. Ans.

In the questions given below, there are two statements marked as Assertion (A) and Reason (R). Read the statements and chose the correct option: Both (A) and (R) are true and (R) is the correct explanation of (A). Both (A) and (R) are true, but (R) is not the correct explanation of (A). (A) is correct, but (R) is wrong. (A) is wrong, but (R) is correct. Assertion (A): The Russian Social Democratic Workers Party had to operate as an illegal organisation. Reason (R): All political parties were illegal in U Russia before 1914.  Option (A) is correct. Explanation: The Russian Social Democratic Workers Party was founded in 1898 by socialists. However, because of government policing, it had to operate as an illegal organisation. Assertion (A): The year 1904 was a particularly bad one for Russian workers. Reason (R): During the 1905 Revolution, the Tsar allowed the creation of an elected consultative U Parliament or Duma.  Option (B) is correct. Explanation: The year 1904 was a particularly bad one for Russian workers. Prices of essential goods rose so quickly that real wages declined by 20 per cent. The membership of workers’ associations rose dramatically. Assertion (A): Industrial equipment disintegrated more rapidly in Russia than elsewhere in Europe. Reason (R): The war also had a severe impact on U industry. Option (A) is correct.

SUBJECTIVE TYPE QUESTIONS Very Short Answer Type Questions Q. 1. Which incident came to be known Sunday? Ans. When four members of the Assembly workers were dismissed, there was industrial action. When the procession

as Bloody E

of Russian a call for of workers

(2 marks each)

led by Father Gapon reached the Winter Palace it was attacked by the police and the Cossacks. Over 100 workers were killed and about 300 wounded. The incident came to be known as Bloody Sunday. (2)

SOCIALISM IN EUROPE AND THE RUSSIAN REVOLUTION

Q. 2. Describe the social and economic conditions of the workers in Russia at the beginning of the 20th U century. Ans. (i) Workers were a divided social group. (ii) The working conditions were very poor. (iii) Women made up 31 per cent of the factory labour force by 1914, but they were paid less than men. (iv) Divisions among workers showed themselves in dress and manners too. (Any Two Points) (1 × 2 = 2) Q. 3. Why had riots at bread shops become common in E the winters of 1916? Ans. Able-bodied men were called up to the war. As a result, there were labour shortages and small

workshops producing essentials were shut down. Large supplies of grain were sent to feed the army. For the people in the cities, bread and flour became scarce. Hence by the winter of 1916, riots at bread shops were common. (2) Q. 4. How did the First World War on the ‘eastern front’ U differ from that on the ‘western front’? Ans. (i) The western front consisted of a smaller area, whereas, the eastern front was huge in size. (ii) In the west, armies fought from trenches stretched along eastern France, whereas, the eastern front lacked the concept of trench warfare like in the West.  (1 × 2 = 2)

Short Answer Type Questions

(3 marks each)

Q. 1. What type of economy existed in Russia before the Revolution ? U [Board Term I, 2016] Ans. Before the Russian Revolution: (i) Vast population was agriculturalists. (ii) Cultivators produced for the market as well as their own needs. (iii) Industry in the form of private properties was found in pockets – factories were set up in 1890s and 1900s. (1×3=3) [CBSE Marking Scheme, 2016] Q. 2. Describe the political conditions in Russia before the Revolution. U [Board Term I, 2016] Ans.  (i) Autocratic rule of the Tsar was not subject to Parliament control. (ii) Political parties were considered illegal. (iii) A vast empire consisted of various nationalities, but the Tsar was insensitive to their demands. [CBSE Marking Scheme, 2016] (1 × 3 = 3) Q. 3. Describe any three changes that took place in the year of 1904 in Russia. 

U [Board Term I, 2015]

Ans. Three changes that took place in Russia in 1904 were: (i) Prices of essential goods rose so quickly that real wages declined by 20%. (ii) The membership of workers’ association increased dramatically. (iii) Four members of the Assembly of Russian Workers were dismissed at the Putilov Iron Works. (iv) Over 1,10,000 workers in St. Petersburg went on strike demanding a reduction in the working day to eight hours, an increase in wages and improvement in working conditions. (Any three) [CBSE Marking Scheme, 2015] (1 × 3 = 3)

31

Commonly Made Error

The students mention only about the political changes.

Answering Tip

Students should mention about the economic,

social and political changes that took place in the year 1904 in Russia.

Q. 4. Mention the three major demands raised by Russian workers who went on strike at St. Petersburg in 1904. R [Board Term I, 2015] Ans. (i) (ii) (iii)

The demands raised by the Russian workers: Reduction in the working day to eight hours. Increase in wages. Improvement in working conditions. [CBSE Marking Scheme, 2015] (1×3=3) Q. 5. In what ways were working population of Russia different from other countries in Europe before 1917 ? A [Board NCT Term I, 2014] [NCERT] Ans. (i) The working population in European countries were a lot more united than those in Russia. The condition of Russian people, especially those of the working population like the farmers and the factory workers, was deplorable as compared to other European countries. (ii) It was mainly due to the autocratic government of Tsar Nicholas-II, who antagonised these people day-by-day by his corrupt and oppressive policies. (iii) The peasants worked as serfs on the land and much of their produce went into the hands of landowners and the privileged classes. The nobility, the Crown and the Orthodox Church owned large properties.

32

Oswaal CBSE Question Bank Chapterwise & Topicwise, SOCIAL SCIENCE, Class-IX

(iv) Although, these peasants were generally deeply religious yet they had no respect for the nobility. In European countries, the peasants respected nobles and fought for them. But in Russia, peasants wanted the land of the nobles to be given to them.  (Any three) (1 × 3 = 3) Q. 6. Describe the incident known as the ‘Bloody Sunday’. State any two events after the Bloody Sunday which led to the Revolution of 1905 in Russia.  U [Board Term I, 2014] Ans. The incident of attack by police over protesting workers demanding improvement in working

condition in Russia which left many workers killed and wounded, is known as the ‘Bloody Sunday’. Events after that which led to the Revolution of 1905 in Russia: (i) Strike took place all over the country, universities closed down. (ii) Lawyers, doctors, engineers and other middle class workers established the Union of Unions and demanded a Constituent Assembly. (1 + 2 = 3) [CBSE Marking Scheme, 2014]

Long Answer Type Questions Q. 1. Explain any five socio-economic conditions of Russia at the beginning of the twentieth century.  U [Board Term I, 2016] Ans. Socio-economic conditions of Russia: (i) Social inequality was very prominent among the working class. (ii) Economically, Russia was going through a very difficult period. (iii) The population had doubled and the economic conditions turned from bad to worse. (iv) Most people were agriculturists. (v) Industries were found in pockets and craftsmen undertook much of the production. (vi) Most industries were the private properties of industrialists. (vii) Workers were divided on the basis of their occupation and skills. (viii) Women made up 31 per cent of the factory labour, but they were paid less than men. (ix) Like workers, peasants too were divided. (Any five points) [CBSE Marking Scheme, 2016] (1×5=5)

Commonly Made Error

Students only mention about the occupation

practiced by Russian population in the 20th century.

Answering Tip

The students are required to mention about the overall conditions of Russia in the 20th century.

Q. 2. Highlight the social and political conditions that led to the Russian Revolution. R  [Board Term I, 2016] Ans. (i) Autocratic rule of Tsars. (ii) Conditions of peasants, who earned their living through agriculture.

(5 marks each)

(iii) Status of industry—factory workers and craftsmen were almost in equal numbers. (iv) Condition of workers—their working hours were sometimes up to 15 hours. (v) Status of nobles—got their power by serving Tsar. Peasants wanted land of nobles.  [CBSE Marking Scheme, 2016] (1 × 5 = 5)

Commonly Made Error

The students answer only one part of the question.

Answering Tip

Mention both the social and political conditions that led to the Russian Revolution.

Q. 3. How did the First World War affect the Soviet? A Ans. (i) In Russia, the war was initially popular and people rallied around Tsar Nicholas II. (ii) Russia’s armies lost badly in Germany and Austria between 1914 and 1916. (iii) The war also had a severe impact on industry. Russia’s own industries were few in number and the country was cut off from other suppliers of industrial goods by German control of the Baltic Sea. (iv) Industrial equipment disintegrated more rapidly in Russia than elsewhere in Europe. (v) By 1916, railway lines began to break down. Ablebodied men were called up to the war. As a result, there were labour shortages and small workshops producing essentials were shut down.  (1 × 5 = 5)

SOCIALISM IN EUROPE AND THE RUSSIAN REVOLUTION

33

Topic-3 The February Revolution in Petrograd Revision Notes  In the winter of 1917, conditions in the capital, Petrograd, were grim.  In February 1917, food shortages were deeply felt in the workers’ quarters. The winter was very Scan to know more about cold – there had been exceptional frost and heavy snow. this topic  On 22nd February, a lockout took place at a factory on the right bank. The next day, workers in fifty factories called a strike in sympathy.  In many factories, women led the way to strikes. This came to be called the ‘International Women’s Day.’ The February  Finally, on Sunday, 25th February, the Government suspended the Duma. Revolution  Demonstrators returned in force to the streets of the left bank on the 26th February. The streets thronged with people raising slogans about bread, wages, better hours and democracy.  By that evening, soldiers and striking workers had gathered to form a ‘Soviet’ or ‘Council’ in the same building as the Duma met. This was the Petrograd Soviet.  Finally the Tsar abdicated on 2nd March.  Soviet leaders and Duma leaders formed a Provisional Government to run the country.  Petrograd had led the February Revolution that brought down the monarchy in February 1917.

After February  Army officials, landowners and industrialists were influential in the Provisional Government. But the liberals as well as socialists among them worked towards an elected government.  In April 1917, the Bolshevik leader Vladimir Lenin returned to Russia from his exile.  Three demands of Lenin’s ‘April Theses’: • He felt, it was time for the Soviets to take over power. • Land should be transferred to the peasants. • Banks should be nationalised.

The Revolution of October 1917  As the conflict between the Provisional Government and the Bolsheviks grew, Lenin feared the Provisional Government would set up a dictatorship.  On 16th October 1917, Lenin persuaded the Petrograd Soviet and the Bolshevik Party to agree to a socialist seizure of power.  A Military Revolutionary Committee was appointed by the Soviet under Leon Trotsky to organise the seizure.  At a meeting of the All Russian Congress of Soviets in Petrograd, the majority approved the Bolshevik action.

Key Terms  The April Theses: They were a series of ten directives issued by the Bolshevik leader Vladimir Lenin upon his April 1917 return to Petrograd from his exile.

Key Fact

Lenin's eldest brother, Aleksandr, a student at the University of St. Petersburg, was hanged for conspiring with a revolutionary terrorist group that plotted to assassinate Emperor Alexander III.

Key Dates  22nd to 27th February, 1917: February Revolution.  2nd March, 1917: Abdication of the Tsar.  3rd April 1917: Return of Lenin and April Theses.

34

Oswaal CBSE Question Bank Chapterwise & Topicwise, SOCIAL SCIENCE, Class-IX

 5th May 1917: Formation of new Provisional Government.  3rd June 1917: First All-Russian Congress of Soviets announced in Petrograd.

Example

How was a Socialist State established by Lenin? Explain Answer: Step 1: Centralized planning with a system of fiveyear plan.



Step 2: Private property were abolished and property was socially controlled. Step 3: An extended schooling system was developed.



OBJECTIVE TYPE QUESTIONS A Multiple Choice Questions Q. 1. In April 1917, the Bolshevik leader ________ returned to Russia from his exile. R (A) Czar Nicholas II (B) Lenin (C) Rasputin (D) Marfa Ans. Option (B) is correct. Q. 2. Look at the picture given below and identity the personality. A

(C) She threw a bomb at the Russian Tsar. (D) None of the above Ans. Option (B) is correct.

B Assertion and Reason MCQs

(A) (B) (C) (D)

(A) Lenin addressing workers in April 1917. (B) Kerenskii addressing workers in April 1917 (C) Stalin addressing workers in April 1917 (D) None of the above Ans. Option (A) is correct. Q. 3. What was the impact of the First World War on Russia? E (A) Russia had lost about seven million soldiers. (B) Its crops and buildings were destroyed badly. (C) Heavy industrial loss created an acute food scarcity. (D) All of the above Ans. Option (D) is correct. Q. 4. Who was Marfa Vasileva? R (A) She was the leader of the revolutionaries. (B) She was a brave worker who organised a successful strike.

(1 mark each)

In the questions given below, there are two statements marked as Assertion (A) and Reason (R). Read the statements and chose the correct option: Both (A) and (R) are true and (R) is the correct explanation of (A). Both (A) and (R) are true, but (R) is not the correct explanation of (A). (A) is correct, but (R) is wrong. (A) is wrong, but (R) is correct.

Q. 1. Assertion (A): In the winter of 1917, conditions in the capital, Petrograd, were grim. Reason (R): The layout of the city seemed to U emphasis the divisions among its people.  Ans. Option (A) is correct. Q. 2. Assertion (A): On Sunday, 25 February 1917, the government suspended the Duma. Reason (R): The government tried to control the situation and called out the cavalry once again. E Ans. Option (B) is correct. Explanation: After the beginning of the First World War and several large defeats of the Russian army, the serious political conflict appeared between the Duma and executive power. On 25 February 1917, Emperor Nicholas II signed a decree on the dissolution of the Duma till April 1917. Q. 3. Assertion (A): In September, Lenin began discussions for an uprising against the government. Reason (R): As the conflict between the Provisional Government and the Bolsheviks grew, Lenin feared the Provisional Government would set up a A dictatorship. Ans. Option (A) is correct.

SOCIALISM IN EUROPE AND THE RUSSIAN REVOLUTION

35

SUBJECTIVE TYPE QUESTIONS Very Short Answer Type Question R Q. 1. Who was Lenin? Ans. Lenin was the leader of the radical socialist Bolshevik Party. He rose to prominence during the Russian Revolution of 1917. He served as the first and founding head of government of Soviet Russia from 1917 to 1924. (2)

(2 marks each)

Q. 2. Why did the Tsar dismiss the first Duma within 75 days and re-elect the second Duma within three A months?  Ans. The Tsar dismissed the first Duma within 75 days and re-elected the second Duma within three months because he did not want any questioning of his authority or any reduction in his power. He changed the voting laws and packed the third Duma with conservative politicians. Liberals and revolutionaries were kept out. (2)

Short Answer Type Questions R Q. 1. Explain the Lenin’s ‘April Theses’.  [Board Term I, 2016] OR What were the main demands of April Theses?  [Board Term I, 2014] OR Describe the three major demands of Bolsheviks during 1917. OR Explain Lenin’s ‘April Theses’. What were its effects on the party members?

Ans. Demands of ‘April Theses’: (i) Lenin returned to Russia from exile and felt that it was time for the Soviets to take over power. He declared that war be brought to a close, land be transferred to the peasants and banks nationalised. He also argued to rename Bolshevik Party as Communist Party. (ii) Started planning on uprising against government with the support of army. (iii) Military Revolutionary Committee was planned to seize the power and bring the city under control. (iv) Initially, the members were surprised and thought that the time was not right for Socialist Revolution, but the developments in the subsequent months changed their attitude. [CBSE Marking Scheme, 2012] (1×3=3)

Commonly Made Error

The students tend to answer the demands of

the April Theses without knowing the meaning of the same.

(3 marks each)

Answering Tip

April Theses were a series of ten directives

issued by Lenin. A thorough reading of the topic is advisable before attempting the topic.

Q. 2. Describe the conditions of the working women during the years from 1900 to 1930 in Russia. U [Board Term I, 2015] [NCERT] Ans. Conditions of the working women in Russia: (i) Took active part in the revolution and often inspired their male co-workers. (ii) Women made up 31 per cent of factory labour. (iii) They were less than their male counterparts. (iv) During February Revolution, the women workers led the way to strikes. (v) 22nd February was called International Women’s Day. (Any three) (1 × 3 = 3) [CBSE Marking Scheme, 2015] Q. 3. How did the Tsarist autocracy collapse in 1917? Explain. A [Board Term I, 2015] OR State the reasons for the collapse of Tsarist autocracy in 1917. [Board Term I, 2014] OR Why did Tsarist autocracy collapse in 1917?  [Board Term I, 2013] [NCERT] Ans. Causes of Tsarist autocracy collapse were : (i) The Industrial Workers in Russia got very low wages and had very long working hours, sometimes up to 15 hours. (ii) The workers demanded higher wages and reduction in working hours, but their demands were not met and they became dissatisfied.

36

Oswaal CBSE Question Bank Chapterwise & Topicwise, SOCIAL SCIENCE, Class-IX

(iii) Most of the peasants were landless and very poor. Due to the autocratic government of Tsar Nicholas II, the condition of the Russian people, farmers and factory workers was highly miserable. As a result, his Autocracy collapsed in 1917. (iv) The peasants worked as serfs on the land. Most of the produce went to the privileged classes and the landowners. As a result, farmers suffered from hunger. Later, due to these oppressive policies and out of frustration, the farmers refused to pay rent to the landlords. (v) The workers could not form any trade unions and political parties to express their grievances. The workers did not get even the minimum fixed wages. They used to work for 12–15 hours a day, due to no limit on working hours. (vi) Karl Marx taught the people to raise a standard revolt. (vii) The Revolution of 1905 also proved to be a dress rehearsal of the Revolution of 1917. (viii) Tsar’s participation and defeat in the First World War served the last reason of failure of his autocracy in 1917. (Any three) (1 × 3 = 3)

Commonly Made Error

The students explain majorly about the poor conditions of peasants under Tsarist rule.

Answering Tip

Condition of industrial workers, influence of philosophical ideas also needs to be explained.

Q. 4. What happened after the February Revolution? E Ans. (i) The February Revolution saw the fall of the monarchy. After the fall of monarchy a Provincial Government was formed in Russia. Lenin returned from his exile and propagated three points from April thesis. (ii) The Provincial Government,with Kerensky as its head, saw Lenin as a threat and started arresting the Bolsheviks and resisted the spread of their ideas. (iii) Lenin feared Kerinsky's dictatorship and convinced the Bolsheviks to stage an uprising.In October 1917, the Bolsheviks, with the help of Leon Trotskii, seized power from the Provincial Government.  (1 x 3 = 3)

U Q. 5. Why was the Tsar abdicated? Ans. Reasons for Tsar's abdication: (i) By 1917, most Russians had lost all faith in the leadership ability of the Tzar. (ii) Government corruption was rampant. (iii) The Russian economy was severely battered by the World War I. (1 x 3 = 3)

Long Answer Type Questions Q. 1. Describe February Revolution of 1917 and October Revolution of 1917 in brief. U  [Board Term I, 2015] [NCERT] Ans. February Revolution: (i) Factory locked out on the right bank on 22nd February. (ii) Sympathy strike was done by workers in 50 factories on 23rd February. (iii) On 25th February, Government suspended the Duma and demonstrators came on roads. (iv) Police Headquarters were ransacked by Workers. Regiment supported the Workers and they formed the Petrograd Soviet. (v) On 2nd March, the Tsar abdicated and Soviet and Duma leaders formed the Provisional Government. October Revolution: (i) On 16th October, Vladimir Lenin, the Bolshevik leader persuaded the Petrograd Soviet. (ii) A Military Revolutionary Committee was appointed by the Soviet. (iii) The Military Committee seized the government offices and arrested ministers. (iv) The Winter Palace was shelled.

(5 marks each)

(v) Ministers of the Provisional Government surrendered. (vi) The Bolsheviks gained control. [CBSE Marking Scheme, 2015] (2½×2=5)

Commonly Made Error

The students mentioned incorrect date/ year.

Answering Tip

Events should be mentioned in a chronological order.

Q. 2. Mention the main events of the February Revolution of 1917. U [Board Term I, 2014] OR State any five events that led to the February Revolution in Petrograd. Ans. Main events of the February Revolution were : (i) In February 1917, there was a severe food shortage in the workers’ quarters.

SOCIALISM IN EUROPE AND THE RUSSIAN REVOLUTION

(ii) All the factories and workers’ quarters were located on the right bank of the River Neva. On the left bank, there were the Winter Palace, official buildings and the Palace where Duma met. (iii) On 22nd February, a lockout took place at a factory leading to a strike by the Workers. The next day, workers in fifty factories called a strike in sympathy. (iv) The demonstrating workers ultimately crossed the river and surrounded the official buildings in Petrograd. (v) The Government imposed a curfew and called out the cavalry and police to keep check on them. [CBSE Marking Scheme, 2014] (1 × 5 = 5) Q. 3. What

changes

were

seen

October Revolution of 1917?



in

Russia

after A

[Board Term I, 2014] OR What are the main changes brought about by the Bolsheviks immediately after the October Revolution?  [Term I, KVS 2019, NCT-2014]



37

OR Relate the changes which followed the October Revolution in Russia. [DDE Term I, 2014] [Board 2013] OR Describe any five changes brought about by the Bolsheviks immediately after the October Revolution.

Ans. Changes were: (i) Most of the industries and banks were nationalised in November 1917. (ii) Land was declared social property. (iii) In cities, Bolsheviks enforced the partition of large houses according to family requirements. (iv) Banned the use of the old titles of aristocracy. (v) New uniforms were designed for the army and officials. (1×5=5) [CBSE Marking Scheme, 2014]

Topic-4 Changes after October and the Global Influence Revision Notes Changes after October  The Bolsheviks were totally opposed to private property. Most industries and banks were nationalised in November 1917.

Scan to know more about this topic

 Land was declared social property and peasants were allowed to seize the land of the nobility.  In cities, Bolsheviks enforced the partition of large houses according to family requirements. Changes after They banned the use of the old titles of aristocracy. October  The Bolshevik Party was renamed as the Russian Communist Party (Bolshevik).  In November 1917, the Bolsheviks conducted the elections to the Constituent Assembly, but they failed to gain majority support.  In the years that followed, the Bolsheviks became the only party to participate in the elections to the All Russian Congress of Soviets, which became the Parliament of the country. Russia became a One-Party State.

The Civil War  During 1918 and 1919, the ‘Greens’ (Socialist Revolutionaries) and ‘Whites’ (Pro-Tsarists) controlled most of the Russian Empire.  As these troops and the Bolsheviks fought a civil war, looting, banditry and famine became common.  By January 1920, the Bolsheviks controlled most of the former Russian Empire. They succeeded due to cooperation with non-Russian nationalities and Muslim Jadidists.  Most non-Russian nationalities were given political autonomy in the Soviet Union (USSR) – the state the Bolsheviks created from the Russian Empire in December 1922.

Making a Socialist Society  A process of centralised planning was introduced. Officials assessed how the economy could work and set targets for a five-year period. On this basis, they made the Five Year Plans.  Industrial production increased between 1929 and 1933 by 100 per cent in the case of oil, coal and steel.  An extended schooling system developed and arrangements were made for factory workers and peasants to enter universities.

38

Oswaal CBSE Question Bank Chapterwise & Topicwise, SOCIAL SCIENCE, Class-IX

 Crèches were established in factories for the children of women workers.  Cheap public health care was provided. Model living quarters were set up for workers.

Stalinism and Collectivisation  By 1927–1928, the towns in Soviet Russia were facing an acute problem of grain supplies. The Government fixed prices at which grain must be sold, but the peasants refused to sell their grain to government buyers at these prices.  Stalin, who headed the party after the death of Lenin, introduced firm emergency measures.  From 1929, the Party forced all peasants to cultivate in collective farms (Kolkhoz).  Those who resisted collectivisation were severely punished. Many were deported and exiled.  In spite of collectivisation, production did not increase immediately and due to bad harvests of 1930–1933 over 4 million people died.

The Global Influence of the Russian Revolution and the USSR  In many countries, communist parties were formed like the Communist Party of Great Britain.  Many non-Russians from outside the USSR participated in the Conference of the Peoples of the East (1920) and the Bolshevik-founded Comintern (an international union of pro-Bolshevik socialist parties).  By the time of the outbreak of the Second World War, the USSR had given socialism a global face and world stature.

Scan to know more about this topic

Global Influence

Key Terms  The Bolsheviks: It was the communist party of Russia that was formed in the year 1917 by Lenin.  Soviet: Council of workers.  Serfdom: Russian type of feudalism under which peasants worked for the landlord in exchange for food and shelter.

Key Dates  1917–1920: Civil War broke out in Russia.  1919: Formation of Comintern.  1929: Beginning of collectivisation.  1991: Break up of Soviet Union.

OBJECTIVE TYPE QUESTIONS A Multiple Choice Questions Q. 1. Who were known as The ‘Greens’? R (A) The Bolsheviks (B) The pro-Tsarists (C) The Socialist Revolutionaries (D) None of the above Ans. Option (C) is correct. Explanation: The Greens were armed peasant groups which fought against all governments in the Russian Civil War from 1917 to 1922. Q. 2. By which name the well-to-do Peasants in Russia called? R (A) Kulaks (B) Kolkhoj (C) Serfs (D) None of these Ans. Option (A) is correct.

(1 mark each)



Explanation: Kulaks was a title given to a wealthy or prosperous peasant, generally characterised as one who owned a relatively large farm and several head of cattle and horses and who was financially capable of employing hired labour and leasing land. Q. 3. The Government tried to control the demonstrators and called out the: U (A) workers (B) army (C) cavalry (D) infantry Ans. Option (C) is correct.

B Assertion and Reason MCQs

(A)

In the questions given below, there are two statements marked as Assertion (A) and Reason (R). Read the statements and chose the correct option: Both (A) and (R) are true and (R) is the correct explanation of (A).

SOCIALISM IN EUROPE AND THE RUSSIAN REVOLUTION

(B)

Both (A) and (R) are true, but (R) is not the correct explanation of (A). (C) (A) is correct, but (R) is wrong. (D) (A) is wrong, but (R) is correct. Q. 1. Assertion (A): Production increased rapidly due to Stalin’s Collectivisation Programme. Reason (R): There was bad harvest during 1930– U 1933.  Ans. Option (D) is correct. Q. 2. Assertion (A): The period of the early Planned Economy was linked to the disasters of the collectivisation of agriculture.



39

Reason (R): The government fixed prices at which grain must be sold, but the peasants refused to sell their grain to government buyers at these prices. E

Ans. Option (A) is correct. Q. 3. Assertion (A): In spite of collectivisation, production did not increase immediately. Reason (R): The bad harvests of 1930–1933 led to one of most devastating famines in Soviet history E when over 4 million died. Ans. Option (A) is correct.

SUBJECTIVE TYPE QUESTIONS Very Short Answer Type Questions Q. 1. Why were the Bolsheviks totally opposed to E private property? Ans. The Bolsheviks were totally opposed to private property as they thought that private ownership of the means of production provided the basis of political power. By nationalising it, they undermined the opposition. (2) Q. 2. Mention any two changes that the Bolsheviks got R in the Russian society in 1917. Ans. (i) Most industry and banks were nationalised in November 1917. (ii) Land was declared social property and peasants were allowed to seize the land of the nobility. (iii) In cities, Bolsheviks enforced the partition of large houses according to family requirements. They banned the use of the old titles of aristocracy. (Any Two Points) (1 x 2 = 2) Q. 3. What was the main aim of Stalin’s collectivisation program? U Ans. The aim of Stalin’s collectivisation program was to increase the agricultural production. The Party forced all peasants to cultivate in collective farms

(kolkhoz). The bulk of land and implements were transferred to the ownership of collective farms. Those who resisted collectivisation were severely punished. Many were deported and exiled. (2) Q. 4. Why was centralised planning introduced in Russia? E Ans. Centralised planning was introduced in Russia on the basis of which the government made the Five Year Plans. The government fixed all prices to promote industrial growth during the first two ‘Plans’. (1927–1932 and 1933–1938). Centralised planning led to economic growth. Industrial production increased (between 1929 and 1933 by 100 per cent in the case of oil, coal and steel). New factory cities came into being. (2) Q. 5. What were Shaukat Usmani’s observation of Soviet Russia in 1920? R Ans. Shaukat Usmani observed that; ‘For the first time in our lives, we were seeing Europeans mixing freely with Asians. On seeing the Russians mingling freely with the rest of the people of the country we were convinced that we had come to a land of real equality.’ (2)

Short Answer Type Questions Q. 1. What were the three main causes of the Civil War between Bolsheviks and the Russian army, the non-Bolshevik socialists?  U [Board Term-I, 2016] OR Explain any three reasons which led to the Civil War between the Bolsheviks and the Russian army of non-Bolsheviks.

(2 marks each)

(3 marks each)

Ans. (i) When Bolshevik ordered land distribution, the Russian army began to break up. (ii) Non-Bolshevik socialists, liberals and supporters of autocracy condemned this. (iii) Their leaders moved to South Russia and organised trips to fight the Bolsheviks. (iv) These troops and Bolsheviks fought a Civil War and looting and famine became common. (Any three) (1 × 3 = 3) [CBSE Marking Scheme, 2016]

40

Oswaal CBSE Question Bank Chapterwise & Topicwise, SOCIAL SCIENCE, Class-IX

Q. 2. Explain measures taken by Stalin to solve the R problem of acute shortages of grains. [Board Term-I, 2016] Ans. (i) Stalin confiscated supplies to prevent rich peasants and landlords from indulging in blackmarketing. (ii) 'Kulaks' were raided. (iii) Collective farms system known as 'Kolkhoz' was introduced. (iv) Party members supervised and enforced grain collection. (Any three) (1 × 3 = 3) [CBSE Marking Scheme, 2016] Q. 3. Identify the conditions of workers in Socialist Society of Russia. A [Board Term-I, 2016] Ans. (i) Rapid construction led to poor working conditions. Workers lived hard lives and the result was 550 stoppages of work in the first year alone. (ii) An extended schooling system developed and arrangements were made for factory workers and peasants to enter universities. (iii) Creches were established in factories for the children of women workers. (iv) Cheap public healthcare was provided. Model living quarters were set up for workers. 

Q. 5. Who were Soviets? What was their role in the Revolution? A [Board Term I, 2013] Ans. Soviet was a council of striking workers and soldiers who organised February Revolution. (i) The Petrograd Soviet led the revolution. (ii) It helped the Bolsheviks to seize power. [CBSE Marking Scheme, 2013] (1+2=3)

Detailed Answer:



(Any three) (1×3=3)

Q. 4. What agricultural changes were introduced in R Soviet Union after 1917? Explain.

(ii) These peasants had very small holdings of land, which were not productive. So, a new method of agriculture was introduced that was called the collective farm. (iii) Rich farmers opposed this type of farming. They were dealt with harshly by the government. Thus, landlords were oppressed. [CBSE Marking Scheme, 2014] (1 × 3 = 3)



[Board Term-I, 2014]

Ans. Changes: (i) Large estates of Church, landlords, nobility, etc., were taken away by the government and distributed to peasants.

(i) (ii) (iii) (iv)

Soviet was the collective name used for several organisations in Russia. It consisted of a council of workers and soldiers. They came up in 1905 after a textile strike. The main head of Soviet was the Bolshevik. The prominent soviet leaders got arrested and imprisoned. They came back in 1917 and challenged the powers of Nicholas II. The Petrograd Soviet led the revolution. The Petrograd Soviet also led the February Revolution to bring down the monarchy in February 1917. The Second Revolution which is known as the Bolshevik Revolution broke out on October 25, 1917. The four-fold programme of this party was: Land to tiller. Food to the hungry. Power to the Soviets. Withdrawal from the First World War.

Long Answer Type Questions Q. 1. Explain any five features of Stalin’s collectivisation programme for peasants in Russia. U [Board Term I, 2015] OR Explain Stalin’s collectivisation programme.  [Board Term I, 2013] OR Describe any five points of Stalin’s collectivisation programme. OR Write a few lines on Stalin’s collectivisation programme. [NCERT] Ans. (i) Party forced all peasants to cultivate in collective farms. (ii) The bulk of land and implements were transferred to the ownership of collective farms. (iii) Peasants worked on the land and the Kolkhoz profit was shared. (iv) Enraged peasants resisted the authorities and destroyed their livestock.

(5 marks each)

(v) Those who resisted collectivisation were severely punished. (vi) Stalin’s Government allowed some independent cultivation but treated such cultivators unsympathetically. (Any five) [CBSE Marking Scheme, 2015] (1×5=5)

Commonly Made Error

Collectivisation Policy is seen as a Short Term Policy by many students.

Answering Tip

Students should know that Collectivisation Policy was part of the first marked by Stalin.

Five Year Plan

SOCIALISM IN EUROPE AND THE RUSSIAN REVOLUTION

Q. 2. ‘By the end of the twentieth century the international reputation of the USSR as a socialist country had declined. Explain the statement. 

U [Board Term I, 2015]

Ans. The above statement is true because: (i) The style of government in the USSR was not keeping in with the ideals of the Russian Revolution. (ii) All was not well in the Soviet Union; dictatorial Rule of Communist Party began. (iii) There was no democracy, only one Party Rule of Communist Party prevailed. (iv) Repressive policies like collectivisation became unpopular. (v) Life of the individual was controlled by the State. This made the people unhappy. [CBSE Marking Scheme, 2015] (1 × 5 = 5) Q. 3. What

changes

were

seen

in

October Revolution of 1917?

Russia

after A

[Board Term I, 2014] OR





What are the main changes brought about by the Bolsheviks immediately after the October Revolution?  [Term I, KVS 2019, NCT-2014] OR Relate the changes which followed the October Revolution in Russia. [DDE Term I, 2014] [Board 2013] OR Describe any five changes brought about by the Bolsheviks immediately after the October Revolution. [Board Term I, 2012]

Ans. Changes were: (i) Most of the industries and banks were nationalised in November 1917. (ii) Land was declared social property. (iii) In cities, Bolsheviks enforced the partition of large houses according to family requirements. (iv) Banned the use of the old titles of aristocracy. (v) New uniforms were designed for the army and officials. (1×5=5) [CBSE Marking Scheme, 2014]

COMPETENCY BASED QUESTIONS Case based MCQs

I. Read the source given below and answer the E (1+1+1+1=4) following questions:

After February In April 1917, the Bolshevik leader Vladimir Lenin returned to Russia from his exile. He and the Bolsheviks had opposed the war since 1914. Now he felt it was time for Soviets to take over power. He declared that the war be brought to a close, land be transferred to the peasants and banks be nationalised. These three demands were Lenin's 'April Theses'. He also argued that the Bolshevik Party rename itself the Communist Party to indicate its new radical aims. Most others in the Bolshevik Party were initially surprised by the April Theses. They thought that the time was not yet ripe for a socialist revolution and the Provisional Government needed to be supported. But the developments of the subsequent months changed their attitude. 1. When did Bolshevik leader Vladimir Lenin return to Russia from his exile? (A) In April 1918 (B) In April 1915 (C) In April 1916 (D) In April 1917 Ans. Option (D) is correct.

41

(4 marks each)



2. How many demands were there in Lenin's 'April Theses'? (A) Three (B) Five (C) Four (D) Two Ans. Option (A) is correct.

Explanation: Lenin's April Theses contained three demands: First, he declared that the war be brought to an end; second, he wanted the lands to be transferred to the peasants; third, the banks had to be nationalised.



3.  Lenin argued that the Bolshevik Party should rename itself as: (A) Communist Party (B) Socialist Party (C) Russian Social Democratic Labour Party (D) Socialist Revolutionary Party Ans. Option (A) is correct.

Explanation: Lenin also argued that the Bolshevik Party should rename themselves as the Communist Party to indicate its new radical aims.



4. Petrograd had led the February Revolution that brought down the: (A) slavery in the February 1917. (B) monarchy in February 1917. (C) clergy in February 1917. (D) nobility in February 1917. Ans. Option (B) is correct.

42

Oswaal CBSE Question Bank Chapterwise & Topicwise, SOCIAL SCIENCE, Class-IX

II. Read the source given below and answer the U following questions: (1+1+1+1=4) One of the groups which looked to change society were the liberals. Liberals wanted a nation which tolerated all religions. We should remember that at this time European states usually discriminated in favour of one religion or another (Britain favoured the Church of England, Austria and Spain favoured the Catholic Church). Liberals also opposed the uncontrolled power of dynastic rulers. They wanted to safeguard the rights of individuals against governments. They argued for a representative elected parliamentary government, subject to laws interpreted by a welltrained judiciary that was independent of rulers and officials. However, they were not 'democrats'. They did not believe in universal adult franchise, that is, the right of every citizen to vote. They felt men of property mainly should have the vote. They also did not want the vote for women.

1. __________ become one of the most significant and powerful ideas to shape society in the twentieth century, through the revolution in Russia.

(A) Communism

(B) Socialism

(C) Dictatorship

(D) Democracy

Ans. Option (B) is correct.

2. Liberals wanted a nation which tolerated:

(A) no religion

(D) one religion

(C) few religions

(B) all religions

Ans. Option (D) is correct.



Explanation: One of the groups which looked to change society were the liberals. Liberals wanted a nation which tolerated all religions. 3. Which countries favoured the Catholic Church?

(A) France and Spain

(B) Spain and Italy

(C) Austria and Spain

(D) Spain and Germany

Ans. Option (C) is correct.

4. What was the suffragette?

(A) Women’s movement (B) Property tax (C) Land acquisition Ans. Option (A) is correct.

(D) Proletariat class

Subjective based Questions I.

Read the source given below and answer the E following questions: (1+1+2=4) The year 1904 was a particularly bad one for Russian workers. Prices of essential goods rose so quickly that real wages declined by 20 per cent. The membership of workers' associations increased dramatically. When four members of the Assembly of Russian Workers, which had been formed in 1904, were dismissed at the Putilov Iron Works, there was a call for industrial action. Over the next

few days over 1,10,000 workers in St. Petersburg went on strike demanding a reduction in the working day to eight hours, an increase in wages and improvement in working conditions. When the procession of workers led by Father Gapon reached the Winter Palace it was attacked by the police and the Cossacks. Over 100 workers were killed and about 300 wounded. The incident, known as Bloody Sunday, started a series of events that became known as the 1905 Revolution. Strikes took place all over the country and universities closed down when student bodies staged walkouts, complaining about the lack of civil liberties. Lawyers, doctors, engineers and other middleclass workers established the Union of Unions and demanded a constituent assembly. 1. In the year 1904, real wages of the Russian workers declined by what percentage? Ans. In 1904, the real wages of the Russian workers declined by 20 per cent. 2. What was the demand of the workers on strike at St. Petersburg? Ans. At St. Petersburg workers on strike were demanding a reduction in the working hours to eight hours. 3. Name a few series of events that took place during the 1905 Revolution. Ans. Strikes took place all over the country and universities closed down when student bodies staged walkouts, complaining about the lack of civil liberties. Lawyers, doctors, engineers and other middle-class workers established the Union of Unions and demanded a constituent assembly. II. Read the source given below and answer the following questions: (1+1+1+1=4) E When the Bolsheviks ordered land redistribution, the Russian army began to break up. Soldiers, mostly peasants, wished to go home for the redistribution and deserted. Non-Bolshevik socialists, liberals and supporters of autocracy condemned the Bolshevik uprising. Their leaders moved to south Russia and organised troops to fight the Bolsheviks (the 'reds'). During 1918 and 1919, the 'greens' (Socialist Revolutionaries) and 'whites' (pro-Tsarists) controlled most of the Russian empire. They were backed by French, American, British and Japanese troops–all those forces who were worried at the growth of socialism in Russia. As these troops and the Bolsheviks fought a civil war, looting, banditry and famine became common. Supporters of private property among 'whites' took harsh steps with peasants who had seized land. Such actions led to the loss of popular support for the non-Bolsheviks. By January 1920, the Bolsheviks controlled most of the former Russian empire. They succeed due to cooperation with non-Russian nationalities and Muslim jadidists.

SOCIALISM IN EUROPE AND THE RUSSIAN REVOLUTION

Cooperation did not work where Russian colonists themselves turned Bolshevik. In Khiva, in Central Asia, Bolshevik colonists brutally massacred local nationalists in the name of defending socialism. In this situation, many were confused about what the Bolshevik government represented.

43

soldiers, mostly peasants, wished to go home for the redistribution. 2. Who were the ‘Red’? Ans. The Bolsheviks. 3. Who all condemned the Bolshevik uprising? What did the Bolsheviks do in Khiva? Ans. Non-Bolshevik socialists, liberals and supporters of autocracy condemned the Bolshevik uprising.



1. Why did the Russian army began to break up when the Bolsheviks ordered land redistribution? Ans. When the Bolsheviks ordered land redistribution, the Russian army began to break up because the



In Khiva, in Central Asia, Bolshevik colonists brutally massacred local nationalists in the name of defending socialism.

Map Work

l Outline

Political Map of World (For locating and labelling / Identification); l Major countries of First World War; l Central Powers: Germany, Austria, Hungary, Turkey (Ottoman Empire); l Allied Powers: France, England, Russia, USA

Syllabus

Q. Locate and label the following items on the given outline map of world : Central Powers: (a) Germany (b) Austria – Hungary (c) Turkey (Ottoman Empire) Allied Powers: (i) France (ii) England (iii) Russia Ans.

(1 Mark each)

(iv) USA

Austria-Hungary Russia Germany (iii) England (iv)

France . ..

(ii) (a) (i) (b)

Turkey

(c)

. ..

(USA)

Map not to scale



Study Time Max Time: 3:30 hr Max Questions: 81

CHAPTER

3

Learning Objectives

NAZISM AND THE RISE OF HITLER Analyse the manipulated control of situations led by an individual. Analyse the role of the “Treaty of Versailles in that led to the rise of Hitler. l Examine the circumstances that led to the rise and fall of Hitler l Discuss the critical significance of Nazism in shaping the politics of the modern world. l Appraise the war compensation compelled on Germany in the name of the “treaty of Versailles led to the rise of Hitler l Compare and contrast the Nazi ideology with fascism of Mussolini. l l

Topic-1 Birth of the Weimar Republic Revision Notes

TOPIC - 1 Birth of the Weimar Republic .... P. 44 TOPIC - 2 Hitler’s Rise to Power

.... P. 49

TOPIC - 3 The Nazi Worldview

.... P. 54

TOPIC - 4

 Germany, a powerful empire in the early years of the twentieth century, Youth, Ordinary people and fought the First World War (1914–1918) alongside the Austrian empire and the Crimes against Humanity against the Allies (England, France and Russia.) .... P. 57  The defeat of Imperial Germany and the abdication of the Emperor gave an opportunity to parliamentary parties to recast German polity.  A National Assembly met at Weimar and established a democratic constitution with a federal structure.  Deputies were now elected to the German Parliament or Reichstag, on the basis of equal and universal votes cast by all adults including women.  The War Guilt Clause held Germany responsible for the war and damages the Allied countries suffered. The Allied armies occupied Rhineland in the 1920s.  Many Germans held the new Weimar Republic responsible for not only the defeat in the war but the disgrace at Versailles.

The Effects of the War

   

The war had a devastating impact on the entire continent both psychologically and financially. From a continent of creditors, Europe turned into one of the debtors. The First World War left a deep imprint on European society and polity. Soldiers came to be placed above civilians.

Political Radicalism and Economic Crises

 The birth of the Weimar Republic coincided with the revolutionary uprising of the Spartacist League on the pattern of the Bolshevik Revolution in Russia.  Those opposed to this – such as the Socialists, Democrats and Catholics – met in Weimar to give shape to the Democratic republic.  The Weimar Republic crushed the uprising with the help of a war veterans organisation called ‘Free Corps’.  Political radicalisation heightened by the economic crisis of 1923.

Scan to know more about this topic

German Economic crises 1923

NAZISM AND THE RISE OF HITLER

45

46

Oswaal CBSE Question Bank Chapterwise & Topicwise, SOCIAL SCIENCE, Class-IX

 The image of Germans carrying cart loads of currency notes to buy a loaf of bread was widely publicised evoking worldwide sympathy. This crisis came to be known as hyperinflation; a situation when prices arise phenomenally high.

The Years of Depression  The years between 1924 and 1928 saw some stability. German investments and industrial recovery were totally dependent on short-term loans, largely from the USA. This support was withdrawn when the Wall Street Exchange crashed in 1929.  On one single day, 24th October, 13 million shares were sold. This was the start of the ‘Great Economic Depression’.  Over the next three years, between 1929 and 1932, the national income of the USA fell by half. Factories shut down, exports fell, farmers were badly hit and speculators withdrew their money from the Scan to know more about Market. The effects of this recession in the US economy were felt worldwide.  The German economy was worst hit by the economic crisis. Workers lost their jobs or were paid reduced wages.  As jobs disappeared, the youth took to criminal activities and total despair became common place.

this topic

Weimar Republic

 Politically too, the Weimar Republic was fragile. The Weimar Constitution had some inherent defects, which made it unstable and vulnerable to dictatorship.  Another defect was Article 48, which gave the President the powers to impose emergency, suspend civil rights and rule by decree.  Yet, the crisis could not be managed. People lost confidence in the democratic parliamentary system, which seemed to offer no solutions.

Key Fact

The Weimar Republic was established as a representative democracy which aimed to give genuine power to all German adults. However, it had major flaws that contributed to its downfall in 1933–34.

Key Terms  The Great Depression: A worldwide economic slump lasting from 1929 to 1935.  Reichstag: Name given to the German Parliament.  Proletarianisation: Process of becoming impoverished to the level of working classes.

Key Dates

   

1918: Proclamation of the Weimar Republic. 1919: Germany signed the Treaty of Versailles. 1929: Wall street exchange crashed. 1929: The start of Great Economic Depression in USA.

OBJECTIVE TYPE QUESTIONS Multiple Choice Questions Q. 1. What was the main reason for Germany’s defeat in the First World War? (A) It had weak military strength. (B) The allies were strengthened by the entry of US. (C) It was not prepared for a massive war. (D) It did not have support from Japan. U Ans. Option (B) is correct.

(1 mark each)

Q. 2. What was the name of the war veteran organisation through which the Weimar Republic crushed the uprising? R (A) The Free Corps (B) The Corps Free (C) The Corps (D) Revolutionaries Ans. Option (A) is correct. Explanation: The Free Corps was a war veterans organisation which helped the Weimar Republic to crush the uprising of the workers or socialists.

NAZISM AND THE RISE OF HITLER

Q. 3. Who supported the Weimar Republic? R (A) Socialist (B) Socialist and Democrats (C) Only Communists (D) Socialists, Catholics and Democrats Ans. Option (D) is correct. Explanation: The Socialists, Catholics and Democrats, supported the Weimar Republic and they were mockingly called the 'November Criminals'.

Assertion and Reason

In the questions given below, there are two statements marked as Assertion (A) and Reason (R). Read the statements and choose the correct option. Options:

(A) Both (A) and (R) are true and (R) is the correct explanation of (A).



(B) Both (A) and (R) are true and (R) is not the correct explanation of (A).



(C) (A) is correct but (R) is wrong.

(D) (A) is wrong but (R) is correct. Q. 1. Assertion (A): The Nuremberg Tribunal sentenced only eleven Nazis to death. Many others were U imprisoned for life.



47

Reason (R): The punishment of the Nazis was far short of the brutality and extent of their crimes.

Ans. Option (B) is correct. Explanation: The Allies did not want to be as harsh on defeated Germany as they had been after the First World War. Q. 2. Assertion (A): All joined the war enthusiastically hoping to gain from a quick victory. Reason (R): Little did they realise that the war would stretch on, eventually draining Europe of all its resources. Ans. Option (A) is correct. Explanation: Germany made initial gains by occupying France and Belgium. However, the Allies, strengthened by the US entry in 1917, won, defeating Germany and the Central Powers in November 1918. Q. 3. Assertion (A): The war had a devastating impact on the entire continent both psychologically and U financially. Reason (R): From a continent of creditors, Europe turned into one of the debtors. Ans. Option (A) is correct.

SUBJECTIVE TYPE QUESTIONS Very Short Answer Type Questions R Q. 1. Who were called the November Criminals? Ans. Those who supported the Weimar Republic, mainly Socialists, Catholics and Democrats, became easy targets of attack in the conservative nationalist circles. They were mockingly called the ‘November criminals’. (2)

Q. 2. Why did the Communists and Socialists become E irreconcilable enemies? Ans. The anguished Spartacists founded the Communist Party of Germany. Communists and Socialists henceforth became irreconcilable enemies and could not make common cause against Hitler.  (2) Q. 3. Political radicalisation was only heightened by the A economic crisis of 1923. Explain Ans. The political radicalisation was heightened by the economic crisis of 1923. As Germany refused to pay the war reparations, France occupied its leading industrial area, Ruhr. Germany retaliated with printing paper currency recklessly. With too

(2 marks each)

much printed money in circulation, the value of the German mark fell.(2) R Q. 4. What was the Dawes Plan? Ans. (i) Dawes Plan was a plan in 1924 that successfully resolved the issue of World War I reparations that Germany had to pay. (ii) It ended a crisis in European diplomacy following World War I and the Treaty of Versailles. The basic idea behind the plan was to make it easier for Germany to pay reparations.  (1 x 2 = 2)

Q. 5. What led to the start of Great Economic Depression E of 1929? Ans. The Wall Street Exchange crashed in 1929. Fearing a fall in prices, people made frantic efforts to sell their shares. On one single day, 24 October, 13 million shares were sold. This was the start of the Great Economic Depression.  (2)

48

Oswaal CBSE Question Bank Chapterwise & Topicwise, SOCIAL SCIENCE, Class-IX

Short Answer Type Questions Q. 1. Describe any three effects of the Great Depression (1929–1932) on Germany. U  [Board Term I, 2016] OR What was the impact of the ‘Great Economic Crisis’ on the economy of Germany? Explain.

(iv) It lost 75 per cent of its iron and 26 per cent of its coal to France, Poland, Denmark and Lithuania. (Any three) (1×3=3) [CBSE Marking Scheme, 2013]

Commonly Made Error

Ans. Great Economic Depression: (i) Industrial output fell by 40 per cent; workers lost their jobs and they were paid low wages. (ii) Sharp fall in agricultural prices (iii) Women were severely affected. 1×3=3  [CBSE Marking Scheme, 2016] Q. 2. Describe any three inherent defects in the Weimar Constitution that made it vulnerable to dictatorship. A [Board Term I, 2016] OR Describe any three factors which made the Weimar Republic politically fragile. [Board Term I, 2015] OR State three reasons responsible for the failure of the Weimar Republic.  [Board Term I, 2014] OR State any three weaknesses of the Weimar Republic. OR State any three factors which made the Weimar Republic politically fragile. OR Describe the problems faced by the Weimar Republic.  (NCERT) Ans. Defects: (i) The Weimar Constitution was based on proportional representation. (ii) Article 48 gave President the power to impose emergency, suspend civil rights. (iii) Thus, in a short time, 20 different cabinets were formed and people lost faith. (1×3=3) [CBSE Marking Scheme, 2016] Q. 3. Describe any three conditions of the Treaty of Versailles. R [Board Term I, 2016] OR Explain any three effects of the Treaty of Versailles over Germany. [Board Term I, 2014] OR

State any three major effects of the First World War on Germany.  [Board Term I, 2014]

Ans. Impact of the Treaty of Versailles over Germany: (i) Germany lost its overseas colonies. (ii) It was demilitarised. (iii) The Allied armies occupied the resource-rich Rhineland.

(3 marks each)



The students write about the reasons for the rise of Nazism in Germany.

Answering Tip

The students should mention about the

economic loss that Germany suffered due to the terms laid in the Treaty of Versailles.

Q. 4. Name the country that defeated Germany in the First World War. State two effects of defeat on Germany. R [DDE Term I, 2014] Ans. US joined the Allies in 1917 and defeated Germany and the Central Powers. Effects: (i) The defeat of Germany resulted in the abdication of the German Emperor. This gave an opportunity to the parliamentary parties to bring a change in German politics. (ii) A democratic Constitution with a federal structure was formed by the National Assembly, which met at Weimar and the Weimar Republic came into existence. (1+2=3) Q. 5. State the verdict of Nuremberg Tribunal. Why did the Allies avoid hard punishment on Germany? R [DDE Term I, 2014] Ans. The Nuremberg Tribunal, which was set up to prosecute the Nazis for committing grave crime against humanity which involved killing of innocent civilians in Europe, sentenced only 11 Nazis to death. Some were given life imprisonment. Allies avoided harsh punishment on Germany because: (i) They did not want to repeat the mistakes committed after the First World War where they imposed harsh terms on Germany by virtue of Treaty of Versailles, which resulted in the rise of Hitler. (ii) Treaty of Versailles was physiologically damaging for the Germans and also proved to be a national shame. The treaty had sown the seeds of the Second World War. This made the Allies cautious enough of not being very harsh on Germans again. (1+2=3)

NAZISM AND THE RISE OF HITLER

Commonly Made Error

The Nuremberg Tribunal is taken to be as a normal tribunal formed to bring criminals to punishment.

49

Answering Tip

Students must be aware of the fact that

Nuremberg Tribunal was formed to conduct trials for crimes committed in Germany during the Holocaust, before attempting the question.

Topic-2 Hitler’s Rise to Power Revision Notes  The crisis in the economy, polity and society formed the background to Hitler’s rise to power. Born in 1889 in Austria, Hitler spent his youth in poverty.  In 1919, he joined a small group called the ‘German Workers’ Party. He subsequently took over the organisation and renamed it as the ‘National Socialist German Workers’ Party. This party later came to be known as the ‘Nazi Party’.  It was during the Great Depression that Nazism became a mass movement.  By 1932, the Nazi Party had become the largest party with 37 per cent votes.  Hitler effectively mobilised popular support in Germany: (i) Hitler was a powerful speaker. His passion and his words moved people. (ii) He promised to build a strong nation. (iii) He also promised to undo the injustice of the Versailles Treaty and restore the dignity of the German people. (iv) He promised employment for those looking for work and a secure future for the youth. (v) He promised to weed out all foreign influences and resist all foreign conspiracies against Germany. (vi) He understood the significance of rituals and spectacle in mass mobilisation. Nazis held massive rallies and public meetings to demonstrate the support for Hitler and instil a sense of unity among the people. (vii) The red banners with the Swastika, the Nazi salute, and the ritualised rounds of applause after the speeches were all part of this spectacle of power. The Destruction of Democracy  On 30th January 1933, President Hindenburg offered the Chancellorship, the highest position in the Cabinet of Ministers, to Hitler.  The Fire Decree of 28th February 1933 indefinitely suspended civic rights like Freedom of Speech, Press and Assembly that had been guaranteed by the Wiemar Constitution.  On 3rd March 1933, the famous ‘Enabling Act’ was passed. This Act established dictatorship in Germany. It gave Hitler all powers to sideline the Parliament and rule by decree.

R econstruction  In foreign policy also, Hitler acquired quick successes. He pulled out of the League of Nations in 1933, reoccupied the Rhineland in 1936, and integrated Austria and Germany in 1938 under the slogan, ’One People, One Empire and One Leader.’  In September 1940, a ‘Tripartite Pact’ was signed between Germany, Italy and Japan, strengthening Hitler’s claim to international power.  By the end of 1940, Hitler was at the pinnacle of his power. He attacked the Soviet Union in June 1941.  Meanwhile, the USA had resisted involvement in the war. It was unwilling to once again face all the economic problems that the First World War had caused.  When Japan extended its support to Hitler and bombed the US base at Pearl Harbour, the US entered the Second World War.  The war ended in May 1945 with Hitler’s defeat and the US dropping of the atom bomb on Hiroshima in Japan.

50

Oswaal CBSE Question Bank Chapterwise & Topicwise, SOCIAL SCIENCE, Class-IX

Key Terms  Gestapo: The Secret State Police in Nazi Germany.  Holocaust: The persecution and mass murder of Jews by German Nazis between 1933 and 1945.  Concentration camp: A camp where people were isolated and detained without due process of law. Typically, it was surrounded by electrified barbed wire fences.

Key Personalities  Hindenburg: He was elected as the President of Germany in 1925  Hitler: Hitler rose to power as the leader of the Nazi Party, becoming the chancellor in 1933. He was the leader of Germany's Nazi Party.

Key Dates

        

1933: Hitler was made Chancellor of Germany. He pulled out of the League of Nations. 1934: Hitler became the President of Germany. 1936: Hitler reoccupied Rhineland. 1938: Integration of Germany and Austria by Hitler. 1939: Germany invaded Poland. 1940: A Tripartite Pact was signed between Germany, Italy and Japan. 1940–1944: Ghettoisation of Jews. June, 1941: Germany attacked the USSR. 1945: Hitler committed suicide by gunshot in Berlin.

Example How was the Nazi Party formed? Answer: Step 1: The disintegration of Weimar Republic led to the formation of Nazi Party after the First World War. Step 2: Hitler enrolled for the army when the First World War broke. He also earned medals for bravery.



Step 3: The German defeat horrified him and Versailles Treaty made him furious. Later, he joined a small group called the ‘German Workers' Party.



Step 4: Subsequently, he took over the organisation and renamed it the ‘National Socialist German Workers' Party’. This party came to be known as 'Nazi Party'.

OBJECTIVE TYPE QUESTIONS Multiple Choice Questions Q. 1. Match the column and choose the correct option: R  Column-I

Column-II

(a)

1889

(i)

Hitler was born

(b)

1919

(ii)

The Nazi Party was formed

(c)

28 February 1933

(iii)

The First Decree

(d)

3 March 1933

(iv)

The Enabling Act was passed

(1 mark each)

Options: (A) a-i, b-ii, c-iii, d-iv (B) a-ii, b-i, c-iv, d-iii (C) a-iv, b-i, c-iii, d-ii (D) a-i, b-iv, c-ii, d-iii Ans. Option (A) is correct. Q. 2. In September________ Germany invaded Poland. R  (A) 1938 (B) 1939 (C) 1940 (D) 1941 Ans. Option (B) is correct. Explanation: In September 1939, Germany invaded Poland. This started a war with France and England.

NAZISM AND THE RISE OF HITLER

Q. 3. Look at the picture and identify the correct option:

51

Assertion and Reason

E



In the questions given below, there are two statements marked as Assertion (A) and Reason (R). Read the statements and choose the correct option.



(A) Both (A) and (R) are true and (R) is the correct explanation of (A).



(B) Both (A) and (R) are true but (R) is not the correct explanation of (A).



(C) (A) is correct but (R) is wrong.

(D) (A) is wrong but (R) is correct. Q. 1. Assertion (A): Hitler was empowered to sideline U Parliament and rule by decree. Reason (R): The Enabling Act of 1935 established democracy in Germany. Ans. Option (C) is correct. Explanation: On 3 March 1933, the famous Enabling Act was passed. This Act established dictatorship in Germany. It gave Hitler all powers to sideline Parliament and rule by decree.

The poster announces: Options: (A) Your Volkswagen (B) Freedom is Ours (C) Free and Young Germany (D) None of the above Ans. Option (A) is correct Explanation: Such posters suggested that owning a car was no longer just a dream for an ordinary worker.

Q. 2. Assertion (A): Hitler attacked the Soviet Union in E June 1941. Reason (R): Hitler now moved to achieve his longterm aim of conquering Eastern Europe. Ans. Option (A) is correct. Q. 3. Assertion (A): The US entered the Second World R War. Reason (R): When Japan extended its support to Hitler and bombed the US base at Pearl Harbor. Ans. Option (A) is correct.

SUBJECTIVE TYPE QUESTIONS Very Short Answer Type Questions R Q. 1. Who was Hjalmar Schacht? Ans. Hjalmar Schacht was an economist whom Hitler had assigned the responsibility of economic recovery. Hjalmar Schacht aimed at full production and full employment through a state-funded workcreation programme. This project produced the famous German superhighways and the people’s car, the Volkswagen. (2) Q. 2. By the end of 1940, Hitler was at the pinnacle of his E power. Ans. By the end of 1940, Hitler was at the pinnacle of his power: (i) Resources were to be accumulated through expansion of territory. In September 1939, Germany invaded Poland.

(2 marks each)

(ii) In September 1940, a Tripartite Pact was signed between Germany, Italy and Japan, strengthening Hitler’s claim to international power. Puppet regimes, supportive of Nazi Germany, were installed in a large part of Europe. (1 x 2=2) Q. 3. What do you know about the Fire Decree of 28 A February 1933? Ans. The Fire Decree of 28 February 1933 indefinitely suspended civic rights like freedom of speech, press and assembly that had been guaranteed by the Weimar constitution. It also permitted the regime to arrest and incarcerate political opponents without specific charge and dissolve political organisations. (2)

52

Oswaal CBSE Question Bank Chapterwise & Topicwise, SOCIAL SCIENCE, Class-IX

Short Answer Type Questions Q. 1. Explain the circumstances under which Nazism became popular in Germany. U, A [Board Term I, 2016, 2015] OR What is Nazism? How did it become popular in Germany? Explain.  [Board Term I, 2014] OR Discuss why did Nazism become popular in Germany by 1930. [Board Term I, 2013] [NCERT] Ans. Circumstances under which Nazism became popular were: (i) Hitler was a great orator. His passion and words moved people. (ii) He promised to build a strong nation. (iii) He promised to restore dignity of the Germans and undo the injustice of the Treaty of Versailles. (iv) He promised employment for those looking for work and a secure future for the youth. (v) He promised to weed out all foreign influences and resist all foreign conspiracies against Germany. (vi) Hitler devised a new style of politics. Nazis held massive rallies and public meetings as to show the strength and confidence for Hitler. (vii) The red banners with the Swastika, ritualised rounds of applause and the Nazi salute after the speeches, were all part of this spectacle of power. (viii) Nazi propaganda projected Hitler as a Messiah or a Saviour. (Any three) (1×3=3) [CBSE Marking Scheme, 2016]

(iii) The Soviet Red Army hounded out the retreating German Soldiers until they reached Berlin, establishing Soviet hegemony over the entire Eastern Europe. (1×3=3)  [CBSE Marking Scheme, 2016] Q. 3. How did the Nazis demonstrate support for Hitler ? A [Board Term I, 2016] Ans. (i) The Nazis held massive rallies and public meetings to demonstrate the support for Hitler and instil the sense of unity among the people. (ii) The red banners with the Swastika, the Nazi salute and the ritualised rounds of applause after the speeches were all part of this spectacle of power. (iii) Nazi propaganda skilfully projected Hitler as a Messiah, a Saviour, as someone who had arrived to deliver people from their distress.  [CBSE Marking Scheme, 2016] (1×3=3) Q. 4. Explain three reasons that led to the German invasion of Soviet Union. R [Board Term I, 2015] Ans. Reasons leading to the German invasion of Soviet Union were: (i) Hitler dreamt of settling the German with pure Aryan blood in the Asian steppe region and of using the Jews and Asians as Slaves. (ii) He wanted to ensure food supplies for the Germans. The Germans and Russians had serious differences over the Balkan, where no clear line demarcation had been worked out. (iii) Hitler wanted to eradicate communism. (1×3=3)

Commonly Made Error

The reasons for popularity of Nazism is restricted only to Hitler's participation in it.

(3 marks each)

[CBSE Marking Scheme, 2015]

Commonly Made Error

Students mention that the Nazism became

popular with the German invasion of Soviet Union.

Answering Tip

Answering Tip

Students should know that the Great

Depression that hit the country was apparently the biggest reason for Nazism's popularity.

Q. 2. What was the impact of Hitler’s attack on Soviet Union in June 1941 ? R [Board Term I, 2016] Ans. (i) In this historic blunder, Hitler exposed the German Western Front to British aerial bombing and the Eastern Front to the powerful Soviet Armies. (ii) The Soviet Red Army inflicted a crushing and humbling defeat on Germany at Stalingrad.



The students need to elaborate Hitler's philosophy about Aryan race.

Q. 5. In what ways did the Nazi state seek to establish total control over its people?

E

Ans. The Nazi state tried to establish total control over its people in the following way: (i) Special surveillance and security forces were created to control the society, in the way Nazis wanted.

NAZISM AND THE RISE OF HITLER

(ii) Apart from the already existing regular police in green uniform and the SA or the Storm Troopers, these included the Gestapo, the SS (the protection squads), criminal police and the Security Service.

(iii) It was the extra-constitutional powers of these newly organised forces that gave the Nazi state its reputation as the most dreaded criminal state. 

Long Answer Type Questions Q. 1. State any five steps taken by Hitler to establish his dictatorial rule in Germany. U  [Board Term I, 2016] OR Explain any five features of political policy adapted by Hitler after coming to power in 1933.  [Board Term I, 2015] Ans. Features of political policy adapted by Hitler: (i) Suspended civil rights and introduced the Enabling Act. (ii) Communists sent to concentration camps. (iii) All political parties and trade unions except the Nazi Party were banned in Germany. (iv) State control over economy, media, army and judiciary. (v) Creation of special surveillance and security forces. (1 × 5 = 5) [CBSE Marking Scheme, 2016 & 2015] Q. 2. Mention five important consequences of Nazism in Germany.  A  [Board Term I, DDE-2016, 2014] Ans. Consequences of Nazism: (i) It pulled the country out of economic crisis. (ii) Hitler tried to make Germany a powerful country, enhanced its military. (iii) All political parties except Nazi party were banned. (iv) The Socialists, the Communists and the Jews were given harsh treatment as they were blamed of German defeat in World War I. (v) Trade unions were banned. (vi) A massive programme of militarism was launched and preparation for war began in a big way. (Any Five) (1×5=5) [CBSE Marking Scheme, 2016] Q. 3. Explain briefly the main causes of the rise of Nazi dictatorship in Germany. A [Board Term I, 2013] Ans. The main causes of rise of Nazi Dictatorship in Germany were as follows: (i) The First World War defeat.

53

(1×3=3)

(5 marks each)

(ii) Weakness of Weimar Republic: The Weimar Constitution had some inherent defects which made it unstable and vulnerable to dictatorship. Due to proportional representation, One Party Rule became an impossible task, leading to a rule by coalitions. Another defect was Article 48, which gave the President the Power to impose emergency, suspend civil rights and rule by decree. (iii) Treaty of Versailles: Germany was forced to sign the Treaty of Versailles with the Allied Powers.

This made Germany to lose its overseas colonies, a tenth of its population, 13 per cent of its territories and many more. The Allied Powers demilitarised Germany. The Allied armies also occupied the resource rich Rhineland.



The War Guilt Clause forced Germany to pay £ 6 billion as War Damages to the Allied Powers.

(iv) The Economic crisis: German economy was worst hit. Industrial production was reduced to 40 per cent. Due to Great Economic Depression, the national income of the USA fell by half, leading to shut down of the factories. The exports fell down and farmers suffered the most during the depression. (v) Mass unemployment: Workers of Germany lost their jobs and were paid reduced wages. Unemployed youth played cards or lined up at local employment exchange. Jobs disappeared and youth took to criminal activities. (vi) German people thought that Hitler would be a good dictator as he promised to undo the injustice of Treaty of Versailles. He also promised to give employment to those looking for work. Later, the Nazi Party became the largest Party with 37 per cent votes in Reichstag. (Any five) (1×5=5)  (CBSE Marking Scheme 2013)

54

Oswaal CBSE Question Bank Chapterwise & Topicwise, SOCIAL SCIENCE, Class-IX

Topic-3 The Nazi Worldview Revision Notes  Nazi ideology was synonymous with Hitler’s world view.  In this view blond, blue-eyed, Nordic German Aryans were at the top, while Jews were located at the lowest rank. They came to be regarded as an anti-race, the arch-enemies of the Aryans.  All other coloured people were placed in between, depending upon their external features.  The other aspect of Hitler’s ideology related to the geopolitical concept of Lebensraum, or living space. He believed that new territories had to be acquired for settlement. This would enhance the area of the mother country, while enabling the settlers on new lands to retain an intimate link with the place of their origin.

Scan to know more about this topic

The main Principles of Nazi Ideology

stablishment of the Racial State E  Nazis wanted only a society of ‘pure and healthy Nordic Aryans’. Only they were seen as worthy of prospering and multiplying against all others who were classed as ‘undesirable’.  Jews were not the only community classified as ‘undesirable’, many Gypsies and Blacks living in Nazi Germany were considered as racial ‘inferiors’ who threatened the biological purity of the superior Aryan race.  Jews remained the worst sufferers in Nazi Germany. They had been stereotyped as killers of Christ and usurers. They lived in separately marked areas called ‘Ghettos’.  From 1933 to 1938, the Nazis terrorised, pauperised and segregated the Jews, compelling them to leave the country.  The next phase, 1939–1945, aimed at concentrating them in certain areas and eventually killing them in gas chambers in Poland.

The Racial Utopia  Genocide and war became two sides of the same coin. Poland was divided and much of North-Western Poland was annexed to Germany.  People of Poland were forced to leave their homes and properties.  Members of the Polish intelligentsia were murdered in large numbers. Polish children who looked like Aryans were forcibly snatched from their mothers and examined by ‘race experts’.

Key Personalities  Charles Darwin: Darwin was a natural scientist who tried to explain the creation of plants and animals through the concept of evolution and natural selection.  Herbert Spencer: He was an English philosopher who initiated a philosophy called 'Social Darwinism'.

Key Terms

 Genocidal: Killing on a large-scale leading to destruction of large sections of people.  Jungvolk: A separate section for Nazi boys up to 14 years of age.  Pauperised: Reduce to absolute poverty.  Usurers: Moneylenders charging excessive interest; often used as a term of abuse.

OBJECTIVE TYPE QUESTIONS Multiple Choice Questions Q. 1. _________ was a natural scientist who tried to explain the creation of plants and animals through

(1 mark each)

the concept of evolution and natural selection. R (A) Hitler (B) Darwin (C) Herbert Spencer (D) None of these Ans. Option (B) is correct.

NAZISM AND THE RISE OF HITLER

Q. 2. The night of November________ is remembered R as ‘the night of broken glass’. (A) 1938 (B) 1935 (C) 1939 (D) 1941 Ans. Option (A) is correct. Q. 3. Some of the largest ghettos and gas chambers, the General Government also served as the killing R fields for the_______. (A) Jews (B) Polish (C) Germans (D) All of the Above Ans. Option (A) is correct.

Assertion and Reason

In the questions given below, there are two statements marked as Assertion (A) and Reason (R). Read the statements and choose the correct option.



(A) Both (A) and (R) are true and (R) is the correct explanation of (A).



(B) Both (A) and (R) are true but (R) is not the correct explanation of (A).



55

(C) (A) is correct but (R) is wrong.

(D) (A) is wrong but (R) is correct. Q. 1. Assertion (A): Jews remained the worst sufferers in E Nazi Germany. Reason (R): Until medieval times Jews survived mainly through trade and moneylending. Ans. Option (B) is correct.

Explanation: Nazi hatred of Jews had a precursor in the traditional Christian hostility towards Jews. They had been stereotyped as killers of Christ and usurers. Q. 2. Assertion (A): Nordic German Aryans were at the top, while Jews were located at the lowest rung. Reason (R): Nazi ideology was synonymous with A Hitler’s worldview. Ans. Option (A) is correct. Q. 3. Assertion (A): Poland became the laboratory for R experimentation. Reason (R): Hitler intended to extend German boundaries by moving eastwards, to concentrate all Germans geographically in one place. Ans. Option (A) is correct.

SUBJECTIVE TYPE QUESTIONS Very Short Answer Type Questions Q. 1. What was Nazi’s argument against the pure race? R  Ans. The Nazi argument was simple: the strongest race would survive and the weak ones would perish. The Aryan race was the finest. It had to retain its purity, become stronger and dominate the world. (2) R Q. 2. Who were Nordic German Aryans? Ans. Nordic German Aryans lived in north European countries and had German or related origin. They alone were considered 'desirable'. Only they were seen as worthy of prospering and multiplying against all others who were classed as 'undesirable'. This meant that even those Germans who were seen as impure or abnormal had no right to exist. (2)

Q. 3. How did the Jews suffer from 1933 to 1945 in the E hands of Nazi? Ans. From 1933 to 1938 the Nazis terrorised, pauperised and segregated the Jews, compelling them to leave the country. The next phase, 1939–1945, aimed at concentrating them in certain areas and eventually killing them in gas chambers in Poland. (2)

(2 marks each)

R Q. 4. What did the Nuremberg Laws state? Ans. (i) Only persons of German or related blood would henceforth be German citizens enjoying the protection of the German empire. (ii) Marriages between Jews and Germans were forbidden. (iii) Extramarital relations between Jews and Germans became a crime. (iv) Jews were forbidden to fly the national flag.  (Any Two Points) (1 x 2 = 2)

Q. 5. What do you know about the Ghettoisation of U 1940–1944?  Ans. (i) From September 1941, all Jews had to wear a yellow Star of David on their breasts. (ii) This identity mark was stamped on their passport, all legal documents and houses. They were kept in Jewish houses in Germany and in ghettos like Lodz and Warsaw in the east. (iii) Jews had to surrender all their wealth before they entered a ghetto. Soon the ghettos were brimming with hunger, starvation and disease due to deprivation and poor hygiene.  (Any Two Points) (1 x 2 = 2)

56

Oswaal CBSE Question Bank Chapterwise & Topicwise, SOCIAL SCIENCE, Class-IX

Short Answer Type Questions Q. 1. Explain Hitler’s ideology related to the geopolitical concept of Lebensraum.

R

[Board Term I, 2015] Ans. Hitler’s geopolitical concept of Lebensraum: (i) He believed that new territories had to be acquired for settlement. (ii) This would enhance the area of the mother country, while enabling the settlers on new land to retain an intimate link with the place of their origin. (iii) It would also enhance the material resources and power of the German nation. (1×3=3) [CBSE Marking Scheme, 2015]

Commonly Made Error

Lebensraum was taken as a new policy given by Nazism by many students.

Answering Tip

Students should know that Lebensraum was

the basic principle of Nazi foreign policy and is a geopolitical concept.

Q. 2. Describe the ideology of racial hierarchy propounded by Hitler. U [Board Term I, 2015]   Ans. Ideology of racial hierarchy propounded by Hitler: ref to Long Ans 3. Q. 3. Explain briefly the theory of Herbert Spencer used by Hitler. U [Board Term I, 2013]

(3 marks each)

(ii) According to this idea, only those species survived on Earth which could adapt them to changing climate conditions. (iii) Used by racist thinkers and politicians to justify imperial rule over conquered people. According to Nazi argument, the strongest race would survive and the weak ones would perish. (1×3=3) [CBSE Marking Scheme, 2013] Q. 4. How were the ideas of Darwin and Herbert Spencer adopted by Hitler or Nazis ? Explain. A Ans. (i) Hitler’s racism was heavily borrowed from thinkers like Charles Darwin and Herbert Spencer. (ii) The ideas written by Darwin in ‘The Origin of Species’ by means of natural selection was used by Hitler to justify his imperial rule over conquered people. (iii) Ref to Q3 point (ii) and (iii) (1×3= 3) Q. 5. Jews were not the only community classified as E ‘undesirable’. Explain. Ans. Jews were not the only community classified as ‘undesirable’: (i) Many Gypsies and blacks living in Nazi Germany were considered as racial ‘inferiors’ who threatened the biological purity of the ‘superior Aryan’ race. They were widely persecuted. (ii) Even Russians and Poles were considered subhuman and hence undeserving of any humanity. (iii) When Germany occupied Poland and parts of Russia, captured civilians were forced to work as slave labour. Many of them died simply through hard work and starvation.  (1×3= 3)

Ans. Theory: (i) Herbert Spencer propounded the idea of 'survival of the fittest'.

Long Answer Type Questions Q. 1. Highlight any five peculiar features of Nazi R [Board Term I, 2014] thinking. OR What are the peculiar features of Nazi thinking ? [NCERT] Ans. The peculiar features of Nazi thinking are: (i) There was no equality between people, but only a racial hierarchy. In this view, blond, blue-eyed, Nordic German Aryans were at the top, while the Jews were located at the lowest level. All other coloured people were placed in between, depending upon their external features.

(5 marks each)

(ii) The Aryan Race was the finest. It had to retain its purity, become stronger and dominate the world. (iii) Nazis glorified war. Their only aim was to unite all people of the Aryan race under one state that is Germany. (iv) Hitler believed that new territories had to be acquired for settlement. It would enhance the material resources and power of the German nation. (v) Nazis wanted only a society of pure and healthy Nordic Aryans. They alone were considered ‘desirables’.

NAZISM AND THE RISE OF HITLER

(vi) Nazis hated Jews. They terrorised, pauperised and segregated them and compelled them to leave the country. (vii) The Nazi argument was simple; the strongest race would survive and the weak ones would perish. (viii) The Aryan race was the finest. It had to retain its purity, become stronger and dominate the world.  (Any five) (1×5=5) Q. 2. How did the Nazis proceed to realise their murderous racial ideology by eliminating the undesirables? Explain. U [DDE Term I, 2014] Ans. (i) Once in power, the Nazis quickly began to implement their dream of creating an exclusive racial community of pure Germans by physically eliminating all those who were seen as ‘undesirable’ in the extended Empire as they were mentally or physically unfit Germans, Gypsies, Blacks, Russians and Poles.

57

(ii) But, Jews remained the worst sufferers in Nazi Germany. They were stereotyped as ‘killers of Christ and usurers’. (iii) Until medieval times, Jews were barred from owning land. They survived mainly through trade and money lending. They lived in separately marked areas called ‘Ghettos’. (iv) They were often persecuted through periodically organised violence and expulsion from land. All this had a precursor in the traditional Christian hostility towards Jews for being the killers of Christ. (v) However, Hitler’s hatred of the Jews was based on pseudo-scientific theories of race, which held that conversion was no solution to ‘the Jewish problem’. It could be solved only through their total elimination. (1 × 5 = 5)

Youth, Ordinary People and the Crimes Against

Topic-4 Humanity Revision Notes

 Hitler was fanatically interested in the youth of the country. He felt that a strong Nazi society could be established only by teaching children Nazi ideology. This required a control over the child both inside and outside school.  Effects of Nazism on the School System: Scan to know more about (i) All schools were ‘cleansed’ and ‘purified’. This meant that teachers who were Jews or this topic seen as politically unreliable were dismissed. (ii) Children were first segregated – Germans and Jews could not sit together or play together. (iii) Subsequently, undesirable children – Jews, the physically handicapped, Gypsies were Life in Nazi thrown out of schools. Germany (iv) ‘Good German’ children were subjected to a process of Nazi schooling, a prolonged Scan to know period of ideological training. more about this topic (v) School textbooks were rewritten. Racial science was introduced to justify Nazi ideas of race. (vi) Children were taught to be loyal and submissive, hate Jews and worship Hitler. (vii) Even the function of sports was to nurture a spirit of violence and aggression among children. Hitler believed that boxing could make children iron-hearted, strong and Nazi Propaganda masculine.  Youth organisations were made responsible for educating German youth in the ‘The Spirit of National Socialism’. Ten year old German kids had to enter Jungvolk.  At 14, all boys had to join the Nazi Youth Organisation – Hitler Youth – where they learnt to worship war, glorify aggression and violence, condemn democracy and hate Jews, communists, Gypsies and all those categorised as ‘undesirable’.  At the age of 18, the youth had to serve in the Armed Forces and enter one of the Nazi organisations. The Youth League of the Nazis was founded in 1922.

The Nazi Cult of Motherhood  Children in Nazi Germany were repeatedly told that women were radically different from men.  Boys were taught to be aggressive, masculine and steel-hearted and girls were told to become good mothers and rear pure-blooded Aryan children.  Girls had to maintain purity of the race, distance from Jews, look after their home and teach their children Nazi values. But all mothers were not treated equally.

58

Oswaal CBSE Question Bank Chapterwise & Topicwise, SOCIAL SCIENCE, Class-IX

 Honour Crosses were awarded to encouraged women to produce more children.

The Art of Propaganda  Nazis termed mass killings as special treatment, final solution (for the Jews), euthanasia (for the Disabled), selection and disinfections.  ‘Evacuation’ meant deporting people to gas chambers. Gas chambers were labelled as ‘disinfection-areas’, and looked like bathrooms equipped with fake shower heads.  Nazi ideas were spread through visual images, films, radio, posters, catchy slogans and leaflets.  Orthodox Jews were stereotyped and marked and were referred to as vermin, rats and pests.  The Nazis made equal efforts to appeal to all the different sections of the population.

Nazi's Propaganda  They sought to win their support by suggesting that Nazis alone could solve all their problems.  Many saw the world through Nazi eyes and spoke their mind in Nazi language. They felt hatred and anger surge inside them when they saw someone who looked like a Jew. But not every German was a Nazi.

Knowledge about the Holocaust  The Nazi killing operation was also called the Holocaust. It comes from the Greek word ‘Holo’ and ‘Kaustos’, which literally means completely burnt. It is used to describe the mass murder of Jews by German Nazis between 1933 and 1945.

Key Terms  Jungvolk: A separate section for Nazi boys upto 14 years of age.  Holocaust: Mass murder of Jews by German Nazis.

Key Date  1933 to 1945: The mass murder of Jews by German Nazis.

OBJECTIVE TYPE QUESTIONS Multiple Choice Questions Q. 1. Name the film which was made to create hatred R for Jews.  (A) The Eternal Jew (B) The External Jew (C) The Jews (D) Mein Kampf Ans. Option (A) is correct. Explanation: The most infamous film ‘The Eternal Jew’ was an antisemitic Nazi propaganda film, presented as a documentary. Q. 2. What was the name given to gas chambers by R Nazis? (A) Environment friendly (B) Pollutant (C) Ghetto (D) Disinfection areas Ans. Option (D) is correct. Explanation: The gas chambers were labelled as ‘disinfection-areas’, and looked like bathrooms equipped with fake shower heads.

(1 mark each)

Q. 3. Look at the picture and identify it: 

(A) A Nazi poster attacking Jews (B) The German professor refusing to teach Jews

U

NAZISM AND THE RISE OF HITLER

(C) German banker sitting on a pile of money (D) None of the above Ans. Option (A) is correct. Explanation: Caption above reads: ‘Money is the God of Jews. In order to earn money he commits the greatest crimes. He does not rest until he can sit on a big sack of money until he has become the king of money.

Assertion and Reason

In the questions given below, there are two statements marked as Assertion (A) and Reason (R). Read the statements and choose the correct option.



(A) Both (A) and (R) are true and (R) is the correct explanation of (A).



(B) Both (A) and (R) are true but (R) is not the correct explanation of (A).



(C) (A) is correct but (R) is wrong.



(D) (A) is wrong but (R) is correct.

59

Q. 1. Assertion (A): Hitler was fanatically interested in U the youth of the country. Reason (R): He felt that a strong Nazi society could be established only by teaching children Nazi Ideology. Ans. Option (A) is correct. Q. 2. Assertion (A): Nazi ideas were spread only through E newspapers. Reason (R): The Nazis made equal efforts to appeal to all the different sections of the population. Ans. Option (D) is correct. Explanation: Nazi ideas were spread through visual images, films, radio, posters, catchy slogans and leaflets. Q. 3. Assertion (A): Not every German was a Nazi. A Reason (R): Many organised active resistance to Nazism, braving police repression and death. Ans. Option (A) is correct. Explanation: The large majority of Germans, however, were passive onlookers and apathetic witnesses.

SUBJECTIVE TYPE QUESTIONS Very Short Answer Type Questions Q. 1. What was the function of sports in schools under E Nazism? Ans. (i) The function of sports was to nurture a spirit of violence and aggression among children. (ii) Hitler believed that boxing could make children iron hearted, strong and masculine. (1 x 2 = 2) Q. 2. What was the purpose of Jungvolk in Nazi U Germany? Ans. All the boys aged over ten joined the youth group called Jungvolk. Here they learnt to worship war, glorify aggression and violence, condemn democracy and hate Jews, communists, Gypsies and all those categorised as ‘undesirable. This was largely done to inculcate the spirit of the Nazi idealogy very early on. (2)

Q. 3. Hitler said: ‘In my state the mother is the most important citizen.’ How true was this statement? 

U

Ans. In 1933, Hitler said: ‘In my state the mother is the most important citizen.’

(i) But in Nazi Germany all mothers were not treated equally. (ii) Women who bore racially undesirable children were punished and those who produced racially desirable children were awarded. (1 x 2 = 2) Q. 4. How were the Aryan’ women who deviated from the prescribed code of conduct treated in Nazi E Germany? Ans. All ‘Aryan’ women who deviated from the prescribed code of conduct were: (i) Publicly condemned, and severely punished. (ii) Those who maintained contact with Jews, Poles and Russians were paraded through the town with shaved heads, blackened faces and placards hanging around their necks announcing ‘I have sullied the honour of the nation’. (iii) Many received jail sentences and lost civic honour as well as their husbands and families for this ‘criminal offence’. (Any Two Points) (1 x 2 = 2)

Short Answer Type Questions Q. 1. Describe the role of youth organisations in Nazi Germany. U [Board Term I, 2016] OR

(2 marks each)

(3 marks each)

Mention three youth organisations that were responsible for educating German youth in the spirit of National Socialism. [Board Term I, 2015] OR

60

Oswaal CBSE Question Bank Chapterwise & Topicwise, SOCIAL SCIENCE, Class-IX

Describe the Germany.

youth

organisations in Nazi [Board Term I, 2013]

OR Explain any three methods adopted by Hitler for educating German youth in the spirit of National Socialism.



Ans. Methods: (i) Youth organisations were made responsible for educating German young in the spirit of National Socialism. Ten years old had to enter ‘Jungvolk’ the Nazi youth group. (ii) At 14, all the boys had to join Nazi youth organisation called ‘Hitler Youth’ where they learnt to worship war, glorify aggression and violence, condemn democracy and hate Jews, communists and gypsies. (iii) After a period of rigorous ideological and physical training, they joined the ‘Labour Service’, and later, they joined the armed forces and entered one of the youth organisations. (1 × 3 = 3) [CBSE Marking Scheme, 2016] Q. 2. Explain any three effects of Nazism on the school system.  [Board Term I, 2015] Ans. Effects of Nazism on the School System: (i) Hitler was fanatically interested in the youth of the country. He believed in teaching the Nazi ideology to the students. (ii) Jewish teachers who were seen as politically unreliable were dismissed from service. (iii) Children were segregated – German and Jews could not sit or play together. (iv) Good German children were subjected to a process of Nazi schooling, a prolonged period of ideological training.

Q. 3. Describe the role of propaganda films in creating hatred for Jews. R [Board Term I, 2013] Ans. (i) Most infamous film was the ‘Eternal Jew’ in which orthodox Jews were stereotyped and marked. (ii) They were shown with flowing beards wearing kaftans. (iii) They were referred to as vermin, rats and pests. Their movements were compared to those of rodents. [CBSE Marking Scheme, 2015] (1 × 3 = 3) Q. 4. Why was Nazi propaganda effective in creating a hatred for the Jews? Explain any five reasons. R [NCERT] Ans. Reasons: (i) The Nazi regime used language and media with care. (ii) They used chilling terms. They never used the words ‘kill’ or ‘murder’. (iii) Mass killing was termed as special treatment leading towards the final solution (for the Jews). (iv) Nazi ideas were spread through visual images, films, radio, posters, etc. (v) Propaganda films were made to create hatred for Jews. (vi) They were referred to as vermin, rats, pests. Nazism worked on minds of the people. (Any three) (1×3=3)

Commonly Made Error

(v) School textbooks were rewritten to glorify and justify the Nazi Ideas of racial superiority. (vi) Children were taught to be loyal, submissive and hate the Jews. (vii) Sports were introduced to nurture violence and aggression among students. Hitler believed that ‘Boxing could make children iron hearted, strong and masculine. (Any three points) [CBSE Marking Scheme, 2015] (1 × 3 = 3)

Commonly Made Error

The students mention about the general upbringing of the children.

Answering Tip

The education system has to be talked about keeping in mind Hitler's doctrine of the spread of Nazism.

The students only explain about mass killing of the Jews.

Answering Tip

Students should mention the points related to the spread of Nazism.

Q. 5. Many saw the world through Nazi eyes and spoke their mind in Nazi language. R

Ans. Many saw the world through Nazi eyes and spoke their mind in Nazi language: (i) They felt hatred and anger surge inside them when they saw someone who looked like a Jew. (ii) They marked the houses of Jews and reported suspicious neighbours. (iii) They genuinely believed Nazism would bring prosperity and improve general well-being. (1 × 3 = 3)

NAZISM AND THE RISE OF HITLER

Long Answer Type Questions Q. 1. Highlight five main features of Nazi schooling to R establish a strong Nazi Society.  [Board Term I, 2016] OR Highlight five important features of education imparted in Nazi schools. [Board Term I, 2015] Ans. Important features of education imparted in Nazi schools: (i) Children were segregated. The Germans and the Jews could not sit or play together. Subsequently, undesirable children like Jews and the physically handicapped; the Gypsies were thrown out of schools. (ii) All schools were cleansed or purified. This meant that teachers who were Jews or seen as politically unreliable were dismissed. (iii) Good German children were subjected to a process of Nazi schooling, a prolonged period of ideological training. (iv) Boxing was introduced as Hitler believed that it could make children iron-hearted, strong and masculine. (v) School text books were rewritten. (vi) Children were taught to be loyal and to hate Jews and worship Hitler. (Any five) (1×5=5) [CBSE Marking Scheme, 2016] Q. 2. Explain the status of women in the German Society under Nazism. A [Board Term I, 2016, 2012] OR How did the assigned role for women by Nazis help in the establishment of the racial state? How were women perceived in Nazi R [Board Term I, 2014] Germany?  OR Explain Hitler’s ideology regarding women and his policy towards them. OR Explain what role women had in Nazi society. [NCERT]

(5 marks each)

Ans. Status of women in Germany: (i) Children in Nazi Germany were told that women were radically different from men. (ii) While boys were taught to be aggressive, masculine and steel-hearted, girls were told to be good mothers and rear pure-blooded Aryan children. They had to teach Nazi values to their children. (iii) Women bearing undesirable children were punished and those bearing desirable were awarded. (iv) Girls had to maintain the purity of race. (v) They had to keep distance from the Jews, look after their home and teach Nazi values to their children. (vi) To encourage women to produce more children, a bronze cross was given for four, silver for six and gold for eight or more children. (vii) Those who maintained contacts with the Jews, Poles or Russians were severely punished. (Any five) [CBSE Marking Scheme, 2016] (1×5=5) Q. 3. Mention three ways in which the world gained R knowledge about Holocaust.

[Board Term I, 2014] OR



What event in history is known as the Holocaust? How did the world come to know about the Holocaust ? Explain.

Ans. The persecution and mass murder of Jews by German Nazis between 1933 and 1945 is known as the Holocaust. Sources: (i) Information and full horror of the Holocaust was revealed after Hitler’s death. (ii) From diaries, notebooks, left behind by many Ghetto inhabitants. (iii) Many of the Jews survived to tell the stories. (iv) Memoirs, fictions, documentaries and poetry in many parts of the world also revealed information about Holocaust. [CBSE Marking Scheme, 2014] (1+4=5)

COMPETENCY BASED QUESTIONS Case based MCQs I.

Read the source given below and answer the following questions:

A + U (1+1+1+1=4)

61

(4 marks each)

In May 1945, Germany surrendered to the Allies. Anticipating what was coming, Hitler, his propaganda minister Goebbels and his entire family committed suicide collectively in his Berlin bunker in April. At the end of the war, an

62

Oswaal CBSE Question Bank Chapterwise & Topicwise, SOCIAL SCIENCE, Class-IX

International Military Tribunal at Nuremberg was set up to prosecute Nazi War Criminals for Crimes against Peace, for War Crimes and Crimes Against Humanity. Germany's conduct during the war, especially those actions which came to be called Crimes Against Humanity, raised serious moral and ethical questions and invited worldwide condemnation. What were these acts? Under the shadow of the Second World War, Germany had waged a Genocidal war, which resulted in the mass murder of selected groups of innocent civilians of Europe. The number of people killed included 6 million Jews, 2,00,000 Gypsies, 1 million Polish Civilians, 70,000 Germans who were considered mentally and physically disabled, besides innumerable political opponents. Nazis devised an unprecedented means of killing people, that is, by gassing them in various killing centres like Auschwitz. The Nuremberg Tribunal sentenced only eleven leading Nazis to death. Many others were imprisoned for life. The retribution did come, yet the punishment of the Nazis was far short of the brutality and extent of their crimes. The Allies did not want to be as harsh on defeated Germany as they had been after the First World War.

1. When did Germany surrender to the Allies?

(A) In July 1945

(B) In May 1945

(C) In March 1945

(D) In June 1945

(C) Polish Civilians (D) Mentally and physically disabled Germans Ans. Option (B) is correct. II. Read the source given below and answer the following questions:

E (1+1+1+1=4)

Political Radicalism and Economic Crises

Political Radicalisation was only heightened by the economic crisis of 1923. Germany had fought the war largely on loans and had to pay war reparations in Gold. This depleted gold reserves at a time resources were scarce. In 1923 Germany refused to pay, and the French occupied its leading industrial area, Ruhr, to claim their coal. Germany retaliated with passive resistance and printed paper currency recklessly. With too much printed money in circulation, the value of the German Mark fell. In April the US Dollar was equal to 24,000 Marks, in July 3,53,000 Marks, in August 46,21,000 Marks and at 9,88,60,000 Marks by December, the figure had run into trillions. As the value of the Mark collapsed, prices of goods soared. The image of Germans carrying cart-loads of currency notes to buy a loaf of bread was widely publicised evoking worldwide sympathy. This crisis came to be known as hyperinflation, a situation when prices rise phenomenally high.

1. Germany had fought the War largely on loans and had to pay war reparations in:

Ans. Option (B) is correct.

(A) Gold

(B) Silver



(C) Currency

(D) Coal



Explanation: The unconditional surrender of the German Third Reich was signed in the early morning hours of Monday, May 7, 1945, at Reims in northeastern France. 2. Killing on a large scale of people leading to destruction is called:

(A) accidental War.

(B) suicidal War.

(C) genocidal War.

(D) artificial War.

Ans. Option (C) is correct.

3. Germany's conduct during the war, especially those actions which came to be called Crimes Against______ raised serious questions and invited worldwide condemnation.

(A) Humanity

(B) Countries

(C) People

(D) All of the above

Ans. Option (A) is correct.

4. Which category of people were the biggest victim of Nazis? (A) Gypsies (B) Jews

Ans. Option (A) is correct. 2. Germany retaliated with passive resistance and printed ___________ currency recklessly. (A) plastic

(B) paper

(C) bronze

(D) copper

Ans. Option (B) is correct.



Explanation: In 1923 Germany refused to pay and the French occupied its leading industrial area, Ruhr, to claim their coal. Germany retaliated with passive resistance and printed paper currency recklessly. 3. What is the currency of Germany called? (A) Rupee

(B) Pound

(C) Dollar

(D) Mark

Ans. Option (D) is correct. 4. When was the US Dollar equal to 46,21,000 Marks? (A) In April

(B) In July

(C) In August

(D) In December

Ans. Option (C) is correct.

NAZISM AND THE RISE OF HITLER

Subjective based Questions I.

Read the source given below and answer the following questions:

E (1+1+2=4)

The crisis in the economy, polity and society formed the background to Hitler's rise to power. Born in 1889 in Austria, Hitler spent his youth in poverty. When the First World War broke out, he enrolled for the army, acted as a messenger in the front, became a corporal and earned medals for bravery. The German defeat horrified him and the Versailles Treaty made him furious. In 1919, he joined a small group called the German Workers' Party. He subsequently took over the organisation and renamed it the National Socialist German Workers' Party. This Party came to be known as the Nazi Party. In 1923, Hitler planned to seize control of Bavaria, march to Berlin and capture power. He failed, was arrested, tried for treason and later released. The Nazis could not effectively mobilise popular support till the early 1930s. It was during the Great Depression that Nazism became a mass movement. As we have seen, after 1929, banks collapsed and businesses shut down, workers lost their jobs and the middle classes were threatened with destitution. In such a situation Nazi Propaganda stirred hopes of a better future. In 1928, the Nazi Party got no more than 2.6 per cent votes in the Reichstag – The German Parliament. By 1932, it had become the largest Party with 37 per cent votes.

1. When did Hitler enrol in the army?

Ans. Hitler enrolled in the Army when the First War

63

Broke out. He acted as a messenger in the front.

2. When did Hitler join the German Workers' Party?

Ans. Hitler joined the German Workers' Party in 1919.

3. What did Hitler rename the German Workers' Party?

Ans. Hitler renamed it as the National Socialist German Workers' Party. II.

Read the source given below and answer the following questions:

U (1+1+2=4)

In May 1945, Germany surrendered to the Allies. Anticipating what was coming, Hitler, his propaganda minister Goebbels and his entire family committed suicide collectively in his Berlin bunker in April. At the end of the war, an International Military Tribunal at Nuremberg was set up to prosecute Nazi war criminals for Crimes against Peace, for War Crimes and Crimes Against Humanity. Germany’s conduct during the war, especially those actions which came to be called Crimes Against Humanity, raised serious moral and ethical questions and invited worldwide condemnation.

1. When did Germany surrender to the Allies?

Ans. Germany surrendered to the Allies in 1945.

2. Who was Hitler’s propaganda minister?

Ans. Goebbels was Hitler’s propaganda minister.

3. What raised serious moral and ethical questions against Germany and invited worldwide condemnation.

Ans. Germany’s conduct during the war, especially those actions which came to be called Crimes Against Humanity, raised serious moral and ethical questions and invited worldwide condemnation. 

UNIT-II

Contemporary India- I

CHAPTER

4

Learning Objectives

Study Time Max Time: 1:30 hr Max Questions: 37

INDIA : SIZE & LOCATION Examine how the location of an area impacts its climate and time with reference to longitude and latitude. l Explore and analyze the trading and cultural relationships of India with its neighboring countries. l Evaluate the situation & reasons that made 82.5E* longitude as Time meridian of India. l Examine how location of India enables its position as a strategic partner in the subcontinent. l

TOPIC - 1 Location and Size

Topic-1 Location and Size

.... P. 64

TOPIC - 2 India and the World; India's Neighbours .... P. 68

Revision Notes

 India is considered as one of the ancient civilisations in the world.  India has achieved multifaceted socio-economic progress in the varied domains like: field of agriculture, industry, technology, overall economic development etc.

Location

India lies in the Northern Hemisphere. The mainland extends between latitudes 8°4'N and 37°6'N and longitudes 68°7'E and 97°25’E. The Tropic of Cancer (23°30'N) divides the country into almost two equal halves. To the southeast and southwest of the mainland, lie the Andaman and Nicobar islands and the Lakshadweep islands in Bay of Bengal and Arabian Sea respectively.  Indira Point is the southernmost point of the Indian Union. It got submerged under the sea water in 2004 during the Tsunami.    

Scan to know more about this topic

Location and Extent of India

Size

The total area of Indian landmass is 3.28 million square km. India’s total area accounts for about 2.4 per cent of the total geographical area of the world. India is the seventh largest country of the world. India has a land boundary of about 15,200 km and the total length of the coast line of the mainland including the Andaman and Nicobar and the Lakshadweep is 7,516.6 km.  The Northwest, North and Northeast boundaries of India are characterised by the young fold mountains.  The latitudinal and longitudinal extent of the mainland is about 30°.  Time along the Standard Meridian of India (82°30'E) which passes through Mirzapur in Uttar Pradesh, is taken as the standard time for the whole country. It is also called IST.    

Key Terms  Equator: An imaginary line drawn around the middle of the earth that divides it into two equal halves.  Latitude: Latitude is the measurement of distance north or south of the Equator. It is measured with 180 imaginary lines that form circles around Earth east-west, parallel to the Equator.

INDIA: SIZE AND LOCATION

65

66

Oswaal CBSE Question Bank Chapterwise & Topicwise, SOCIAL SCIENCE, Class-IX

 Longitude: Longitude measures distance east or west of the prime meridian.  Strait: A narrow channel of sea which separates two land masses.

Example

India has a long coastline which is advantageous. Explain. Answer Step 1: The total length of the coastline of the mainland, including Andaman and Nicobar and Lakshadweep, is 7,516.6 km. Step 2: It is advantageous because of the following reasons:



Step 3: India's central location and long coastline have helped in interacting with the world. Step 4: Major ports on the eastern and the western coast help in the import and export of goods from India, developing its trade. Step 5: The coastline has contributed in the exchange of ideas and commodities.





OBJECTIVE TYPE QUESTIONS Column-I

Multiple Choice Questions Q. 1. What is the western-most longitude of India? R (A) 68°7’ E in Gujarat (B) 68°7’ N in Gujarat (C) 68°7’ S in Gujarat (D) 68°7’ E in Mumbai  Ans. Option (A) is correct. Explanation: The western-most longitude of India is 68°7' E. It passes through Gujarat (Kutch area). Q. 2. What is the eastern-most longitude of India? (A) 97°25’N (B) 97°25’E R (C) 97°25’W (D) 97°25’S Ans. Option (B) is correct. Explanation: The eastern-most longitude of India is 97°25'E. It is based in Arunachal Pradesh. Q. 3. In which year did Indira Point submerge under water due to Tsunami? (A) In 2014 (B) In 2004 R (C) In 1994 (D) In 1894 Ans. Option (B) is correct. Explanation: On 26th December 2004 Tsunami caused wide spread damage in coastal areas of India. The 'Indira Point' (Southern most point of India) in Andaman and Nicobar Islands got submerged in water due to Tsunami waves. Q. 4. Andaman and Nicobar Islands lie in which direction with respect to the mainland of India? (A) North-east direction to the mainland of India. (B) South direction to the mainland of India. (C) East direction to the mainland of India. (D) South-east direction to the mainland of India. R

Ans. Option (D) is correct. Q. 5. Match the column with the correct option:

(1 mark each)

E

Column-II

(a)

Standard Meridian of India

(i)

Passes through Mirzapur in Uttar Pradesh

(b)

Equator

(ii)

An imaginary line that divides the Earth

(c)

Tropic of Cancer

(iii)

23°30’N

(d) India’s coastline (iv) 7,516.6 km (A) a-i, b-ii, c-iii, d-iv (B) a-ii, b-i, c-iv, d-iii (C) a-i, b-ii, c-iii, d-iv (D) a-iv, b-i, c-ii, d-iii Ans. Option (A) is correct.

Assertion and Reason Direction: In the following questions, a statement of Assertion (A) followed by a statement of Reason (R). Mark the correct choice as:

(A)  Both Assertion (A) and Reason (R) are true, and Reason (R) is the correct explanation of Assertion (A).



(B) Both Assertion (A) and Reason (R) are true but Reason (R) is not the correct explanation of Assertion (A).



(C) Assertion (A) is true but Reason (R) is false.



(D) Assertion (A) is false but Reason (R) is true.

Q. 1. Assertion (A): The Indian Ocean is named after U India.

Reason (R): India has a strategic location along the trans-Indian Ocean routes. Ans. Option (A) is correct. Explanation: India is strategically located at the centre of the trans-Indian ocean routes which connects the European country in the west and the countries of the East Asia.

INDIA: SIZE AND LOCATION

Q. 2. Assertion (A): There would be the least difference in the duration between day time and night time A at Kanyakumari.

Reason (R): It is located far down south of the country. Ans. Option (B) is correct. Explanation: The day and night are nearly of the same duration at the Equator. As Kanyakumari is quite near the equator at 8°4' N the day-night difference is hardly one hour there and therefore, the duration of day and night is hardly felt at Kanyakumari which is located down South of the country.

67

Q. 3. Assertion (A): India is a Vast country. 

E



Reason (R): The Tropic of Cancer (23°30'N) divides the country into almost equal parts. Ans. Option (B) is correct. Explanation: India is a vast country geographically. It lies in the Northern hemisphere. The mainland extends between latitudes 8°4'N and 37°6'N and longitudes 68°7'E and 97°25'E. The Tropic of Cancer (23°30'N) divides the country into almost two equal parts.

SUBJECTIVE TYPE QUESTIONS Very Short Answer Type Questions Q. 1. From Gujarat to Arunachal Pradesh, there is a time lag of two hours. Explain. U Ans. There is a time lag of two hours from Gujarat to Arunachal Pradesh because of the longitudinal extent of India. The Sun rises two hours earlier in Arunachal Pradesh than Gujarat. This is because Gujarat is situated in the extreme west of India and Arunachal Pradesh is situated in the extreme east of India.(2) Q. 2. Why is India called a subcontinent? Give four points. E Ans. India is termed a subcontinent because of the following reasons: (i) India is a vast country. It covers 2.4% of the Earth's total land area. (ii) It has a long land frontier that is about 15,200 km. (iii) It also possesses a coastline which is about is 7,516.6 km including the Andaman and Nicobar and Lakshadweep. (iv) Its natural frontiers like the Himalayas in North and the Deccan Peninsula provides it a unique identity on globe.  (0.5 x 4=2)

Commonly Made Error

The students are unable to write the points related to the location of India.

Answering Tip

Ans. (i) Lakshadweep. (ii) It lies in the Arabian Sea.

The students should have a look at maps showing the geographical aspects of India's location so as to get a clear understanding.

Q. 3. Why was 82°30'E chosen as the Standard Meridian of India? Explain the reason. U Ans. (i) India is vast country in terms of size. So, different regions will have a different time. To avoid such confusion, it is necessary to have a standard meridian. (ii) It helps in maintaining uniformity of time across the country. (iii) It is also necessary to maintain harmony among the people. (1 x 2=2) Q. 4. Explain the size and geographical extent of India?  R Ans. Size and geographical extent: (i) The mainland extends between latitude 8°4'N and 37°6'N and longitude 68° 7'E and 97° 25'E. (ii) The land mass of India has an area of 3.28 million sq. km. (iii) India has a land boundary of about 15,200 km and length of coastline including islands is 7,516.6 km.  ( Any Two Points) (1 x 2 = 2)

Short Answer Type Questions Q. 1. If you visit Kavaratti Island, which Union Territory of India you will be going to? Mention its location.  A [Board Term I, 2016] [NCERT]

(2 marks each)

(3 marks each)

(iii) It is situated approximately 400 km off India's west coast. [CBSE Marking Scheme, 2016] (1×3=3)

68

Oswaal CBSE Question Bank Chapterwise & Topicwise, SOCIAL SCIENCE, Class-IX

Q. 2. What is meant by Indian Standard Time? Why do we need a Standard Meridian for India?  U [Board Term I, 2013, 2016] Ans. The time applicable for all over India is Indian Standard Time. To avoid time gap of 2 hours between East and West part of India which may create confusion in working, it is essential to have a Standard Meridian. (1+2=3) [CBSE Marking Scheme, 2013] Q. 3. Explain the major reasons for the two hours time difference in local time between Arunachal Pradesh and Gujarat. A [Board Term I, 2016] OR What is the time lag between Arunachal Pradesh and Gujarat?  OR The Sun rises two hours earlier in Arunachal Pradesh as compared to Gujarat in the West but the watches show the same time.  [NCERT] Ans. (i) Arunachal Pradesh is approximately 30.8° longitudinally. (ii) The Sun rays take 4 minutes to travel 1° longitude. (iii) So, the Sun rays will take 120 min, that is, 2 hours to travel from Arunachal Pradesh to Gujarat. Hence, the time difference of two hours. [CBSE Marking Scheme, 2016] (1 × 3 = 3)

(iii) Kanyakumari is close to the Equator, whereas Kashmir is far from the Equator. As Equator receives direct sunlight, the duration between day and night would hardly be felt at Kanyakumari. On the other hand, Kashmir is at 37° N latitude, which means that it is 37° away from the Equator and receive slanting sunlight. So, there would be a difference in the duration of day and night in Kashmir. This difference causes a time lag between Kashmir and Kanyakumari. [CBSE Marking Scheme, 2015] (1 × 3 = 3)

Commonly Made Error

The students usually write the wrong values of the latitude and longitude while writing this answer.

Answering Tip

Students should learn the correct values of the latitude and longitude before attempting the question.

Q. 5. State the latitudinal and longitudinal extent of India's mainland. What is the latitude of Tropic of Cancer? U [Board Term I, 2015] OR What is the latitudinal and longitudinal extent of India? Name the imaginary line which divides India into North and South. R

Q. 4. Why is the difference between the duration of day and night hardly felt at Kanyakumari but not so in Kashmir? A [Board 2015 Term I, (NCERT)]



Ans. (i) The difference in the duration of day and night at Kanyakumari and Kashmir are respectively due to their latitudinal locations. (ii) Kanyakumari is located closer to the Equator, i.e., 8° away from the Equator.

Ans. Latitudinal extension of India: 8°4’N–37°6’N Longitudinal extension of India: 68°7’E–97°25’E Latitude of Tropic of Cancer: 23°30’N [CBSE Marking Scheme, 2015] (1+1+1=3)

Topic-2 India and the World; India's Neighbours Revision Notes India and the World

 Location of India in the world plays an important role because of the following reasons:





l The Indian landmass has a central location between the East and West Asia.





l India is a Southward extension of the Asian continent.





l The Trans-Indian Ocean routes connecting the countries of Europe in the West and the countries of East Asia provide a strategic central location to India.





l The Deccan Peninsula protrudes into the Indian Ocean which helps it to establish close contact with West Asia, Africa and Europe from the western coast and with Southeast and East Asia from the eastern coast.





l No other country has such a long coastline on the Indian Ocean as India has, and indeed, it is India’s strategic position in the Indian Ocean which justifies the naming of an ocean after it.

INDIA: SIZE AND LOCATION

69

 Distance between India and Europe has been reduced by 7,000 km after the opening of the Suez Canal in 1869.  India is connected with Europe, North America and South America with the Suez Canal and the Cape of Good Hope through the sea routes. Scan to know more about this topic

India’s Neighbours  India has 28 states and 8 Union Territories.  India shares its land boundaries with Pakistan and Afghanistan in the Northwest, China (Tibet), Nepal and Bhutan in the North along with Myanmar and Bangladesh in the East.  Our southern neighbours across the sea consist of the two island countries, i.e., Sri Lanka and Maldives.

India's Neighbours

 Sri Lanka is separated from India by a narrow channel of sea formed by the Palk Strait and the Gulf of Mannar.  India has had strong geographical and historical links with its neighbours.

Key Term  Peninsular Plateau: It is a tableland composed of the old crystalline, igneous and metamorphic rocks.

OBJECTIVE TYPE QUESTIONS Multiple Choice Questions Q. 1. Who are India’s neighbours in the north-west? R (A) Pakistan and Afghanistan (B) Pakistan and China (C) Myanmar and Bangladesh (D) Afghanistan and Bhutan  Ans. Option (A) is correct. Explanation: There are neighbouring countries of India. Pakistan and Afghanistan in the northwest, China, Nepal and Bhutan in the north; Myanmar and Bangladesh in the east and Sri Lanka and Maldives in South. Q. 2. Which two states of India share their land R boundary with Bangladesh?  (A) Rajasthan and Assam (B) West Bengal and Jharkhand (C) West Bengal and Assam (D) Assam and Uttar Pradesh Ans. Option (C) is correct. Explanation: Bangladesh is situated in the east. Hence, it shares its land boundary with West Bengal, Assam, Meghalaya, Mizoram and Tripura. Q. 3. Which country among India’s neighbours is the smallest? ( R, E ) (A) Sri Lanka (B) Myanmar (C) Nepal (D) Bhutan Ans. Option (D) is correct. Explanation: Amongst all the above options, Bhutan is the smallest as the area of Sri Lanka is 65,610 sq. km, Myanmar is 6,76,578 sq. km, Nepal is 1,47,181 sq. km and Bhutan is 38,394 sq. km. Q. 4. Which country shares the longest boundary with India? ( U, R) (A) Sri Lanka (B) Bangladesh (C) China (D) Myanmar

(1 mark each)

Ans. Option (B) is correct. Explanation: India shares its land boundary with seven countries. Among all the above options, length of Bangladesh–India border is 4096.7 km. Q. 5. What separates Sri Lanka and India through a narrow channel of sea? (A) Arabian Sea (B) Persian Gulf (C) Palk Strait and the Gulf of Mannar (D) Palk Strait and Persian Gulf Ans. Option (C) is correct.

Assertion and Reason Direction: In the following questions, a statement of Assertion (A) is followed by a statement of Reason (R). Mark the correct choice as:

(A)  Both Assertion (A) and Reason (R) are true, and Reason (R) is the correct explanation of Assertion (A).



(B)  Both Assertion (A) and Reason (R) are true, but Reason (R) is not the correct explanation of Assertion (A).



(C) Assertion (A) is true but Reason (R) is false.



(D) Assertion (A) is false but Reason (R) is true.

Q. 1. Assertion (A): The Indian landmass has a central E location between the East and West Asia.

Reason (R): India is a Northward extension of the Asian Continent. Ans. Option (C) is correct. Explanation: Indian landmass has a central location between the East and West Asia. India is a southward extension of Asian Continent.

70

Oswaal CBSE Question Bank Chapterwise & Topicwise, SOCIAL SCIENCE, Class-IX

Q. 2. Assertion (A): The various passes across the mountains in the north have provided passages to A the ancient travellers.

Reason (R): These routes have contributed in the exchange of ideas and commodities since ancient times. Ans. Option (A) is correct. Q. 3. Assertion (A): Before 1947, there were three types A of states in India. 



Reason (R): Provinces were ruled directly by the British officials and Princely States were ruled by local rulers. Ans. Option (D) is correct. Explanation: Before 1947, there were two types of States in India–(i) the Provinces which were ruled directly to British officials, (ii) Princely States which were ruled by local, hereditary rulers.

SUBJECTIVE TYPE QUESTIONS Short Answer Type Questions Q. 1. When did the Suez Canal start functioning and how did it benefit India? R [Board Term I, 2015]

Q. 3. The central location of India at the head of the Indian Ocean is considered of great significance. Why?

Ans. The Suez Canal started functioning in 1869. Benefits to India:



(i) It reduced the distance between India and Europe by 7,000 km. (ii) The canal is a boon for trade as it had reduced the transportation cost and number of days involved.

[CBSE Marking Scheme, 2015] (1 + 2 = 3)

Commonly Made Error

The students usually write the wrong year of starting the Suez Canal.

U [NCERT]

[Board 2013 Term-I] OR Explain the significance of the location of India at the head of the Indian Ocean. OR Explain the significance of India’s eminent position in the Indian Ocean. OR What is the significance of India's central location? Ans. Significance: (i) The Trans-Indian Ocean route connects countries of Europe in West.

(ii) Countries of East Asia have provided a strategic central location to India.

Answering Tip

(3 marks each)

The economic benefit received due to the construction of the canal needs to be described.

(iii) Deccan Plateau protrudes into the Indian Ocean. It helps to establish a close contact with West Asia, Africa and Europe.

Q. 2. Which neighbouring island country lies to the South-east of India? Name the water bodies that separate this island country from India.  R [Board Term I, 2015]

(iv) No country other than India has such a long coastal line on the Indian Ocean which provides significance and favourable trading ports and commercial hubs for many MNCs and countries.

Ans. Sri Lanka is the neighbouring island country that lies to the South-east of India.

(v) The coastal regions of our country provide important hinterlands for the ports.



The two water bodies that separate this island country from India are:

(vi) The sea routes have contributed in the exchange of ideas and commodities.



(i) The Palk Strait, and



(ii) The Gulf of Mannar.

(vii) The ideas of Upanishads and Ramayana, Indian numerals and the decimal system could reach to many parts of the world.



(1+2=3)

[CBSE Marking Scheme, 2015]

(viii) Spices, muslin and other merchandises were taken to other countries. (ix) Influence of Greek sculpture and styles of domes and minarets can be seen in our country. (Any six) (½ × 6 = 3)

INDIA: SIZE AND LOCATION

Commonly Made Error

The students give details about the geographical location of India.

Answering Tip

The benefits that India has on the economic front should be explained.

Q. 4. Why is the Indian Ocean named after our country? Give three reasons. U Ans. Indian Ocean is named after India because : (i) India has the longest coastline on the Indian Ocean. (ii) India has a central location between East and West Asia. (iii) India's southernmost extension Deccan Peninsula protrudes into Indian Ocean that makes it significant to international trade done through Indian Ocean. (1 × 3 = 3)

Long Answer Type Question Q. 1. Name the states that share common borders with following neighbours. (i) Pakistan, (ii) China, (iii) Myanmar and (iv) Bangladesh, (v) Nepal. Ans. (i) Pakistan: Jammu and Kashmir, Ladakh, Punjab, Rajasthan and Gujarat are the states which share common border with Pakistan. (ii) China: Ladakh, Himachal Pradesh, Sikkim and Arunachal Pradesh are the states which share common border with China.

(5 marks)

(iii) Myanmar: Arunachal Pradesh, Nagaland, Manipur, Mizoram are the states which share common border with Myanmar. (iv) Bangladesh: West Bengal, Mizoram, Meghalaya, Tripura, and Assam are the states which share common border with Bangladesh. (v) Nepal: Uttarakhand, Uttar Pradesh, Bihar, West Bengal and Sikkim are the states which share common border with Nepal. (5)

COMPETENCY BASED QUESTION Case based MCQs

I. Read the source given below and answer the A (1+1+1+1=4) following questions:  India's Neighbours India occupies an important strategic position in South Asia. India has 28 states and 8 Union Territories. India shares its land boundaries with Pakistan and Afghanistan in the northwest, China (Tibet), Nepal and Bhutan in the north and Myanmar and Bangladesh in the east. Our southern neighbours across the sea consist of the two island countries, namely Sri Lanka and Maldives. Sri Lanka is separated from India by a narrow channel of sea formed by the Palk Strait and the Gulf of Mannar, while Maldives Islands are situated to the south of the Lakshadweep Islands. India had strong geographical and historical links with her neighbours. 1. Who are India’s neighbours in the north-west? (A) Pakistan and Afghanistan (B) Pakistan and China (C) Myanmar and Bangladesh (D) Afghanistan and Bhutan Ans. Option (A) is correct. 2. Which two states of India share their land boundary with Bangladesh? (A) Rajasthan and Assam

71

(4 marks)

(B) West Bengal and Jharkhand (C) West Bengal and Assam (D) Assam and Uttar Pradesh Ans. Option (C) is correct. 3. What do we call a narrow channel of sea which separates two land masses? (A) Strait (B) Gulf (C) Peninsula (D) Delta Ans. Option (A) is correct. 4. My friend hails from a country which does not share land boundary with India. Identify the country. (A) Bhutan (B) Tajikistan (C) Bangladesh (D) Nepal Ans. Option (B) is correct.

Subjective based Questions

I. Read the source given below and answer the following questions:  (1+1+2 =4) India’s contact with the world has continued through ages but her relationships through the land routes are much older than her maritime contacts. The various passes across the mountains in the north have provided passages to the ancient travellers, while the oceans restricted such interaction for a long time. These routes have contributed in the exchange of ideas and commodities since ancient times.

72



Oswaal CBSE Question Bank Chapterwise & Topicwise, SOCIAL SCIENCE, Class-IX

Ans. The various passes across the mountains of Himalayas have provided passages (Passes) to the ancient travellers. 2. Name an epic and a story. (A) upanishad, epic (B) epic, stories (C) upanishad, stories (D) epic, tale

The ideas of the Upanishads and the Ramayana, the stories of Panchatantra, the Indian numerals and the decimal system thus could reach many parts of the world. The spices, muslin and other merchandise were taken from India to different countries. On the other hand, the influence of Greek sculpture and the architectural styles of dome and minarets from West Asia can be seen in different parts of our country. 1. Which geographical feature has provided passage to the ancient travellers in India?

Ans. Ramayana is an epic and Panchatantra are stories. 3. Name any two things that India traded with the world. Ans. Spices, muslin and other merchandise. ( Any Two)

Map Work

Syllabus

India-States with Capitals, Tropic of Cancer, Standard Meridian (Location and Labelling) Neighbouring Countries. l

Indian States and Union Territories with their capitals: 68°

72°

76°

80°

84°

88°

92°

96°

36°

TAJIKISTAN

INDIA– Political

36°

Scale

AFGHANISTAN

LADAKH

KABUL ISLAMABAD

SRINAGAR

32°

km100

Standard Meridian

J&K

LEH HIMACHAL PRADESH

PA K I S TA N

PUNJAB

SHIMLA DEHRA DUN

CHANDIGARH

UTTARA

KHAND

0

1: 35.07 million 200

400 km

REFERENCES State Boundary

International Boundary Country Capital Jammu and Kashmir Island

J&K I.

Islands

HARYANA

SIKKIM

DELHI 28°

U T TA R PRADESH

JAIPUR

RAJASTHAN

GANGTOK

20°

Diu DAMAN

D

I

MAHARASHTRA

RANCHI

A

28°

ITANAGAR

BENGAL KOLKATA

MANIPUR24° r

Tropic of Cance

AIZAWL MIZORAM

DHAKA

New Moore (India)

NAGALAND

IMPHAL

G L A D E SH B A NTR IPURA AGARTALA

ODISHA

RAIPUR

DADRAAND NAGAR HAV ELI Silvassa AND DAMAN AND DIU

RA A L P D ESH

SHILLONG

MEGHALAYA

ND KHA WEST JHAR

CH HA TT IS GA RH

N

I

CH

AS SA M KOHIMA

MADHYA PRADESH BHOPAL

NA

THIMPHU

BIHAR PATNA

GANDHINAGAR

A

RU

DISPUR

LUCKNOW

24°

GU JA RAT

BHUTAN

32°

State Headquarter

Is.

MYANMAR

I.

20°

BHUBANESHWAR

MUMBAI

A R A B I A N S E A PANAJI GOA

16°

KA RN AT AK A

TELANGANA HYDERABAD

Yanam (Puducherry)

KAVARATTI

F

ANDHRA PRADESH

B

E

N

G

A

L

TA M IL NADU

Preparis I.

Coco Is. ar) (Myanm Narcondam (India)

CHENNAI

Barren (India)

PUDUCHERRY

LA RA KE

Mahe (Puducherry)

16°

O

AMARAVATI

BENGALURU 12°

YANGON

Y

A

B

12°

PORT BLAIR

Karaikal (Puducherry)

8° 8°

Map not to scale I N D I A N 72°

76°

O C E A N 80°

84°

88°

Indira Point

Projection : Lambert Conical Orthomorphic 92°

INDIA: SIZE AND LOCATION

73

Artificial Intelligence Understanding the location of India in the world and to understand its importance using Google Maps Using the concept of maps in Artificial Models for understanding India’s location, size in the world. Objectives:  Understanding what India’s location in the world (Latitudinal and longitudinal extent).  Understanding size of India and its position in the world in terms of size.  Understanding the relationship between distance and time through different longitudes.  Understanding the importance of India’s central location in the World for trading.  Understanding India’s relations with its neighbouring countries. Material Required: Textbook, globe, World Map, PPT, Pen, paper, white board/smart board, marker, Laptops and Internet connection. PARAMETERS Chapter Covered

DESCRIPTION

AI CONCEPTS INTEGRATED

Chapter 1: India Size and Location

Name of the Book Social Science Contemporary India - 1, Class 9 NCERT Subject and Artificial Intelligence Integrated

Understanding the location of India in the world and to understand its importance using Google Maps Using the concept of maps in Artificial Models for understanding India’s location, size in the world

Learning Objectives

l l l l l

 nderstanding what India’s location in the world (Latitudinal U and longitudinal extent) Understanding Size of India and its position in the world in terms of size Understanding the relationship between distance and time through different longitudes Understanding the importance of India’s central location in the World for trading Understanding India’s relations with its neighbouring countries.

Time Required

3 periods of 40 minutes each

Classroom Arrangement

Flexible

Material Required

Textbook, globe, World Map, PPT, Pen, paper, white board/smart board, marker, Laptops and Internet connection.

Pre – Preparation Activities

Video showing India’s location in the world, Latitudinal, longitudinal extent and standard Meridian of India. Scan to know more about this topic

Previous Knowledge

Scan to know more about this topic

The students shall be asked questions such as What is latitude and longitude? What is Indian Peninsula? Why is India called a subcontinent? Why do we have standard time? Which are the neighbouring countries of India?

https://datavizcatalogue. com/ https://mapwith.ai/rapid# background=OpenTopo Map&disable_features= boundaries&map=4.00/1 9.59/59.61 http://ncase.me/loopy/

74

Oswaal CBSE Question Bank Chapterwise & Topicwise, SOCIAL SCIENCE, Class-IX

Methodology

The teacher will discuss about Using AI Model l  Latitudinal & longitudinal extent of India through Hands on datavizcatalogue And Google Map activity using map & atlas l Indian sub-continent using discussion and map activity l Indian’s position in the world in terms of size through bar graph l Importance of Indian Standard Meridian using video l Importance of India’s location through Discussion/brainstorming l Neighbouring countries through l Visual technique using ppt & map of India

Learning Outcomes

At the end of the lesson students will be able to: l Appreciate the importance of India’s relation with the world through ages and its size through bar graph and will learn to use datavizcatalogue. l  Develop the skill to locate important latitudes & longitudes passing through India & the neighbouring countries of India on the map. l Appreciate & understand the implication of vast longitudinal & latitudinal extent of India. l Appreciate the reason why an ocean is named after India using google map.

Follow up Activities

The students are asked l To make a map of India showing different states and UTs. l To label and locate all the neighbouring countries, Bay of Bengal, Arbian sea and Indian ocean on map. l To find out latitudinal and longitudinal extent of your state.

Reflections

The students will be asked to identify India’s neighbouring countries, Indian states and UTs on a globe. To identify and names the states through which Standard Meridian is passing 

Study Time Max Time: 2:45 hr Max Questions: 64

CHAPTER

5

Learning Objectives

PHYSICAL FEATURES OF INDIA l Justify why India is a sub- continent. l Examine the geological process that played a crucial role in the formation of diverse physical features in India. l Analyse the conditions and relationships of the people living in different physiographic areas. l Examine various environmental issues.

Major Physiographic Divisions

Topic-1 and The Himalayan Mountains Revision Notes



TOPIC - 1 Major Physiographic Divisions and The Himalayan Mountains  .... P. 75 TOPIC - 2 The Northern Plains and The Peninsular Plateau .... P. 80 TOPIC - 3

 I ndia is a large landmass formed during different geological periods which The Indian Desert , The Coastal has influenced its relief. Plains and The Islands  Besides geological formations, a number of processes such as weathering, .... P. 84 erosion and deposition are also responsible for creating and modifying the relief to its present form.  The Gondwana land includes today Africa, South America, Australia, Antarctica, the Indian subcontinent and the Arabian Peninsula.  Geologically, the Peninsular Plateau constitutes one of the ancient landmasses on the earth’s surface. The Himalayas and the Northern Plains are the most recent landforms.  Most volcanoes and earthquakes in the world are located at plate margins, but some do occur within the plates.

Major Physiographic Divisions

 The physical features of India can be grouped under the following physiographic divisions:





     

The Himalayan Mountains The Northern Plains The Peninsular Plateau The Indian Desert The Coastal Plains The Islands

The Himalayan Mountains  The Himalayas, geologically young and structurally fold mountains, stretch over the northern borders of India.

Scan to know more about this topic

6 Physiographic Divisions of India Scan to know more about this topic

 These mountain ranges run in a West–East direction from the Indus to the Brahmaputra.  An arc is formed by the mountains that cover a distance of about 2,400 km.  The attitudinal variations are greater in the eastern half than those in the western half.  The Himalayas consist of three parallel ranges in its longitudinal extent.

The Himalayan Mountains

76

Oswaal CBSE Question Bank Chapterwise & Topicwise, SOCIAL SCIENCE, Class-IX

PHYSICAL FEATURES OF INDIA













 

77

Great or Inner Himalayas or the Himadri : The northern-most range, consisting of the loftiest peaks with an average height of 6,000 m.  Himachal or Lesser Himalaya : The range lying to the South of the Himadri forms the most rugged mountain system. The altitude varies between 3,700 to 4,500 metres and the average width is 50 km. The ranges are mainly composed of highly compressed and altered rocks.  Shivaliks: The outermost range of the Himalayas. Their width varies from 10–50 km and has an altitude varying between 900 to 1,100 m. These ranges are composed of unconsolidated sediments brought down by rivers from the main Himalayan ranges located farther north. The longitudinal valleys lying between the lesser Himalayas and the Shivaliks are known as ‘Duns’. Apart from longitudinal divisions, the Himalayas have also been divided by river valleys on the basis of regions from West to East. 

Key Terms  Gondwana land: It was an ancient supercontinent that broke up about 180 million years ago. The continent eventually split into landmasses that we recognise today as Africa, South America, Australia, Antarctica, the Indian subcontinent and the Arabian Peninsula.  Purvanchal: Mountains along the eastern boundary of India are called the Purvanchal.

OBJECTIVE TYPE QUESTIONS Multiple Choice Questions R Q. 1. Which is the oldest landmass in India? (A) The Deccan Plateau (B) Jack Hills (C) Eurasia (D) The Peninsular Plateau Ans. Option (D) is correct. Explanation: The Peninsular Plateau of India is one of the ancient (old) landmass on Earth's surface in India, which was also part of Gondwanaland. Q. 2. The whole mountain system of Himalaya U represents a very________ topography.  (A) youthful (B) skilled (C) simple (D) All of the Above Ans. Option (A) is correct. Q. 3. Mountain ranges in the Eastern part of India forming its boundary with Myanmar are R collectively called: (A) Himachal (B) Uttarakhand (C) Purvanchal (D) None of the above Ans. Option (C) is correct. Explanation: As the question suggests the eastern part of India and location of Myanmar is also in east. Hence, the mountain range is known as 'Purvanchal'. Q. 4. How long are the east-west distances covered by R the Himalayas?  (A) 1,400 kms (B) 2,500 kms (C) 4,200 kms (D) 2,040 kms Ans. Option (B) is correct. Explanation: The Himalaya themselves stretch uninterruptedly for about 1550 miles i.e., 2500 km from Nanga Parbat in west (Pakistan) to Namcha

(1 mark each)

Barwa in east (Tibet). Q. 5. Name the part of the Himalayas lying between the R Kali and Tista rivers.  (A) Nepal Himalaya (B) Kanchenjunga (C) Mt. Everest (D) Mount Kailash Ans. Option (A) is correct. Explanation: The part of Himalayas lying between Satluj and Kali is Kumaon; between Kali and Teesta is Nepal Himalayas and between Teesta and Dihang is Assam Himalayas.

Assertion and Reason Directions: In the following questions, a statement of Assertion (A) if followed by a statement of Reason (R). Mark the correct choice as:

(A)  Both Assertion (A) and Reason (R) are true, and Reason (R) is the correct explanation of Assertion (A).



(B)  Both Assertion (A) and Reason (R) are true, but Reason (R) is not the correct explanation of Assertion (A).



(C) Assertion (A) is true, but Reason (R) is false.



(D) Assertion (A) is false, but Reason (R) is true.

Q. 1. Assertion (A): The Himalayas represent the loftiest and one of the most rugged mountain barriers of E the world. 

Reason (R): The Himalayas are the most recent landforms. Ans. Option (B) is correct. Explanation: The Himalayas are the most recent young landforms. The Himalayas represent the loftiest and one of the most rugged mountain barriers of the world composed of rocks. They form an arc, which covers distance of about 2400 km.

78

Oswaal CBSE Question Bank Chapterwise & Topicwise, SOCIAL SCIENCE, Class-IX

Q. 2. Assertion (A): Geologically, the Peninsular Plateau was supposed to be one of the most unstable land

Q. 3. Assertion (A): The longitudinal valley lying between lesser Himalaya and the Shiwaliks is

U

blocks.

Reason (R): From the view point of geology, Himalayan mountains form an unstable zone. Ans. Option (D) is correct. Explanation: Geologically, the Peninsular Plateau constitutes one of the ancient landmasses on Earth's surface and they were supposed to be one of the most stable land blocks. Whereas, the Himalaya and the Northern Plains are most recent landforms so from the point of geology, Himalayan mountains form an unstable zone.

A

known as Duns. 

Reason (R): Dehra Dun, Kotli Dun and Patli Dun are some of the well-known Duns. Ans. Option (B) is correct. Explanation: In general Duns refer to the longitudinal valleys which are formed as a result of folding when Eurasian and Indian Plate collide. They are generally formed between Lesser Himalayas and Shivaliks. These valleys are deposited with coarse alluvium which is brought down by the Himalayan rivers. Examples of some Duns are Dehra Dun, Patli Dun and Kotli Duns, which are very famous.

SUBJECTIVE TYPE QUESTIONS Very Short Answer Type Questions Q. 1. Why does India have diversity in its relief? 

R

Ans. Reasons for diversity in relief:

(i) Different geological periods. (ii) Different

geological

processes—weathering,

erosion and deposition. (1 x 2 = 2) Q. 2. Compare the Shiwalik and the Himachal Mountain ranges on the basis of composition and altitude. R Ans. S. No.

Himachal Range

Shiwalik Range

The range lying to the South of the Himadri forms the most rugged mountain system and is known as Himachal range.

The outer-most range of the Himachal is called the Shivalik range.

(ii)

The altitude varies between 3,700 and 4,500 metres and the average width is 50 km.

They extend over a width of 10-50 km and have an altitude varying between 900 and 1,100 m.

(iii)

The ranges are mainly composed of highly compressed and altered rocks.

These ranges are composed of unconsolidated sediments brought down by rivers from the main Himalayan ranges.

(i)



(2 marks each)

Q. 3. Which range of Himalayas lies between Himadri and Shivaliks? Mention any one feature of this range of Himalayas. E Ans. The range that lies between the Himadri and Shivaliks is Himachal. Features:

(i) Composed of highly compressed and altered rocks.



(ii) Consists of famous valleys of the Kangra and Kullu which are known for hill stations.



(Any One Point) (1 + 1 = 2)

Q. 4. Find out the names of the glaciers and passes that lie in the Great Himalayas.  R [NCERT] Ans. Glaciers lying in the Great Himalayas are: Gangotri, Chaturangi, Bhagirathi, Kharak, Satopanth, Kamet, Milam and Pandari. (Any Two) Passes lying in Great Himalayas are: Karakoram, Shiplika, Bomdila, Nathula, etc.

(Any Two Points) (1 x 2 = 2)



(Any Two) (0.4 x 5 = 2)

Q. 5. Find out the names of the states where highest peaks are located. R [NCERT] Ans. Some of the highest peaks and their states of location are:

(i) kanchenjenga in Sikkim.



(ii) Nanga Parbat in Ladakh.

(iii) Nanda Devi in Uttarakhand. (iv) Kamet in Uttarakhand and Namcha Barwa in Arunachal Pradesh. 

(Any Two Points) (1 x 2 = 2)

PHYSICAL FEATURES OF INDIA

Short Answer Type Questions Q. 1. Describe any three features of the Kashmir Himalayas. U [Board Term I, 2016] Ans. (i) Kashmir or Northwestern Himalayas comprise a series of ranges such as Karakoram, Ladakh, Zaskar and Pirpanjal. (ii) Between the Greater Himalayas and the Pirpanjal range lies the famous valley of Kashmir and Dal Lake. (iii) This place is famous for kahwa.  [CBSE Marking Scheme, 2016] (1×3=3) Q. 2. Why are the Shiwalik ranges prone to landslides and earthquakes? Give reasons. U  [Board Term I, 2016] Ans. (i) Shivalik is the Southern range of the Himalayas. It is a discontinuous range as it disappears in the East. (ii) This region is made of loose unconsolidated deposits brought down by rivers from the main Himalayan ranges. (iii) The region is prone to widespread erosion, landslides and earthquakes.  [CBSE Marking Scheme, 2016] (1×3=3)

Commonly Made Error

The students are not aware of what are the Himalayan foothills.

Answering Tip

The students need to mention the geographical,

climatic and other features that leads to the occurrence of landslides.

Q. 3. Describe the extent of the Himalayas in the Northeast. R [Board Term I, 2016] Ans. (i) The Brahmaputra River makes the easternmost boundary of the Himalayas. (ii) Beyond the Dihang Gorge, the Himalayas bend sharply to the South. (iii) General direction is from South-west to North-east.

(3 marks each)

(iv) Eastern Hills or the Hills running through North-eastern states are known by different local names such as Mizo Hills, Naga Hills, etc. (Any three) [CBSE Marking Scheme, 2016] (1×3=3) Q. 4. Describe any three features of the Purvanchal Range of the Himalayas.  U [Board Term I, 2014] OR What are the Purvanchal Hills? Mention any two features of these hills.  [Board Term-I, DDE 2014] OR Which part of the Himalayas form the Purvanchal? State any two characteristics of the Purvanchal. Ans. Beyond the Dihang Gorge, the Himalayas bend sharply to the South and spread along the eastern boundary of India. These are known as the Purvanchal. Characteristics: (i) Mostly composed of strong sedimentary rocks. (ii) It is covered with dense forests which mostly run parallel to ranges and valleys. (iii) It consists of Patkai Hills, Naga Hills, the Manipur and the Mizo Hills. (Any two) (1+2=3) Q. 5. Name the southernmost range of Himalaya. Mention any four features of this range. U

[Board Term I, DDE-2014] OR Describe any three features of the Shivalik Range. [Board Term I, 2013] Ans. The southern-most range of the Himalayas is called the Shiwaliks. Features: (i) The outer-most range of the Himalayas is called the Outer Himalayas or Shivaliks. (ii) They extend over a width of 10–15 km. (iii) Their altitude varies between 900–1,100 m. (iv) They are discontinuous ranges and composed of unconsolidated sediments, gravel and alluvium brought down by the rivers from the main Himalayan ranges located farther north. (v) Longitudinal valleys known as Duns lie between the lesser Himalayas and Shivaliks. For example, Dehradun, Kotli Dun and Patli Dun. (Any four) (1+4×½=3)

Long Answer Type Questions Q. 1. Give an account of the four divisions of Himalayas from West to East along with Purvanchal hills respectively. U [Board Term I, 2013]

79

(5 marks each)

Ans. Punjab Himalayas: Lies between Indus and Satluj rivers.

80

Oswaal CBSE Question Bank Chapterwise & Topicwise, SOCIAL SCIENCE, Class-IX

S.No.

Himadri Range

Shivalik Range

1.

It is the innermost or the northernmost range of the Himalayas. It has an average height of 6,000 m

It is the outermost range of the Himalayas.

The core of this part of Himalayas is composed of granite. It contains all prominent Himalayan peaks.

This range is composed of unconsolidated rocks.

Example: It includes peaks like K2, Kangchenjunga, Mt. Everest , etc.

Example: Dehradun.



Kumaon Himalayas: Lies between Satluj and Kali rivers.



Nepal Himalayas: Lies between Kali and Tista rivers.



Assam Himalyas: Lies between Tista and Dihang rivers.

2.



Purvanchal Hills: North–eastern extension of the Himalayas.

3.

[CBSE Marking Scheme, 2013] (1×5=5) 4.

Q. 2. What are Duns? Differentiate between the Inner Himalayas and the Lesser Himalayas. U Ans. The longitudinal valleys lying between the Lesser Himalayas and the Shivaliks are known as Duns. (i) The northern most range is known as the Greater or Inner Himalayas or the ‘Himadri’.

5.

(ii) It is the most continuous range consisting of the loftiest peaks with an average height of 6,000 m. (iii) It contains all the prominent Himalayan peaks.

Commonly Made Error

(vi) This range consists of the famous valley of Kashmir, Kangra and Kulu. (Any four) (1+4=5) Q. 3. State the differences between the Himadri range and Shiwalik range. U Ans. The differences between the Himadri and Shiwalik range are:

Presence of longitudinal valleys or Duns like Dehradun, Kotli Dun and Patli Dun are one of the most prominent features of the Shivalik range.

(1×5=5)

(iv) The range lying to the south of the Himadari is the most rugged mountain system and is known as Himachal or the Lesser Himalayas. (v) Continuous altered rocks with altitude between 3,700 to 4,500 m.

Its height varies between 900 to 1,100 m.

The students usually make errors in the figures of the altitudes.

Answering Tip

The differences between the two ranges should be explained on the basis of location , soil found, etc.

Topic-2 The Northern Plains and The Peninsular Plateau Revision Notes The Northern Plains  The Northern Plains have been formed by the interplay of the three major river systems, i.e., the Indus, Ganga and Brahmaputra along with their tributaries.  With rich soil cover, combined with adequate water supply and favourable climate, it is agriculturally a very productive part of India.  The Northern Plains are broadly divided into three sections—Punjab Plain, Ganga Plain and Brahmaputra Plain.  Bhangar is the largest part of the Northern Plains, formed of older alluvium.  Majuli in the Brahmaputra River is the largest inhabited riverine island in the world.

Scan to know more about this topic

The Northern Plains

Scan to know more about this topic

The Peninsular Plateau  The Peninsular Plateau is a tableland composed of the old Crystalline, Igneous and Metamorphic rocks.  This plateau consists of two broad divisions — the Central Highlands and the Deccan Plateau.

The Peninsular Plateau

PHYSICAL FEATURES OF INDIA

81

 The part of the Peninsular Plateau lying to the North of the Narmada River covering a major area of the Malwa Plateau is known as the Central Highlands.  The Deccan Plateau is a triangular landmass that lies to the South of the River Narmada.  The Western Ghats and the Eastern Ghats mark the western and the eastern edges of the Deccan Plateau respectively.  The highest peaks of the Western Ghats are the Anai Mudi (2,695 metres) and the Doda Betta (2,637 metres).  Mahendragiri (1,501 m) is the highest peak in the Eastern Ghats.

Key Facts  The Indus–Ganga plains, also known as the "Great Plains", are large floodplains of the Indus, Ganga and the Brahmaputra river systems.

Key Terms

    

Bhabar: Bhabar is a belt of pebbles extending from 8-16 km in width in which stream disappears. Terai: Terai is a wet, swampy, marshy region with thick forests and wildlife. Bhangar: Bhangar is a terrace-like feature made of old alluvium. It contains calcareous deposits called Kankar. Khadar: Khadar is the flood plain which is renewed every year and is very fertile. Doab: It is made up of two words- ‘do’ meaning two and ‘ab’ meaning water.

Example . Give an account of the Deccan Plateau. Q Answer:

Step 1: It is a triangular landmass that lies to the South of the River Narmada. The Satpura range flanks its broad base in the North, while the Mahadev forms its eastern extensions.



Step 2: The Deccan Plateau is higher in the West and slopes gently Eastwards. It is separated by a fault from the Chota Nagpur Plateau. Three prominent

hill ranges from the West to East are the Garo, Khasi and the Jaintia Hills. The Western Ghats and the Eastern Ghats mark the western and eastern edges of the Deccan Plateau respectively. The Western Ghats lies parallel to the Western Coast and Eastern Ghats lies parallel to the Eastern Coast.

Step 3: An extension of the plateau is also visible in the Northeast. It is locally known as the Meghalaya, Karbi-Anglong Plateau and North Cachar Hills.

OBJECTIVE TYPE QUESTIONS Multiple Choice Questions Q. 1. In which state Garo, Khasi and Jaintia hills are R located?  (A) Himachal Pradesh (B) Assam (C) Meghalaya (D) Punjab Ans. Option (C) is correct. R Q. 2. What is the width of the bhabhar?  (A) 8 to 19 km (B) 8 to 16 km (C) 16 to 30 km (D) 8 to 26 km Ans. Option (B) is correct. Explanation: A narrow belt lying parallel to Shivalik is known as Bhabhar. Width of this belt is about 8 to 16 km.

(1 mark each)

Q. 3. Name the mountain range which bounds the R Central Highlands on the northwest.  (A) Western Ghats (B) Eastern Ghats (C) Vindhya Range (D) The Aravallis Ans. Option (C) is correct. Explanation: The Central Highlands is bounded in the North by the Northern Plains, in the west by the Aravallli Range and in the northwest by Vindhya Range. Q. 4. Which plateau lies between the Aravalli and the R Vindhyan Range? (A) Malwa Plateau (B) Deccan Plateau (C) Peninsular Plateau (D) Chota Nagpur Plateau Ans. Option (A) is correct.

82

Oswaal CBSE Question Bank Chapterwise & Topicwise, SOCIAL SCIENCE, Class-IX

R Q. 5. The highest peak in the Eastern Ghats is:  [NCERT] (A) Anai Mudi (B) Mahendragiri (C) Kanchenjunga (D) Khasi Ans. Option (B) is correct. Explanation: Mahendragiri is the highest peak of the Eastern Ghats. Its elevation is 1501 meters.

Assertion and Reason Directions: In the following questions, a statement of Assertion (A) if followed by a statement of Reason (R). Mark the correct choice as:

(A)  Both Assertion (A) and Reason (R) are true, and Reason (R) is the correct explanation of Assertion (A).



(B)  Both Assertion (A) and Reason (R) are true, but Reason (R) is not the correct explanation of Assertion (A).



(C) Assertion (A) is true, but Reason (R) is false.



(D) Assertion (A) is false, but Reason (R) is true.

Q. 1. Assertion (A): According to the variations in relief features, the Northern plains can be divided into four regions.

U



Reason (R): The rivers coming from southern mountains are involved in depositional work. ns. Option (C) is correct. A



Explanation: The Northern Plains can be divided into four regions. The Bhabar, Terai, Bhangar and Khadar and the rivers coming from Northern mountains are involved in depositional work.

Q. 2. Assertion (A): The Western ghats are higher than E the Eastern Ghats. 

Reason (R): Their average elevation is 700-1500 metres as against 500 metres of the Eastern Ghats. Ans. Option (C) is correct. Explanation: The Western Ghats are higher than the Eastern Ghats. Their average elevation is 900 to 1600 metres as against 600 metres of the Eastern Ghats. Q. 3. Assertion (A): With a rich soil cover combined with adequate water supply and favourable climate the Northern Plain is an agriculturally a productive U part of India.

Reason (R): The Northern Plain being 2400 km long and 240 to 320 km broad, is a densely populated physiographic division. Ans. Option (A) is correct. Explanation: Northern Plains have been formed by the flooding of three major rivers, i.e., the Indus, Ganga and Brahmaputra. These are pevennial river which flows throughout the year which makes soil rich for the agricultural Production in India. Because of its high fertility and most ideal for cultivation, these plains are densely populated.

SUBJECTIVE TYPE QUESTIONS Short Answer Type Questions Q. 1. Why are the Northern Plains agriculturally productive parts of India? Explain. U OR Describe any three characteristics of the Northern Plains of India. R  [Board Term I, 2016] Ans. The Northern Plains are agriculturally a productive part of India because of: (i) Rich and fertile alluvial soil cover. (ii) Adequate water supply. (iii) Favourable climate. (iv) Level land. [CBSE Marking Scheme, 2016] Q. 2. Broadly divide the Northern Plains on the basis of location. U Ans. The Northern Plains on the basis of location are divided into: (i) Punjab Plains: Formed by the Indus and its tributaries, the larger part of this Plain lies in Pakistan. This section of the plain is dominated by the Doabs.

(3 marks each)

(ii) Ganga Plains: It is spread over North India in Haryana, Delhi, U.P., Bihar, partly Jharkhand and West Bengal. (iii) The Brahmaputra Plains: They cover the areas of Assam and Arunachal Pradesh. (1 × 3 = 3)

Commonly Made Error

Students do not provide correct data pertaining to the extent of these plains.

Answering Tip

The students should classify all three broad

division of the Northern Plains and also practice map work for better understanding of relief features.

PHYSICAL FEATURES OF INDIA

Q. 3. Name the part of the Northern Plains formed of newer, younger deposits of the flood plains in India. Mention any two characteristics of it. U Ans. This part is known as Khadar. Characteristics: (i) It lies in flood plains. (ii) It is renewed almost every year. (iii) It is very fertile. (Any two) (1 + 2 = 3) Q. 4. Write any three differences between Bhangar and Khadar. OR Distinguish between Bhangar and Khadar. 

U [NCERT]

S. Bhangar Khadar No. 1. Formed of older Renewed every year. alluvium. 2. Lies above flood plains of Is newer, younger rivers. deposit of flood plains. 3.

Contains calcareous Ideal for intensive deposits locally known agriculture. as Kankar.

4.

Less fertile



More fertile

Q. 5. How was the ‘Peninsular Plateau’ formed? Name the two broad divisions of this plateau. Write one characteristic of each division. U Ans. Peninsular Plateau was formed due to the breaking and drifting of the Gondwana land. The two broad divisions of this plateau are the Central Highlands and the Deccan Plateau. The Central Highlands lies to the North of the Narmada River covering a major area of the Malwa Plateau. The Deccan Plateau is a triangular landmass that lies to the South of the river Narmada. (1+1+1=3)

Commonly Made Error

The students generally make mistakes in writing the answers point wise.

Answering Tip

Explain widely about the Peninsular Plateau

and its two broad divisions that are; the Central Highlands and the Deccan Plateau. Also discuss its stretch and special characteristics of the Peninsular Plateau.

(Any three) (1×3=3)

Long Answer Type Questions Q. 1. Name the two Ghats that mark the edges of the Deccan Plateau. Distinguish between the two by giving three characteristics of each. U [Board Term I, 2015] OR Describe the features of the Western Ghats and the Eastern Ghats in reference to height, slope, continuity, rivers and vegetation.  [(NCT 2014) Board Term I, 2013] OR Distinguish between Western Ghats and Eastern Ghats. (KVS, 2019) [NCERT] Ans. The two Ghats—the Western and the Eastern Ghats mark the edges of the Deccan Plateau. S. No 1.

2.

Western Ghats

Eastern Ghats

Their height is 900 —1,600 m. Anai Mudi (2,695 m) is the highest peak of the Western Ghats.

The Eastern Ghats are lower than the West. It ranges from 600—900 m. Mahendragiri (1,501 m) is the highest peak of this region.

They have steep slopes They have as height increases slope. from north to south.

a

gentle

83

(5 marks each)

3.

The Western Ghats are The Eastern Ghats are continuous and can be discontinuous, irregular crossed through the and dissected by rivers. passes only.

4.

They have evergreen to The Eastern Ghats have deciduous forests. scrub vegetation due to overgrazing and deforestation.

(Any three) (KVS Marking Scheme, 2019) (2+3=5) Q. 2. Distinguish between Bhabar and Terai. U [Board Term I, 2014] Ans. S. No 1.

Bhabar

Terai

It lies to the south of the Shivalik range.

The belt exist to the South of Bhabar area.

2.

The width ranges between 8 to 16 km.

It is almost parallel to the Bhabar.

3.

The area is highly coarse in nature due to many pebbles and ‘kankars’ found over here.

The area has highly fine sediments due to the deposition made by several streams.

84

Oswaal CBSE Question Bank Chapterwise & Topicwise, SOCIAL SCIENCE, Class-IX

4.

Vegetation found here is very less.

Very dense vegetation is found in Terai region.

5.

Main feature is that river disappears in the Bhabar region because big pores are present in it.

Since the river reemerges back in this region, the area becomes highly swampy and marshy.



(1×5=5)

Q. 3. Write a short note on the Northern Plains.  R  [Board Term I, NCT-2014]

OR Give an account of the Northern Plains of India.

[NCERT] Ans. The Northern Plains: (i) The Northern Plains have been formed from the alluvium deposited by the mountain rivers. (ii) They are located between the Himalayan Rivers in the North and the Peninsular Plateau in the South. (iii) They turned the soil fertile, on the land surface for growing a rich harvest of variety of crops. This led to the development of the Indus River Valley Civilization.

(iv) They are made up of deposits of alluvium and cover an area of 7 lakh sq. km. (v) They are densely populated physiographic division of India and are believed to be highly productive in terms of agriculture. (vi) The North Indian Plains or the Great Indians Plains have the Indus river system in the West and the Ganga–Brahmaputra river system in the East. (vii) On the basis of difference of relief, they are divided into four parts—Bhabar, Terai, Bhangar and Khadar. (Any five)

(1×5=5)



Commonly Made Error

Students usually discuss the reliefs of the Northern Plains.

Answering Tip

The students need to provide entire information related to formation, location, agricultural aspect, etc., about northern plains of India.

The Indian Desert , The Coastal Plains and The

Topic-3 Islands Revision Notes The Indian Desert

 The Indian Desert lies towards the western margins of the Aravalli Hills. It is an undulating sandy plain covered with sand dunes.  The region is characterized by arid climate, very low rainfall below 150 mm per year with scanty vegetation cover.

Scan to know more about this topic

The Indian Desert

 Luni is the only large river in this region.

The Coastal Plains  The Peninsular Plateau is flanked by stretch of narrow coastal strips, running along the Arabian Sea in the west and the Bay of Bengal in the east.

Scan to know more about this topic

 The Western Coast consists of three sections— Konkan Coast, Kannad Plain and Malabar Coast.  The Eastern Coast is divided into the Northern Circars and Coromandel Coasts.

The Islands  The small coral islands, the Lakshadweep Islands group, lies close to the Malabar Coast of Kerala.

The Coastal Plains

 Kavaratti Island is the administrative headquarters of Lakshadweep.  The elongated chain of islands extending from north to south is located in the Bay of Bengal. These are Andaman and Nicobar Islands.  These islands lies close to the Equator and experience equatorial climate and have thick forest cover.  India’s only active volcano is found on the Barren Island in Andaman and Nicobar group of Islands.

PHYSICAL FEATURES OF INDIA

85

Key Terms  Barchan: A crescent-shaped sand dune with the convex side in the direction of the wind.  Western Coastal Plain: A thin strip of coastal plain between the Western Ghats and the Arabian Sea.  Eastern Coastal Plain: A wide stretch of landmass of India, lying between the Eastern Ghats and the Bay of Bengal.  Coral polyps : Short-lived microscopic organisms, which live in colonies.

OBJECTIVE TYPE QUESTIONS Multiple Choice Questions Q. 1. The western coastal strip, south of Goa is referred U to as:  (A) Coromandel (B) Konkan (C) Kannad (D) Northern Circar Ans. Option (C) is correct. Explanation: The Western coastal strip south of Goa is referred to as Kannad, which – lies between Goa and Karnataka. Q. 2. A landmass bounded by sea on three sides is R [NCERT] referred to as:  (A) Coast (B) Island (C) Peninsula (D) None of the above Ans. Option (C) is correct. R Q. 3. Name the only active volcano in India.  (A) Majuli Island (B) Barren island (C) Venadu Island (D) Kutch Island Ans. Option (B) is correct. Explanation: Barren Island is the only active volcano found in Andaman and Nicobar Islands.

Assertion and Reason Directions: In the following questions, a statement of Assertion (A) if followed by a statement of Reason (R). Mark the correct choice as:

(1 mark each)



(A)  Both Assertion (A) and Reason (R) are true, and Reason (R) is the correct explanation of Assertion (A).



(B)  Both Assertion (A) and Reason (R) are true, but Reason (R) is not the correct explanation of Assertion (A).



(C) Assertion (A) is true, but Reason (R) is false.



(D) Assertion (A) is false, but Reason (R) is true.

Q. 1. Assertion (A): The Western coast, sandwiched between the Western Ghats and the Arabian Sea, U

is a narrow plain.

Reason (R): It consists of three sections. The Northern part of the coast is called the Konkan (Mumbai–Goa), the Central stretch is called the Kannad Plain, while the Southern stretch is referred to as the Malabar Coast. ns. Option (A) is correct. A Q. 2. Assertion (A): The mountains are the major sources of water and forest wealth.

E



Reason (R): The coastal region and island groups provide sites for fishing and port activities. Ans. Option (B) is correct. Explanation: The diverse physical features of the land have immense future possibilities of development.

SUBJECTIVE TYPE QUESTIONS Very Short Answer Type Questions Q. 1. Explain any two differences between the two R island groups of India. Ans. S. Andaman & No. Nicobar Islands (i) Andaman and Nicobar islands are group of islands lying away from the coast of Bay of Bengal.

Lakshadweep Islands Lakshadweep islands are group of islands lying close to the Malabar Coast of Kerala.

(2 marks each)

(ii) These group of islands These group of islands are bigger and are are smaller in size. more numerous and scattered. (iii) These islands are These group of islands formed from Oceanic are made up of small beds which are corals. known as submarine mountains. (Any Two Points) (1 x 2 = 2)

86

Oswaal CBSE Question Bank Chapterwise & Topicwise, SOCIAL SCIENCE, Class-IX

Q. 2. Define coral polyps. R Ans. Coral polyps are short-lived microscopic organisms, which live in colonies. They flourish in shallow, mud-free and warm waters. They secrete calcium carbonate. (2)

Q. 3. What do you know about Kavaratti Islands? U Ans. (i) Kavaratti island is the administrative headquarters of Lakshadweep. (ii) This island group has great diversity of flora and fauna. (2)

Short Answer Type Questions Q. 1. Name two coastal plains of India and describe any two features of each.  [Board Term I, 2016]

Q. 3. Explain the characteristics of the Indian Desert. Ans. The characteristics of the Indian Desert are: (i) Lies towards the western margins of the Aravallis. (ii) Undulating sandy plain covered with sand dunes.

OR

(3 marks each)

How are the Eastern Coastal plains of India different from the Western Coastal plains? State any three points of distinction. U

(iii) Receives low rainfall, below 150 mm per year.

Ans.

(iv) Low vegetation cover because of arid climate.

S. No. (i)

Western Coastal Plains

These lie between These lie between the the Western Ghats Eastern Ghats and the and the Arabian Sea. Bay of Bengal.

(ii)

It consists of three It consists of two sections, sections, i.e., Konkan i.e., Northern Circar and coast, Kannad plains Coromandel Coast. and Malabar coast.

(iii)

(v) Rain fed streams appear but soon disappear into the sand as there is inadequate water.

Eastern Coastal Pains

This coastal plain is This coastal plain is wide. narrow. (1×3=3)

(vi) River Luni is the only large river. (Any Three) (1×3=3) Q. 4. Name the four major states which are the parts of the Eastern Coastal plains. Mention the two divisions of these coastal plains. R

[Board Term I, DDE 2014]

Ans. The plains extend from the state of Tamil Nadu in the southern part to the state of West Bengal in the northern part. Other states include Andhra Pradesh and Odisha.

Q. 2. Which coastal plain lies along the Arabian Sea? State four features of it. U [Board Term I, 2015]



The two divisions of these coastal plains are:



(i) Northern Circars

Ans. The Western coastal plain lies along the Arabian Sea. Features of the Western Coast: (i) The Western Coast lies between the Western Ghats and the Arabian Sea. (ii) It is a narrow plain. (iii) The coastal plain consists of three sections. (iv) The northern part of the coast is the Konkan Coast (Mumbai—Goa). The central stretch is called the Kannad Plain, while the southern stretch is referred to as the Malabar Coast. [CBSE Marking Scheme, 2015] 1+2=3

(ii) Coromandel Coast (2+1=3) Q. 5. Name the major island groups lying in the Arabian Sea. Explain any two major features of it.  R [Board Term I, 2013]

Commonly Made Error

The students get confused in the stretch of the coastal plains.

Answering Tip

Since, Arabian Sea is in West, the Western Coast will lie along the Arabian Sea.

OR Name the Island group located in the Arabian Sea. State any two features of these Islands. OR Name the island group of India having coral origin. [NCERT] Ans. Lakshadweep Islands group is located in the Arabian Sea. Features : (i) This group of Islands is made up of small corals. Lakshadweep is famous for corals. (ii) Earlier they were known as Laccadive, Minicoy and Amindivi. (1 + 2 = 3)

PHYSICAL FEATURES OF INDIA

COMPETENCY BASED QUESTIONS

I. Read the source given below and answer the U (1+1+1+1=4) questions that follow: The Himalayas, geologically young and structurally fold mountain stretch over the northern borders of India. These mountain ranges run in a west-east direction from the Indus to the Brahmaputra. The Himalayas represent the loftiest and one of the most rugged mountain barriers of the world. They form an arc, which covers a distance of about 2,400 Km. Their width varies from 400 Km in Kashmir to 150 Km in Arunachal Pradesh. The altitudinal variations are greater in the eastern half than those in the western half. The Himalaya consists of three parallel ranges in its longitudinal extent. A number of valleys lie between these ranges. The northern-most range is known as the Great or Inner Himalayas or the Himadri. It is the most continuous range consisting of the loftiest peaks with an average height of 6,000 metres. It contains all prominent Himalayan peaks. The folds of the Great Himalayas are asymmetrical in nature. The core of this part of Himalayas is composed of granite. It is perennially snow bound, and a number of glaciers descend from this range. how

old

are

(4 marks each)

4. The Himalayas consists of ............... parallel ranges.

Case based MCQs

1. Geologically Mountains?

the

Himalayan

(A) two

(B) four

(C) three

(D) six

Ans. Option (C) is correct.

Explanation: The Himalayas consist of three parallel ranges Himadri, Himachal and Shivaliks. II. Read the source given below and answer the E (1+1+1+1=4) questions that follow: The Indian desert lies towards the western margins of the Aravalli Hills. It is an undulating sandy plain covered with sand dunes. This region receives very low rainfall below 150 mm per year. It has arid climate with low vegetation cover. Streams appear during the rainy season. Soon after they disappear into the sand as they do not have enough water to reach the sea. Luni is the only large river in this region. Barchans (crescent-shaped dunes) cover larger areas but longitudinal dunes become more prominent near the Indo- Pakistan boundary. If you visit Jaisalmer, you may go to see a group of barchans.

1. The Indian Desert lies towards the western margins of: (A) Aravalli Hills (B) Nilgiri Hills (C) Shivalik Hills (D) Mahendragiri Hills

(A) Young and fold

(B) Ancient and fold

Ans. Option (A) is correct.

(C) Old and fold

(D) Modern and fold



2. The Indian Desert is also known as:

Ans. Option (A) is correct.

(A) Gobi Desert

(B) Sahara Desert



(C) Thar Desert

(D) Sonoran Desert

Explanation: The Himalayas are considered geologically young and structurally fold mountains because they have been formed few million years ago.

Ans. Option (C) is correct.

2. The Northern-most range of Himalayan Mountains is called:

Explanation: Thar Desert is an undulating sandy plain covered with sand dunes. It receives very low rainfall below 150 mm per year. It has arid climate with low vegetation cover.

3. This region receives very low rainfall below ............... per year. (A) 150 mm (B) 150 cm (C) 100 mm (D) 50 cm Ans. Option (A) is correct.

(A) Greater Himalayan (B) Inner Himalayan (C) Himadri (D) All the above Ans. Option (D) is correct. 3. The range lying to the ............... of the Himadri forms the most rugged mountain system called Himachal. (A) east

(B) north

(C) south

(D) west

Ans. Option (C) is correct.

87



Explanation: The Indian Desert receives very low rainfall because it is located near the Equator.

4. Barchans are ............... shaped dunes. (A) round

(B) crescent

(C) semi-circle

(D) hexagon

Ans. Option (B) is correct.

88

Oswaal CBSE Question Bank Chapterwise & Topicwise, SOCIAL SCIENCE, Class-IX

Subjective based Questions I. Read the source given below and answer the A (1+1+2=4) questions that follow: The Ganga plain extends between Ghaggar and Teesta rivers. It is spread over the states of North India, Haryana, Delhi, U.P., Bihar, partly Jharkhand and West Bengal to its East, particularly in Assam lies the Brahmaputra plain. The northern plains are generally described as flat land with no variations in its relief. It is not true. These vast plains also have diverse relief features. According to the variations in relief features, the Northern plains can be divided into four regions. The rivers, after descending from the mountains, deposit pebbles in a narrow belt of about 8 to 16 km in width lying parallel to the slopes of the Shivaliks. It is known as bhabar. All the streams disappear in this bhabar belt. South of this belt, the streams and rivers re-emerge and create a wet, swampy and marshy region known as Terai. This was a thickly forested region full of wildlife. The forests have been cleared to create agricultural land and to settle migrants from Pakistan after partition. Dudhwa National Park is in this region.

4. The Northern Plains have been formed by the interplay of the three major rivers systems, with their tributaries. Name them. Ans. The Indus, the Ganga and the Brahmaputra . II. Read the source given below and answer the U (1+1+2=4) questions that follow: The Peninsular Plateau is flanked by stretch of narrow coastal strips, running along the Arabian Sea in the west and the Bay of Bengal in the east. The western coast, sandwiched between the Western Ghats and the Arabian Sea, is a narrow plain. It consists of three sections. The northern part of the coast is called the Konkan (Mumbai - Goa), the central stretch is called the Kannad Plain, while the southern stretch is referred to as the Malabar coast. The plains along the Bay of Bengal are wide and level. In the northern part, it is referred to as the Northern Circar, while the southern part is known as the Coromandel Coast. Large rivers, such as the Mahanadi, the Godavari, the Krishna and the Kaveri have formed extensive delta on this coast. Lake Chilika is an important feature along the eastern coast. 1. Which lake is an important feature along the Eastern coast?

1. The Ganga Plain extends between which two rivers?

Ans. Lake Chilika is an important feature along the Eastern Coast.

Ans. The Ganga plain extends between Ghaggar and Teesta rivers.

2. What is the Northern part of the Western Coast called?

2. How are the northern plains generally described?

Ans. The Konkan Coast 3. Which four rivers form the extensive delta in this coast?

Ans. The northern plains are generally described as flat land with no variations in its relief. 3. What is Bhabar? Ans. Bhabar is a narrow belt of about 8 to 16 km in width lying parallel to the slopes of the Shivaliks.

Ans. The Mahanadi, the Godavari, the Krishna and the Kaveri form the extensive delta in Coromandel Coast plains.

Map Work

Syllabus

l Mountain Ranges: The Karakoram, The Zaskar, The Shivalik, The Aravali, The Vindhya, The Satpura, Western & Eastern Ghats l Mountain Peaks – K2, Kanchan Junga, Anai Mudi l Plateau - Deccan Plateau, Chotta Nagpur Plateau, Malwa Plateau; l Coastal Plains - Konkan, Malabar, Coromandal & Northern Circar (Location and Labelling);

PHYSICAL FEATURES OF INDIA

89



Study Time Max Time: 1:30 hr Max Questions: 46

CHAPTER

6

Learning Objectives

DRAINAGE Justify the statement that the rivers are lifeline of economy with reference to India. l Examine the information about different lakes and infer on their contribution to Indian ecology. l Distinguish between the rivers of north and south India. l Analyse the flow of different rivers of India to infer on their impact on livelihood. l

Topic-1 Major Rivers and Tributaries Revision Notes

TOPIC - 1 Major Rivers and Tributaries .... P. 90 TOPIC - 2 Lakes .... P. 96 TOPIC - 3 Role of Rivers in the Economy & River Pollution .... P. 98

Drainage Patterns  The term Drainage describes the river system of an area.  The area drained by a single river system is called a Drainage Basin.  Any elevated area, such as a mountain or upland, separates two drainage basins. Such upland is known as a Water Divide.  The world’s largest Drainage Basin is that of the Amazon River.

Drainage System in India  The Drainage systems of India are mainly controlled by the broad relief features of the subcontinent.  The Indian rivers are divided into two major groups: the Himalayan rivers and the Peninsular rivers.

(i) The Himalayan Rivers

 The major Himalayan rivers include the Indus, the Ganga and the Brahmaputra. These rivers are long and are joined by many large and important tributaries.  Most of the Himalayan rivers are perennial, therefore, they have water throughout the year.



(a) The Ganga River System

 The headwaters of the Ganga, called the Bhagirathi, is fed by the Gangotri Glacier and joined by the Alaknanda at Devaprayag in Uttarakhand.  The Ganga is joined by many tributaries from the Himalayas, such as the Yamuna, the Ghaghara, the Gandak and the Kosi.  The delta formed by the Ganga and the Brahmaputra rivers is known as the Sunderban Delta.  The length of the river Ganga is over 2,500 kms.

(b) The Brahmaputra River System  The Brahmaputra rises in Tibet, east of Mansarovar Lake, very close to the sources of the Indus and the Satluj.

DRAINAGE

91

92

Oswaal CBSE Question Bank Chapterwise & Topicwise, SOCIAL SCIENCE, Class-IX

 Brahmaputra is known as Tsang Po in Tibet and Jamuna in Bangladesh.  The Brahmaputra has a braided channel in its entire length in Assam and forms many Riverine Islands.  Every year, during the rainy season, the river overflows its banks, causing widespread devastation due to floods in Assam and Bangladesh.

(ii) The Peninsular Rivers  The main water divide in Peninsular India is formed by the Western Ghats, which runs from North to South, close to the Western Coast. A large number of Peninsular rivers are seasonal, as their flow is dependent on rainfall.  Major rivers of the peninsula are the Narmada and the Tapi, Mahanadi, Godavari, Krishna and Kaveri. They flow eastwards and drain into the Bay of Bengal. These rivers make deltas at their mouths.  The Narmada and the Tapi are the only long rivers, which flow westward and make estuaries.

(a) The Narmada Basin  The Narmada rises in the Amarkantak Hills in Madhya Pradesh.  The Narmada basin covers parts of Madhya Pradesh and Gujarat.

(b) The Tapi Basin  The Tapi rises in the Satpura ranges, in the Betul district of Madhya Pradesh.  Its basin covers parts of Madhya Pradesh, Gujarat and Maharashtra.

Scan to know more about this topic

(c) The Godavari Basin  The Godavari is the largest Peninsular river.  It rises from the slopes of the Western Ghats in the Nashik District of Maharashtra. Its length is about 1,500 km.

The Himalayan Rivers

 The drainage basin of the Godavari River is the largest among the Peninsular rivers.  Important tributaries of the Godavari are the Purna, Wardha, Pranhita, Manjra, Wainganga and Penganga.  The Godavari River is also known as the ‘Dakshin Ganga’ because of its length and the area it covers.

(d) The Mahanadi Basin  The Mahanadi rises in the highlands of Chhattisgarh.  It flows through Odisha to reach the Bay of Bengal. (e) The Krishna Basin  Rising from a spring near Mahabaleshwar, the Krishna flows for about 1,400 km and reaches the Bay of Bengal.  Tungabhadra, Koyana, Ghatprabha, Musi and Bhima are some of its tributaries.

(f) The Kaveri Basin  The Kaveri rises in the Brahmagri range of the Western Ghats and it reaches to the Bay of Bengal.  The main tributaries are the Amravati, the Bhavani, the Hemavati and the Kabini.  Its basin drains parts of Karnataka, Kerala and Tamil Nadu.

Key Terms  Perennial: It is a stream or river that exhibits continuous flow of water throughout the year.  Dendritic: The stream with its tributaries which resembles the branches of a tree. Thus, the name Dendritic.  Trellis: A river joined by its tributaries, at approximately right angles, develops a Trellis pattern.  Rectangular: A drainage pattern that develops on a strongly joint rocky terrain.  Radial: The Radial pattern develops when streams flow in different directions from a central peak or domelike structure.  Tributary: A river or stream that flows into a larger river or lake.  Estuary: The tidal mouth of a large river, where the tide meets the stream.

DRAINAGE

OBJECTIVE TYPE QUESTIONS Multiple Choice Questions Q. 1. Yamuna, Ghaghara and Kosi are important R tributaries of which river?  (A) Ganga (B) Kaveri (C) Indus (D) Satluj Ans. Option (A) is correct. Q. 2. The river Narmada has its source at: R [NCERT] (A) Satpura (B) Brahmagiri (C) Amarkantak (D) Slopes of the Western Ghats Ans. Option (C) is correct. Explanation: The source of the Narmada is a small reservoir, known as the Narmada Kund. It is located at Amarkantak on the Amarkantak Plateau in the Anuppur District on Shahdol zone of eastern Madhya Pradesh. Q. 3. Which one of the following is the longest river of R [NCERT] the Peninsular India? (A) Narmada (B) Krishna (C) Godavari (D) Mahanadi Ans. Option (C) is correct.

(1 mark each)



(A) Both (A) and (R) are true and (R) is the correct explanation of (A).



(B) Both (A) and (R) are true and (R) is not the correct explanation of (A).



(C) (A) is correct but (R) is wrong.



(D) (A) is wrong but (R) is correct.

Q. 1. Assertion (A): The Ganga is joined by many tributaries from the Himalayas, a few of them being major rivers, such as the Yamuna, the Ghaghara, the Gandak and the Kosi. 

U



Reason (R): The Ghaghara, the Gandak and the Kosi rise in the Nepal Himalaya. Ans. Option (A) is correct. Q. 2. Assertion (A): The Himalayan rivers perform intensive erosional activity in their upper courses and carry huge loads of silt and sand.

E



Reason (R): The Himalayan rivers have long courses from their source to the sea. Ans. Option (A) is correct. Q. 3. Assertion (A): A large number of the Peninsular Rivers are seasonal. 

U



Assertion and Reason

93

In the questions given below, there are two statements marked as Assertion (A) and Reason (R). Read the statements and choose the correct option.

Reason (R): Their flow is dependent on glacier melt. Ans. Option (C) is correct. Explanation: A large number of the Peninsular Rivers are seasonal, as their flow is dependent on rainfall.

SUBJECTIVE TYPE QUESTIONS Very Short Answer Type Questions Q. 1. Name the two head streams of the Ganga. Where do they meet to form the Ganga?

U

[NCERT] Ans. (i) Alaknanda and Bhagirathi are the two head streams of Ganga.

(2 marks each)

(ii) In Tibet, it carries a smaller volume of water and less silt, but in India, it carries a large volume of water and a considerable amount of silt.

(1 x 2 = 2)

(1 x 2 = 2)

Q. 3. In which state of India, river Brahmaputra has a braided channel in its entire length? Why does this river carry a large volume of water and silt in

Q. 2. Explain any two main features of the Brahmaputra River System. R

Ans. The Brahmaputra has a braided channel in its entire length in the state of Assam.

(ii) They join at Devprayag in Uttarakhand.

Ans. Characteristics of the Brahmaputra River System: (i) The Brahmaputra rises in Tibet, flows eastwards parallel to the Himalayas and enters India in the state of Arunachal Pradesh. At this point, it is joined by Dibang and Lohit.

India compared to Tibet?



E

This river carries a larger volume of water and silt in India, as here, it passes through a region of high rainfall, whereas, in Tibet, the river carries a smaller volume of water and less silt, as it is a cold and dry area. (1 + 1 = 2)

94

Oswaal CBSE Question Bank Chapterwise & Topicwise, SOCIAL SCIENCE, Class-IX

Q. 4. Differentiate between the Godavari River System. Ans. S. No.

Ganga River System

Ganga

and

the R

Godavari River System

(iv)

Its tributaries are: Its tributaries are: Purna, Ghaghara, Kosi, Wardha, Pranhirta, etc. Gandak, Son, Betwa, Chambal, etc.



(Any Two) (1 x 2 = 2)

(i)

It is a Himalayan river. It is a Peninsular river.

Q. 5. How do the Himalayan Rivers perform erosional

(ii)

At Haridwar, the It rises from the slopes Ganga emerges from of the Western Ghats the mountains onto in Nashik district of the plains Maharashtra.

Ans. (i) In the upper course, rivers flow with great speed from higher to lower levels. They cause erosion.

(iii) The length of the Its length is 1,500 km. Ganga River System is over 2,500 km.

activities and form depositional features?

(ii) In the middle and the lower course, they carry lots of sediment like silt and sand. Hence, the speed slows down. Besides, absence of slope in the lower course causes deposition and formation of various features.  (1 x 2 = 2)

Short Answer Type Questions Q. 1. Which river is known as the Dakshin Ganga? State any two characteristics of it. R  [Board Term I, 2016, 2013] OR Why is the Godavari River also known as ‘Dakshin Ganga’? [Board Term I, 2015] Ans. The Godavari. Characteristics are: (i) It rises from the slopes of the Western Ghats in Nashik District of Maharashtra. (ii) Length is about 1,500 km. (iii) The basin covers Maharashtra, Madhya Pradesh, Odisha and Andhra Pradesh. (iv) Tributaries are the Purna, the Wardha, the Pranhita, the Manjra, the Wainganga and the Penganga. (Any two) [CBSE Marking Scheme, 2016] (1+2=3)

Students generally forget to write actual length of the river and name of the states covered.

Q. 3. Differentiate between the Narmada and Tapi Basin.  U [Board Term I, 2016, 2014] S. No.

Dakshin implies to South and Godavari River is the longest river of South.

Q. 2. Describe any three important features of the Mahanadi Basin. R [Board Term I, 2016] Ans. (i) The Mahanadi rises in Chhattisgarh. (ii) It flows through Odisha, for 860 km till it falls into the Bay of Bengal.

Narmada Basin

Tapi Basin

(i)

Narmada River originates from the Amarkantak Hills.

(ii)

It is very long and it It is very short in length as flows towards West compared to the Narmada and joins Arabian Sea. River.

Tapi River originates from the Satpura Ranges (in Betul district).

(iii) It covers part of It covers parts of Madhya Madhya Pradesh and Pradesh, Gujarat and Gujarat. Maharashtra.

(1×3=3)

Commonly Made Error

Students mostly define basin and give examples of tributaries of the two rivers.

Answering Tip

Answering Tip

(3 marks each)

(iii) Its basin is shared by Maharashtra, Chhattisgarh, Jharkhand and Odisha. (1 × 3 = 3)  [CBSE Marking Scheme, 2016]

Commonly Made Error

E



Discuss the rivers and the regions covered by them.

Q. 4. Why does river Brahmaputra has less silt in Tibet than India? Name the state where it forms Riverine Island. A [Board Term I, 2016] Ans. Brahmaputra has less silt in Tibet, as it is a cold and dry area. In India, the river carries a large volume

DRAINAGE

of water and considerable silt because it passes through a region of heavy rainfall. The Brahmaputra has a braided channel in its entire length in Assam and forms many Riverine Islands there. (1½+1½=3) Q. 5. Differentiate between the West and East flowing Peninsular Rivers.  U [Board Term I, DDE-2014] OR Compare the East-flowing and West-flowing rivers of the Peninsular Plateau. [NCERT]

Ans. S. No.

East-flowing Rivers

East flowing rivers are : West flowing rivers Mahanadi, Godavari, are: Narmada, Luni, Krishna and Kaveri. Ghaggar and Tapi.

(ii)

They drain into the They originate in the Bay of Bengal. central part and drain into the Arabian Sea.

(iii) They are numerous They are numerous in number and form in number and form deltas at their mouth. estuaries at their mouth. They have numerous They are devoid of any tributaries. remarkable tributary.



(Any three) (1 × 3 = 3)

Long Answer Type Questions

(iii) These rivers are not very developed. (iv) These rivers flow in troughs. (v) Peninsular rivers are not navigable because a large number of these rivers are seasonal. The flow of the Peninsular rivers is dependent upon rainfall and they generally dry up in summer season. River beds are uneven, rocky and have sharp edges. Construction of dams has also made navigation difficult.

[CBSE Marking Scheme, 2016] (1 × 5 = 5)

Q. 2. Discuss the significant differences between the Himalayan and the Peninsular Rivers. U [Board Term I, 2014] [NCERT] Ans. S. No. (i)

The Himalayan Rivers

The Peninsular Rivers

The Himalayan Rivers rise in the snowcovered mountains and they flow throughout the year.

The mountains in which the Peninsular Rivers rise are not snow covered. Hence, they dry up in summer.

(5 marks each)

S. No.

The Himalayan Rivers

The Peninsular Rivers

(ii)

The Himalayan Rivers flow in levelled Northern Plains. Therefore, they are quite useful for navigation and irrigation.

The Peninsular Rivers flow on uneven rocky surface. Therefore, they are neither navigable nor useful for irrigation.

(iii) The Himalayan Rivers bring with them fertile alluvium which they deposit in the IndoGangetic plains.

The Peninsular Rivers do not bring with them enough alluvium. As the current is swift, the deposition activity is negligible.

Ans. (i) The coastal plains between the Western Ghats and the Arabian Sea are very narrow. Hence, the coastal rivers are short. (ii) The main West flowing rivers are the Sabarmati, Mahi, Bharathapuzha, Periyar, Narmada, Tapi, etc.

West-flowing Rivers

(i)

(iv)

Q. 1. From the point of view of agriculture, western Peninsular rivers are less important and moreover these are not navigable. Analyse the reasons behind it.  U [Board Term I, 2016]

95

(iv) Canals have been dug As the terrain is rocky to use the water of these and the banks of rivers for irrigation. these rivers are high, canals cannot be dug. However, dams are built to store the flood water for irrigation with the help of small channels. (v)

Many important towns and centres of trade are situated on the banks of these rivers.

Very few important towns and centres of trade are situated on the banks of these rivers.

(vi) The porous soil absorbs a lot of water which is later used as ground water by digging wells and tube wells for domestic and irrigation purposes. 

The rocky soil does not absorb any water and hence no wells can be dug. All the water flows down the sea at once and at the same time. (Any five) (1×5=5)

96

Oswaal CBSE Question Bank Chapterwise & Topicwise, SOCIAL SCIENCE, Class-IX

Q. 3. Define water divide. What are the different patterns of flow made by a river? U  [Board Term I, NCT 2014] OR Describe the four drainage patterns. Ans. An elevated area such as a mountain or an upland that separates two drainage basins is called Water Divide. The four drainage patterns are as follows: (i) Dendritic: The dendritic pattern develops where the river channels follow the slope of the terrain. The stream with its tributaries

resembles the branches of a tree, thus, the name dendritic. (ii) Trellis: A river joined by its tributaries, at approximately right angles, develops a trellis pattern. A trellis drainage pattern develops where hard and soft rocks exist parallel to each other. (iii) Rectangular: A rectangular drainage pattern develops on a strongly jointed rocky terrain. (iv) Radial: The radial pattern develops when stream flows in different directions from a central peak or dome-like structure. 

(1 + 4 = 5)

Topic-2 Lakes Revision Notes Lakes  Lake is a large area of water surrounded by land. Lakes of large extent are called the seas, like the Caspian, the Dead and the Aral Sea.

Scan to know more about this topic

 India has many lakes. These differ from each other in the size and other characteristics. Most lakes are permanent, while others contain water only during the rainy season.  Lakes are formed by the following processes: the action of glaciers and ice sheets, by wind, river action and human activities.

Wular Lake

 Most of the freshwater lakes are in the Himalayan region. They are of glacial origin.  The Wular Lake in Jammu and Kashmir is the largest freshwater lake in India.  Apart from natural lakes, the damming of the rivers for the generation of hydel power has also led to the formation of lakes such as Guru Gobind Sagar Lake (Bhakra Nangal Project).  Importance of lakes:





A lake helps to regulate the flow of a river.







During heavy rainfall, it prevents flooding, and during the dry season, it helps to maintain an even flow of water.







Lakes can also be used for developing hydel power.







They moderate the climate of the surroundings; maintain the aquatic ecosystem, enhance natural beauty, help develop tourism and provide recreation.

Key Terms  Ox-bow lake: It is a U- shaped water body formed when a meandering river is cut off from the mainstream.  Lagoon: When the lake is formed by spits and bars in coastal areas, it is called a lagoon.  Glacial lake: A lake formed by melting of the glacier is called a glacial lake.

DRAINAGE

OBJECTIVE TYPE QUESTIONS

Assertion and Reason

In the questions given below, there are two statements marked as Assertion (A) and Reason

(1 mark each)

(R). Read the statements and choose the correct option.

Multiple Choice Questions Q. 1. Which one of the following lakes is a salt water R [NCERT] lake?  (A) Sambhar (B) Wular (C) Dal (D) Gobind Sagar Ans. Option (A) is correct. Explanation: The Sambhar lake in Rajasthan, is a salt water lake. Its water is used for producing salt. Q. 2. In which of the following states is the Wular lake R [NCERT] located?  (A) Rajasthan (B) Uttar Pradesh (C) Punjab (D) Jammu and Kashmir Ans. Option (D) is correct. Q. 3. Lakes of large extent are called the seas, like: R (A) the Caspian. (B) the Dead. (C) the Aral seas. (D) All of the Above Ans. Option (D) is correct.

97



(A) Both (A) and (R) are true and (R) is the correct explanation of (A).



(B) Both (A) and (R) are true and (R) is not the correct explanation of (A).



(C) (A) is correct but (R) is wrong.



(D) (A) is wrong but (R) is correct.

Q. 1. Assertion (A): Lakes can also be used for E

developing hydel power. 

Reason (R): Guru Gobind Sagar is a natural lake. Ans. Option (C) is correct. Explanation: Gobind Sagar lake is a man made reservoir built in 1976; its source is the hydel dam at Bhakra. Q. 2. Assertion (A): Most of the fresh water lakes are in A

the Himalayan region. Reason (R): They are of glacial origin. Ans. Option (A) is correct.

Explanation: Wular Lake is one of the largest fresh water lake, located in the state of Jammu & Kashmir.

SUBJECTIVE TYPE QUESTIONS Short Answer Type Questions Q. 1. How are man-made lakes created? For what purpose are they used? Give an example. U  [Board Term I, 2015] Ans. Causes of creating man-made lakes: (i) Man-made lakes are created for hydro-electric power supply, recreational purposes, industrial use, agricultural use or domestic water supply. (ii) Man-made lakes are formed by stopping the natural flow of a river by constructing dams resulting into collection of water on one side. The Gobind Sagar Lake is an example of manmade lake.  [CBSE Marking Scheme, 2015] (2+1=3) Q. 2. ‘Lakes are of great value to human beings.’ Explain the statement with three examples. U [Board Term I, 2015] OR How can you develop lakes as a source of employment? [Board Term I, DDE-2014] OR State three economic benefits of lakes. [Board Term I, 2013] [NCERT]

(3 marks each)

Ans. (i) (ii) (iii)

Importance of Lakes: Help to regulate the flow of a river. Prevent flooding. They help to maintain even flow of water during dry season. (iv) They can be used to develop hydel power. (v) Moderate the climate of surroundings. (vi) Maintain the aquatic ecosystem. (vii) Help to develop tourism and provide recreation facilities. (viii) Can be used in irrigation and other agricultural activities. (Any three) [CBSE Marking Scheme, 2015] (1×3=3) Q. 3. How does a man-made lake help a river to regulate its flow? U [Board Term I, 2014] Ans. Man-made lakes help a river to regulate its flow in the following manner: (i) Man-made lakes are generally created by construction of dams. (ii) The reservoirs thus created help to control the flow of the rivers.

98

Oswaal CBSE Question Bank Chapterwise & Topicwise, SOCIAL SCIENCE, Class-IX

(iii) They help in regulating the flow of water to be released to downstream regions as per the requirement. (iv) In this manner, they also help in the prevention of flooding. (Any three) (1×3=3) Q. 4. Below are given names of a few lakes of India. Group them under two categories, natural and created by human beings. U [NCERT]

OR Make a list of natural and artificial lakes with the help of an atlas. [NCERT] Ans.

S.No.

Natural Lakes

Man-Made Lakes

(i)

Wular

Gobind Sagar

(ii)

Dal

Rana Pratap Sagar

(a) Wular

(b) Dal

(iii)

Nainital

Nizam Sagar

(c) Nainital

(d) Bhimtal

(iv)

Bhimtal

Nagarjuna Sagar

(e) Gobind Sagar

(f) Loktak

(v)

Loktak

Hirakund

(g) Barapani

(h) Chilika

(vi)

Barapani

(i) Sambhar

(j) Rana Pratap Sagar

(vii)

Chilika

(k) Nizam Sagar

(l) Pulicat

(viii)

Sambhar

(m) Nagarjuna Sagar

(n) Hirakund

(ix)

Pulicat

Topic-3 Role of Rivers in the Economy and River Pollution Revision Notes Role of Rivers in the Economy  Rivers are of great value to human beings:





Rivers have been of fundamental importance throughout the human history.







Water from the rivers is a basic natural resource, essential for various human activities.







The river banks have attracted settlers from ancient times.







Using rivers for irrigation, navigation, hydro-power generation is of special significance – particularly to a country like India, where agriculture is the major source of livelihood of the majority of its population.

Scan to know more about this topic

what Causes RIver Pollution

River Pollution  The growing domestic, municipal, industrial and agricultural demand for water from rivers naturally affects the quality of water.





As a result more and more water is being drained out of the rivers reducing their volume.







On the other hand, a heavy load of untreated sewage and industrial effluents are emptied into the rivers.







This affects not only the quality of water but also the self-cleaning capacity of the river.

 The increasing urbanisation and industrialisation are responsible for the rising pollution level of many rivers.  Concern over rising pollution in our rivers led to the launching of various action plans to clean the rivers.

Key Terms  Irrigation: Supply of water for agriculture purpose from canals, wells and tanks artificially or manually.  Hydro-power: Power derived from running or falling water.

DRAINAGE

99

Example

Describe the major steps taken by the Government of India to control the pollution level in the River Ganga. Answer: Step 1: Major steps taken by Government of India to control the pollution level in the River Ganga. Step 2: Treating of waste before dumping in the river. Step 3: Taking projects to control river water pollution like Ganga Action Plan. The activities of Ganga Action Plan (GAP) Phase-I, initiated in 1985, were declared closed on 31st March, 2000.

Step 4: The Steering Committee of the National River Conservation Authority reviewed the

progress of the GAP and did necessary correction on the basis of lesson learnt and experiences gained from GAP Phase-I. These have been applied to the major polluted rivers of the country under the NRCP. The Ganga Action Plan (GAP) Phase-II, has been merged with the NRCP. The expanded NRCP now covers 152 towns located along 27 interstate rivers in 16 states.

Step 5: Under this action plan, pollution abatement works are being taken up in 57 towns. A total of 215 schemes of pollution abatement have been sanctioned. So far, 69 schemes have been completed under this action plan. A million litres of sewage is targeted to be treated.

OBJECTIVE TYPE QUESTIONS Multiple Choice Questions Q. 1. Water from the rivers is a basic _______ resource, U essential for various human activities. (A) natural (B) man made (C) extracted (D) None of the Above Ans. Option (A) is correct. Q. 2. The activities of Ganga Action Plan (GAP) phase-I, R initiated in: (A) 1985 (B) 1984

(1 mark each)

(C) 1986 (D) 1989 Ans. Option (A) is correct. Explanation: Ganga Action Plan (GAP) is an initiative taken by the Government of India to control the pollution level of River Ganga. Ganga Action Plan (GAP) phase I, initiated in 1985. Q. 3 The Ganga water is able to dilute and assimilate pollution loads within ____ km of large cities. R (A) 30 (B) 20 (C) 10 (D) 5 Ans. Option (B) is correct.

SUBJECTIVE TYPE QUESTIONS Short Answer Type Questions Q. 1. What is self-cleansing capacity of a river? How does it get affected by pollution? U [Board Term I, 2016] Ans. (i) A fast-flowing river with lots of volume of water can dilute pollution load. This is known as the self-cleansing capacity of the river. (ii) Pollution slows the speed of rivers affecting self cleansing capacity of the river. (iii) Pollution increases the volume of load in the river. [CBSE Marking Scheme, 2016] (1 × 3 = 3) Q. 2. Why is pollution rate of Ganga River increasing day by day? U [Board Term I, 2016] Ans. The pollution rate of Ganga River is increasing because: (i) Heavy load of untreated sewage and industrial effluents is being emptied into the Ganga. (ii) Increasing urbanization has raised the pollution level of the river.

(3 marks each)

(iii) Domestic, municipal, industrial and agricultural wastage are emptied into the river.  [CBSE Marking Scheme, 2016] (1 × 3 = 3) Q. 3. “Rivers have been of fundamental importance throughout the human history.” Justify this statement in three points. U [Board Term I, 2015] Ans.  Rivers have been of fundamental importance throughout the human history: (i) Water is a basic natural resource, essential for various human activities. (ii) Therefore, river banks attracted settlers since ancient times. These settlements have now become big cities. (iii) Use of rivers has been taking place for irrigation, navigation, hydro-power generation and trade since ages. [CBSE Marking Scheme, 2015] (1×3=3)

100

Oswaal CBSE Question Bank Chapterwise & Topicwise, SOCIAL SCIENCE, Class-IX

Q. 4. What is meant by Drainage? Explain any four benefits of river. U [Term I, DDE 2014] OR State some economic benefits of rivers. [NCERT] Ans. The River System of an area is called Drainage. Benefits of River: (i) Rivers provide water for survival and growth of all organisms. (ii) Rivers provided ideal conditions for the early man to lead a settled life. (iii) Rivers have built flood plains, deltas and provide fertile soil for agriculture. (iv) Water from rivers is used for drinking, irrigation and for generating hydro-electricity. (v) Man is able to supplement his food supply with the fish in the rivers. Rivers also serve as natural waterways. (vi) The world’s earliest civilizations developed in the river valleys, e.g., The Nile Valley Civilization, The Indus Valley Civilization, etc. river valleys are the world’s best agricultural lands and are densely populated regions. (Any four) (1+ 4×½=3 )

Answering Tip

Commonly Made Error



of river.

Long Answer Type Questions

Ans. The Steering Committee of the National River Conservation Authority reviewed the progress of the GAP and the necessary corrections on the basis of the lessons learnt and experiences gained from GAP phase – I. These have been applied to the major polluted rivers of the country under NRCP due to high pollution level in maximum rivers. The Ganga Action Plan (GAP) Phase – II has been merged with the NRCP. The expanded NCP now covers 152 towns located along 27 Interstate rivers in 16 states. Under this action plan, pollution abatement works are being taken up in 57 towns. Achievements: A total of 215 schemes of pollution abatement have been sanctioned. So far, 69 schemes have been completed under this action plan. A million litres of sewage is targeted to be treated. [CBSE Marking Scheme, 2015] (5)

Students mention the basic reasons of pollution.

Answering Tip

Students generally write the common benefits

Q. 1. The activities of Ganga Action Plan (GAP) Phase – I, initiated in 1985 were declared merge with National River Conservation Plan (NRCP). Why? Write its major achievements. U [Board Term I, 2015]

importance since ancient times and the major civilizations that developed around the rivers.

Q. 5. Explain any three causes of River Pollution in India. U Ans. Three causes of River pollution are: (i) Growing domestic, municipal, industrial and agricultural demand for water affects the quality of water in the rivers. (ii) A heavy load of untreated sewers and industrial effluents further increases river pollution. (iii) Increasing urbanization and industrialization. (1×3=3)

Commonly Made Error

Mention the major points, rivers have been of

Major cause of river pollution deals with urbanization and industrialization.

(5 marks each)

Q. 2. What is water pollution? Describe the main sources of pollution in the river Ganga. U Ans. When any unwanted waste material is added to water which makes it poisonous and useless, it is called as water pollution.

Main sources of pollution in the River Ganga :

Ref to Short Ans 2.  (1+4=5) Q. 3. Mention the main features of the Ganga Action Plan. U Ans. Main features of the Ganga Action Plan : (i) Sewage flowing into the river is to be diverted to other locations for treatment and conversion into energy source. (ii) Steps have been taken to supply safe drinking water by constructing electric crematoriums and separating bathing ghats. (iii) The diversion of several major drains carrying wastes into the river has been completed in cities like Varanasi, Patna and Kanpur and pollution level has been decreased. (iv) Infrastructure capable of diverting and treating 835 million litres per day of domestic waste or sewage has been created.

DRAINAGE

(v) The Ganga Action Plan has been merged with National River Conservation Plan. It now

covers 152 towns located along 27 Inter-state rivers in over 16 states. (1×5=5)

COMPETENCY BASED QUESTIONS Case based MCQs

(4 marks each)

Subjective based Questions

Read the source given below and answer the A (1+1+1+1=4) questions that follow: The main water divide in Peninsular India is formed by the Western Ghats, which runs from north to south close to the western coast. Most of the major rivers of the Peninsula, such as the Mahanadi, the Godavari, the Krishna and the Kaveri flow eastwards and drain into the Bay of Bengal. These rivers make deltas at their mouths. There are numerous small streams flowing west of the Western Ghats. The Narmada and the Tapi are the only long rivers, which flow west and make estuaries. The drainage basins of the peninsular rivers are comparatively smaller in size. [NCERT] 1. The main water divide in Peninsular India is formed by the Western Ghats, which runs from ..................... close to the Western Coast. (A) South to East (B) North to South (C) East to West (D) North to West Ans. Option (B) is correct. 2. The major rivers of the peninsula flow ....................... . (A) westwards (B) southwards (C) eastwards (D) northwards Ans. Option (C) is correct. 3. These rivers make deltas at their ..................... . (A) source (B) mouth (C) banks (D) None of the above. Ans. Option (B) is correct. 4. The ..................... are the only long rivers, which flow West and make estuaries. (A) Narmada and Tapi (B) Krishna and Kaveri (C) Ganga and Yamuna (D) Mahanadi and Kaveri



Ans. Option (A) is correct.





101

Read the source given below and answer the U (1+1+2=4) questions that follows: India has many lakes. These differ from each other in size and other characteristics. Most lakes are permanent; some contain water only during the rainy season, like the lakes in the basins of inland drainage of semi-arid regions. There are some lakes which are the result of the action of glaciers and ice sheets, while others have been formed by wind, river action and human activities. A meandering river across a floodplain forms cut- offs that later develops into ox-bow lakes. Spits and bars form lagoons in the coastal areas, e.g., the Chilika lake, the Pulicat lake and the Kolleru lake. Lakes in the region of inland drainage are sometimes seasonal; for example, the Sambhar lake in Rajasthan, which is a salt water lake. Its water is used for producing salt. Most of the freshwater lakes are in the Himalayan region. They are of glacial origin. In other words, they formed when glaciers dug out a basin, which was later filled with snowmelt. The Wular lake in Jammu and Kashmir, in contrast, is the result of tectonic activity. It is the largest freshwater lake in India. The Dal lake, Bhimtal, Nainital, Loktak and Barapani are some other important freshwater lakes. [NCERT] Answer the following questions: 1. How do the Indian lakes differ from each other? Ans. India has many lakes. These differ from each other in size and other characteristics.

2. Where is Sambhar lake situated?

Ans. Sambhar lake is situated in Rajasthan.

3. Name any two freshwater lake in India.

Ans. The Dal lake, Bhimtal, Nainital, Loktak and Barapani are some other important freshwater lakes. (Any Two)

102

Oswaal CBSE Question Bank Chapterwise & Topicwise, SOCIAL SCIENCE, Class-IX

Map Work

Syllabus

Rivers: (Identification only) The Himalayan River System-The Indus, The Ganges, and The Setlej , The Peninsular Rivers-The Narmada, The Tapi, The Kaveri, The Krishna, The Godavari, The Mahanadi l Lakes: Wular, Pulicat, Sambhar, Chilika l



Study Time Max Time: 1:30 hr Max Questions: 48

CHAPTER

7

Learning Objectives

Topic-1

CLIMATE Examine and analyse the factors that determine the climate of India. Discuss the mechanisms of monsoons in Indian subcontinent. l Analyse and infer the reasons behind the wide difference between day and night temperatures at different geographical locations of India. l To interpret how monsoon acts as a Unifying bond. l l

Climate of India and Climate Controls

Revision Notes

TOPIC - 1 Climate of India and Climate Controls.... P. 103 TOPIC - 2 The Seasons of India .... P. 109

Concept:  Weather describes the day-to-day meteorological conditions such as wind, temperature, cloudiness, moisture, rainfall, etc., affecting a place. Climate is the average weather, usually taken over 30-years period for a particular region and time.  ‘Monsoon’ refers to the seasonal reversal in the wind direction during the year. The two important elements of climate are temperature and precipitation. Indian Climate: Scan to know  Despite an overall unity in the general pattern, there are perceptible regional variations in more about this topic climatic conditions within the country.  Let us take two important elements – temperature and precipitation, and examine how they vary from place to place and season to season.  In summer, the mercury occasionally touches 50°C in some parts of the Rajasthan Desert, whereas it may be around 20°C in Pahalgam in Jammu and Kashmir. On a winter night, temperature at Drass in Jammu and Kashmir may be as low as minus 45°C. Thiruvananthapuram, Weather and Climate on the other hand, may have a temperature of 22°C.  There are variations not only in the form and types of precipitation but also in its amount and the seasonal distribution.  The climate of India is described as the ‘monsoon’ type. In Asia, this type of climate is found mainly in the south and the southeast.  Climate refers to the sum total of weather conditions and variations over a large area for a long period of time (more than thirty years).  Weather refers to the state of the atmosphere over an area at any point of time. The elements of weather and climate are the same, i.e. temperature, atmospheric pressure, wind, humidity and precipitation.  India has both tropical and subtropical types of climate.  The climate of India is described as the ‘monsoon’ type. In Asia, this type of climate is found mainly in the south and the southeast.

104

Oswaal CBSE Question Bank Chapterwise & Topicwise, SOCIAL SCIENCE, Class-IX

CLIMATE

105

Climate Control: Scan to know more about  Based on climatic differences, the world can be divided into a number of climatic regions. Each this topic climatic region has its own characteristic vegetation and wildlife. The climatic conditions also influence the lifestyles of the people living in these regions. Factors Affecting India’s Climate:  The factors affecting the climate of a place are referred to as ‘controls’. There are six major controls of the climate of any place. These are: latitude, altitude, pressure and wind system, Six Major Climatic Controls distance from the sea (continentality), ocean currents and relief features. (i) Latitude: Almost half of the country, lying south of the Tropic of Cancer, belongs to the tropical area. All the remaining area, north of the Tropic, lies in the sub-tropics. Therefore, India’s climate has characteristics of tropical as well as subtropical climates. (ii) Altitude: It is another factor controlling the climate of a place. Altitude refers to the height of a place above sea level. The higher one travels into the Troposphere, the lower the temperature becomes. (iii) Pressure and Wind: The pressure and wind conditions over India are unique. During winter, there is a high-pressure area north of the Himalayas. In summer, a low-pressure area develops over interior Asia as well as over northwestern India. This causes a complete reversal of the direction of winds during summer.  The differences in air pressures near the Equator and the Poles are the main factors that influence global pressure and wind systems. In India, the Tropic of Cancer passes through the central part of the country, from the Rann of Kutch in the west to Mizoram in the east. India has both Tropical and Subtropical types of Climate. (iv) Distance from Sea: The distance of a place from the sea is another important factor that regulates the climate of a place. As the distance from the sea increases, the weather conditions become extreme. Places away from the sea have very hot summers and very cold winters. This condition is known as Continentality. (v) Ocean currents: Ocean currents along with onshore winds affect the climate of the coastal areas, For example, any coastal area with warm or cold currents flowing past it, will be warmed or cooled if the winds are onshore. (vi) Relief: Relief too plays a major role in determining the climate of a place. High mountains act as barriers for cold or hot winds; they may also cause precipitation if they are high enough and lie in the path of rainbearing winds. The leeward side of mountains remains relatively dry. Various disasters and its preventive protocols  Earthquakes: • Identify safe spots in your home, workplace or school (under sturdy furniture, doorways, or against interior walls). • Secure heavy items such as bookshelves, cabinets and water heaters to the walls • Keep emergency supplies such as food, water, flashlights and first aid kits in accessible locations. • Develop a family or workplace emergency plan, and practice it regularly.  Floods: • Be aware of areas prone to flooding in your community and avoid them during heavy rainfalls. • If possible, elevate your home’s foundation or install flood-resistant barriers. • Create a household emergency plan that includes evacuation routes, contact information and emergency supplies. • Monitor weather forecasts and warnings, and follow the instructions of local authorities.  Hurricanes: • Install storm shutters or board up windows to protect against high winds and flying debris. • Secure loose outdoor items, and bring indoors any lightweight objects that could become projectiles. • Plan for evacuation if necessary, and identify a safe place to stay during the storm. • Keep emergency supplies on hand, including food, water, first aid kit, flashlights, batteries and a portable radio.  Wildfires: • Clear flammable vegetation and debris from around your home or property. • Create a defensible space around your home by using fire-resistant materials for roofing, walls and landscaping. • Have an evacuation plan, and be ready to leave if ordered to do so by authorities. • Keep emergency supplies in a readily accessible location, and monitor the progress of the fire through local news and social media.  Tornadoes: • Identify a safe room or shelter in your home, and make sure everyone knows where it is. • Listen to local weather forecasts and warnings, and take shelter immediately if a tornado warning is issued.

106

Oswaal CBSE Question Bank Chapterwise & Topicwise, SOCIAL SCIENCE, Class-IX

• Secure outdoor objects that could become projectiles, and move vehicles into a garage or other protected area. • Keep an emergency supply kit in your safe room or shelter, including food, water, first aid kit, and flashlights.

Key Terms  Elements of weather: Temperature, Atmospheric pressure, Wind, Humidity and Precipitation are the elements of Weather and Climate.  Altitude: Altitude refers to the height of a place above the Sea level.  Continentality: As the distance from the sea increases, the weather conditions become extreme. Places away from the sea have very hot summers and very cold winters. This condition is known as Continentality.

OBJECTIVE TYPE QUESTIONS Multiple Choice Questions R Q. 1. Which is the coldest place in India?  (A) Shimla (B) Drass (C) Kashmir (D) Mussoorie Ans. Option (B) is correct. Explanation: Situated at an elevation of 3350 m, Drass is the coldest place in India and second to the coldest inhabited region on Earth. The Average temperature on a winter night may be as low as minus 45°C. Q. 2. Which part of India experiences the highest range R of temperature in a day? (A) Gujarat (B) Madhya Pradesh (C) Uttar Pradesh (D) Thar Desert in Rajasthan Ans. Option (D) is correct. R Q. 3. India lies in the region of __________winds. (A) North easterly (B) South westerly (C) Trade winds (D) None of the Above Ans. Option (A) is correct. Explanation: India lies in the region of North easterly winds. These winds originate from the subtropical high-pressure belt of the northern hemisphere. Q. 4. What is the first step in developing a disaster prevention protocol? (A) Identify the most common types of disasters that could affect the community (B) Assess the risks associated with each type

of disaster

(C) Create a communication plan for the community (D) Build a resource inventory for emergency

response teams

Ans. Option (A) is correct.

(1 mark each)

Q. 5. What is the purpose of a disaster prevention protocol? (A) To minimize the impact of disasters on the community (B) To prepare emergency responders for

disaster response

(C) To provide medical supplies for disaster victims (D) To allocate financial resources for disaster

recovery

Ans. Option (A) is correct.

Assertion and Reason In the questions given below, there are two statements marked as Assertion (A) and Reason (R). Read the statements and choose the correct option.

(A) Both (A) and (R) are true and (R) is the correct explanation of (A). (B) Both (A) and (R) are true and (R) is not the correct explanation of (A). (C) (A) is correct but (R) is wrong. (D) (A) is wrong but (R) is correct.

Q. 1. Assertion (A): Air temperature generally decreases U from the Equator towards the Poles. 

Reason (R): Due to the curvature of the Earth, the amount of Solar energy received varies according to latitude.

Ans. Option (A) is correct.

Explanation: Temperature decreases from Equator towards the Poles because it receives less sunlight. As we move away from Equator the temperature decreases and hence, the Poles receives very little sunlight.

Q. 2. Assertion (A): Any coastal area with warm or cold currents flowing past it, will be warmed or cooled E if the winds are onshore. 

Reason (R): Ocean currents along with onshore winds affect the climate of the Coastal areas.

Ans. Option (A) is correct.

CLIMATE

107

SUBJECTIVE TYPE QUESTIONS Very Short Answer Type Questions Q. 1. The elements of weather and climate are the U same. Name any two such elements. Ans. The elements of weather and climate are the same, i.e. temperature, atmospheric pressure, wind, humidity and precipitation. (Any Two)  (1 x 2 = 2) Q. 2. There are variations not only in the form and types of precipitation but also in its amount and U the seasonal distribution. Explain. Ans. There are variations not only in the form and types of precipitation but also in its amount and the seasonal distribution. While precipitation is mostly in the form of snowfall in the upper parts of Himalayas, it rains over the rest of the country. The annual precipitation varies from over 400 cm in Meghalaya to less than 10 cm in Ladakh and Western Rajasthan. (2) Q. 3. Give reasons as to why: The Tamil Nadu coast E [NCERT] receives winter rainfall. Ans. Tamil Nadu gets a large portion of its rain during October and November. In general, coastal areas experience less contrasts in temperature conditions.

Seasonal contrasts are more in the interior of the country.(2) Q. 4. How do relief play a major role in determining U the climate of a place? Ans. Relief plays a major role in determining the climate of a place. High mountains act as barriers for cold or hot winds; they may also cause precipitation if they are high enough and lie in the path of rain-bearing winds. The Leeward side of mountains remains relatively dry. (2) Q. 5. India’s climate has characteristics of Tropical as well as Subtropical Climates.

Commonly Made Error

Students mention only the state and write the reason that it receives highest rainfall.

Answering Tip

Students need to mention that Mawsynram is

located in the Khasi Hills and receives highest rainfall as this place is enclosed by hills on three sides and the monsoon gets trapped in these hills.

E

Ans. The Tropic of Cancer passes through the middle of the country from the Rann of Kutch in the west to Mizoram in the east. Almost half of the country, lying South of the Tropic of Cancer, belongs to the Tropical area. All the remaining area, North of the Tropic, lies in the Sub-tropics. Therefore, India’s climate has characteristics of Tropical as well as Subtropical Climates. (2)

Short Answer Type Questions Q. 1. Where is Mawsynram located  ? Why does Mawsynram receive the highest amount of R rainfall? Ans. (i) Mawsynram is located in the southern ranges of the Khasi Hills at a height of 1,500 m above the sea level. It receives the highest rainfall in the world. Annual rainfall is about 1,140 cm. (ii) This place receives the highest amount of rainfall because it is enclosed by hills on three sides. The relief features give this place a tunnel-shaped location. The Bay of Bengal monsoon is trapped in these hills. (iii) The winds try to get out of it, but are forced to pour down there. (1½+1½=3)

(2 marks each)

(3 marks each)

Q. 2. What influence do the Himalayas have on India’s climate?

R

Ans. The Himalayas, the lofty mountains, have

provided India with a compact physical setting.

The Himalayas protect the sub-continent from the northern winds. These cold and chilly winds originate near the Arctic Circle and blow across central and eastern Asia. (3)

Q. 3. “India

has diverse climatic conditions.” Explain by giving two examples each of temperature and precipitation. A OR Examine the regional variations in climatic conditions within the country with reference to temperature and precipitation. (NCERT) Ans. Temperature: (i) In May, 45°C temperature is common to be experienced in North–western parts of India. (ii) In December and January, the temperature decreases from South to North, e.g., Chennai 25°C and in Northern plains 10°C.

108

Oswaal CBSE Question Bank Chapterwise & Topicwise, SOCIAL SCIENCE, Class-IX

Precipitation: (i) In June and July, windward side of Western Ghats receives very heavy rainfall. (ii) Mawsynram receives the highest average rainfall in the world. (iii) Rajasthan and parts of Gujarat get scanty rainfall. (1×3=3)

Commonly Made Error

Students forgot to mention months.

Answering Tip

Mention examples of two places in India with

differences in temperature and rainfall. Q. 4. What are the factors affecting the climate of India? U OR What are the controls affecting the climate of India? (NCERT) Ans. Factors affecting the climate of India are: (i) Latitude: The temperature decreases from the Equator towards the Poles. Places located on lower latitudes are hotter than the places located on higher latitudes. Solar energy is present at a higher rate more in lower latitudes. (ii) Altitude: At higher altitudes, the atmosphere becomes less dense and temperature decreases. Hills are cooler during summers, e.g., climate of Shimla is cooler than that of Delhi. (iii) Distance from the sea: Sea has a moderating influence on the climate. As the distance from the

ocean or sea increases, its moderating influence decreases and extreme weather conditions are experienced, i.e., very hot in summer and very cold in winter. (iv) Ocean currents: Ocean currents along with onshore winds affect the climate of the coastal regions. Any coastal area with warm or cold currents flowing fast, will be cooled or warmed if the winds are onshore.  (Any three) (1×3=3) Q. 5. What are some important items to keep in an emergency kit? Ans. It’s important to keep emergency supplies on hand in case of a natural disaster. Some essential items to include in your emergency kit are food, water, first aid kit, flashlights, batteries, and a portable radio. You should also consider including any necessary medications, personal hygiene items, and important documents such as identification and insurance papers. For a more comprehensive list of items to include in your emergency kit. (3) Q. 6. What are the most common types of disasters that could affect our community/region? Ans. The most common types of natural disasters that can affect communities include earthquakes, floods, hurricanes, wildfires, and tornadoes. However, the specific risks associated with each type of disaster can vary depending on the location and other factors. It’s best to consult with local authorities or disaster management agencies to identify the specific risks in your area and develop a plan to prepare for them. (3)

Long Answer Type Questions Q. 1. How can we involve the community in disaster U prevention and response efforts? Ans. Community involvement is crucial in disaster prevention and response efforts. One way to involve the community is to organize training sessions or drills to educate people on how to respond in case of a natural disaster. We can also encourage people to develop their own emergency plans and kits, and share information about local risks and hazards. Additionally, we can work with local authorities or disaster management agencies to establish community-based early warning systems, evacuation plans, and other preparedness measures. (5) Q. 2. What measures can we take to minimize the economic impact of each type of disaster on businesses and individuals in our community? U

(5 marks each)

Ans. Natural disasters can have a significant economic impact on businesses and individuals in a community. While it’s impossible to completely eliminate the economic impact of a disaster, there are measures that can be taken to minimize it. For example, businesses can develop continuity plans to ensure that critical operations can continue during and after a disaster. Individuals can also take steps to protect their property and assets, such as purchasing insurance or investing in disasterresistant building materials. Additionally, local authorities or disaster management agencies may offer financial assistance or other resources to help businesses and individuals recover from the economic impact of a disaster. (5)

CLIMATE

Q. 3. Study the data carefully and answer the following questions: 



(a) Which station has the higher range of temperature and why? (b) Which station receives the higher amount of rainfall? Name the two most rainy months of this station. Ans. (a) Delhi has the higher range of temperature as it is not a coastal area or we can say it is far away from

109 A

the sea. So, the temperature is not moderate in this station. (b) Chennai receives higher amount of rains. The two most rainy months of this station are October and November. (2½+2½=5)

Topic-2 The Seasons of India Revision Notes  The changes in weather conditions of India can be broadly categorised into four distinct patterns. These are the cold weather season, the hot weather season, the advancing monsoon and the retreating monsoon. The Cold Weather Season:  India has cold weather in the winter that begins in mid-November and lasts till February. The Scan to know more about Northern parts of India are the first to experience the cold. December and January are the this topic coldest months in Northern India. The southern parts of India do not experience very low temperatures.  Frost is a common phenomenon in the Northern plains, while the higher slopes of the Himalayas experience snowfall.  During winter, the north–east trade winds blow over the country. In most parts of India, The Seasons of the weather during the winter is dry. In some parts, like the coast of Tamil Nadu, the winds India blow from the sea to the land. Hence, these places receive some amount of rainfall during the winter.  Other characteristic feature of the winter season experienced over the Northern Plains is the inflow of cyclonic disturbances from the West and the North–West. The winter rainfall is locally known as Mahawat. The Hot Weather Season:  The sea has a moderating influence over the Peninsular region of India. Hence, there are no drastic seasonal variations in temperature pattern, and winters are not as distinct as in the north. The hot weather season in India refers to the summers that begin in March and lasts till Mid-June.  In India, summer is caused due to the apparent northward movement of the Sun, which shifts the Global Heat Belt northward. ‘Loo’ are strong, gusty, hot, dry winds that blow during the day over North and North-western India.  In northern India, dust storms are also very common during the month of May. This is also the season for localised thunderstorms, associated with violent winds, torrential downpours, often accompanied by hail. Pre -monsoon showers are common, especially in Kerala and Karnataka and are often referred to as ‘Mango Showers,’ as they help in the early ripening of mangoes.

110

Oswaal CBSE Question Bank Chapterwise & Topicwise, SOCIAL SCIENCE, Class-IX

Advancing Monsoon:  The advancing monsoon in India refers to the rainy season that lasts for a period of four months from June to September. The trade winds from the Southern Hemisphere, which originate over the warm subtropical areas of the southern oceans, bring the South-West Monsoon Winds into India.  Another characteristic feature of the monsoon is the ‘Breaks’ in rainfall and refers to the alternating wet and dry spells of rains. The Indian monsoon is very unpredictable. These breaks occur due to the movement of the Monsoon Trough.  The farming schedule of millions of farmers all over the country, which is dependent on the monsoons, can sometimes get disturbed. Retreating Monsoon:  The retreating monsoon refers to the transition season that lasts from October to December.  The months of October–November form a period of transition from the Hot Rainy Season to Dry Winter conditions. The main cause of this transition is the movement of the Sun towards the South. By the beginning of october, the monsoon withdraws from the Northern Plains. The retreating monsoon season is marked by clear skies and a rise in the temperatures. The land is still moist and the weather becomes hot and humid. The days can become quite oppressive. This is commonly known as ‘October Heat’.  Parts of western coast and northeastern India receive over about 400 cm of rainfall annually. However, it is less than 60 cm in western Rajasthan and adjoining parts of Gujarat, Haryana and Punjab.  Rainfall is equally low in the interior of the Deccan plateau, and east of the Sahyadris.  A third area of low precipitation is around Leh in Jammu and Kashmir.  The rest of the country receives moderate rainfall. Snowfall is restricted to the Himalayan region.  Owing to the nature of monsoons, the annual rainfall is highly variable from year to year. Variability is high in the regions of low rainfall, such as parts of Rajasthan, Gujarat and the leeward side of the Western Ghats.  As such, while areas of high rainfall are liable to be affected by floods, areas of low rainfall are drought-prone. Monsoon as a Unifying Bond:  Because of the unpredictable nature of the monsoon, the distribution of rainfall varies drastically from region to region. Major variations can also be seen in the annual precipitation from year to year. The phenomenon of monsoon is an integral part of the Indian landscape, animal and plant life, agricultural calendar, and the life of the people and their festivities.  Year after year, people of India from north to south and from east to west, eagerly awaits the arrival of the monsoon.  These monsoon winds bind the whole country water to set the agricultural activities in motion.

Key Terms  Loo: ‘Loo’ are strong, gusty, hot, dry winds that blow during the day over North and North-Western India.  Mango Showers: Pre– monsoon showers are often referred to as ‘Mango Showers,’ as they help in the early ripening of mangoes.  Advancing Monsoon: An Advancing Monsoon in India refers to the Rainy Season that lasts for a period of 4 months from June to September.  Retreating Monsoon: The Retreating Monsoon refers to the transition season that lasts from October to December.

OBJECTIVE TYPE QUESTIONS Multiple Choice Questions Q. 1. Mango showers occur in which group of two R states?  (A) Kerala and Karnataka (B) Kerala and Tamil Nadu (C) Karnataka and Andhra Pradesh (D) Tamil Nadu and Kerala Ans. Option (A) is correct.



(1 mark each)

Explanation: Pre-monsoon showers are common in Kerala and Karnataka and are often referred as ‘Mango Showers’ as they help in the early ripening of mangoes. R Q. 2. Monsoon arrive in India approximately in: [NCERT] (A) Early May (B) Early July (C) Early June (D) Early August Ans. Option (B) is correct.

CLIMATE

Q. 3. The wind blowing in the Northern Plains in R [NCERT] summers is known as: (A) Kaal Baisakhi (B) Loo (C) Trade Winds (D) None of the above Ans. Option (B) is correct. Explanation: Loo are strong, gusty, hot, dry winds blowing during the day over the north and northwestern India.

Assertion and Reason In the questions given below, there are two statements marked as Assertion (A) and Reason (R). Read the statements and choose the correct option.

(A) Both (A) and (R) are true and (R) is the correct explanation of (A).



(B) Both (A) and (R) are true and (R) is not the correct explanation of (A).



(C) (A) is correct but (R) is wrong.



(D) (A) is wrong but (R) is correct.

111

Q. 1. Assertion (A): From March to May, it is hot weather season in India. 

A

Reason (R): Due to the apparent northward movement of the Sun, the global heat belt shifts northward.

Ans. Option (A) is correct.

Explanation: The hot weather season in India begin in march and lasts till mid-June. Summer is caused due to the southward movement of the sun, which shifts the Global Heat Belt.

Q. 2. Assertion (A): The Monsoon winds bind the whole country by providing water to set the agricultural activities in motion.

E

Reason (R): The unifying influence of the monsoon on the Indian subcontinent is quite perceptible.

Ans. Option (A) is correct.

SUBJECTIVE TYPE QUESTIONS Very Short Answer Type Questions Q. 1. Highlight any three characteristics of cold R weather season in India. Ans. Characteristics of cold weather season in India are: (i) Duration of this season is mid November in the Northern Plains and stays till February. December and January are the coldest months in the Northern parts of the country. (ii) The temperature decreases from South to North. The days are warm and nights are cold. (iii) Frost is common in the North and the higher slopes of the Himalayas experience snowfall.  (Any Two) (1 x 2 = 2) Q. 2. Why do the Western Ghats receive more rainfall U than the Eastern Ghats? Explain briefly Ans. (i) The Western Ghats receive rainfall from the Arabian Sea monsoon winds. (ii) They do not allow these winds to cross over without shedding their moisture on the western slopes. (iii) A part of these winds that reaches the Eastern Ghats is almost dry. (Any Two) (1 x 2 = 2)

Q. 3. What do you understand by ‘Burst of Monsoons’? U  Ans. Around the time of its arrival, the normal rainfall increases suddenly and continues constantly for several days. This is known as the ‘Burst’ of the Monsoon.  (2)

Q. 4. What do you know about the arrival of monsoons E in India? Ans. (i) The monsoon arrives at the southern tip of the Indian peninsula generally by the first week of June. (ii) Subsequently, it proceeds into two – the Arabian Sea branch and the Bay of Bengal branch.  (1 x 2 = 2)

Q. 5. Why the peninsular region does not have a wellE defined cold season? Ans. The peninsular region does not have a well- defined cold season. There is hardly any noticeable seasonal change in temperature pattern during winters due to the moderating influence of the sea. (2)

Short Answer Type Questions Q. 1. Describe the climatic conditions of India during Retreating Monsoon. A Ans. Climatic conditions during Retreating Monsoons:

(2 marks each)

(3 marks each)

(i) This is the transition period during the months of October and November. (ii) The season changes from hot and rainy to dry winter conditions. The retreat of the monsoon is marked by

112

Oswaal CBSE Question Bank Chapterwise & Topicwise, SOCIAL SCIENCE, Class-IX

clear skies and rise in temperature. Day temperatures are high and nights are cool and pleasant. (iii) Low–pressure conditions get transferred to the Bay of Bengal by early November. (iv) Cyclonic depressions in the Andaman Sea cause heavy rainfall and are destructive tropical cyclones.  (Any three) (1×3=3) Q. 2. Explain the terms: R (a) Loo, (b) Kal Baisakhi, (c) Mango showers. OR Define ‘Kal Baisakhi’ and ‘Loo’. In which parts of India are they experienced? Ans. (a) Loo: Strong, dusty, hot and dry winds blowing during the day over the North and North- western India. (b) Kal Baisakhi: Localised thunderstorms, associated with violent winds and torrential downpours. In West Bengal, these storms are known as Kal Baisakhi. (c) Mango showers: Pre- monsoon showers which help in the early ripening of mangoes are called ‘Mango Showers’. (1×3=3) Q. 3. Why does Mumbai receive rainfall in summer while Chennai receive rainfall in winter? A Ans. (i) Mumbai receives rainfall in summer from the Arabian Sea Branch from June to September. (ii) Mumbai receives rainfall in summer as it is located on the western coast.

(iii) It is on the windward side of Western Ghats. Chennai receives rainfall in winter as : (i) It is located on the eastern coast. (ii) The Arabian Sea Branch becomes offshore. (iii) The Bay of Bengal Branch moves parallel to the Eastern Coast. (iv) Chennai receives rainfall from North-east monsoons and by cyclones from October to December. (Any three) (3) Q. 4. Withdrawal of the monsoon is a more gradual U process . Explain. Ans. (i) The withdrawal of the monsoon begins in northwestern states of India by early September. (ii) By mid-October, it withdraws completely from the northern half of the peninsula. (iii) The withdrawal from the southern half of the peninsula is fairly rapid. By early December, the monsoon has withdrawn from the rest of the country. (1 x 3 = 3)

Q. 5. Why is the annual rainfall in India highly variable E from year to year. Ans. Owing to the nature of monsoons, the annual rainfall is highly variable from year to year. Variability is high in the regions of low rainfall such as parts of Rajasthan, Gujarat and the leeward side of the Western Ghats. (3)

Long Answer Type Questions Q. 1. Describe any five characteristic features of the hot weather season in India. A (Board Term II, 2015) Ans. (i) March to May. (ii) Low pressure develops extending from Thar Desert, in the North–west to Patna and Chota Nagpur Plateau in the East and South–east. (iii) Temperature ranges between 38 degree Celsius to 45 degree Celsius in the North. (iv) Pre–monsoon showers are common; are known as Kal Baisakhi, Mango Showers. (v) Striking feature: Loo, a hot, dry, dusty and strong wind blows over the North and North-west India, dust storms. (1×5=5) (CBSE Marking Scheme, 2015)

Commonly Made Error

Students commonly make some errors like they do not mention the features or characteristics as per region wise.

(5 marks each)

Answering Tip

Discuss the features of summer season in India and also classified the characteristics of the hot summer season. And also mention the region wise temperature variations.

Q. 2. Differentiate between the Cold Weather Season and the Hot Weather Season of India by explaining two distinctive features of each. A Ans. Cold Weather Season: (i) It begins from mid November in northern India and stays till February. (ii) The temperature decreases from South to North. (iii) Days are warm and nights are cold. Hot Weather Season: (i) It begins from March and stays till May. (ii) Experiences rising temperature and falling air pressure in North India. (iii) Hot and dry wind called ‘Loo’ blows during the day. (2½+2½=5)

CLIMATE

COMPETENCY BASED QUESTIONS Case based MCQs I. Read the source given below and answer the questions that follow:  (1+1+1+1=4) The cold weather season begins from midNovember in northern India and stays till February. December and January are the coldest months in the northern part of India. The temperature decreases from south to the north. The average temperature of Chennai on the eastern coast, is between 24° C - 25° Celsius, while in the northern plains, it ranges between 10°C and 15° Celsius. Days are warm and nights are cold. Frost is common in the north and the higher slopes of the Himalayas experience snowfall. During this season, the northeast trade winds prevail over the country. They blow from land to sea and hence, for most part of the country, it is a dry season. Some amount of rainfall occurs on the Tamil Nadu coast from these winds as, here they blow from sea to land. In the northern part of the country, a feeble highpressure region develops, with light winds moving outwards from this area. Influenced by the relief, these winds blow through the Ganga valley from the west and the northwest. The weather is normally marked by clear sky, low temperatures and low humidity and feeble, variable winds. A characteristic feature of the cold weather season over the northern plains is the inflow of cyclonic disturbances from the west and the northwest. These low-pressure systems, originate over the Mediterranean Sea and western Asia and move into India, along with the westerly flow. They cause the much-needed winter rains over the plains and snowfall in the mountains. Although the total amount of winter rainfall locally known as ‘mahawat’ is small, they are of immense importance for the cultivation of ‘rabi’ crops. 1. In cold weather season of winter, the temperature decreases from: (A) South to the North (B) East to the West (C) South to the East (D) North to the West Ans. Option (A) is correct. 2. Which one of the following characterizes the cold weather season in India? (A) Warm days and warm nights (B) Warm days and cold nights (C) Cold days and cold nights (D) Cold days and warm nights Ans. Option (B) is correct. 3. Some amount of rainfall occurs on the ________ coast from the northeast trade winds, as they blow from the sea to land. (A) Coromandel (B) Malabar (C) Kerala (D) Tamil Nadu Ans. Option (D) is correct.

113

(4 marks each)



4.  Winter rainfall called _________ is of immense importance for the cultivation of _________ crops. (A) Monsoon Showers, Rabi Crops (B) Mango Showers, Kharif Crops (C) Mahawat, Rabi (D) Kal Baisakhi, Kharif Explanation: The winter rainfall in Northern India is locally known as Mahawat. Ans. Option (C) is correct. Subjective based Questions I. Read the source given below and answer the questions that follow:  (1+1+2=4) There are six major controls of the climate of any place. They are: latitude, altitude, pressure and wind system, distance from the sea (continentality), ocean currents and relief features. Due to the curvature of the earth, the amount of solar energy received varies according to latitude. As a result, air temperature generally decreases from the equator towards the poles. As one goes from the surface of the earth to higher altitudes, the atmosphere becomes less dense and temperature decreases. The hills are therefore cooler during summers. The pressure and wind system of any area depend on the latitude and altitude of the place. Thus, it influences the temperature and rainfall pattern. The sea exerts a moderating influence on climate: As the distance from the sea increases, its moderating influence decreases and the people experience extreme weather conditions. This condition is known as continentality (i.e., very hot during summers and very cold during winters). Ocean currents along with onshore winds affect the climate of the coastal areas. For example, any coastal area with warm or cold currents flowing past it, will be warmed or cooled if the winds are onshore. Finally, relief too plays a major role in determining the climate of a place. High mountains act as barriers for cold or hot winds, they may also cause precipitation if they are high enough and lie in the path of rain bearing winds. The Leeward side of mountains remains relatively dry. 1. How does the curvature of the Earth affect climate? Ans. Due to the curvature of the earth, the amount of solar energy received varies according to latitude. As a result, air temperature generally decreases from the equator towards the poles. 2. Which side of the mountain remains relatively dry? Ans. The leeward side of the mountains remains relatively dry. 3.  What is the condition known as when it is very hot during summers and very cold during winters? Ans. As the distance from the sea increases, its moderating influence decreases and the people

114

Oswaal CBSE Question Bank Chapterwise & Topicwise, SOCIAL SCIENCE, Class-IX

experience extreme weather conditions. This condition is known as continentality. II. Read the source given below and answer the questions that follow:  (1+1+2=4) The summer months experience rising temperature and falling air pressure in the northern part of the country. Towards the end of May, an elongated low-pressure area develops in the region extending from the Thar Desert in the northwest to Patna and Chota Nagpur Plateau in the east and southeast. Circulation of air begins to set in around this trough. A striking feature of the hot weather season is the ‘loo’. These are strong, gusty, hot, dry winds blowing during the day over the north and northwestern India. Sometimes they even continue until late in the evening. Direct exposure to these winds may even prove to be fatal. Dust storms are very common during the month of May in northern India. These storms bring temporary relief as they lower the temperature and may bring

light rain and cool breeze. This is also the season for localised thunderstorms, associated with violent winds, torrential downpours often accompanied by hail. In West Bengal, these storms are known as the ‘Kal Baisakhi’. Towards the close of the summer season, premonsoon showers are common, especially in Kerala and Karnataka. They help in the early ripening of mangoes, and are often referred to as ‘mango showers’. 1. The rising temperature and falling air pressure in summer is seen in which part of the country? Ans. The rising temperature and falling air pressure in summer is seen in northern part of the country. 2. What is the name given to the wind which blows in the northern plains in summers? Ans. Loo 3.  Mango showers occur in which group of two states? Ans. Mango Showers are Pre-monsoon showers which are common in the states of Karnataka and Kerala.

Map Work

Syllabus

l

Annual rainfall in India, Monsoon wind direction.

CLIMATE

115

Artificial Intelligence Understand monsoon winds with the help of Data Acquisition and Data Exploration. Objectives:  To identify various factors affecting the arrival and retreating of monsoon winds.  Students will also be able to understand the relation between temperature and atmospheric pressure.  To understand the concept of system map. Material Required:  Textbooks, pen and paper, smart board/screen and projector, thematic map of rainfall in India  Internet access for data acquisition and laptops. PARAMETERS

DESCRIPTION

Chapter Covered

Chapter 4: Climate

Name of the Book

Contemporary India – I, Class 9, NCERT

AI CONCEPTS INTEGRATED

Subject and Artificial Monsoon winds with the help of Data Acquisition and Data Intelligence Exploration Integrated Learning Objectives

l l l

 o identify various factors affecting the arrival and retreating of T monsoon winds. Students will also be able to understand the relation between temperature and atmospheric pressure. To understand the concept of system map. The system map Loopy will be able to make them understand the cause and effect relationship of temperature and pressure.

Time Required

3-4 Periods of 40 minutes each

Classroom Arrangement

Flexible Classroom Arrangement

Data acquisition and Data exploration https:// datavizcatalogue. com

116

Oswaal CBSE Question Bank Chapterwise & Topicwise, SOCIAL SCIENCE, Class-IX

Material Required

l l

Pre – Preparation Activities

 extbooks, pen and paper, smart board/screen and projector, T thematic map of rainfall in India Internet access for data acquisition and laptops

Students will be shown a video in which regional variation in climatic conditions will be described. It will make them curious to learn the reason behind these variations. Scan to know more about this topic

Video Time -2:00 Min to 5:15 Min Previous Knowledge

Students will be asked few questions – l Students will be asked about their personal experience about the variation in weather conditions of a place if they have experienced while travelling from one place to another. l They will be asked few questions to know their previous knowledge about the winds and pressure belts.

Methodology

Students will be shown the loopy of the given link to make them understand the relation between atmospheric pressure and winds (using the LOOPY) https://bit.ly/3b7bzQf The students will be asked to collect month wise data of the average temperature and rainfall of the city they live in for a particular year. Students will be asked to present their collected data of temperature and rainfall through a bar graph or line graph.

Learning Outcomes

l l l



Data Acquisition Data exploration https:// datavizcatalogue. com

 tudents will be able to understand the relation between S temperature pressure and rainfall. They will also be able to represent their collected data through Bar or Line graph. They will also be able to learn the factors Indian Climate is affected with

Follow up Activities

Make a graphical representation of your data. A follow up worksheet will be given in which they have to study the table and give the answers.

Reflections

A discussion with students how climate affects vegetation, behaviour http://www. of people, clothing style, cuisine, festivals and lifestyle of people. inklewriter.com/ After the discussion students will be asked to write a report on the basis of their learning using inkelwriter.



Study Time Max Time: 1:30 hr Max Questions: 41

CHAPTER

8

Learning Objectives

POPULATION l Examine the reasons behind

the uneven distribution of population in India with specification to UP & Rajasthan and Mizoram and Karnataka. TOPIC - 1 Population: Size and Distribution.... P. 117

Population: Size and

Topic-1 Distribution

TOPIC - 2 Population Growth and Processes of Population Change .... P. 120

Revision Notes  Population Size and Distribution:  Population is the pivotal element in social studies. Population is the point of reference from which all other elements are observed.  A Census is an official enumeration of the population of a country that is recorded periodically. The First census in India was held in 1872. Census has been held regularly after every 10 years.  The Census provides answers to three primary questions about the population: • Population size and distribution Scan to know more about • Population growth and processes of population change this topic • Characteristics or qualities of the population  Almost half of India’s population lives in the following five states: Uttar Pradesh, Maharashtra, Bihar, West Bengal and Andhra Pradesh. Uttar Pradesh is the most populated state of India with 199 million people, according to the Census of 2011. Population density is the number Population of India of persons that live in one square kilometre of an area.  On the other hand, the Himalayan state of Sikkim has a population of just about 0.5 million and Lakshadweep has only 60 thousand people.  Population density is calculated as the number of persons per unit area.  India is one of the most densely populated countries of the world.  Rugged terrain and unfavourable climatic conditions are the main reasons for the sparse population in some areas. Moderate to low rainfall and less fertile soils also influence the population density.

Key Terms  Census: A Census is an official enumeration of the population of a country that is recorded periodically.  Population Density: Population density is calculated as the number of persons per unit area.

118

Oswaal CBSE Question Bank Chapterwise & Topicwise, SOCIAL SCIENCE, Class-IX

POPULATION

OBJECTIVE TYPE QUESTIONS Multiple Choice Questions Q. 1. What was the population density of India R according to 2011 census?  (A) 82 person/km2 (B) 382 person/km2 (C) 85 person/km2 (D) 282 person/km2 Ans. Option (B) is correct. Explanation: The population density of India in 2011 was 382 per sq km-decadal growth of 17.72 percent. The density of population increased in all States and Union Territories between 1991 and 2011. Q. 2. Which state has the lowest population in India? R  (A) Sikkim (B) Assam (C) Andhra Pradesh (D) ) Karnataka Ans. Option (B) is correct. Explanation: Sikkim is the least populous state of India. It has a population of just about 0.5 million. U Q. 3. Study the given picture carefully: POPULATION

119

(1 mark each)

What is India’s share of World’s Area and Population? (A) 2.4% and 17.5% (B) 17.5% and 2.4% (C) 2.4% and 17.5% (D) 97.6% and 82.5% Ans. Option (C) is correct.

Assertion and Reason In the questions given below, there are two statements marked as Assertion (A) and Reason (R). Read the statements and choose the correct option.

(A) Both (A) and (R) are true and (R) is the correct explanation of (A).



(B) Both (A) and (R) are true and (R) is not the correct explanation of (A).



(C) (A) is correct but (R) is wrong.



(D) (A) is wrong but (R) is correct.

Q. 1. Assertion (A): Human beings are producers and consumers of Earth’s resources.

Rest of the world, 82.5%

U



Reason (R): The Census of India provides us with information regarding the population of our country. Ans. Option (B) is correct. India, 17.5%



AREA Rest of the world, 97.6%

Explanation: Human beings are producers and consumers of Earth’s resources. Therefore, it is important to know how many people are there in a country, where do they live, how and why their numbers are increasing and what are their characteristics.

Q. 2. Assertion (A): Population is the pivotal element in social studies.

E

India, 2.4%

Fig. India’s Share of World’s Area and Population

Reason (R): It is the point of reference from which all other elements are observed and from which they derive significance and meaning. Ans. Option (A) is correct.

SUBJECTIVE TYPE QUESTIONS Short Answer Type Questions Q. 1. Why is population the pivotal element in social



studies? U (Board Term II, 2017) OR



(3 marks each) Population is the pivotal element in social studies. Support your answer giving three points. (Board 2015, Term II)

120

Oswaal CBSE Question Bank Chapterwise & Topicwise, SOCIAL SCIENCE, Class-IX

Ans. People are important to develop the economy and society. The people make and use resources and are themselves resources with varying quality. Population is the pivotal element in social studies. It is the point of reference from which all other elements are observed and from which they derive significance and meaning.  (CBSE Marking Scheme, 2017) 3 Q. 2. Census is primarily concerned with which three main questions about the population? Explain. R  Ans. The three main questions on which census is primarily concerned with are: (i) Population size and distribution. (ii) Population growth and processes of population change. (iii) Characteristics or qualities of the population. (1×3=3)

Q. 3. Describe the three population density zones of India. A Ans. The three population density zones of India are: (i) High-density zone: The Northern Plains, above 500 people per sq. km. (ii) Moderate or medium density zone: Mountain region, 250–500 people per sq. km. (iii) Low-density zone: Plateau region, below 250 people per sq. km. (1×3=3)

Commonly Made Error

Students mostly get confused about the exact figure.

Answering Tip

Three density zones include high, low and moderate density zone.

Long Answer Type Questions

(v) Political factors: States providing security, peace and stability also become very dense in the long run. (1×5=5)

Q. 1. “Distribution of population in India is uneven.“ Prove this statement with examples. A (Board Term II, 2013) Ans. (i) P  hysical factors: Rugged and mountainous terrain as in Sikkim or lack of means of transport as in Arunachal Pradesh are primarily responsible for sparse population. (ii) Harsh climatic conditions: Snow-covered regions like Ladakh and very hot regions such as Rajasthan invite less population. (iii) Plain terrain: Places with rich, fertile soil, good rainfall and moderate climate like Kerala and Uttar Pradesh make highly populated states. (iv) Economical factors: Regions become densely populated where ample scope of starting business and industries prevail. With good transport, trade and communication these regions become acutely dense.

(5 marks each)

Commonly Made Error

Students were not able to mention the

correct reason for the uneven population distribution.

Answering Tip

Mention the reasons for uneven population

distribution depending on factors such as physical, climatic, political, economic, etc.

Population Growth and Processes of Population

Topic-2 Change Revision Notes

Different patterns of Population Growth:  In a population, some people are born, some die, some migrate internally or internationally. These three processes are known as birth rate, death rate and migration, respectively.  Birth rate: It is the number of live births per thousand persons in a year.  Death rate: It is the number of deaths per thousand persons in a year.  Migration: Migration is the movement of people across regions and territories. It can be internal as well as international.

Scan to know more about this topic

Population size and Distribution

POPULATION

121

Factors affecting the size of a population:  Population grows through births and immigration and declines through deaths and emigration.  Internal migration influences the distribution of population within the nation.  In India, most migrations have been from rural to urban areas because of the “Push” factor in rural areas.  These are adverse conditions of poverty and unemployment in the rural areas and the “pull” of the city in terms of increased employment opportunities and better living conditions.  The urban population has increased from 17.29% of the total population in 1951 to 31.80% in 2011.  There has been a significant increase in the number of ‘million plus cities’ from 35 to 53 in just one decade, i.e., 2001 to 2011.

Age Composition of Population:  Age composition indicates the number of people that belong to different age groups.  The population of a nation is generally grouped into three broad categories : • Children (Below 15 years) • Working age adults (15-59 years) • Aged (Above 59 years)  Dependency ratio is the ratio of the dependent population to the working-age population of the country. The percentage of children and the aged affect the dependency ratio.  Sex ratio is the ratio of males to females in a population. It is calculated as the number of females per 1000 males of the population. The sex ratio in India has always remained unfavourable to females.

Key Terms  Annual growth rate: The rate or pace of population increase. It is studied in per cent per annum.  Immigration: When people come to a country, it is called immigration.  Emigration: When people of a country leave that country, it is called emigration.  Composition of population: The age composition of a population refers to the number of people in different age groups in a country.

OBJECTIVE TYPE QUESTIONS Multiple Choice Questions

(1 mark each)

Q. 2. Look at the Table: India: Sex Ratio 1951-2011. Census year

Q. 1. Identify the most significant feature of the Indian R population. (A) The size of its infant population

Sex ratio (Females per 1000 Males)

1951

946

1961

941

(B) The size of its old age population (C) The size of its adolescent population

1971

930

1981

934

1991

929

2001

933

2011

943

(D) The size of its male population Ans. Option (C) is correct. Explanation: The most significant feature of the Indian population is the size of its adolescent population. It constitutes one-fifth of the total population of India. Adolescents are, generally, grouped in the age group of 10 to 19 years. They are the most important resource for the future.

What is the sex ratio of females (per 1,000 Males) in the year 1961? (A) 946 (C) 930 Ans. Option (B) is correct.

E

(B) 941 (D) 934

122

Oswaal CBSE Question Bank Chapterwise & Topicwise, SOCIAL SCIENCE, Class-IX

Q. 3. Study the given picture carefully:

write with understanding in any language, is treated as literate.

U

Assertion and Reason In the questions given below, there are two statements marked as Assertion (A) and Reason (R). Read the statements and choose the correct option.

Fig. India Age Composition Which age composition contains senior citizens of India? (A) Adults (B) Children (C) Aged (D) None of the above Ans. Option (C) is correct. Q. 4. The magnitude of population growth refers to: E [NCERT]  (A) the total population of an area. (B) the number of persons added each year. (C) the rate at which the population increases. (D) the number of females per thousand males. Ans. Option (B) is correct. Explanation: The absolute numbers added each year or decade is the magnitude of increase. It is obtained by simply subtracting the earlier population from the later population. Q. 5. According to the Census 2001, a “literate” person is U [NCERT] one who: (A) can read and write his/her name. (B) can read and write any language. (C) is 7 years old and can read and write any language with understanding. (D) knows the 3 ‘R’s (reading, writing, arithmetic). Ans. Option (C) is correct. Explanation: According to the Census of 2001, a person aged 7 years and above who can read and



(A) Both (A) and (R) are true and (R) is the correct explanation of (A).



(B) Both (A) and (R) are true and (R) is not the correct explanation of (A).



(C) (A) is correct but (R) is wrong.



(D) (A) is wrong but (R) is correct.

Q. 1. Assertion (A): The numbers, distribution and composition of population are constantly U changing. 

Reason (R): This is the influence of the interaction of the three processes, namely—births, deaths and migrations. Ans. Option (A) is correct. Q. 2. Assertion (A): The Government of India initiated a comprehensive Family Planning Programme in E 1952.

Reason (R): The Family Welfare Programme has sought to promote responsible and planned parenthood on a voluntary basis. Ans. Option (A) is correct. Q. 3. Assertion (A): A well-educated healthy population A

provides potential power. 

Reason (R): People are the nation’s most valuable resource. Ans. Option (A) is correct. Explanation: A well-educated healthy population is important for economic growth, poverty reduction and reducing inequality. By investing in these areas country can create a more prosperous and equitable society for all.

SUBJECTIVE TYPE QUESTIONS Very Short Answer Type Questions

Q. 1. Why is the rate of population growth in India U [NCERT] declining since 1981? Ans. Since 1981, birth rates in India have started declining gradually resulting in a gradual decline in the rate of population growth. The three reasons are as follows:

(2 marks each)

(i) A downward trend of crude birth rate. (ii) An increase in the mean age of marriage. (iii) Improvement in the quality of life, particularly education of females in a country.  (Any Two Points) (1 x 2 = 2)

123

POPULATION

R [NCERT]

Q. 2. Define age structure.

Ans. Age structure is a summary of the number of individuals in the population. Age structure is useful in understanding and predicting population growth. The population of a nation is, generally, grouped into three broad categories: (i) Children (generally below 15 years) (ii) Working Age (15–59 years) (iii) Aged (Above 59 years)

(Any Two) (1 x 2 = 2)

Q. 3. What are the significant features of the National R Population Policy 2000? Ans. Features of the National Population Policy 2000: (i) Reduce infant mortality rate to below 30 per 1000 live births. Reduce maternal mortality ratio to below 100 per 100,000 live births.

against all vaccine preventable diseases. (Any Two) (1 x 2 = 2) Q. 4. Give any two reasons responsible unfavourable sex ratio in India.

for U

Ans. Reasons: (i) Lesser care of female children. (ii) Greater risk to women’s lives, especially at the time of child birth. (iii) Women are also killed or forced to die by the dowry seekers. (iv) Due to illiteracy. (v) Lack of medical facilities for women, etc. (Any Two) (1 x 2 = 2) Q. 5. Describe the trends of migration in India.

U

Ans. In India, most migrations have been from rural to urban areas because of the push factor in rural areas. ‘Push’ factors include adverse conditions of poverty and unemployment in the rural areas and the ‘pull’ of the city in terms of increased employment opportunities and better living conditions.  (2)

(ii) Promote delayed marriage for girls, not earlier than age 18. (iii) Imparting free and compulsory school education up to 14 years of age. (iv) Achieving universal immunisation of children

Short Answer Type Questions Q. 1. What have been the effects of rural-urban migration in India? Explain with examples. U  (Board Term II, 2017) Ans. In India, most migrations have been from rural to urban areas because of the “push” factor in rural areas. These are adverse conditions of poverty and unemployment in the rural areas and the “pull” of the city in terms of increased employment opportunities and better living conditions.  (CBSE Marking Scheme, 2017) (3)

Commonly Made Error

The students tend to take the ‘push’ and ‘pull’ as one and the same thing.

Answering Tip

Push factors encourage people to leave their

points of origin and settle elsewhere; while pull factors attract migrants to new areas.

Q. 2. Mention three main features of population growth. U (Board Term II, 2016) OR What are the three aspects of population that need to be considered? (Board Term II, 2016)

(3 marks each) OR

Explain the process of population change. OR What are the three major components of population growth?

(NCERT) OR



Explain the following terms:

(i) Birth Rate, (ii) Death Rate, (iii) Migration. (NCERT)



Ans. The three features/components of population growth are birth rate, death rate and migration. (i)  Birth Rate: Birth rate is the number of live births per thousand persons in one year. It is a major component of growth, because in India birth rates have always been higher than death rates. (ii)  Death Rate: It is the number of deaths per thousand persons in a year. The main cause of the rate of growth of the Indian population has been the rapid decline in death rates. (iii)  Migration: Migration is the movement of people across regions and territories. Migration can be internal (within the country) or international (between the countries).

(1×3=3)

124

Oswaal CBSE Question Bank Chapterwise & Topicwise, SOCIAL SCIENCE, Class-IX

Q. 3. What is Dependency Ratio? Why is Dependency Ratio higher in India? 

R

Ans. The ratio of people of dependent age (below 15 years and above 60 years) to people of economically active ages (15–59 years) is called Dependency Ratio.

The Dependency Ratio is higher in India because a larger number of children (34.4%) and aged population (6.9%) are found dependent upon the working population (58.7%).



The economically unproductive population needs food, clothing, education and medicare which has to be provided by the working population.



(1½×1½=3)



Q. 4. “Cities are growing faster than the capacity of the economy to support them.” Explain its effects. A (Board Term II, 2013)



Ans. A large number of migrants are being absorbed in mega cities like Delhi, Kolkata and Mumbai. It has created many problems like: (i) Shortage of water supply (ii) Irregular supply of electricity (iii) Lack of education (iv) Inadequate transport (v) Environmental Pollution (CBSE Marking Scheme, 2013) (3) Q. 5. Distinguish between ‘absolute number and the annual growth rate’ with reference to population growth. R Ans. Absolute Number: The magnitude of increase in population in each year or decade minus the earlier population from the current population. Annual Growth Rate: Increase of persons for every 1000 persons in the base population in a given year. (1½+1½=3)

Long Answer Type Questions Q. 1. “There has been a sudden abrupt rise in population of India since 1951.“ Support the statement with reasons. A (Board Term II, 2016) Ans. (i) India’s population has been steadily increasing from 361 million in 1951 to 1210 million in 2011. (ii) It reveals that from 1951 to 1981, the annual rate of population growth was steadily increasing, which explains the rapid increase in population from 361 million in 1951 to 683 million in 1981. (iii) Despite the decline in growth ratio, the number of people being added every decade was steadily increasing. (iv) Since 1981, however, the rate of growth started declining gradually. During this period, birth rates declined rapidly. (v) Still 182 million people were added to the total population in the 1990s alone (an annual addition larger than ever before). (1×5=5)

Q. 2. How does migration play a significant role in changing the composition and distribution of the population of a place? Explain.

Students tend to commit some error to

OR population change”. Explain.



India’s population has been steadily increasing

from 361 million in 1951 to 1210 million in 2011 and also put table that is from 1951 to 2011.

(NCERT)

Ans. (i) Migration is an important determinant of population change. It changes not only the population size, but also the population composition of urban and rural populations in terms of age and sex composition. (ii) In India the rural–urban migration has resulted in a steady increase in the percentage of population in cities and towns. (iii) The urban population has increased from 17.29% of total population in 1951 to 31.80% in 2011. (iv) There has been a significant increase in a number

recognise the sudden abrupt rise in population.

Answering Tip

U

(Board Term II, 2016)

“Migration is an important determinant of

Commonly Made Error

(5 marks each)

of million plus cities from 35 to 53 within 10 years

from 2001 to 2011. (1¼×4=5) (CBSE Marking Scheme, 2016)

Q. 3. “Describe the main features of the adolescent population in India”. A (Board Term II, 2016)

POPULATION

Ans. (i) The most significant feature of the Indian population is the size of its adolescent population. (ii) It constitutes one-fifth of the total population of India. (iii) Adolescents are generally grouped in the age group of 10 to 19 years. (iv) They are the most important resource for the future. (v) Nutrition requirements of adolescents are higher than those of a normal child or adult. (vi) Poor nutrition can lead to deficiency and stunted growth. (vii) But in India, the diet available to adolescents is inadequate in all nutrients.

(Any five) (1×5=5)

(CBSE Marking Scheme, 2016)

Q. 4. What is growth of population? Explain the two ways by which we can express population change. U (Board Term II, 2013) Ans. Growth of population : It refers to the change in the number of inhabitants of a country /territory during a specific period of time, say during the last 10 years. Population change can be expressed in the following ways: (i) In terms of absolute numbers, and (ii) In terms of percentage change per year. (iii) The absolute numbers added each year or decade is the magnitude of increase. It is obtained by simply subtracting the earlier population from the later population. It is referred to as the absolute increase. (iv) The rate or the pace of population increase is the other important aspect. It is studied in percent per annum. This is referred to as the annual growth rate. (Any two) (3+2=5)

COMPETENCY BASED QUESTIONS world’s population?

I. Read the source given below and answer the questions that follow:

(4 marks each)

I1. When was India’s population 17.5 percent of the

Case based MCQs

125

U (1+1+1+1=4)

India’s population as on March 2011 stood at 1,210.6 million, which accounted for 17.5 per cent of the world’s population. These 1.21 billion people are unevenly distributed over our country’s vast area of 3.28 million square km, which accounts for 2.4 per cent of the world’s area. The 2011 Census data revealed that Uttar Pradesh with a population size of 199 million is the most

(A) 2000

(B) 2011

(C) 2012

(D) 2001

Ans. Option (B) is correct.

2. Which state of India had population size of 199 million in the Census of 2011?

(A) Madhya Pradesh

(B) West Bengal

(C) Maharashtra

(D) Uttar Pradesh

Ans. Option (D) is correct.

Explanation: The 2011 Census data revealed that Uttar Pradesh with a population size of 199 million

populous state of India. Uttar Pradesh accounts

is the most populous state of India.

for about 16 per cent of the country’s population. On the other hand, the Himalayan state of Sikkim

3. Population of Uttar Pradesh accounts for about

has a population of just about 0.6 million and

................ percent of the country’s population.

Lakshadweep has only 64,429 people.

(A) 16

(B) 18

Almost half of India’s population lives in just five

(C) 13

(D) 15

states. These are Uttar Pradesh, Maharashtra, Bihar,

Ans. Option (A) is correct.

West Bengal and Andhra Pradesh. Rajasthan, the



biggest state in terms of area, has only 5.5 per cent of the total population of India.

4. The biggest state in terms of area has only ................ percent of the total population of India.

(A) 3.5

(B) 6.5

(C) 5.5 Ans. Option (C) is correct.

(D) 4.5

126

Oswaal CBSE Question Bank Chapterwise & Topicwise, SOCIAL SCIENCE, Class-IX

change. It changes not only the population size but

Subjective based Questions

also the population composition of urban and rural populations in terms of age and sex composition.

I. Read the source given below and answer the questions that follow:

U (1+1+2=4)



The third component of population growth is

questions that follow:

migration. Migration is the movement of people

The percentage of population that is economically

across regions and territories. Migration can

active is an important index of development. The

be internal (within the country) or international

distribution of the population according to different

(between the countries).

types of occupation is referred to as the occupational

Internal migration does not change the size of

structure. An enormous variety of occupations are

population within the nation. Migration plays a

classified as primary, secondary and tertiary.

very significant role in changing the composition

Primary activities include agriculture, animal

and distribution of population.

husbandry,

In India, most migrations have been from rural to

quarrying,

fishing,

Secondary

mining

activities

and

include

work, etc. Tertiary activities include transport,

areas. These are adverse conditions of poverty and

communications, commerce, administration and

unemployment in the rural areas and the “pull” of the city in terms of increased employment

other services.

opportunities and better living conditions.

The proportion of people working in different

Migration

is

an

important

determinant

activities varies in developed and developing

of

countries.

population change. It changes not only the

proportion of population dependent on secondary

urban population has increased from 17.29 per

and tertiary sectors is about 13 and 20 per cent

cent of the total population in 1951 to 31.80 per

respectively. There has been an occupational shift

cent in 2011. There has been a significant increase

in favour of secondary and tertiary sectors because

in the number of ‘million plus cities’ from 35 to 53

of growing industrialisation and urbanisation in

in just one decade, i.e., 2001 to 2011.

2. What factor plays a very significant role in changing the composition and distribution of

recent times. 1. What is an important index of development? Ans. The percentage of population that is economically active is an important index of development.

the composition and distribution of population. 3. Migration

is

an

important

determinant

of

population change. Explain. Ans. Migration is an important determinant of population

2. How are occupations classified?

Ans. Occupations are, classified as primary, secondary and tertiary.

population. Ans. Migration plays a very significant role in changing

high

the population is engaged only in agriculture. The

percentage of population in cities and towns. The

migration .

a

primary activities. In India, about 64 per cent of

migration has resulted in a steady increase in the

Ans. The third component of population growth is

have

higher proportion of their workforce engaged in

and sex composition. In India, the rural-urban

growth?

nations

activities. Developing countries tend to have a

of urban and rural populations in terms of age

1. What is the third component of population

Developed

proportion of people in secondary and tertiary

population size but also the population composition



forestry, etc.

manufacturing industry, building and construction

urban areas because of the “push” factor in rural



A (1+1+2=4)

found in any country. Occupations are, generally,

the population, but influences the distribution of



II. Read the source given below and answer the



3. Why has there been an occupational shift in favour of secondary and tertiary sectors?

Ans. There has been an occupational shift in favour of secondary and tertiary sectors because of growing industrialisation and urbanisation in recent times.

POPULATION

127

Map Work The states having highest and lowest densities of population.



Syllabus

l

Population densities of all states.





UNIT-III

Democratic Politics—I

CHAPTER

9

Learning Objectives

Study Time Max Time: 3:30 hr Max Questions: 81

WHAT IS DEMOCRACY? WHY DEMOCRACY? l Examine and analyse the working structure of the governments of India and North Korea. l Analyse and infer on the different historical processes and forces that have contributed for the promotion of democracy.

Topic-1 Democracy and its Features

TOPIC - 1 Democracy and its Features .... P. 128 TOPIC - 2 Need and Broader Meaning of Democracy .... P. 134

Revision Notes What is Democracy?  The word ‘Democracy’ has been derived from a Greek word ‘Demokratia’. ‘Demos’ means people and ‘Kratia’ means rule. So, democracy is the rule by the people.  Democracy is a form of government in which the rulers are elected by the people.  Democracy is the most prevalent form of government in the world today.

Democratic Government

(i) Government formed by people’s representatives. (ii) Representatives elected by free and fair election. (iii) Citizens have right to oppose any government action and policy. (iv) Citizens have right to protest, as long as the mode of protest is within the boundaries of law.

Non-Democratic Government Scan to know

(i) Rule by force, by a person or by a group of person.



(ii) No opposition is permitted.



(iii) Citizens have no rights.



(iv) Citizens cannot resort to any method of protest.

Features of Democracy

more about this topic

Key Features of Democracy?

Major Decisions by Elected Leaders  In Pakistan, General Pervez Musharraf led a military coup in October 1999. He overthrew a democratically elected government and declared himself the ‘Chief Executive’ of the country. Later, he changed his designation to the President and in 2002 held a referendum in the country that granted him five-year extension.  In August 2002, he issued a ‘Legal Framework Order’ that amended the Constitution of Pakistan. According to this Order, the President can dismiss the national and provincial assemblies.  Though Pakistan has had elections — the elected representatives have fewer powers. The final power vests with military officers and General Musharraf himself.

WHAT IS DEMOCRACY ? WHY DEMOCRACY ?

129

130

Oswaal CBSE Question Bank Chapterwise & Topicwise, SOCIAL SCIENCE, Class-IX

 Clearly, there are many reasons why Pakistan under General Musharraf should not be called a democracy. Among many reasons, the most significant is the power to take final decision rests with army officials and with General Musharraf and none of them was elected by the people.  But in a democracy, the final decision-making power must vest with those who are elected by the people.

Free and Fair Electoral Competition  In China, elections are regularly held after every five years for electing the country’s parliament, called Quanguo Renmin Daibiao Dahui (National People’s Congress). The National People’s Congress has the power to appoint the President of the country.  Only those, who are members of the Chinese Communist Party or eight smaller parties allied to it, were allowed to contest elections held in 2002–2003. The government is always formed by the Communist Party.  Mexico got its independence in 1930, holds elections after every six years to elect its President. The country has never been under a military or dictator’s rule. But in the year 2000, every election was won by a party called PRI (Institutional Revolutionary Party).  Opposition parties in Mexico did contest elections, but never managed to win. It was because the PRI played many dirty tricks to win elections. l But in a democracy, there must be a free and fair election where those currently in power have a fair chance of losing.

One Person, One Vote, One Value  Many instances of denial of equal right to vote in the world: l Until 2015, in Saudi Arabia women did not have the right to vote. l Estonia has made its citizenship rules in such a way that people belonging to Russian minority find it difficult to get the right to vote. l In Fiji, the electoral system is such that the vote of an indigenous Fiji has more value than that of an IndianFijian.  Democracy is based on a fundamental principle of political equality. Thus, in a democracy, each adult citizen must have one vote and each vote must have one value.

Rule of Law and Respect for Rights  Zimbabwe attained independence from White minority rule in 1980. Since then, the country has been ruled by ZANU-PF, the party that led the freedom struggle. Its leader, Robert Mugabe, ruled the country since independence till 2017. Elections have been held regularly and always won by ZANU-PF.  Over the years, his government changed the constitution several times to increase the powers of the President and make him less accountable. Opposition party workers were harassed and their meeting disrupted. He was forced out of office in 2017.  The example of Zimbabwe shows that popular approval of the rulers is necessary in a democracy, but it is not sufficient. If we wish to assess a democracy, it is important to look at the elections.  Thus, in a democratic government, rules within limits are set by constitutional law and citizens’ rights.

Key Fact  Democracy provides an environment that respects human rights and fundamental freedoms, and in which the freely expressed will of people is exercised.

Key Terms  Universal Adult Franchise: Any person who is above 18 years of age has the right to vote, irrespective of caste, colour, status, religion, etc.  Constitutional monarchy: A government headed by a king or queen whose powers are limited by a Constitution.

OBJECTIVE TYPE QUESTIONS Multiple Choice Questions Q. 1. The term ‘democracy’ originated from which R Greek word?  (A) Demoskrati (B) Demokratia (C) Democracia (D) Kratia R

(1 mark each)

Ans. Option (B) is correct. Explanation: The word democracy comes from the Greek words “demos”, meaning people, and “kratos” meaning power; so democracy can be thought of as “power of the people”.

WHAT IS DEMOCRACY ? WHY DEMOCRACY ?

U

Q. 2. Study the given cartoon carefully:

(A) a-i, b-ii, c-iii, d-iv (C) a-iii, b-i, c-iv, d-ii

131

(B) a-ii, b-i, c-iv, d-iii (D) a-iv, b-ii, c-iii, d-i

Ans. Option (A) is correct.

Assertion and Reason In the following questions, a statement of Assertion (A) is followed by a statement of Reason (R). Mark the correct choice as:



Which of the following options best signifies this cartoon?

(A) Building democracy (B) Structured democracy (C) Democratic statue (D) Democratic structure



(A)  Both Assertion (A) and Reason (R) are true, and Reason (R) is the correct explanation of Assertion (A).



(B)  Both Assertion (A) and Reason (R) are true, but Reason (R) is not the correct explanation of Assertion (A).



(C) Assertion (A) is true, but Reason (R) is false.



(D) Assertion (A) is false, but Reason (R) is true.

Q. 1. Assertion (A): Democracy is a form of government in which the rulers are elected by the people.  U

Reason (R): Democracy is people’s rule.

Ans. Option (A) is correct.

Ans. Option (A) is correct.

Explanation: This cartoon was titled ‘Building Democracy ‘and was first published in a Latin American publication. R Q. 3. Which of these is a non-democratic country? (A) USA (B) Africa (C) India (D) Pakistan

Explanation: Democracy is a form of government in which the common people hold political power and can rule either directly or through elected representatives. As Abraham Lincoln said that democracy is a government of the people, by the people and for the people.

Ans. Option (D) is correct.

Q. 2. Assertion (A): In Pakistan, General Pervez Musharraf led a military coup in October 1999. 





Explanation: Pakistan has a federal parliamentary republic type of government.

Q. 4. Who appoints the President of China?  (A) People (B) UNO (C) National People’s Congress (D) Monarch

U, R

Ans. Option (C) is correct. Explanation: The President is elected by the National People’s Congress (NPC), China’s highest state body, which also has the power to remove the president and other state officers from office. Q. 5. Match the correct columns: Column I

Column II

(a)

1999

(i)

General Pervez Musharraf led a military coup

(b)

1930

(ii)

Mexico got its independence

(c)

Until 2015

(iii) Saudi Arabia women did not have the right to vote.

(d) 1980

(iv) Zimbabwe attained independence from White minority rule

Reason (R): He was forced to lead a military coup R with the support of the people.

Ans. Option (C) is correct.

Explanation: General Pervez Musharraf had overthrown Nawaz Sharif in 1999. He overthrew a democratically elected government and declared himself the ‘Chief Executive’ of the country. There was no consent taken from the citizens.

Q. 3. Assertion (A): China is a democratic country.

Reason (R): In China, elections are regularly held after every five years for electing the country’s A parliament. 

Ans. Option (D) is correct.

Explanation: Though the elections are held regularly in China, it is not a democratic country. China is a communist country. Like other democratic countries, any political party cannot participate in the elections of China. The party must be affiliated to the Communist party of China to participate in the elections.

132

Oswaal CBSE Question Bank Chapterwise & Topicwise, SOCIAL SCIENCE, Class-IX

SUBJECTIVE TYPE QUESTIONS Very Short Answer Type Questions Q. 1. Why can’t the democratic set-up in China be U called a true democracy? Explain. Ans. Democracy in China cannot be called a true democracy because: (i) In China, elections do not offer a serious choice. (ii) They have to choose the ruling party and the candidates approved by it. (1 x 2 = 2) Q. 2. Are the following statements in keeping with U democracy as a value? Why? (a) Father to daughter: I don’t want to hear your opinion about your marriage. In our family, children marry where the parents tell them to. (b) Teacher to Student: Don’t disturb my concentration by asking me questions in the classroom. Ans. (a) Father’s statement is undemocratic because every adult has the right to choose his/her life partner. The father does not have the right to impose his choice on his daughter. (b) This statement is also against democracy. The student has the right to ask question. It is undemocratic if this right is denied to the student. (1 + 1 = 2) Q. 3. Citizens of a non-democratic nation can bring about a change in the political system of the country. Write any two traits which can help them U to do so. Ans. Traits are:

(2 marks each)

(i) They should be allowed to actively participate in the politics. (ii) They can form associations. They can protest and take other political actions. (iii) They can express opinions in public. (Any Two Points) (1 x 2 = 2) Q. 4. Why was every election held in Mexico was won by the party called PRI? Explain any two E reasons. Ans. Every election was won by PRI because: (i) Opposition did not contest and never managed to win. (ii) All those who were employed in the government office had to attend party meetings. (iii) Teachers of government schools used to force parents to vote for PRI. (iv) Media did not support the opposition. (Any Two Points) (1 x 2 = 2) Q. 5. Examine the two common features of democratic R countries. Ans. Following are the features of democratic countries: (i) Freedom of speech, expression and thought is a feature of democracy. (ii) Elections are held at regular intervals and are fair and free. (iii) The dignity of an individual is recognised. (Any Two Points) (1 x 2 = 2)

Short Answer Type Questions

(3 marks each)

Q. 1. ‘Major decisions in a democracy must be taken by elected leaders.’ Justify the statement by giving two reasons and a suitable example. U (Board Term I, 2016)

Q. 2. ‘Popular governments can also be undemocratic.’ Justify the statement by giving a suitable example.

Ans. In Pakistan, after passing a law, elections are held to form national and state assemblies. (i) So now, elected representatives must have some powers, but final power vests with military officers. (ii) Those who take final decisions are not elected by the people. (iii) They have formally elected a parliament and government, but the real power is in the hands of those who are not elected. (1 × 3 = 3) [CBSE Marking Scheme, 2016]

Ans. (i) In Zimbabwe, opposition party workers harassed the other party workers and disrupted their meetings.



U (Board Term I, 2016)

(ii) Public protests against government was declared illegal. (iii) There is a law that limits the right to criticise the President. [CBSE Marking Scheme, 2016] (1 × 3 = 3)

WHAT IS DEMOCRACY ? WHY DEMOCRACY ?

Q. 3. Can Zimbabwe be called a democratic country? Give any three examples to support your answer.



U (Board Term I, 2016) OR Write the name of the leader ruling Zimbabwe since 1980. Why cannot his rule be called democratic? Explain any two reasons.

Ans. The leader ruling Zimbabwe since 1980 was Robert Mugabe. His rule cannot be called democratic because of the following reasons: (i) Unfair practices used in elections. (ii) Changes made in the Constitution to increase the powers of the President. (iii) This made him less accountable. (1×3=3) [CBSE Marking Scheme, 2016] Q. 4. How did General Pervez Musharraf become the President of Pakistan? R (Board Term I, 2016, 2015) Ans. General Pervez Musharraf became the President of Pakistan in the following manner : (i) In Pakistan, General Musharraf led a military coup in October 1999. (ii) He overthrew a democratically elected government and declared himself the ‘Chief Executive’ of the country. (iii) Later he changed his designation to the President, and in 2002 he held a referendum in the country that granted him a five year extension. (CBSE Marking Scheme, 2016) (1 × 3 = 3)

Q. 5. Democracy is based on the fundamental principle of political equality. Give two instances of denial of equal right to vote. 

U (Board Term I, 2016) OR Give three examples to prove that political equality is still missing in many countries of the world. (Board Term I, 2015)



Ans. Examples to prove that political equality is still missing in many countries are as follows : (i) Until 2015, in Saudi Arabia women did not have the right to vote. (ii) In Fiji, the electoral system is such that the vote of an indigenous Fiji has more value than that of an Indian-Fiji. (iii) In Estonia, Russian minority finds it difficult to get the right to vote. (1 × 3 = 3) [CBSE Marking Scheme, 2015]

Commonly Made Error

Ans. The rights which the people of non-democratic countries cannot enjoy are: (i) People cannot choose/change rulers. (ii) No real freedom to express opinions. (iii) No freedom to form political associations. (iv) No freedom to organise protests/action. (v) No right to question authority. [CBSE Marking Scheme, 2016] (1 × 5 = 5)

Students only define the democracy but they leave common features of democracy.

Answering Tip

Long Answer Type Questions Q. 1. Enlist the rights/freedoms which people do not enjoy in non-democratic countries. U (Board Term I, 2016)

133

Democracy means for the people, by the people. People also have the freedom to express themselves.

(5 marks each)

(iv) Laws that limits the right to criticise the president were passed. (v) Television and radio are controlled by the government. (vi) Government has ignored some court judgements and went against it. (vii) Government harasses those journalists who go against it. (Any five) (1×5=5) [CBSE Marking Scheme, 2015] Q. 3. Why Pakistan under General Musharraf cannot be called a democracy? State any five reasons. U (Board Term 1, 2014)

Q. 2. Highlight any five unfair practices used by the President of Zimbabwe to win the elections. R (Board Term 1, 2015)

Ans. Pakistan under General Musharraf cannot be called a democracy because :

Ans. Unfair practices used by the President : (i) His government has changed the Constitution several times to increase his power. (ii) Opposition party workers are harassed and their meetings disrupted. (iii) Public protests and demonstrations against the government are declared illegal.

(ii) The power to take final decisions vests with army officials and General Musharraf.

(i) People in Pakistan have elected representatives to the National and Provincial Assemblies, but those elected representatives are not the real leaders.

(iii) They formally have an elected Parliament and government. The work of the civilian cabinet is

134

Oswaal CBSE Question Bank Chapterwise & Topicwise, SOCIAL SCIENCE, Class-IX

supervised by the National Security Council which is dominated by military officials. (iv) The real power in Pakistan is in the hands of military officers and not with locally elected representatives. (v) This cannot be called ‘people’s rule’. General Musharraf was responsible for the coup. (1 × 5 = 5)

Commonly Made Error

In this question students only explain about General Musharraf.

Answering Tip

Pakistan under General Musharraf cannot be

called democracy and the main reason was that final decisions for the country lies with General Musharraf.

Q. 4. “Legal Framework Order of August 2002 in Pakistan was non-democratic in nature.” Justify by giving five reasons. U (Board Term I, 2013) Ans. Reasons are: (i) President can dismiss the National/Provincial Assemblies. (ii) Work of civilian cabinet was supervised by military officers. (iii) National Security Council was dominated by military officers.

(iv) In spite of having elected representatives, final power vests with military officers. (v) Final decisions were taken by those not elected by people. (1×5=5) [CBSE Marking Scheme, 2013] Q. 5. Why can’t Zimbabwe be considered a truly democratic country under Robert Mugabe? State any five reasons. R Ans. Reasons are: (i) The President is powerful and less accountable. (ii) Opposition party workers are harassed. (iii) Public protests and demonstrations are declared illegal. (iv) Media and press are controlled by the government. (v) The government pressurizes the judges and has ignored several court judgements.  (1×5=5)

Commonly Made Error

Students do not pay much focus on Zimbabwe.

Answering Tip

Students should know that opposition parties worker in Zimbabwe were harassed, public protest was illegal. Also, media was under government’s control.

Topic-2 Need and Broader meaning of Democracy Revision Notes Arguments against Democracy      

Leaders keep changing in a democracy. This leads to instability. Democracy is all about political competition and power play. There is no scope for morality. So many people have to be consulted in a democracy that it leads to delays. Elected leaders do not know the best interest of the people. It leads to bad decisions. Democracy leads to corruption as it is based on electoral competition. Ordinary people don’t know what is good for them; they should not decide anything.

Scan to know more about this topic

Why Democracy?

Arguments for Democracy  A democratic government is a better government because it is a more accountable form of government.  Democracy improves the quality of decision making.  Democracy provides a method to deal with differences and conflicts.  Democracy enhances the dignity of citizens. Democracy is better than other forms of government because it allows to correct its own mistakes.

Broader Meaning of Democracy  In countries which we call as democratic, all the people do not rule. A majority is allowed to take decisions on behalf of all the people. Even the majority does not rule directly. The majority of people rule through their elected representatives.

WHAT IS DEMOCRACY ? WHY DEMOCRACY ?

135

 This has become necessary because: l Modern democracies involve such a large number of people that it is physically impossible for Scan to know more about all of them to sit together and take a collective decision. this topic l Even if they could, the citizens do not have the time, the desire or the skills to take part in all the decisions.  It gives us a clear but minimal understanding of democracy and helps us to distinguish democracies from non-democracies. Representative Democracy  A democratic decision involves consultation with and consent of all those who are affected by that decision.  This can apply to a government or a family or any other organisation. Thus, democracy is also a principle that can be applied to any sphere of life.  In a democracy, every citizen must be able to play an equal role in decision making. The working structure of the Government of India and Government of North Korea to have a better and deep understanding of the functioning of both the governments and how they are different from each other in various ways. Serial No.

India

i.

India is a Sovereign, Socialist, Secular, North Korea is an "independent socialist state" with a Democratic Republic with a parliamentary totalitarian dictatorship. form of government which is federal in structure with unitary features.

North Korea

ii.

There is a Council of Ministers with the Prime The head of government is the premier, assisted by Minster as its head to advice. The President several vice-premiers and a cabinet, the members who is the constitutional head of the country. of which are appointed by the Supreme People’s Assembly (SPA).

iii.

Under the Constitution, there are three primary branches of government: the legislative, the executive and the judiciary, whose powers are vested in a bicameral Parliament, President, aided by the Council of Ministers, and the Supreme Court respectively.

In the North Korean , the Cabinet is the administrative and executive body. The North Korean government consists of three branches: administrative, legislative, and judicial. They are not independent of each other, but all branches are under the exclusive political leadership of the Workers' Party of Korea.

iv.

The President of India is the head of state and the Commander-in-Chief of the Indian Armed Forces, whilst the elected Prime Minister acts as the head of the executive and is responsible for running the Union government.

In practice, the government is under one-man leadership of Kim Jong-Un. He is the supreme commander of the Korean People’s Army and general secretary of the Korean Workers’ Party (KWP).

There are several key differences between the two. Let us have a look at few of them:  The Indian Parliament is made up of two houses - the Lok Sabha (lower house) and the Rajya Sabha (upper house) - and members are elected through a democratic voting system. The judiciary is independent, and the Supreme Court is the highest court in the country.  In contrast, North Korea is a totalitarian state with a single-party system. The head of state and government is the Supreme Leader. The Workers’ Party of Korea is the only dominating political party , and the country operates a command economy, with the government controlling all economic activity. There is no independent judiciary in North Korea, and the legal system is controlled by the government.  Overall, while India operates under a democratic system where the citizens have a voice in governance; North Korea operates under an authoritarian system where the government has complete control over the population.

Voting rights of the Indian population Vs the population of Iran  There are significant differences in the voting rights of India and Iran and their way of approaching electoral systems and the level of inclusivity they offer to their respective citizens.  India, as the world’s largest democracy, has a robust and inclusive electoral system. Universal adult suffrage is guaranteed under the Indian Constitution, granting voting rights to all citizens above the age of 18. This inclusive approach ensures that every eligible citizen, regardless of their gender, religion, caste, or socioeconomic background, has the right to vote. India conducts periodic elections at the national, state, and local levels, allowing citizens to choose their representatives through a secret ballot system. The Election Commission of India oversees the electoral process, ensuring free and fair elections.

136

Oswaal CBSE Question Bank Chapterwise & Topicwise, SOCIAL SCIENCE, Class-IX

 In contrast, Iran’s voting rights are subject to certain limitations and restrictions. While the country holds elections, including presidential and parliamentary polls, the process is tightly controlled by the theocratic regime. Iran follows an Islamic Republic System, where the Supreme Leader holds ultimate authority, and the Guardian Council, a body of clerics, screens candidates for elections. This vetting process excludes candidates deemed unfit or opposed to the principles of the Islamic Republic, limiting the choices available to voters. Additionally, certain groups, such as religious and ethnic minorities, face challenges in exercising their voting rights.  In conclusion, while India upholds a comprehensive and inclusive voting rights framework, Iran’s system places certain restrictions on the electoral process, limiting the choices available to its citizens. Both countries face their own unique challenges, and ongoing efforts are needed to ensure that voting rights are protected, expanded, and made accessible to all eligible individuals, regardless of their background or beliefs.

Key Terms  Minimal democracy: A system of government in which citizens give teams of political leaders, the right to rule in periodic elections.  Representative democracy: A type of democracy founded on the principle of elected officials representing a group of people, as opposed to direct democracy.

Example

‘Democracy is an ideal form which is not limited to the government alone. It goes far beyond and has a broader meaning.’ Justify. Answer: Step 1: In order to justify the statement, we can make use of the term democracy for the organisation other than the government. Step 2: Democratic family: All sit down and take a decision. Everyone’s opinion matters.



Step 3: Democratic temperament: The teacher should allow students to ask questions in class.



Step 4: One leader and his family members take decisions – this is not democracy.



Step 5: True democracy will come to the country only when no one goes hungry to bed.

OBJECTIVE TYPE QUESTIONS Multiple Choice Questions Q. 1. Which is the most common form of democracy in U today’s world? (A) Representative Democracy (B) Direct Democracy (C) Presidential Democracy (D) Participatory Democracy Ans. Option (A) is correct. Explanation: The most common form of democracy is representative democracy. Modern democracies involve such a large number of people that it is physically impossible for them to sit together and take a collective decision. Q. 2. On what does the fate of a democratic country A mainly depend on? (A) Constitution (B) Politicians (C) What we as responsible citizens do (D) Foreign policies Ans. Option (C) is correct.



(1 mark each)

Explanation: Citizens have a responsibility to participate in their government by registering to vote and voting in elections. By voting, citizens have a voice in their government and help ensure that the democratic representative system of government is maintained. Hence, the role of citizens is vital for the success of the democracy. Q. 3. What is the basis of a good democratic government? U  (A) It functions according to its intuition. (B) It allows only a few people to enjoy rights (C) It consists of financially sound group of people (D) It is accountable Ans. Option (D) is correct. Explanation: A democratic government is an accountable form of government which improves the quality of decision making, provides a method to deal with differences and conflicts and also enhances the dignity of citizens. Q. 4. How is the head of state selected in India? (A) Elected by the people (B) Appointed by the Prime Minister

WHAT IS DEMOCRACY ? WHY DEMOCRACY ?

(C) Chosen by the Parliament (D) Hereditary monarchy Ans. Option (A) is correct. Explanation: The head of state in India is elected by an Electoral College consisting of elected members of both houses of Parliament and of Legislative Assemblies of States. Q. 5. How are laws created and enforced in India? (A) By the Prime Minister (B) By the President (C) By the Parliament (D) By the military Ans. Option (C) is correct. Q. 6. What is the process for voter registration in India? (A) Automatic registration (B) Voluntary registration (C) Compulsory registration (D) Biometric registration Ans. Option (B) is correct. Explanation: In India, voter registration is voluntary. Citizens above the age of 18 can register to vote by filling out a form and submitting it to their local election office or online through the National Voter Service Portal. Biometric registration is also available as an option for voters. Q. 7. What is the minimum age for voting in Iran? (A) 16 years (B) 18 years (C) 21 years (D) 25 years Ans. Option (B) is correct. Explanation: Citizens over the age of 18 are eligible to vote in Iran. Therefore, the minimum age for voting in Iran is 18 years.

137

Assertion and Reason In the following questions, a statement of Assertion (A) is followed by a statement of Reason (R). Mark the correct choice as:

(A)  Both Assertion (A) and Reason (R) are true, and Reason (R) is the correct explanation of Assertion (A).



(B)  Both Assertion (A) and Reason (R) are true, but Reason (R) is not the correct explanation of Assertion (A).



(C) Assertion (A) is true, but Reason (R) is false.



(D) Assertion (A) is false, but Reason (R) is true.

Q. 1. Assertion (A): Democracy is better than other forms of government. 

U

Reason (R): It allows us to correct its own mistakes.

Ans. Option (A) is correct. Q. 2. Assertion (A): Clearly, democracy is not a magical solution for all the problems. 

E

Reason (R): It has not ended poverty in our country and in other parts of the world.

Ans. Option (A) is correct.

Explanation: It is indeed the most convenient way of governance. Democratic government is not the solution to all the problems world is facing right now. The efforts made in democratic governments to reduce poverty are not effective at all. The inequalities between poor and rich are still increasing.

SUBJECTIVE TYPE QUESTIONS Very Short Answer Type Questions Q. 1. How the principles of democracy can be applied to all spheres of life? 

U

Ans. The principles of democracy can be applied to all spheres of life: (i) A democratic decision involves consultation with and consent of all those who are affected by that decision. (ii) Those who are not powerful, have the same say in taking the decision as those who are powerful. This can apply to a government or a family or any other organization. (iii) Thus, democracy is also a principle that can be applied to any sphere of life. (Any Two) (1 × 2 = 2) Q. 2. “Democracy is based on consultation and discussion.” Explain this statement. E

(2 marks each)

Ans. This statement states that: (i) Democratic decisions always involve many persons, discussions and meetings. (ii) When a number of people put their head together, they are able to point out their mistakes. (iii) It takes time, but there is a big advantage in taking time over important decisions. (Any Two) (1 × 2 = 2) Q. 3. ‘‘Democracy is better than any other form of government, because it allows to correct its own mistakes.’’ Justify the statement with the help of three examples. E Ans. Examples: (i) In a democracy, mistakes cannot be hidden for long. (ii) There is space for public discussion.

138

Oswaal CBSE Question Bank Chapterwise & Topicwise, SOCIAL SCIENCE, Class-IX

(iii) There is room for correction. (iv) Either ruler has to change their decisions or the rulers can be changed.  (Any Two) (1 × 2 = 2)

(iii) This takes time, but reduces the chances of a rash/ irresponsible decision. (Any Two) (1 × 2 = 2)

Q. 4. ‘Democracy improves the quality of decision making.’ Explain. U Ans. Reasons are: (i) A democratic decision involves many persons, discussion, debates, etc. (ii) A number of people together can point out the possible mistakes in any decision.

Q. 5. How do voting rights in India compare to those in Iran? Ans. India has a more comprehensive and inclusive approach to voting rights than Iran, which places certain restrictions on the electoral process and limits voter choices through a vetting process. (2)

Short Answer Type Questions Q. 1. “Democracy is considered the best form of government and is clearly better than any other alternative.” A (Board Term I, 2016) Ans. Democracy is a better form of government because: (i) It offers better chances of a good decision. (ii) It respects people’s wishes and allows different kinds of people to live together. (iii) It offers more dignity to all citizens and allows a way of correcting mistakes. (1×3=3) [CBSE Marking Scheme, 2016]

Commonly Made Error

Students generally define democracy and are unable to explain it with suitable examples.

Answering Tip

Students must know that democracy allows better chance and respect to people’s wish.

Q. 2. What do you understand by the “broader meaning of democracy”? Explain in three points. U (Board Term I, 2016) Ans. The broader meaning of democracy signifies that : (i) Democracy is a principle that can be applied to any sphere of life. (ii) The most common form of democracy in today’s world is rule through people’s elected representatives. (iii) Every democracy has to try to realise the ideals of a democratic decision making. This cannot be achieved once and for all. It requires a constant effort to save and strengthen democratic forms of decision making. (iv) Democracy depends on active political participation by all the citizens. That is why, a study of democracy must focus on democratic politics. (Any three) (1×3=3)

(3 marks each)

Q. 3. How does democracy provide a method to deal with differences and conflicts? U (Board Term I, 2015)



OR ‘Democracy keeps a diverse country like India together.’ How? (Board Term I, 2014)

Ans. A method provided by the democracy is: (i) No one is a permanent winner or loser in a democracy as the government keeps on changing every 5 years. (ii) Different groups can live together peacefully as India is democratic and has diversity. (iii) Democracy keeps us together — all are equal in the eyes of laws. There is no discrimination in democracy. [CBSE Marking Scheme, 2015] (1 × 3 = 3) Q. 4. What distinguishes democracy from other forms of government? Explain by giving three points. U (Board Term I, 2015) Ans. Democracy is different from other forms of government because here: (i) Citizens must take part in politics. (ii) Democracy depends on active political participation by all citizens. (iii) The fate of the country depends on not just who rules, but also on what we as responsible citizens do. [CBSE Marking Scheme, 2015] (1×3=3)

Commonly Made Error

Students are unable to understand the various forms of government.

Answering Tip

Students should know that in democracy, the

country is governed by its own people with active political participation.

WHAT IS DEMOCRACY ? WHY DEMOCRACY ?

Q. 5. What weaknesses of democracy do you observe in India? U (DDE–2014) OR Explain the main challenges to democracy. Ans. Although democracy has been accepted as the best form of government in the modern world, yet it has its own problems. They are: (i) Growing economic and social inequalities among the people: Although all the citizens have the right to vote and fight elections, yet only rich people have a chance to win the election. (ii) Role of anti-social elements: Voters are forced to vote for a particular candidate or party. Rigging also takes place during the elections. (iii) Corruption and inefficiency: In many democratic countries of the world, political leaders and government officials are corrupt, dishonest and inefficient. This affects the working of democracy very badly. (iv) Casteism and communalism : There are other big challenges in many democratic countries like India. During election, a large number of voters give vote to the caste and religion of the candidate. (v) Political parties also keep in mind the caste or religion of a person while distributing the tickets of the election. (Any three) (1 × 3 = 3)

Commonly Made Error

Democracy is thought to be the best form of government with no flaws.

Answering Tip

Students must know the biggest flaw of democracy is that corruption is prevalent at every level.

Q. 6. How is the head of state selected in India and North Korea? Ans. In India, the President is the constitutional head of the country and is elected by an Electoral College consisting of elected members of both houses of Parliament and of Legislative Assemblies of States. In North Korea, the head of state and government is the Supreme Leader, who is selected by the Workers’ Party of Korea. (3) Q. 7. How does the judiciary system in India and North Korea work, and what are the major differences between the two countries? Ans. In India, the judiciary is independent, and the Supreme Court is the highest court in the country. The Constitution of India provides for an integrated judicial system with a three-tier structure: the Supreme Court, High Courts, and subordinate courts. The Supreme Court has original, appellate, and advisory jurisdiction. In contrast, North Korea does not have an independent judiciary system. The legal system is controlled by the government, and there is no separation of powers between the legislative, executive, and judicial branches of government. (3) Q. 8. What are the eligibility criteria for voting in India and Iran, and how do they compare? Ans. India has a more comprehensive and inclusive approach to voting rights than Iran. India guarantees universal adult suffrage under the Indian Constitution, granting voting rights to all citizens above the age of 18, regardless of their gender, religion, caste, or socioeconomic background. In Iran, citizens over the age of 18 are eligible to vote, but certain groups such as religious and ethnic minorities face challenges in exercising their voting rights due to discrimination and intimidation.  (3)

Long Answer Type Questions Q. 1. Rulers and the ruled both are responsible for making an ideal democracy. Justify by giving suitable examples. A (Board Term I, 2016) OR Can the citizens of the country turn a simple democracy into a good democracy? Give five reasons in support of your arguments. (Board Term I, 2015) Ans. The citizens of the country can turn a simple democracy into a good democracy by the following ways : (i) What we do as citizens can make a difference in making our country more or less democratic. (ii) This is the strength and weakness of democracy.

139

(5 marks each)

(iii) The fate of our country depends on not only what rulers do, but also on what we as citizens do. (iv) Citizens can take part in politics. (v) Democracy depends on active political participation of all citizens. (1×5=5) [CBSE Marking Scheme, 2015] Q. 2. How far was the Communist Government responsible for the famine that occurred in China in 1958–1961? Explain. U (Board Term I, DDE–2014) Ans. (i) China’s famine of 1958–1961 was the worst recorded famine in the world history. Nearly three crore people died in this famine. During those days,

140

Oswaal CBSE Question Bank Chapterwise & Topicwise, SOCIAL SCIENCE, Class-IX

India’s economic condition was not much better than China. Yet, India did not have a famine of the kind in China. (ii) The Communist Government of China did shut down the gates for international business (import primarily) so, when they had poor food production and also they were not accepting international import, it led to problems which promoted the famine of 1958. (iii) The government wanted to revolutionise farming by discarding existing knowledge in favour of new techniques. Farmers were ordered to ‘closeplant’ (sowing millions of seeds of different species together in a small area) and ‘deep-plough’ (digging the ground much deeper to encourage deep root growth). Both these experiments failed and entire plantings yielded next to nothing. Farmers were forbidden to use chemical fertilisers and large amounts of land were left fallow, with poor results. (iv) A more tangible cause of famine was grain procurement by the state. From 1953, all Chinese farmers were required to sell grains to the government, at prices and levels decided by the government. Some of these grains were redistributed back to the farmers. (v) Most, however, was either sent to the cities (the so-called ‘urban food bias’), sold as export grains or distributed as foreign aid, to create the illusion of a booming economy. By the summer of 1959, however, food shortages had reached a critical point. Mao Zedong’s response was to attack his critics rather than relax his policies. (1 × 5 = 5)

Commonly Made Error

In this question, students only explain the causes of famine in China.

Answering Tip

Students should know that major concern

included that if China had democratic government, so many people would not have lost their lives in the famine.

Q. 3. Write any three arguments in favour and two against democracy as a form of government. U (Board Term I, 2014) Ans. Arguments in favour of democracy : (i) Democratic form of government is more accountable. (ii) It improves the quality of decision making. (iii) It provides a method to deal with differences and conflicts. (iv) It enhances the dignity of citizens. (v) It allows us to correct our mistakes. (Any three) Arguments against democracy :

(i) Change of leaders leads to instability. (ii) It involves only political competition with no scope for morality. (iii) Consulting more people leads to delays. (iv) It leads to corruption. (v) Ordinary people don’t know that what is good for them. (Any two) (3 + 2 = 5) Q. 4. What are some key differences between a democratic country like India and a totalitarian dictatorship like North Korea? Ans. India operates under a democratic system where citizens have a voice in governance, while North Korea operates under an authoritarian system where the government has complete control over the population. India is a Sovereign Socialist Secular Democratic Republic with a parliamentary form of government which is federal in structure with unitary features. The Parliament is made up of two houses - the Lok Sabha (lower house) and the Rajya Sabha (upper house) - and members are elected through a democratic voting system. The judiciary is independent, and the Supreme Court is the highest court in the country. In contrast, North Korea is a totalitarian state with a singleparty system. The head of state and government is the Supreme Leader. The Workers’ Party of Korea is the only political party allowed, and the country operates a command economy, with the government controlling all economic activity. There is no independent judiciary in North Korea, and the legal system is controlled by the government. (5) Q. 5. What are the key differences between India and Iran’s approach to voting rights? Ans. There are significant differences between India and Iran’s approach to voting rights. While India upholds a comprehensive and inclusive voting rights framework, Iran’s system places certain restrictions on the electoral process, limiting the choices available to its citizens. Iran follows an Islamic Republic system, where the Supreme Leader holds ultimate authority, and the Guardian Council screens candidates for elections. This vetting process excludes candidates deemed unfit or opposed to the principles of the Islamic Republic, further limiting voter choices. Additionally, certain groups such as religious and ethnic minorities face challenges in exercising their voting rights in Iran.  (5) Q. 6. What role does the Election Commission of India play in overseeing the electoral process? Ans. The Election Commission of India oversees the electoral process in India. The commission is an independent body responsible for conducting free and fair elections at the national, state, and local levels. Its role includes preparing electoral rolls, supervising polling stations, monitoring election

WHAT IS DEMOCRACY ? WHY DEMOCRACY ?

expenses, and enforcing the model code of conduct to ensure a level playing field for all candidates. The commission also has the power to cancel elections in case of malpractice or irregularities and can order

repolling in affected areas. Overall, the Election Commission of India plays a crucial role in ensuring that India’s electoral process is transparent and democratic. (5)

COMPETENCY BASED QUESTIONS Case based MCQs I. Read the source given below and answer the C (1+1+1+1=4) questions that follow: China’s famine of 1958–1961 was the worst recorded famine in world history. Nearly, three crore people died in this famine. During those days, India’s economic condition was not much better than China. Yet, India did not have a famine of the kind China had. Economists think that this was a result of different government policies in the two countries. The existence of democracy in India made the Indian Government respond to food scarcity in a way that the Chinese government did not. They point out that no large-scale famine has ever taken place in an independent and democratic country. If China too had multiparty elections, an opposition party and a press free to criticise the government, then so many people may not have died in the famine. This example brings out one of the reasons why democracy is considered the best form of government. Democracy is better than any other form of government in responding to the needs of the people. A non-democratic government may and can respond to the people’s needs, but it all depends on the wishes of the people who rule. If the rulers don’t want to, they don’t have to act according to the wishes of the people. A democracy requires that the rulers have to attend to the needs of the people. A democratic government is a better government because it is a more accountable form of government. 1. When did the worst famine struck China? (A) 1955–1958 (B) 1958 –1961 (C) 1956–1959 (D) 1959 –1962 Ans. Option (B) is correct. Explanation: Sixty years ago China was in the middle of the world’s largest famine: between the spring of 1958 and the end of 1961, some 30 million Chinese starved to death and about the same number of births were lost or postponed. 2. During those days, how was the economic condition of India? (A) It was much better than China (B) It was little better than China

141

(4 marks each)

(C) It was same like China (D) It was not much better than China Ans. Option (D) is correct. 3. Democracy is better that any other form of government in responding to ............... of the people. (A) wants (B) needs (C) requirements (D) aspirations Ans. Option (B) is correct. 4. A democratic government is better than any other government because it is: (A) non-accountable government (B) semi-accountable government (C) accountable (D) None of the above Ans. Option (C) is correct.

Subjective based Questions I. Read the source given below and answer the questions that follow:  (1+1+2=4) Rule of law and respect for rights Zimbabwe attained independence from White minority rule in 1980. Since then, the country has been ruled by ZANU-PF, the party that led the freedom struggle. Its leader, Robert Mugabe, ruled the country since independence. Elections were held regularly and always won by ZANUPF. President Mugabe was popular but also used unfair practices in elections. Over the years his government changed the Constitution several times to increase the powers of the President and make him less accountable. Opposition party workers were harassed and their meetings disrupted. Public protests and demonstrations against the government were declared illegal. There was a law that limited the right to criticise the President. Television and radio were controlled by the government and gave only the ruling party’s version. There were independent newspapers but the government harassed those journalists who went against it. The government ignored some court judgments that went against it and pressurised judges. He was forced out of office in 2017. 1. Who was Robert Mugabe? Ans. Robert Gabriel Mugabe was a Zimbabwean politician. He was the President of Zimbabwe from

142

Oswaal CBSE Question Bank Chapterwise & Topicwise, SOCIAL SCIENCE, Class-IX

1987 to 2017. Before that, he was Prime Minister, the head of government, after being elected in 1980. For many years before he resigned, Mugabe ruled his country in the style of a dictator.

2. Which nation justified the example that ‘popular governments can be undemocratic’? Ans. Zimbabwe justified the example that ‘popular governments can be undemocratic.

3. What do you know about ZANU-PF?

Ans. The Zimbabwe African National Union – Patriotic Front (ZANU–PF) is a political organisation which has been the ruling party of Zimbabwe since independence in 1980 II. Read the source given below and answer the questions that follow:  (1+1+2=4) In Pakistan, General Pervez Musharraf led a military coup in October 1999. He overthrew a democratically elected government and declared himself the ‘Chief Executive’ of the country. Later he changed his designation to President and in 2002 held a referendum in the country that granted him a five-year extension. Pakistani media, human rights organisations and democracy activists said that the referendum was based on malpractices and fraud. In August 2002, he issued a ‘Legal

Framework Order’ that amended the Constitution of Pakistan. According to this Order, the President can dismiss the national and provincial assemblies. The work of the civilian cabinet is supervised by a National Security Council which is dominated by military officers. After passing this law, elections were held to the national and provincial assemblies. So, Pakistan has had elections, elected representatives have some powers. But the final power rested with military officers and General Musharraf himself. 1. What was the outcome of the military coup led by General Pervez Musharraf in October 1999? Ans. General Pervez Musharraf overthrew a democratically elected government and declared himself the ‘Chief Executive’ of the country. 2. The Referendum of 2002 granted him what? Ans. The referendum allowed Pervez Musharraf to continue as the President of Pakistan for five years. 3. According to the ‘Legal Framework Order’, who could dismiss the national and provincial assemblies?  Ans. According to the ‘Legal Framework Order’, the President could dismiss the national and provincial assemblies 

Study Time Max Time: 3:30 hr Max Questions: 81

CHAPTER

10 Learning Objectives

Topic-1

CONSTITUTIONAL DESIGN l Comprehend the purpose of

constitution. l Enumerate the essential features that need to be kept in mind while drafting any constitution. l Examine the guiding values that created the Indian constitution. l Comprehend the roles and responsibilities as citizens of India.

Democratic Constitution in South Africa

Revision Notes

TOPIC - 1 Democratic Constitution in South Africa .... P. 143 TOPIC - 2 Why do we Need a Constitution? Making of the Indian Constitution .... P. 148

There are certain basic rules that the citizens and the government have to follow. All such rules together are called the Constitution. As the supreme law of the country, the Constitution determines the rights of citizens, the powers of the government and how the government should function.

Democratic Constitution in South Africa Struggle against Apartheid

 Apartheid was the system of racial discrimination or segregation on the grounds of race unique to South Africa. The Europeans imposed this system on South Africa. The system of apartheid divided the people and labelled them on the basis of their skin colour.



 The apartheid system was particularly oppressive for the blacks. • The non-whites did not have the voting rights. • They were forbidden from living in white areas. • They could work in white areas only if they had a permit. • Trains, buses, taxis, hotels, hospitals, schools and colleges, libraries, cinema halls, theatres, beaches, swimming pools, public toilets, were all separate for the whites and blacks. This was called ‘segregation’. • They could not even visit the churches where the whites worshipped. • Blacks could not form associations or protest against the terrible treatment.



Scan to know more about this topic

Constitution of South Africa

 The African National Congress (ANC) was the umbrella organisation that led the struggle against the policies of segregation. This included many workers’ unions and the Communist Party. Many sensitive whites also joined the ANC to oppose apartheid and played a leading role in this struggle.

Towards a New Constitution

 Finally, at the midnight of 26 April 1994, the new national flag of the Republic of South Africa was unfurled, marking the democracy in the world. The apartheid government came to an end, paving way for the formation of a multi-racial government.

144

Oswaal CBSE Question Bank Chapterwise & Topicwise, SOCIAL SCIENCE, Class-IX

CONSTITUTIONAL DESIGN



145

 After two years of discussion and debate, they came out with one of the finest constitutions the world has ever had. The characteristics of the South African Constitution are : •

This constitution gave its citizens the most extensive rights available in any country.



Together, they decided that in the search for a solution to the problems, nobody should be excluded; no one should be treated as a demon.



They agreed that everybody should become part of the solution.

Key Terms

 Treason: The offence of attempting to overthrow the government of the state for which the offender owes allegiance.



 Apartheid: The official policy of racial separation and ill-treatment of blacks followed by the Government of South Africa between 1948 and 1989.

OBJECTIVE TYPE QUESTIONS Multiple Choice Questions Q. 1. Study the given picture carefully: 

This picture is related to which country? (A) South Africa (B) England (C) India (D) Australia Ans. Option (A) is correct. Q. 2. Study the given picture carefully:

U

(1 mark each)

Q. 3. The apartheid government came to an end, paving way for the formation of a ________ government. U  (A) multi–racial (B) multi-faced (C) multi-ideated (D) Both B & C Ans. Option (A) is correct. Q. 4. On what charge was Nelson Mandela tried by the R white South African Government?  (A) Practicing discrimination (B) Treason (C) Amend the Constitution (D) Seeking foreign help Ans. Option (B) is correct. Q. 5. Which of these tactics did the white racist Government of South Africa use to continue to U rule? 

U

(A) Stealing (C) Teasing Ans. Option (D) is correct.

(B) Laughing (D) Detaining

Assertion and Reason Directions: In the following questions, a statement of Assertion (A) is followed by a statement of Reason (R). Mark the correct choice as:

This image captures the spirit of which country? (A) America (B) South Africa (C) Australia (D) India Ans. Option (B) is correct. Explanation: This image captures the spirit of South Africa. South Africans call themselves a ‘rainbow nation’.



(A) Both Assertion (A) and Reason (R) are true, and Reason (R) is the correct explanation of Assertion (A).



(B) Both Assertion (A) and Reason (R) are true, but Reason (R) is not the correct explanation of Assertion (A).



(C) Assertion (A) is true, but Reason (R) is false.

(D) Assertion (A) is false, but Reason (R) is true. Q. 1. Assertion (A): The non-whites did not have the U voting rights.  Reason (R): The white rulers treated non-whites as inferiors.

146

Oswaal CBSE Question Bank Chapterwise & Topicwise, SOCIAL SCIENCE, Class-IX

Ans. Option (A) is correct.

Explanation: The apartheid system was particularly oppressive for the blacks.

Q. 2. Assertion (A): The South African constitution E inspires democrats all over the world.



Reason (R): A state denounced by the entire world till recently as the most undemocratic one is now seen as a model of democracy.

Ans. Option (A) is correct.

SUBJECTIVE TYPE QUESTIONS Short Answer Type Questions Q. 1. Explain the term apartheid. What were its U (Board Term-I, 2016) implications? Ans. Apartheid was the system of racial discrimination unique to South Africa. Implications: (i) White Europeans imposed this system on South Africa. (ii) White rulers treated non-whites as inferiors. (iii) Non-whites did not have the right to vote. They were forbidden to live in white areas. They were allowed to work there only with a permit. (iv) Trains, buses, hospitals, schools, cinemas, public places and public toilets were all separate for the whites and blacks.  (Any two)  (1 + 2 = 3) (CBSE Marking Scheme, 2016) Q. 2. Explain any three characteristics of South African U (NCT–2014, Term I) Constitution.  Ans. Characteristics: (i) Emergence of the new democratic South Africa under black leaders. (ii) Based on equality of all races, men and women. (iii) Extensive rights were given to all the citizens. (iv) Everybody becomes the part of the solution. (v) Determination of the people to work together.  (Any Three Points) (1×3=3) Q. 3. Explain the efforts made to form a new constitution after attaining democracy in South U Africa. Ans. Efforts: (i) They thought of building a new South Africa. (ii) They sat together to draw up a new constitution. (iii) They came out with the finest constitution through discussions. (iv) It provided the most extensive rights to the citizens. (Any Three) (1×3=3)

(3 marks each)

Q. 4. “The South African Constitution inspires democrats all over the world”. Justify the U statement. Ans. Justification of the statement: (i) It gave the citizens the most extensive rights available in any country. (ii) All the communities sat down in an atmosphere of reconciliation and became a part of the solution. (iii) The Constitution was based on equality of sex, democracy and social justice. (1 × 3 = 3) Q. 5. How did the people of South Africa struggle E against apartheid? Ans. Apartheid was the system of racial discrimination, unique to South Africa. Efforts made by the people of South Africa against apartheid are: (i) They launched protests, marches and strikes. (ii) The African National Congress led the struggle. (iii) Many sensitive whites also joined the struggle with the African National Congress. (iv) Many workers’ unions and the Communist Party joined the movement.  (Any Three) (1 x 3 = 3)

Commonly Made Error

The students forget to give the role of the ANC and also forget to give the example.

Answering Tip

Discuss that people launched protests and

strikes and ANC led the struggle joined by majority of the citizens.

CONSTITUTIONAL DESIGN

Long Answer Type Questions Q. 1. Explain the necessity of a constitution in the newly born democracy in South Africa. U (Board Term-I, 2016)  Ans. The necessities of a constitution in the newly born democracy in South Africa are: (i) The oppressor and the oppressed were planning to live together as equals. (ii) It was not going to be easy to trust each other. (iii) Black majority was keen to ensure that the democratic principle of majority rule was not compromised. (iv) The Blacks wanted social and economic rights. (v) White minority was keen to protect its privileges and property. (1 × 5 = 5)  (CBSE Marking Scheme, 2016) Q. 2. Who led the struggle against apartheid? State any four practices followed in the system of apartheid R (Board Term-I, 2015) in South Africa. Ans. Nelson Mandela led the struggle against apartheid. Practices followed in the system of apartheid in South Africa: (i) Blacks were forbidden from living in white areas. (ii) All public places were separate for the whites and blacks. (iii) They could not visit the churches of whites. (iv) They could not form associations or protest against the terrible treatment.  (1 + 4 = 5) (CBSE Marking Scheme, 2015)

Ans. Transition that took place in South Africa: (i) After the emergence of the new democratic South Africa, the black leaders appealed to the fellow blacks to forgive the whites for the atrocities they had committed while in power. (ii) They built a new South Africa based on equality of all races and men and women, on democratic values, social justice and human rights. (iii) The party that ruled through oppression and brutal killings and the party that led the freedom struggle, sat together to draw up a common constitution. (iv) After two years of discussion and debate, they came out with one of the finest constitutions, the world has ever had. (v) This constitution gave to its citizens the most extensive rights available in any country. Together, they decided that in the search for a solution to the problems, nobody should be excluded, no one should be treated as a demon.  (CBSE Marking Scheme, 2015) (1×5=5)

Commonly Made Error

Students did not mention the role of Nelson



Students have to mention the role of Nelson

Mandela. For example; He was sentenced to life imprisonment in 1964 for daring to oppose the apartheid regime in his country.

Students forget to clarify, how the black people

were struggling against white and how they came out of it.

Answering Tip

Mandela.

Answering Tip

(5 marks each)

Q. 3. Explain with five facts of the transition of apartheid to democracy and multi-racial governR ment that took place in South Africa.  (Board Term-I, 2015)

Commonly Made Error

147



Students have to mention how did the historical

enemies succeeded in negotiating a peaceful transition from apartheid to democracy exactly because they were prepared to accept the inherent capacity for goodness in the other. And also a new democratic South Africa was formed where Black and White were treated equally.

148

Oswaal CBSE Question Bank Chapterwise & Topicwise, SOCIAL SCIENCE, Class-IX

Topic-2

Why do we Need a Constitution? Making of the Indian Constitution.

Revision Notes  When we see the example of South Africa, then we understand why we need the constitution and what constitutions do. Scan to know

How was this Compromise to be Implemented?

more about this topic

 The constitution of a country is a set of written rules that are accepted by all people living together in a country. The constitution is the supreme law that determines the relationship among people living in a territory and also the relationship between the people and government. Why do we Need  A constitution does many things: Constitution? l It generates a degree of trust and coordination that is necessary for different kind of people to live together. l It specifies how the government will be constituted, who will have power to take which decisions. l It lays down limits on the powers of the government and tells us the rights of the citizens. l It expresses the aspirations of the people about creating a good society.  The Constitution of India was drawn up under very difficult circumstances.  The country was born through a partition on the basis of religious differences and was a traumatic experience for the people of India and Pakistan.  The British had left it to the rulers of the princely states to decide whether they wanted to merge with India or with Pakistan or remain independent.  The merger of these princely states was a difficult and uncertain task.  When the Constitution was being written, the future of the country did not look as secure as it does today.

The Making of the Constitution  In 1928, Motilal Nehru and the eight other Congress leaders drafted a Constitution for India.  In 1931, the resolution at the Karachi session of the Indian National Congress dwelt on how independent India’s Constitution should look like. Both these documents were committed to the inclusion of Universal Adult Franchise, right to freedom and equality and to protecting the rights of minorities in the Constitution of independent India.  Our leaders gained confidence to learn from other countries, but on our own terms.

The Constituent Assembly  The drafting of the document called the Constitution was done by an assembly of elected representatives called the Constituent Assembly.  Elections to the Constituent Assembly were held in July 1946. The Assembly adopted the Constitution on 26th November 1949, but it came into effect on 26th January 1950. To mark this day, we celebrate January 26 as Republic Day every year.

Guiding Values of the Indian Constitution  First, understand the overall philosophy of what our Constitution is all about.  Read the views of some of our major leaders on our Constitution and read what the Constitution says about its own philosophy.

Scan to know more about this topic

 This is what the preamble to the Constitution does.

The Dream and the Promise  There were many members who followed the vision of Mahatma Gandhi.  This dream of an India that has eliminated inequality was shared by Dr. Ambedkar.

Guiding Values and Philosophy of Indian Constitution

 Dr Ambedkar played a key role in the making of the Constitution, but his vision of removing inequalities from India was different from Gandhiji.

Philosophy of the Constitution  Values that inspired and guided the freedom struggle and were, in turn, nurtured by it, formed the foundation for India’s democracy. These values are embedded in the Preamble of the Indian Constitution.  The Constitution of India begins with a short statement of its basic values. This is called the Preamble to the Constitution.

CONSTITUTIONAL DESIGN

149

Institutional Design  A constitution is not merely a statement of values and philosophy. It is mainly about embodying these values into institutional arrangements.  It is a very long and detailed document. Therefore, it needs to be amended quite regularly to keep it updated. Those who drafted the Indian Constitution felt to make provisions to incorporate changes from time to time. These changes are called Constitutional Amendments.

Roles and Responsibility as a Citizen of India  To abide by the constitution and respect its ideals and institution, the National Flag and the National Anthem.  To cherish and follow the noble ideals which inspired our national struggle for freedom.  To uphold and protect the sovereignty, unity and integrity of India.

Key Terms  Constituent Assembly: An assembly of people’s representatives that drafts a Constitution for a country.  Constitutional Amendment: A change in the Constitution made by the supreme legislative body in a country.  Preamble: An introductory statement in a constitution which states the reasons and guiding values of the Constitution.

Example Assess the Constitution made by the Constituent Assembly of India. Answer: Step 1: It worked in a systematic, open and consensual manner. Step 2: Basic principles were decided and Drafting Committee prepared a draft of the constitution.



Step 3: Discussions of several rounds occurred, clause by clause. Step 4: They worked for 114 days, spread over three years. Step 5: Every document was recorded and preserved to form twelve bulky volumes of the Constituent Assembly debates.

OBJECTIVE TYPE QUESTIONS Multiple Choice Questions Q. 1. Which among the following is not a feature of the U Indian Constitution? (A) Parliamentary form of government (B) Federal form of government (C) Double citizenship (D) A written Constitution Ans. Option (C) is correct. Explanation: Double citizenship means that a person has a citizenship of two countries at the same time. R Q. 2. Why is India called a republic?  (A) Because the head of the state is elected (B) Because it has many cultures (C) Because it has many religions (D) Because it has a multi-party system Ans. Option (A) is correct. Explanation: India is called a republic as the head of the country is elected by people not a monarch.

(1 mark each)

Q. 3. What is meant by Constitutional Amendment? U  (A) Change in the Constitution made by the supreme legislative body (Parliament) in a country.

(B) Change in the Constitution made by the people.

(C) Change in the Constitution made by foreign intervention.

(D) Change in the Constitution made by the High Court.

Ans. Option (A) is correct. Q. 4. Name the great leader of Indian freedom movement who was not the member of the Indian E Constituent Assembly.  (A) Motilal Nehru

(B) Mahatma Gandhi

(C) Dr. Rajendra Prasad

(D) Dr. Bhimrao Ambedkar

Ans. Option (B) is correct.

150

Oswaal CBSE Question Bank Chapterwise & Topicwise, SOCIAL SCIENCE, Class-IX

Q. 5. Who drafted the Constitution for India in 1928? R  (A) Only Motilal Nehru (B) Motilal Nehru and the eight other Congress leaders (C) A few Congress leaders (D) Dr. Rajendra Prasad Ans. Option (B) is correct. Explanation: The Indian Constitution of 1928 was also called Nehru Report. It was prepared by a committee headed by Motilal Nehru.

Assertion and Reason Directions: In the following questions, a statement of Assertion (A) is followed by a statement of Reason (R). Mark the correct choice as:

(A) Both Assertion (A) and Reason (R) are true, and Reason (R) is the correct explanation of Assertion (A).



(B) Both Assertion (A) and Reason (R) are true, but Reason (R) is not the correct explanation of Assertion (A).



(C) Assertion (A) is true, but Reason (R) is false.



(D) Assertion (A) is false, but Reason (R) is true.





Explanation: The Constitution is a backbone to every democracy in the world. The Constitution gives people their rights and puts check on the rulers. It proves the notion of democracy in true sense.

Q. 2. Assertion (A): The British rule had given voting rights only to a few. On that basis the British had R introduced very weak legislatures. 

Reason (R): In 1931, the resolution at the Karachi session of the Indian National Congress dwelt on how independent India’s Constitution should look like.

Ans. Option (B) is correct.

Explanation: The Constitution makers were well aware about the problems to be faced in future. They wanted to make a Constitution which will have solution to all the problems. The British had a weaker legislature. The constituent assembly was aware about it and hence, they were constantly working for a better Constitution to create a stronger base for democracy.

Q. 3. Assertion (A): The Constitution of a country is a set of written rules that are accepted by all the people living together in a country.

Q. 1. Assertion (A): As far back as in 1928, Motilal Nehru and eight other Congress leaders drafted U

the Constitution for India. 

Ans. Option (B) is correct.

Reason (R): The oppressor and the oppressed in this new democracy were planning to live together as equals.



A

Reason (R): Together, while designing the Constitution, Indian leaders decided that in the search for a solution to the problems, nobody should be excluded, no one should be treated as a demon.

Ans. Option (A) is correct.

SUBJECTIVE TYPE QUESTIONS Very Short Answer Type Questions Q. 1. Describe any two salient Constitution of India.

features

of

the R

Ans. Salient features: (i) Sovereign—People are masters and have supreme right to take decisions. (ii) Socialist—Wealth to be shared equally. (iii) Secular—It is based on the freedom of religion. All religions should be treated equally. (iv) Democratic—People have political equality. (v) Republic—Head of the state is a common man elected by the people. (Any Two) (1 x 2 = 2) Q. 2. Why is the Preamble of the Constitution very U important? Ans. Significance of the Preamble: (i) It contains the philosophy on which the entire constitution has been built.

(2 marks each)

(ii) It provides a standard to examine and evaluate any law and action of the government. (1 + 1 = 2) Q. 3. What factors helped the Indian leaders to develop E the Indian Constitution. Explain any two. Ans. Factors: (i) Ideals of the French Revolution. (ii) Parliamentary democracy in Britain. (iii) Bill of Rights in the U.S.  (Any Two) (1 x 2 = 2) Q. 4. Highlight any two ideals enshrined in the Indian R Constitution. Ans. Ideals enshrined in the Indian Constitution are as follows: (i) Justice: Citizens cannot be discriminated on the basis of gender, caste and religion. (ii) Liberty: Citizens are free to express their thoughts. (iii) Equality: Everybody is equal before the law, irrespective of gender, caste and religion.  (Any Two) (1 x 2 = 2)

CONSTITUTIONAL DESIGN

Q. 5. Why does the Indian Constitution need to be amended quite regularly?

to incorporate changes from time to time. It puts limit to what the government can’t and can do by providing some rights to the citizens that cannot be violate. Those who crafted the Indian Constitution felt that it has to be in accordance with people’s aspirations and changes in society. (2)

E

Ans. The Indian Constitution is a very long and detailed document. It is in accordance with people’s aspirational changes in society. There is a provision

Short Answer Type Questions Q. 1. Describe the steps taken to form the Constituent Assembly of India. R (Board Term I, 2015) Ans. (i) Elections to the Constituent Assembly were held in July 1946. (ii) Members of the Assembly were from different sections of the society. (iii) The Indian Constituent Assembly had 299 members. [CBSE Marking Scheme, 2015] (1 × 3 = 3) Q. 2. Examine three basic guidelines the Constituent Assembly agreed upon before the drafting of Indian Constitution. U (Board Term I, 2013) Ans. The Constitution of India is a framework that defines fundamental, political principles, structures, procedures, powers and duties of government institutions. (i) The aim of drafting the Constitution was to set out fundamental rights, directive principles and duties of citizens. (ii) The Constitution of India guarantees right to freedom and equality, justice, fraternity and liberty. (iii) It is aimed to protect the rights of minorities. (iv) It also aimed to offer unbiased political security and to proclaim India as a Sovereign Democratic Republic with the distribution of powers between centre and the states. (v) It also aims at the inclusion of universal adult franchise.  (Any three) (1 × 3 = 3) [CBSE Marking Scheme, 2013] Q. 3. ‘‘The Constitution of India reflects the views of various social groups and political parties.’’ Explain it by giving suitable arguments. U Ans. Arguments: (i) The Constituent Assembly was dominated by Indian National Congress, but the Congress itself included a variety of political groups and opinions. (ii) The Assembly had many members who did not agree with Congress’ point of view. (iii) It also represented members from different languages, groups, castes, classes, religion, etc. (1×3=3)

151

(3 marks each)

Q. 4. Describe the composition of the Constituent Assembly. How much time did it take to frame the Indian Constitution? R (Board Term I, 2015) Ans. Composition of the Constituent Assembly: (i) It consisted of people of all religions and communities. Eminent persons like Maulana Azad, K. M. Munshi were the members of the Assembly. (ii) It also represented the people of all castes and creeds. Frank Anthony and H. P. Modi represented the Christian and the Persian communities respectively. (iii) Women members like Sarojini Naidu, Renuka Rai and Vijayalakshmi Pandit also played an important role in the Constituent Assembly. (Any two) The Constituent Assembly took over three years to frame the Constitution of India. [CBSE Marking Scheme, 2015] (2+1=3)

Commonly Made Error

In this question, students only explain time frame of the Indian Constitution and do not explain the composition of the Constituent Assembly.

Answering Tip

Students should discuss that the Constituent

Assembly consists of people from any religion, community, castes, creeds and even gender.

Q. 5. The Preamble of Indian Constitution provides philosophy and values of constitution. Explain any three values that you derive from it. U  (DDE, Term 1, 2014, Board Term I, 2013) Ans. The Preamble of Constitution encompasses the fundamental values and philosophy on which our constitution is based. The importance and utility of the Preamble have been pointed out in several decisions of the Supreme Court.

152

Oswaal CBSE Question Bank Chapterwise & Topicwise, SOCIAL SCIENCE, Class-IX



The Preamble to a written constitution states all those objectives which our constitution seeks to establish. It also promotes and aids the legal interpretation of the Indian Constitution with ambiguity in the language’. It mainly serves the following purposes: (i) It provides socioeconomic justice for all.

(ii) It provides equality of opportunity. (iii) It is based on the principle of accommodation of opinion. (iv) Fraternity, assuring the dignity of individual. (v) It is based on a sense of social welfare for all. (Any three) (1×3=3) [CBSE Marking Scheme, 2014]

Long Answer Type Questions Q. 1. Formation of Indian Constitution was no less difficult than that of South Africa. Do you agree? Explain with five arguments. U (Board Term I, 2016, 2014) OR “The making of the constitution for a huge and diverse country like India was not an easy affair.” Justify the statement. (Board Term I, 2013) OR Highlight any five difficult circumstances under which the Indian Constitution was drawn up. Ans. Points to argue : (i) Size of the country. (ii) Diversity comparatively larger in India. (iii) Communal attitude among various groups. (iv) Merger of princely states. (v) Partition and violence at the time of independence. [CBSE Marking Scheme, 2016] Q. 2. Explain the importance of institutional design of the Constitution of India. U (Board Term I, 2016) OR Describe how has the Indian Constitution embodied the basic values into institutional arrangements. (Board Term I, 2014) Ans. (i) The Indian Constitution is a detailed document. It needs to be amended to keep it updated. It has provisions to incorporate changes from time-totime. These changes are called ‘constitutional amendments.’ (ii) It lays down the procedure for choosing a representative to govern the country. (iii) It defines ‘who will have how much power to take a particular decision’. (iv) It also limits the powers of the government by providing some rights to the citizens which cannot be violated. (v) It describes institutional arrangements in a legal language. [CBSE Marking Scheme, 2016]

(5 marks each)

Q. 3. Assess the Constitution made by the Constituent Assembly of India.

U

(DDE Term I, 2014, Board Term I)

Ans. Assessing the constitution as: (i) It worked in a systematic, open and consensual manner. (ii) Basic principles were decided and Drafting Committee prepared a draft of the constitution. (iii) Discussions of several rounds occurred, clause by clause. (iv) They worked for 114 days, spread over three years. (v) Every document was recorded and preserved to form twelve bulky volumes of the Constituent Assembly debates. (1×5=5) [CBSE Marking Scheme, 2014] Q. 4. How did Motilal Nehru Report (1928) and resolution at Karachi Session (1931) of the Indian National Congress shape the Indian Constitution? Describe the working of Constituent Assembly which gave sanctity to the Constitution. U (Board 2012, Term I)

Ans. Motilal Nehru Report and Karachi Session shaped the Indian Constitution with the following features: (i) Universal Adult Franchise. (ii) Right to Freedom and Equality. (iii) Protecting rights of minority. Working of Constituent Assembly to give sanctity to the Indian Constitution: (i) It worked in a systematic, open and consensual manner. (ii) Lots of discussion and debates under Drafting Committee have been preserved as Constitutional Assembly debates. (3+2=5) [CBSE Marking Scheme, 2012]

CONSTITUTIONAL DESIGN

COMPETENCY BASED QUESTIONS Case based MCQs

I. Read the source given below and answer the questions that follow: (1+1+1+1=4) Apartheid was the name of a system of racial discrimination unique to South Africa. The white Europeans imposed this system on South Africa. During the seventeenth and eighteenth centuries, the trading companies from Europe occupied it with arms and force, in the way they occupied India. But unlike India, a large number of ‘whites’ had settled in South Africa and became the local rulers. The system of apartheid divided the people and labelled them on the basis of their skin colour. The native people of South Africa are black in colour. They made up about three-fourth of the population and were called ‘blacks’. Besides these two groups, there were people of mixed races who were called ‘coloured’ and people who migrated from India. The white rulers treated all non-whites as inferiors. The non-whites did not have voting rights. The apartheid system was particularly oppressive for the blacks. They were forbidden from living in white areas. They could work in white areas only if they had a permit. Trains, buses, taxis, hotels, hospitals, schools and colleges, libraries, cinema halls, theatres, beaches, swimming pools, public toilets, were all separate for the whites and blacks. This was called segregation. They could not even visit the churches where the whites worshipped. Blacks could not form associations or protest against the terrible treatment. 1. System of racial discrimination is known as: (A) segregation (B) treatment (C) apartheid (D) None of these Ans. Option (C) is correct. 2. Apartheid System divided people on the basis of their: (A) skin colour (B) caste (C) religion (D) occupation Ans. Option (A) is correct. 3. Non-whites did not have right to— (A) form associations (B) vote (C) live or work in white areas

153

(4 marks each)

(D) All of the above Ans. Option (D) is correct. 4. Who were treated as inferiors? (A) Blacks (B) Coloured (C) Whites (D) Both ‘A’ and ‘B’ Ans. Option (D) is correct. Subjective based Questions

I. Read the source given below and answer the following questions: (1+1+2=4) Like South Africa, India’s Constitution was also drawn up under very difficult circumstances. The making of the Constitution for a huge and diverse country like India was not an easy affair. At that time the people of India were emerging from the status of subjects to that of citizens. The country was born through a partition on the basis of religious differences. This was a traumatic experience for the people of India and Pakistan. At least ten lakh people were killed on both sides of the border in partition related violence. There was another problem. The British had left it to the rulers of the princely states to decide whether they wanted to merge with India or with Pakistan or remain independent. The merger of these princely states was a difficult and uncertain task. When the constitution was being written, the future of the country did not look as secure as it does today. The makers of the constitution had anxieties about the present and the future of the country. 1. Like South Africa, which other country’s constitution was drawn under very difficult circumstances? Ans. Like South Africa, India’s Constitution was also drawn up under very difficult circumstances. 2. Why was making of the Constitution for a huge and diverse country like India was not an easy affair? Ans. The making of the Constitution for a huge and diverse country like India was not an easy affair because that time the people of India were emerging from the status of subjects to that of citizens. 3. Why did the makers of constitution had anxieties when the constitution was being written? Ans. When the constitu­tion was being written, the future of the country did not look as secure as it does today. The makers of the constitution had anxieties about the present and the future of the country. 

Study Time Max Time: 2.5 hr Max Questions: 60

CHAPTER

11

ELECTORAL POLITICS Comprehend the concept and system of elections. l Evaluate the conditions that make Elections in India democratic. l Analyse the implications of power of vote and power of recall. l Appraise the role of election commission for the conduct of free and fair elections. l

Learning Objectives

TOPIC - 1 Why Elections?

Topic-1 Why Elections? Revision Notes

.... P. 154

TOPIC - 2 What is Our System of Elections? .... P. 158 TOPIC - 3 What Makes Elections in India Democratic? .... P. 163

 Why do we Need Elections?  In any democracy, elections take place regularly. But elections are also held in many countries that are not democratic.  In most democracies, people rule through their representatives.  The process by which people choose their representatives at regular intervals is known as Election.  In an election the voters make many choices:  They can choose who will make laws for them.  They can choose who will form the government and take major decisions.  They can choose the party whose policies will guide the government and law making.  What Makes an Election Democratic?

Scan to know more about this topic

How representative are selected

 The process of election in democratic countries differs from that of non-democratic countries. In a democratic election, the preferred contestant is elected. The elections are carried out in a free and fair manner.  A simple list of the minimum conditions of a democratic election are:  Everyone should be able to choose his/her representative, i.e., everyone should have one vote and every vote should have equal value. This is termed as Universal Adult Franchise.  There should be parties and candidates to choose from, freedom to contest and a wide choice for people.  Elections must be held at regular intervals.  Candidate preferred by the people should be elected.  Elections should be held in a fair and free atmosphere to be democratic.  Is it Good to have Political Competition?  Actually, elections are all about the political competition. The most obvious form is the competition among political parties. At the constituency level, it takes the form of competition among several candidates. If there is no competition, elections will become pointless.

ELECTORAL POLITICS

155

156

Oswaal CBSE Question Bank Chapterwise & Topicwise, SOCIAL SCIENCE, Class-IX

 There are some demerits and merits of the political competition.

Demerits:



 Creates a sense of disunity and ‘party politics’.



 Parties level allegations against each other by using dirty tricks to win elections.



 Long-term policies cannot be formulated.



 Good people do not enter politics.

 Merits:



Elections are good because they force the ruling party to perform. The government is aware that it will be voted out of power if it does not perform as the people expected.





It forces parties and leaders to perform, so competition is good.

Key Terms  Election: The process by which people choose their representatives at regular intervals is known as election.  Electorate: It refers to the entire body of people who are qualified to vote in the elections for the legislatures or local bodies.  Franchise: It refers to the right of people to vote and elect their representatives to make laws.

OBJECTIVE TYPE QUESTIONS

B Assertion and Reason MCQs

Multiple Choice Questions U Q. 1. Why do we need elections? Give reasons. (A) Through elections people come to know about government policies (B) Through elections we can choose the party whose policies will guide the government and law making. (C) Through elections we can make laws. (D) Through elections we get freedom. Ans. Option (B) is correct. R Q. 2. Who led the ‘Nyaya Yudh’? (A) Bhimrao Ambedkar (B) Chaudhary Devi Lal (C) Dr. Rajendra Prasad (D) Motilal Nehru Ans. Option (B) is correct. Explanation: Chaudhary Devi Lal, then an opposition leader, led a movement called ‘Nyaya Yudh’ (Struggle for Justice) and formed a new party, Lok Dal. Q. 3. What promise was made by Chaudhary Devi Lal R to the farmers and small businessmen?  (A) Land for cultivation (B) Provide loans without any rate of interest (C) Waive the loans of farmers and small businessmen (D) Free land for women Ans. Option (C) is correct. Explanation: In the election campaign, Devi Lal said that if his party won the elections, his government would waive the loans of farmers and small businessmen. He promised that this would be the first action of his government.

(1 mark each)



(A) (B) (C) (D)

In the questions given below, there are two statements marked as Assertion (A) and Reason (R). Read the statements and chose the correct option: Both (A) and (R) are true and (R) is the correct explanation of (A). Both (A) and (R) are true, but (R) is not the correct explanation of (A). (A) is correct but (R) is wrong. (A) is wrong but (R) is correct.

Q. 1. Assertion (A): At the constituency level, elections takes the form of competition among several candidates. Reason (R): Elections are all about political U competition. Ans. Option (A) is correct. Explanation: Elections are all about the political competition. The most obvious form is the competition among political parties. It takes the form of competition among several candidates. Q. 2. Assertion (A): An electoral competition has many demerits. Reason (R): It creates a sense of disunity and A ‘factionalism’ in every locality.  Ans. Option (A) is correct. Q. 3. Assertion (A): Different political parties and leaders often level allegations against one another. Reason (R): Parties and candidates often use dirty U tricks to win elections. Ans. Option (A) is correct.

ELECTORAL POLITICS

157

SUBJECTIVE TYPE QUESTIONS Short Answer Type Questions Q. 1. Name the movement led by Chaudhary Devi Lal of Haryana in 1987. What promise did he make to lure the voters before election? Which political party did he form? R (Board Term II, 2016)

(3 marks each)

(ii) For Lok Sabha elections, the country is divided into 543 constituencies at present. (iii) The basis of the division of the constituencies is on the basis of population.

Ans. (i) The name of this movement was ‘Nyaya Yudh’.

Commonly Made Error

(ii) The popular promise was, if his party won the elections, his government would waive the loans of farmers and small businessmen. (iii) The name of the party that he formed was Lok Dal. [CBSE Marking Scheme, 2016] (1×3=3)



Students are unable to understand the basis of the division of these constituencies.

Answering Tip

Q. 2. What are Constituencies? How many constituencies are there in India for Lok Sabha? What is the basis of the division of these Constituencies? R



Ans. (i) The country is divided into different areas for the purpose of elections. These areas are called electoral constituencies.

Students should know that 543 constituencies

make a Lok Sabha and the division is on the basis of population.

Long Answer Type Questions Q. 1. Write any five competition.

demerits to have political U (Board Term II, 2014, 2013)

Ans. Five demerits to have political competitions are : (i) Parties and candidates often use dirty tricks to win elections. (ii) Different political parties and leaders often level allegations against one another. (iii) Political competition creates a sense of disunity and ‘factionalism’ in every locality. (iv) The pressure to win electoral fights does not allow sensible long-term policies to be formulated. (v) Some good people who may wish to serve the country, do not like the idea of being dragged into unhealthy competition. [CBSE Marking Scheme, 2014] (1×5=5) Q. 2. Why do we need elections? Mention any three demerits of an electoral competition. A (Board Term II, 2013) OR Describe any five demerits of an electoral competition. OR An electoral competition has many demerits. State any five reasons to justify the statements. Ans. Election is a mechanism by which people choose their representatives at regular intervals and change

(5 marks each) them if they wish to do so. Therefore, elections are considered essential for representative democracy.



Three demerits of electoral competition: (i) Creates a sense of disunity and factionalism in every locality. (ii) Political parties and leaders level allegation at each other. (iii) Political parties and candidates often use dirty tricks to win elections. (iv) Pressure to win an election does not lead to the formulation of long-term policies. (v) Good people who may wish to serve the country do not enter this arena. They do not like unhealthy competition. (Any three) (2+3=5)

Commonly Made Error

In this question students only explain election, they don’t explain the demerits of electoral competition.

Answering Tip

Students should know that political parties have an unhealthy competitions with dirty tricks.

158

Oswaal CBSE Question Bank Chapterwise & Topicwise, SOCIAL SCIENCE, Class-IX

Q. 3. What do you understand by election. Explain the nomination process as practised in Indian elections. U Ans. Election: It is a process by which representatives get elected, who will further make policies and rule our country. Nomination Process: (i) Party tickets are given. (ii) Nomination form is filled. (iii) Security amount is deposited. (iv) Nomination papers are scrutinised.  (1+4=5)

Commonly Made Error

Students generally mention the criteria to fulfill the nomination process.

Answering Tip

Students should learn that nomination

process includes imparting party tickets, filling nomination forms, depositing security amount and scrutinising nomination papers.

Topic-2 What is Our System of Elections? Revision Notes  An election is carried out every five years to the Lok Sabha or the Vidhan Sabha, it is known as a general election.  Sometimes, the Lok Sabha and the Vidhan Sabha are dissolved and an election is held before the expiry of their full term of five years. Such an election is called a mid-term election.  Electoral constituencies  In India, we follow an area based system of representation. The country is divided into different areas for purposes of elections. These areas are called Electoral Constituencies. The voters who live in an area elect one representative.  For Lok Sabha elections, the country is divided into 543 constituencies. The representative elected from each constituency is called a Member of Parliament or an MP. Scan to know  Similarly, each state is divided into a specific number of Assembly constituencies. In this more about this topic case, the elected representative is called the Member of Legislative Assembly or an MLA.  The same principle applies for Panchayat and Municipal elections. Each village or town is divided into several ‘wards’ that are like constituencies. Each ward elects one member of the village or the urban local body. Indian Electoral Process  Reserved Constituencies:  The Constitution of India entitles every citizen to elect her/his representative and to be elected as a representative.  The Constitution of India states a special system of reserved constituencies for the Scheduled Castes (SC) and Scheduled Tribes (ST) as well as Other Backward Classes (OBC).  One-third of the seats are reserved in rural and urban local bodies for women candidates.  Voters’ List:  In a democratic election like in our country, the list of those who are eligible to vote is prepared much before the election and given to everyone. This list is officially called the Electoral Roll and is commonly known as the Voters’ List.  The Indian Government has introduced the Election Photo Identity Card [EPIC] System. Every eligible voter on the list is issued a Photo Identity Card. Carrying this EPIC is not mandatory. Instead, voters can provide proof of identity like ration card or driving licence to exercise their right to vote.  Nomination of Candidates:  Any citizen of India who can be a voter can also become a candidate in elections. The only difference is that in order to be a candidate, the minimum age is 25 years.  Political parties nominate their candidates who get the party symbol and their party worker’s support. Party’s nomination is often called ‘party ticket’.  Every person who wishes to contest an election has to fill a ‘nomination form’ and give some money as ‘security deposit’.

ELECTORAL POLITICS

159

 Election Campaign:  The main purpose of election campaign is to have a free and open discussion about who is a better representative, which party will make a better government or what is a good policy.  Sometimes it is necessary to regulate campaigns to ensure that every political party and candidate gets a fair and equal chance to compete. According to our election law:  Political parties or candidates cannot bribe or threaten voters.  They cannot ask for votes on the grounds of caste or religion.  They cannot make use of government resources or places of worship for campaigning.  They cannot spend more than ` 25 lakh per constituency for a Lok Sabha election or more than ` 10 lakh per constituency in a state legislative assembly election.  The Indian Constitution provides equal rights of representation to all the citizens of India.  There is a common Code of Conduct for election campaigns, which all political parties in India have to follow.  Polling and Counting of Votes  The final stage of an election is the day when the voters cast or ‘poll’ their vote. That day is usually called the election day.  Every person whose name is on the voters’ list can go to a nearby polling booth, and cast his/her votes. Nowadays, electronic voting machines (EVMs) are used to record votes.  Once the polling is over, all the EVMs are sealed and taken to a secured place.  A few days later, on a fixed date, all the EVMs from a constituency are opened and the votes secured by each candidate are counted. Within a few hours of counting, all the results are declared and it becomes clear as to who will form the next government.  There is a common Code of Conduct for election campaigns, which all political parties in India have to follow.

Key Terms  Campaigning: It refers to a process by which a candidate tries to persuade the voter to vote for him rather than for others.  Election Photo Identity Card: The voters are required to carry this card when they go out to vote.  Voter’s List: List of those who are eligible to vote, that is prepared before the election.  Electoral Roll: Voter’s list is also known as Electoral Roll.  Election Manifesto: A document published by every political party before elections containing the policies and programmes of that party.  Code of Conduct: A set of norms and guidelines to be followed by political parties and contesting candidates during the election time.

Example Analyse the concept of one person, one vote, one value. Ans. Step 1: The Indian Constitution provides equal rights of representation to all the citizens of India in elections and to choose their representatives. Step 2: All citizens of age 18 years or above are eligible to vote and a citizen aged 25 years or above is also eligible for contesting an election. Step 3: According to Universal Adult Franchise, everyone should have one vote and each note

should have equal value. No one should be denied the right to vote without a good reason.

Step 4: Citizens differ from one another in many ways : some are rich, some are poor, some are highly educated, some are not so educated or not educated at all, some are kind, others are not. But all of them are human beings with their own needs and views.



Step 5: That is why all of them deserve to have an equal say in decisions that affect them especially politics.

160

Oswaal CBSE Question Bank Chapterwise & Topicwise, SOCIAL SCIENCE, Class-IX

OBJECTIVE TYPE QUESTIONS

(R). Read the statements and choose the correct options:

Multiple Choice Questions A Q. 1. How are candidates elected in India?  (A) By face value (B) One who is financially strong (C) One who secures the highest number of votes (D) Both (A) and (C) Ans. Option (C) is correct. Explanation: The candidate who secures the highest number of votes from a constituency is declared elected. Q. 2. For voting, the voter has to show which identity R proof?  (A) Aadhar Card (B) Passport (C) EPIC (Election Photo Identity Card) (D) Driver’s license Ans. Option (C) is correct. Explanation: The government has tried to give the EPIC to every person on the voters list. The voters are required to carry this card when they go out to vote, so that no one can vote for someone else. Q. 3. What is the use of electronic voting machines? U  (A) To record the votes of people (B) To show the power of technology (C) To ensure absolute voting (D) To get on the spot voting results Ans. Option (A) is correct. Explanation: Nowadays electronic voting machines (EVMs) are used to record votes. The machine shows the names of the candidates and the party symbols.

Assertion and Reason In the questions given below, there are two statements marked as Assertion (A) and Reason

(1 mark each)



(A) Both (A) and (R) are true and (R) is the correct explanation of (A).



(B) Both (A) and (R) are true and (R) is not the correct explanation of (A).



(C) (A) is correct but (R) is wrong.



(D) (A) is wrong but (R) is correct.

Q. 1. Assertion (A): For Lok Sabha elections, the country is divided into 543 constituencies. 

E

Reason (R): The representative elected from each constituency is called a Member of Parliament or an MP.

Ans. Option (A) is correct. Q. 2. Assertion (A): The Congress party led by Indira Gandhi gave the slogan of Garibi Hatao (Remove poverty) in the Lok Sabha elections of 1971.

A

Reason (R): The party promised to reorient all the policies of the government to remove poverty from the country.

Ans. Option (A) is correct. Q. 3. Assertion (A): Earlier the voters used to indicate who they wanted to vote for by putting a stamp on the ballot paper.

E

Reason (R): Nowadays electronic voting machines (EVMs) are used to record votes.

Ans. Option (B) is correct.

Explanation: Nowadays electronic voting machines (EVMs) are used to record votes. The machine shows the names of the candidates and the party symbols. All that the voter has to do is to press the button against the name of the candidate she wants to give her vote.

SUBJECTIVE TYPE QUESTIONS Very Short Answer Type Questions Q. 1. Define the following: (a) Ballot Paper (b) Election Day Ans. (a) Ballot Paper: A ballot paper is a sheet of paper on which the names of the contesting candidates along with party name and symbols are listed. Earlier the

(2 marks each)

voters used to indicate who they wanted to vote for by putting a stamp on the ballot paper. (b) Election Day: The final stage of an election is the day when the voters cast or ‘poll’ their vote. That day is usually called the election day. (1 + 1 = 2)

ELECTORAL POLITICS

161

Q. 2. Mention any two restrictions imposed by the Election Commission on the party and the R candidates.

Q. 4. Mention any two legal declarations which every candidate has to make at the time of filling his / R her nomination papers for the election. 

Ans. (i) Bribe or threaten voters. (ii) Appeal to them in the name of religion. (iii) Use government vehicles or government resources. (Any Two Points) (1 x 2 = 2) Q. 3. Why are elections considered essential for any

Ans. Declaration is made regarding: (i) Serious criminal cases pending against the candidates. (ii) Details of the assets and liabilities of candidate and his or her family. (iii) Educational qualifications of the candidates.  (Any Two Points) (1 x 2 = 2) R Q. 5. What is Model Code of Conduct?  Ans. The Model Code of Conduct is a set of guidelines issued by the Election Commission of India for conduct of political parties and candidates during elections. All the political parties in our country have agreed to a Model Code of Conduct for election campaigns.  (2)

E representative democracy? Ans. Elections are considered essential for any representative democracy as: (i) They can choose the one who will form the government and take major decisions. (ii) They can choose the party whose policies will guide the government in law making. (1 x 2 = 2)

Short Answer Type Questions Q. 1. Describe the process of campaigning in election in India. U (Board Term II, 2017) Ans. Campaigning has a criteria in India: (i) Campaign takes place for about two week period between the announcement of list of candidates and the date of polling. (ii) Candidates contact voters, address meetings. (iii) Newspapers and television are full of election related stories.  [CBSE Marking Scheme, 2017] (1×3=3) Q. 2. What is meant by ‘seats‘ in the election? Explain with example. U (Board Term II, 2016) Ans. (i) In India, for Lok Sabha election, the country is divided into 543 constituencies. (ii) Similarly, each state is divided into a specific number of assembly constituencies. Each Parliamentary Constituency has several assembly constituencies. The same principle applies to Panchayat and municipal elections. (iii) Each village or town is divided into several ‘wards‘ that are like constituencies. Each ward elects one member of the village or the urban local body. Sometimes these constituencies are counted as ‘seats‘ for each constituency represents one seat in the assembly. (Any two) Example: When we say a party has won 20 seats. It means that candidates of that party won in 20 assembly constituencies in the state and that it has 20 MLAs in the state assembly. (2+1=3) Q. 3. Define the following: R (i) Election (ii) Election Campaign (iii) Voters Turn Out  (Board Term II, 2016) Ans. (i) Elections: The process by which people choose their representatives at regular intervals is known as election.

(3 marks each)

(ii) Election Campaign: It refers to a process by which a candidate tries to persuade the voter to vote for him rather than for others. (iii) Voters Turn Out: The percentage of eligible voters who cast their votes in an election. (1×3=3) Q. 4. Distinguish between general election and midterm elections. (Board Term II, 2016) Ans. General Elections

Mid Term Elections

(i) General election is a kind of election which is held after certain period of time (5 years in India) for all constituencies at the same day or within few days.

(i) Mid term election is held when a Lok Sabha or a State Assembly dissolve as a whole before the expiring period. It is held to form the new house.

(ii) A general election is the election held after a stipulated period of time to elect all members of a given political body. For example, the Lok Sabha elections that are held in India, every five years.

(ii) If a government (central and state) fails a confidence motion and no other alternative to the government is possible, mid-term elections are held.

(iii) A general election is an election in which all or most members of a given political body are chosen.

(iii) Mid-term elections are those which are conducted before the end of the term of a given government. Such elections cause undesired load of expense over the public.

(1×3=3)

162

Oswaal CBSE Question Bank Chapterwise & Topicwise, SOCIAL SCIENCE, Class-IX

Q. 5. Define the following: R (a) Universal Adult Franchise (b) Election Photo Identify Card (c) Voter’s List (Board Term II, 2014) Ans. (a) Universal Adult Franchise: It is a right granted to all adults – men or women, rich or poor, white or black, to vote for their representatives to run the government. In practice it means that everyone should have one vote and each vote should have equal value. (b) Election Photo Identify Card : This is introduced by the government to stop rigging. The voters are



required to carry this card when they go out to vote so that no one can vote for someone else. The government has tried to give this card to every person on the voters list. But the card is not yet compulsory for voting. (c) Voter’s List: In a democratic election, the list of those who are eligible to vote is prepared much before the election and given to everyone. This list is officially called the electoral roll and is commonly known as the Voters’ List. (Any three) [CBSE Marking Scheme, 2014] (1×3=3)

Long Answer Type Questions Q. 1. Describe

the

various

methods

of

election

campaign used by political parties. A  (Board Term II, 2017) Ans. Methods of election campaign are: (i) Candidates contact the voters personally. (ii) Election meetings are held. (iii) Political parties mobilise their supporters. (iv) Advertisements and articles are published in newspapers. (v) Slogans on big issues are used to attract the voters.  [CBSE Marking Scheme, 2017] (1×5=5)

Commonly Made Error

Students generally explain only one and two methods of campaign.

Answering Tip

Election campaign includes publishing of advertisements and articles and using slogans, etc.

Q. 2. Describe the procedure for nomination of candidates for election in India.

A

(Board Term II, 2016) Ans. (i) Any one, who can be a voter, can also become a candidate in elections. Political parties nominate their candidates who get party symbol and support. Party‘s nomination is often called ‘party ticket‘.

(5 marks each)

(ii) Every person who wishes to contest an election has to fill a ‘nomination form‘ and give some money as a ‘security deposit‘. (iii) Recently, a new system of declaration has been introduced on direction from the Supreme Court. Every candidate has to make a legal declaration, giving full details of: (a) Serious criminal cases pending against the candidate. (b) Details of the assets and liabilities of the candidate and his or her family. (c) Educational qualifications of the candidate. (iv) This information has to be made public. (v) This provides an opportunity to the voters to make their decision on the basis of the information provided by candidates. (1×5=5) Q. 3. Describe the rules and regulations regarding election campaign in India. A (Board Term II, 2015) Ans. No party or candidate should do the following : (i) Use government resources for campaigning. (ii) Bribe or threaten voters. (iii) Appeal to voters in the name of caste or religion (iv) Spend more than the prescribed amount more than ` 10 lakh in assembly election. (v) If any candidate violates the above laws or rules his election can be rejected by the court. [CBSE Marking Scheme, 2015] (1×5=5)

ELECTORAL POLITICS

163

Topic-3 What Makes Elections in India Democratic? Revision Notes Independent Election Commission:  India has a democratic election system. The election system in India is controlled and governed by an independent and very powerful body called the Election Commission (EC). The EC is headed by the Chief Election Commissioner, who is assisted by several Election Commissioners.  The Election Commission of India performs several functions, starting from the announcement of the elections to the final declaration of the result.  It drafts and implements the Model Code of Conduct for elections and takes disciplinary action against parties violating it.

Scan to know more about this topic

What makes Elections in India Democratic?

 The Election Commission is authorised to advise the government on decisions affecting the election and control the transfer of government officials. The Election Commission also has the function of controlling the work of government officials on election duty. The Election Commission has the power to order a re-poll in case it finds evidence of unfair practices during polling.

Popular Participation:

 The quality of the election process can also be checked by seeing the participation of people. Some conclusions about participation in India:

(i) People’s participation in the election is measured by voter turnout figures. Turnout indicates the percentage of eligible voters, who actually cast their vote.

(ii) In India, the poor, illiterate and underprivileged people vote in larger proportion as compared to the rich and privileged sections. (iii) Common people in India feel that through elections they can bring pressure on political parties to adopt policies and programmes favourable to them. (iv) The interest of voters in election related activities has been increasing over the years.

Acceptance of Election Outcome:  One final test of the free and fairness of the election is the outcome of the election. (i) The ruling parties routinely lose elections in India both at the national and state level. (ii) In India, about half of the sitting MPs or MLAs lose elections. (iii) Candidates who are known to have spent a lot of money on ‘buying votes’ and those with known criminal connections often lose elections. (iv) Barring very few disputed elections, the electoral outcomes are usually accepted as ‘people’s verdict’ by the defeated party.

Challenges to Free and Fair Elections  Elections in India are basically free and fair. Sometimes this may not be true for every constituency.  There are many limitations and challenges to Indian elections. These include : (i) Candidates and parties with a lot of money enjoy a big and unfair advantage over smaller parties. (ii) Candidates with criminal connections have been able to push others out of the electoral race and to secure a ‘ticket’ from major parties. (iii) Tickets are distributed to relatives from their families. (iv) Elections offer little choice to ordinary citizens as major parties are quite similar to each other, both in policies and practice. (v) Smaller parties and independent candidates suffer a huge disadvantage compared to bigger parties.

Key Terms  Incumbent: The current holder of a political office.

164

Oswaal CBSE Question Bank Chapterwise & Topicwise, SOCIAL SCIENCE, Class-IX

OBJECTIVE TYPE QUESTIONS Multiple Choice Questions Q. 1. Study the given picture carefully:

E

(1 mark each)

(C) Prime Minister (D) Supreme Court Ans. Option (A) is correct. Q. 5. Match the following Column I with Column II: U  Column-I

This picture is related to which of the following. (A) Public distribution system (B) Election (C) Famine (D) None of the above Ans. Option (B) is correct. U Q. 2. Study the given picture carefully:

Column-II

a.

Election Commission of India

i.

Announcement elections

b.

EVM

ii

Electronic machine used for casting the vote

c.

Turnout

iii.

Indicates the per cent of eligible voters who actually cast their vote.

d.

Incumbent

iv.

The current holder of a political office.

(A) a-i, b-ii, c-iii, d-iv (C) a-iii, b-ii, c-i, d-iv Ans. Option (A) is correct.

of

the

(B) a-iv, b-ii, c-iii, d-i (D) a-ii, b-iii, c-iv, d-i

Assertion and Reason



In the questions given below, there are two statements marked as Assertion (A) and Reason (R). Read the statements and choose the correct options: This picture is related to which of the following? (A) Monarchy (B) Democracy (C) Oligarchy (D) Totalitarianism Ans. Option (B) is correct. Explanation: Titled ‘Electoral Campaigns’, this cartoon was drawn in the Latin American context. Q. 3. Who appoints the Chief Election Commissioner in R India?  (A) The Election Commission (B) The President of India (C) The Supreme Court (D) The Prime Minister Ans. Option (B) is correct. Explanation: The Election Commissioner (CEC) is appointed by the President of India. But once appointed, the Chief Election Commissioner is not answerable to the President or the government. R Q. 4. Who implements the Code of Conduct? (A) The Election Commission (B) General Public



(A) Both (A) and (R) are true and (R) is the correct explanation of (A). (B) Both (A) and (R) are true and (R) is not the correct explanation of (A). (C) (A) is correct but (R) is wrong. (D) (A) is wrong but (R) is correct.

Q. 1. Assertion (A): It is very common now for the Election Commission to reprimand the government and administration for their lapses.

Reason (R): One simple way of checking whether elections are fair or not is to look at who conducts U the elections.

Ans. Option (B) is correct. Explanation: In the last 25 years or so, the Election Commission has begun to exercise all its powers and even expand them. It is very common now for the Election Commission to reprimand the government and administration for their lapses. Q. 2. Assertion (A): More than half of the people identified themselves as being close to one or the E other political party.

Reason (R): One out of every seven voters is a member of a political party.

Ans. Option (A) is correct.

ELECTORAL POLITICS

Q. 3. Assertion (A): A few candidates may win purely on the basis of money power and unfair means. A 

Reason (R): But the overall verdict of a general election still reflects limited preference.

165

Ans. Option (C) is correct. Explanation: A few candidates may win purely on the basis of money power and unfair means. But the overall verdict of a general election still reflects popular preference.

SUBJECTIVE TYPE QUESTIONS Very Short Answer Type Questions Q. 1. The party that wins an election and forms the U

government. Explain. Ans. The party that wins an election and forms government because people have chosen it over its rivals. This is what makes Indian elections democratic. (2) Q. 2. Why do common people in India attach a lot of importance to elections? E Ans. Common people in India attach a lot of importance to elections. (i) They feel that through elections they can bring pressure on political parties to adopt policies and programmes favourable to them. (ii) They also feel that their vote matters in the way things are run in the country. (1 x 2 = 2)

(2 marks each)

Q. 3. People’s participation in election is usually measured by voter turnout figures. Explain. U  Ans. People’s participation in election is usually measured by voter turnout figures. (i) Turnout indicates the per cent of eligible voters who actually cast their vote. (ii) Over the last fifty years, the turnout in Europe and North America has declined. In India, the turnout has either remained stable or actually gone up.  (1 x 2 = 2) U Q. 4. Define Rigging.  Ans. Fraud and malpractices indulged by a party or candidate to increase its votes is called Rigging. It includes stuffing ballot boxes by a few persons using the votes of others; recording multiple votes by the same person; and bribing or coercing polling officers to favour a candidate. (2)

Short Answer Type Questions Q. 1. Mention the role of the Election Commission in the elections. A (Board Term II, 2016) OR Who appoints the Chief Election Commissioner of India? What powers are exercised by the Election Commission of India? (Board Term II, 2015) OR Describe the role of the Election Commission in holding free and fair elections in India. (Board Term II, 2013) OR Mention any three powers and functions of the Election Commission of India. OR Does the Election Commission of India has power to conduct free and fair elections? Support your answer with three arguments.

(iii) The Election Commission has power to implement code of conduct and punish any party who violates it. (iv) Government officials on election duty, work under its control. (v) The Election Commission can order government to follow some guidelines to prevent undue misuse of governmental powers during elections. (Any three) (1+2=3) [CBSE Marking Scheme, 2016]

Commonly Made Error

Ans. The Chief Election Commissioner is appointed by the President of India.

Powers of the Election Commission:

(i) Independent and powerful body. (ii) The Election Commission takes independent decision on all aspects of election.

(3 marks each)

In this question students only define the Election Commission.

Answering Tip

Students should learn that the Election

Commission is an independent powerful body and can also order ruling government to follow guidelines during election time to avoid misuse of power.

166

Oswaal CBSE Question Bank Chapterwise & Topicwise, SOCIAL SCIENCE, Class-IX

Q. 2. Explain any three challenges faced by the election R system in India. Ans. Challenges faced by election system : (i) Candidates and parties with money have unfair advantage over smaller parties. (ii) Candidates with criminal connection push others out of electoral race. (iii) Some families tend to dominate political parties and distributes tickets to relatives. (iv) No real choice is given to voters, as major parties are quite similar in policies and practice. (v) Small parties and independent candidates have disadvantages as compared to bigger parties. (Any three) (1×3=3) Q. 3. Mention any three techniques of election campaign. A Ans. (i) Posturing: A few days before the actual election day, the election campaign begins with posturing. These posters bear the photograph of the candidate along with that of the national leaders of the party and appear on the walls of the bazaars and streets.

(ii) Meetings: Candidates hold party meetings and corner meetings to get support of various groups of people. (iii) Processions: Processions are often taken out and large number of trucks, tongas, cycles and scooters are used to carry the people who shout slogans in favour of their candidates. (iv) Door-to-Door canvassing: This is undertaken by the different candidates and their party men to get the support of the public. (Any three) (1×3=3)

Commonly Made Error

Students only mention about the unfair practices taking place during election.

Answering Tip

Students should learn that door-to-door canvassing, meetings, processions are major techniques viewed during election campaign.

Long Answer Type Questions Q. 1. Analyse the trends of the outcomes of elections in R (Board Term II, 2017) India. Ans. The trends of the outcome of elections in India are given below:

Commonly Made Error

(i) Rich candidates spend money on buying votes. (ii) Candidates having criminal connection often lose elections. (iii) The electoral outcomes are accepted as people’s verdict by the defeated candidate. (iv) Mostly, sitting MLAs and MPs lose elections. (v) Ruling party often loose elections due to incumbency. 

[CBSE Marking Scheme, 2017] (1×5=5)

Q. 2. Describe some unfair practices that take place during elections. A (Board Term II, 2015) Ans. The unfair practices used during elections are: (i) False names are included in the voter list. (ii) Misuse of government machinery. (iii) Abuse of money by rich candidates. (iv) Rigging on the polling day. (v) Use of muscle power by the candidates. [CBSE Marking Scheme, 2015] 5

(5 marks each)

Students do not explain this question because

they cannot differentiate between unfair practice and election campaign.

Answering Tip

The major unfair practices observed during

elections are inclusion of false name in the voter list and rigging and abuse of money.

Q. 3. Do you think educational qualifications should be necessary for the candidate aspiring to stand in elections? Give your views in favour or against it. A Ans. (i) Educational qualifications are not relevant to all kind of jobs. (ii) Relevant qualification for being an MLA or an MP is the ability to understand people’s concerns, problems and the way to solve them. (iii) Putting an educational qualification would go against the spirit of democracy. (iv) If educational qualification was made compulsory, it would mean depriving a majority of the country’s citizens to contest elections.  (4 × 1¼ = 5)

ELECTORAL POLITICS

COMPETENCY BASED QUESTIONS Case based MCQs

I. Read the source given below and answer the U (1+1+1+1=4) questions that follow: Once the election results were announced, the sitting Chief Minister resigned. The newly elected Members of Legislative Assembly (MLAs) of Lok Dal chose Devi Lal as their leader. The Governor invited Devi Lal to be the new Chief Minister. Three days after the election results were declared, he became the Chief Minister. As soon as he became the Chief Minister, his government issued a Government Order waiving the outstanding loans of small farmers, agricultural labourers and small businessmen. His party ruled the State for four years. The next elections were held in 1991. But this time his party did not win popular support. The Congress won the election and formed the government. 1. Where was this State Assembly election held? (A) In Punjab (B) In Haryana (C) In Bihar (D) In Rajasthan Ans. Option (B) is correct. 2. Once the election results were announced, the sitting Chief Minister________ . (A) extended the tenure (B) shifted to another state (C) resigned (D) None of the above Ans. Option (C) is correct. 3. What is the name of Devi Lal’s Party? (A) Lok Dal (B) Janata Lok Dal (C) Congress (D) Bharatiya Janata Party Ans. Option (A) is correct. 4. Devi Lal’s Party and its partners won ................ seats in the State Assembly. (A) 56 out of 80 (B) 96 out of 100 (C) 76 out of 80 (D) 66 out of 90 Ans. Option (C) is correct.

Subjective based Questions

I. Read the source given below and answer the E (1+1+2=4) questions that follow: In our country such campaigns take place for a two-week period between the announcement of the final list of candidates and the date of polling. During this period the candidates contact their voters, political leaders address election meetings and political parties mobilise their supporters. This

167

(4 marks each)

is also the period when newspapers and television news are full of election related stories and debates. But election campaign is not limited to these two weeks only. Political parties start preparing for elections months before they actually take place. In election campaigns, political parties try to focus public attention on some big issues. They want to attract the public to that issue and get them to vote for their party on that basis. 1. What is the minimum age to cast a vote? Ans. The minimum age to cast vote is 18 years. 2. As the new people attain the voting age, where is there name added? Ans. As the new people attain the voting age their name is added to the voter list. 3. What other proofs of identity can be shown by the voter if he/she does not have an EPIC card? Ans. Ration Card and Driving License. OR 1. For how long does election campaign take place? Ans. The election campaign takes place for two weeks period. 2. What is the role of the newspapers and televisions during elections? Ans. The newspapers and television news are full of election related stories and debates. They are used to promote commercial, social and political agendas of the political parties. II. Read the source given below and answer the U (1+1+2=4) questions that follow: We noted above that in a democratic election people should have a real choice. This happens only when there are almost no restrictions on anyone to contest an election. This is what our system provides. Anyone who can be a voter can also become a candidate in elections. The only difference is that in order to be a candidate the minimum age is 25 years, while it is only 18 years for being a voter. There are some other restrictions on criminals, etc., but these apply in very extreme cases. Political parties nominate their candidates who get the party symbol and support. Party’s nomination is often called party ‘ticket’. Every person who wishes to contest an election has to fill a ‘nomination form’ and give some money as ‘security deposit’. Recently, a new system of declaration has been introduced on direction from the Supreme Court. Every candidate has to make a legal declaration, giving full details of: • Serious criminal cases pending against the candidate; • Details of the assets and liabilities of the candidate and his or her family; and

168

Oswaal CBSE Question Bank Chapterwise & Topicwise, SOCIAL SCIENCE, Class-IX

• Educational qualifications of the candidate. This information has to be made public. This provides an opportunity to the voters to make their decision on the basis of the information provided by the candidates. 1. What should be the minimum age of a candidate to contest an election? Ans. The minimum age of a candidate to contest an election should 25 years.

2. What is party ticket? Ans. When a candidate is nominated by the party to fight an election as a representative of the party, he is issued a ticket which is often known as party ticket. 3. Who has introduced a new system of declaration? Ans. The Supreme Court has introduced a new system of declaration.

Artificial Intelligence Data Acquisition & Data Exploration and Inkle Writer integrated with Electoral Politics with special reference to topic: What is our system of Elections? Objective: To get familiarisation with our electoral System and the reason as to why we have chosen this system.  To identify the need for elections in democratic countries like India.  To make them aware about steps involved in election process.  To be able to use AI supported Tool for casting the votes and getting the results.  To make familiar with Data exploration through AI. Material Required: Textbooks, Blackboard, chalk, laptop / desktop, internet connection, Voting List and a marker. PARAMETERS

DESCRIPTION

Chapter Covered

Chapter 3: Electoral Politics

Name of the Book

Democratic Politics – I, Class 9, NCERT

AI CONCEPTS INTEGRATED

Subject and Artificial AI Tools: Data Acquisition & Data Exploration and Inkle Writer Intelligence integrated with Electoral Politics with special reference to topic: What Integrated is our system of Elections? Learning Objectives

l l l l l

To get familiarization with our electoral System and the reason as https://datavizcatalogue.com to why we have chosen this system. To identify the need for elections in democratic countries like India. To make them aware about steps involved in election process. To be able to use AI supported Tool for casting the votes and getting the results. To make familiar with Data exploration through AI.

Time Required

3 Periods of 40 minutes each (Two periods for initial process and casting the vote and one period for Data Exploration of Election results & Declaration of Results)

Classroom Arrangement

Flexible Seating arrangement

Material Required

Textbooks, Blackboard, chalk, laptop / desktop, internet connection, Voting List and a marker.

Pre – Preparation Activities

A video on casting vote with EVM would be shown to students in https://youtu.be/ spOWG6pjBb8 order to make them understand the election process. Activity 1: Students are asked to offer Candidature for Class Representative position. They are asked to fill Nomination Form in stipulated time keeping all the eligibility criteria in mind. A group of students would act as Election Commission who will provide Symbol to the selected candidates. A voter list is also prepared by EC. Selected candidates will start their Campaign and publish their Manifesto.

Previous Knowledge

Discussion about Constituencies and Political parties. (Reference Chapter 3 Class VIII)

ELECTORAL POLITICS

Methodology

Learning Outcomes

Activity 2: Polling Day Students will be asked to cast their vote online thought the link. https://www.zingpoll.com/6xnjjcbl All the aspect of One Person One Vote – Political Right and Free and fair Election would be introduced to students through real life experience. Activity 3: Counting Day Students are asked to collect the data about Election polls basis on their survey. They were asked to go for analysis and exploration of Data using AI tools. https://datavizcatalogue.com Activity 4: Result Declaration Day Result would be declared basis on number of highest votes, which is already counted by App. l l l l l

169

https://www.zingpoll .com/6xnjjcbl

https:// datavizcatalogue.com

 he students would be able to get familiarization with our electoral T System and the reason as to why we have chosen this system. The students would be able to identify the need for elections in democratic countries like India. The students would be able to understand the steps involved in election process. The students would be able to be able to use AI supported Tool for casting the votes and getting the results. The students would be able to make familiar with Data exploration through AI.

Follow up Activities

Students would be asked to submit a report on Indian Election www.inkle.writer System.

Reflections

What role does the Election Commission play in getting free and fair elections through the entire Election process? 

Study Time Max Time: 2 hr Max Questions: 59

CHAPTER

12

WORKING OF INSTITUTIONS Examine the Roles , responsibilities and interdependency of all the 3 organs of the Government. l Examine the rule of law in India and its relevance. l Understand the power and working of Indian Judiciary system and comprehend the hierarchy system of the judiciary in India. l

Learning Objectives

How is a major policy

Topic-1 decision taken?; Parliament Revision Notes

TOPIC - 1 How is a major policy decision taken?; Parliament .... P. 170 TOPIC - 2 Political Executive

.... P. 176

TOPIC - 3 The Judiciary

.... P. 180

A Government Order  On August 13, 1990, the Government of India issued an Order.  It was called an Office Memorandum.  According to this order, other than SC and ST the 27% job reservation benefit will be given to a new third category called Socially and Educationally Backward Classes (SEBC).  Only persons who belong to backward classes, were eligible for this quota.

The Decision Makers

Scan to know more about

 In a Democratic Government, the decision making power is divided in three separate organs – this topic the legislature, the executive and the judiciary.  The legislature makes the laws, the executive implements them, and the judiciary resolves disputes that arise out of major policy decisions. It consists of an assembly of the people’s representatives which has the power to enact laws for a country. Major Policy Decisions  Any major policy decision is conveyed through a government order. A government order is also called an office memorandum. This Office Memorandum was the culmination of a long chain of events.  The Second Backward Classes Commission in India was established in 1979 by the Janata Party Government under the then Prime Minister Morarji Desai. It was popularly known as the Mandal Commission.  As per the Mandal Commission recommendations, a government order announced that 27 per cent of vacancies in civil posts and services under the Government of India would be reserved for Socially and Economically Backward Classes (SEBC). The reservations issue was strongly protested by the people stating that this largely affected everyone’s job opportunities.

Need for the Political Institution

 A democracy works well when political institutions perform functions assigned to them.  Institutions involve Meetings, Committees and Routines.  This often leads to delays and complications.  Some of the delays and complications introduced by institutions are very useful as they provide an opportunity for a wider set of people to be consulted.

WORKING OF INSTITUTIONS

171

172

Oswaal CBSE Question Bank Chapterwise & Topicwise, SOCIAL SCIENCE, Class-IX

 Institutions make it difficult to have a good decision taken very quickly. But, they also make it equally difficult to rush through a bad decision.  There are several such institutions at work : The Prime Minister and the Cabinet are institutions that make all the important policy decisions. The civil servants are responsible for taking steps to implement the policy decisions effectively. The Supreme Court, as an institution, resolves the disputes between the citizens and the government.

Why Do We Need Parliament  The Parliament is a national assembly of elected representatives of the people. The Parliament has the authority of enacting laws. It can add new laws, and change or abolish existing laws. National policy and important public issues are discussed and debated in the Parliament.

Two Houses of Parliament  The Parliament consists of the office of the President of India and two houses—the Rajya Sabha or the Council of States, and the Lok Sabha or the House of the People.

Scan to know more about this topic

 The Rajya Sabha is the Upper House of the Parliament of India. It consists of 250 members, of which 12 are nominated by the President of India. The remainder of the Rajya Sabha is elected by state and territorial legislatures. The term of office is 6 years, and 1/3rd of the members retire Difference every 2 years. The Vice President is the ex-officio Chairman of the Rajya Sabha. between Rajya  The Lok Sabha is directly elected by the people of India. The Lok Sabha can have a maximum Sabha & Lok Sabha of 552 members, including 20 members from the Union Territories and 2 from the Anglo Indian community. The Speaker presides over the sessions.  Though the Rajya Sabha has some special powers, the Lok Sabha has supreme powers. The Lok Sabha can be dissolved by the President. Once the Lok Sabha passes the budget or a law related to money, the Rajya Sabha cannot reject it.

Key Terms  Office memorandum: A communication issued by an appropriate authority stating the policy or decision of the government.  Reservations: A policy that declares some positions in government employment and educational institutions ‘reserved’ for people and communities who have been discriminated against, are disadvantaged and backward.  Lok Sabha: It is the Lower House of the Parliament. The Lok Sabha is directly elected by the people of India.  Rajya Sabha: The Rajya Sabha is the Upper House of the Parliament of India. The Rajya Sabha is indirectly elected by members of state assemblies.  Speaker: He is the presiding officer of the Lok Sabha and is responsible for the efficient conduct of business in the Lok Sabha.  Ordinance: Temporary law promulgated by the President of India on the recommendations of the Union Cabinet. It can only be issued when the Parliament is not in session. It has to be approved by the Parliament within six weeks of its first sitting otherwise the ordinance would be considered null and void.  Money Bills: Bills dealing with money matters like taxes, income, expenditure and grants.

Example ‘‘Parliament is the supreme legislature of India.’’ Justify the statement. Answer:

Step 1: In all democracies, an assembly of elected representatives exercises supreme political authority on behalf of the people.



Step 2: In India, such a national assembly of elected representatives is called Parliament. At the state level, it is called Legislature or Legislative Assembly. Step 3: Parliament is the final authority for making laws in any country. Step 4: Parliaments all over the world can make new laws, change existing laws or abolish existing laws and make new ones in their place.

WORKING OF INSTITUTIONS

OBJECTIVE TYPE QUESTIONS Multiple Choice Questions Q. 1. In a Democratic Government, the decision making R power is divided in the: (A) legislature (B) executive (C) judiciary (D) All of these Ans. Option (D) is correct. Explanation: In a Democratic Government, the decision making power is divided into three separate organs – the legislature, the executive and the judiciary. Q. 2. The Second Backward Classes Commission in R India was established in ________. (A) 1979 (B) 1978 (C) 1977 (D) 1976 Ans. Option (A) is correct. Explanation: The Second Backward Classes Commission in India was established in 1979 by the Janata Party Government under the then Prime Minister Morarji Desai. It was popularly known as the Mandal Commission. R Q. 3. The Rajya Sabha is known as the _______ . (A) upper chamber (B) lower chamber (C) middle chamber (D) None of these Ans. Option (A) is correct.

Assertion and Reason In the questions given below, there are two statements marked as Assertion (A) and Reason (R). Read the statements and choose the correct options:

173

(1 mark each)



(A) Both (A) and (R) are true and (R) is the correct explanation of (A).



(B) Both (A) and (R) are true but (R) is not the correct explanation of (A).



(C) (A) is correct but (R) is wrong.



(D) (A) is wrong but (R) is correct.

Q. 1. Assertion (A): The Lok Sabha exercises supreme R power.

Reason (R): Our Constitution does give the Rajya Sabha some special powers over the states.

Ans. Option (B) is correct.

Explanation: Lok Sabha exercises more powers in money matters. Once the Lok Sabha passes the budget of the government or any other money related law, the Rajya Sabha cannot reject it.

Q. 2. Assertion (A): If the majority of the Lok Sabha members say they have ‘no confidence’ in the Council of Ministers, all ministers including the U

Prime Minister, have to quit.

Reason (R): The Lok Sabha controls the Council of Ministers.

Ans. Option (A) is correct. Q. 3. Assertion (A): Those who run the government can take decisions only so long as they enjoy support A

of the Parliament.

Reason (R): Parliaments all over the world exercise some control over those who run the government.

Ans. Option (A) is correct.

SUBJECTIVE TYPE QUESTIONS Very Short Answer Type Questions Q. 1. The Lok Sabha controls the Council of Ministers. E Explain. Ans. The Lok Sabha controls the Council of Ministers. Only a person who enjoys the support of the majority of the members in the Lok Sabha is appointed the Prime Minister. If the majority of the Lok Sabha members say they have ‘no confidence’ in the Council of Ministers, all ministers including the Prime Minister, have to quit. The Rajya Sabha does not have this power. (2) Q. 2. Define Parliament. Mention any one important R role.  Ans. In India, a national assembly of elected representatives who exercises supreme political authority on behalf of the people is called the Parliament.

(2 marks each)

Role: The Parliament is the final authority for making laws in our country. ( 1+1=2) R Q. 3. Name the two houses of the Parliament. Ans. The two houses of the Parliament are: (i) The Lok Sabha (ii) The Rajya Sabha

(1+1=2)

Q. 4. Which House has more power regarding the U money bill? Give reason. Ans. The Lok Sabha exercises more powers in money matters. Once the Lok Sabha passes the budget of any other money related law, the Rajya Sabha cannot reject it. The Rajya Sabha can only delay it

174

Oswaal CBSE Question Bank Chapterwise & Topicwise, SOCIAL SCIENCE, Class-IX

by 14 days or suggest changes in it. The Lok Sabha may or may not accept these changes. (2)

Ans. An office memorandum is the communication issued by an appropriate authority stating the policy or decision of the government. (2)

Short Answer Type Questions Q. 1. ‘‘The Indira Sawhney and others vs Union of India Case’’, was filed in which concern? Describe it. A (Board Term II, 2016) Ans. (i) The Second Backward Classes Commission in India was established in 1979 by the Janata Party Government under the then Prime Minister Morarji Desai. It was headed by Indian Parliamentarian B.P. Mandal. Hence, it was popularly known as the Mandal Commission. (ii) The Commission was asked to determine the criteria to identify the socially and educationally backward classes in India and recommend steps for their advancement. (iii) The Commission gave its report in 1980 and made many recommendations. One of these was that 27% of government jobs be reserved for the socially and economically backward classes (SEBC). The reservation issue was very strongly protested after the implementation of the Commission Report in 1990. People reacted vehemently because this decision affected thousands of job opportunities. (iv) Some people felt that reservations were necessary to balance the inequality among people of different castes in India. Others felt that reservations were unfair and would deny equal opportunities to those who did not belong to backward communities. Following the nation-wide outrage against the order and protest, a court case was filed against the order. These cases were grouped together by the supreme court and called the ‘Indira Sawhney and others Versus the Union of India Case’. (Any three) (1×3=3) Q. 2. Give any three functions (or responsibilities) of the government. R Ans. (i) Government collects taxes and uses it for administration, defence and development programmes. (ii) Government ensures security to the citizens and provides facilities for education and health. (iii) It formulates and implements several welfare schemes.(1×3=3)

Commonly Made Error

Students mostly mentioned the features of the government when asked about the functions of the government.

R

Q. 5. Define office memorandum. 

(3 marks each)

Answering Tip

Government collects taxes, ensures security of citizens and implements welfare schemes.

Q. 3. “The Lok Sabha controls the Council of Ministers.” Justify this statement.  U (Board Term II, 2017) Ans. (i) Only a person who enjoys the support of the majority of the members in the Lok Sabha is appointed as the Prime Minister. (ii) If the majority of the Lok Sabha members say they have “no confidence” in the Council of Ministers, then all the ministers including the Prime Minister have to quit. [CBSE Marking Scheme, 2017] 1½+1½=3 Q. 4. Describe the role played by the contemporary Prime Minister of India in the implementation of the Mandal Commission Report. A (Board 2015, Term II) Ans. (i) The Prime Minister informed the Parliament about the decision through a statement in both the Houses of Parliament. (ii) The decision of the cabinet was sent to the department of personnel and training. (iii) The senior officers of the department drafted an order in line with the cabinet decision and took the minister’s approval. (1×3=3)

Commonly Made Error

It is assumed by students that the Mandal Commission was established by the Congress Party.

Answering Tip

Students should know that the Mandal

Commission was established by the Janata Party Government under Prime Minister Morarji Desai.

Q. 5. Why do we need a Parliament? Explain any two functions of Parliament. Ans. (i) Parliament is the final authority for making laws. (ii) It exercises control over the working of the government. (iii) It controls all the money that government have.  ( 1×3=3)

WORKING OF INSTITUTIONS

Commonly Made Error

Student are unable to understand the function of the parliament.

Answering Tip

Students should know that parliament exercises

control over the working and also controls money of the government.

Long Answer Type Questions Q. 1. Analyse the reaction of the people after the implementation of the Mandal Commission Report. A (Board Term II, 2014, 2013) Ans. It led to widespread protests, some of which were violent. People reacted strongly because this decision affected thousands of job opportunities. (i) The people felt this would give fair opportunity to those communities who had not been represented in government employment. (ii) Some felt that this was unfair as it would deny equality of opportunity to those who did not belong to backward communities. (iii) They would be denied jobs even they were qualified. (iv) Some felt that this would arouse caste feelings. (v) Others felt that this situation would lead to social division and weaker national unity.

[CBSE Marking Scheme, 2014]

Q. 2. Compare the powers of Lok Sabha and Rajya Sabha. R (Board Term II, 2016) Ans. (i) Any ordinary law needs to be passed by both the houses. But in case of a difference the final decision is taken in a joint session in which the members of both the houses sit together. The view of the Lok Sabha is likely to prevail in such a meeting. (ii) Lok Sabha exercises more powers in money matters. Once the Lok Sabha passes the bill, the Rajya Sabha cannot reject it.

Ans. Election Procedure of the two Houses of Parliament: (i) The Lok Sabha is directly elected by the people and enjoys real power on behalf of the people. (ii) The Rajya Sabha is elected indirectly and mainly looks after the interest of the regions or federal units. The Lok Sabha enjoys supreme powers: (i) If there is a joint session of the two houses, then the will of the Lok Sabha prevails due to its numerical supremacy. (ii) In money matters, the Lok Sabha is supreme as the Rajya Sabha can only delay a money bill for 14 days or give suggestions. (iii) The Lok Sabha controls the Council of Ministers through ‘no-confidence motion’. (CBSE Marking Scheme 2013) (2+3=5)

Commonly Made Error

(v) If the Lok Sabha Passes ”no confidence” motion than the Council of Ministers and the Prime Minister have to quit. The Rajya Sabha does not have this power. (1×5=5) Q. 3. Describe the election procedure of the two houses of Indian Parliament. List three areas where Lok Sabha enjoys supreme powers. A (Board 2013)

(5 marks each)

OR In which three ways is the Lok Sabha more powerful than the Rajya Sabha? (Board Term II, 2013) OR How does the Lok Sabha exercise supreme power over the Rajya Sabha? Explain. (Board Term II) OR Which house of the Parliament is more powerful in India and why? Give any three reasons to justify your answer.  (Board Term II)

(iii) The Rajya Sabha can delay it for only 14 days or suggests changes in it. The Lok Sabha may or may not accept it. (iv) The Lok Sabha controls the Council of Ministers. Only a person who enjoys the support of the Lok Sabha is appointed the Prime Minister.

175

Student are unable to differentiate between the

features and powers of Lok Sabha and Rajya Sabha.

Answering Tip

Understand the difference between the features and powers of Lok Sabha and Rajya Sabha.

176

Oswaal CBSE Question Bank Chapterwise & Topicwise, SOCIAL SCIENCE, Class-IX

Topic-2 Political Executive Revision Notes Political and Permanent Executive  A group of functionaries is called the executive. The executive has two categories: political and permanent. The political executive consists of political leaders who are elected by the people for a specific term.  The permanent executive includes members who are appointed on a long-term basis. The permanent executive is also called the civil services. The political executive has more powers and the final say in all important decisions, as they are the representatives of the people.

Scan to know more about this topic

Political Executive

Prime Minister and Council of Ministers  The political executive includes the Prime Minister and the Council of Ministers. The President appoints the leader of the majority, or the coalition party that has a majority in the Lok Sabha, as the Prime Minister. Prime Minister is the most important political institution in the country.  The remaining ministers are appointed by the President based on the Prime Minister’s advice. These ministers are usually from the party or the coalition that has a majority in the Lok Sabha. The group of minister is called the Council of Ministers. It usually has 60 to 80 Ministers of different ranks. The Council of Ministers includes Cabinet Ministers, Ministers of State with independent charge and other Ministers of State.

Powers of Prime Minister  The Prime Minister has several powers as the Head of the Government. He chairs cabinet meetings, and assigns work to the other ministers. He also has the power to dismiss ministers. If the Prime Minister resigns, the entire ministry is supposed to resign.  The Prime Minister is the most powerful member in the Cabinet. The power of the Prime Minister in all parliamentary democracies of the world has increased so much in the recent decades that parliamentary democracies are sometimes seen as the Prime Ministerial form of Government.

The President  The President is the Head of the State and has nominal powers. The President supervises the overall functioning of all political institutions in the country to achieve the objectives of the state. The President is not directly elected by the people. All the Members of Parliament (MPs) and the members of the state legislative assemblies elect the President.

Powers of President  All major government activities, policy decisions and laws are issued in the name of the President. The President takes all major decisions based on the advice of the council of ministers.  One major decision that the President makes on his or her own is to appoint the Prime Minister. The President appoints the leader of the majority party or coalition that enjoys a majority support in the Lok Sabha as the Prime Minister.

Key Terms  Cabinet : A body of senior ministers who controls important ministers.  Collective responsibility: For any decision or action of the Cabinet, the Council of Ministers is collectively responsible. If any cabinet decisions is not approved by the Parliament, the entire Council of Ministers has to resign.  Emergency: Extraordinary or abnormal situation in a country which can pose threat to the constitution or security.

WORKING OF INSTITUTIONS

OBJECTIVE TYPE QUESTIONS Multiple Choice Questions Q. 1. What is the position of the President in India?  U (A) Nominal head of the state (B) Real head of the state (C) Hereditary head of the state (D) None of the above Ans. Option (A) is correct. Explanation: The President of India is like the Queen of Britain whose functions are to a large extent ceremonial. The President supervises the overall functioning of all the political institutions in the country so that they operate in harmony to achieve the objectives of the State. R Q. 2. What is meant by political executive?  (A) The Speaker (B) Political leaders who are elected by the people for a specific term. (C) Only those political leaders who are in the ruling party. (D) Only those political leaders who are in the opposition party. Ans. Option (B) is correct. Explanation: The President of India is like the Queen of Britain whose functions are to a large extent ceremonial. The President supervises the overall functioning of all the political institutions in the country so that they operate in harmony to achieve the objectives of the State. Q. 3. Study the given Picture carefully and answer the U question that follows:

177

(1 mark each)

Assertion and Reason In the questions given below, there are two statements marked as Assertion (A) and Reason (R). Read the statements and choose the correct options:

(A) Both (A) and (R) are true and (R) is the correct explanation of (A).



(B) Both (A) and (R) are true but (R) is not the correct explanation of (A).



(C) (A) is correct but (R) is wrong.



(D) (A) is wrong but (R) is correct.

Q. 1. Assertion (A): The Prime Minister of a coalition government cannot take decisions as he likes.

U

Reason (R): In recent years the rise of coalition politics has imposed certain constraints on the power of the Prime Minister.

Ans. Option (A) is correct. Q. 2. Assertion (A): While the Prime Minister is the head of the government, the President is the head of the State. 

U

Reason (R): The President can be seen to represent the entire nation.

Ans. Option (B) is correct.

Explanation: The President supervises the overall functioning of all the political institutions in the country so that they operate in harmony to achieve the objectives of the State.

Q. 3. Assertion (A): Jawaharlal Nehru, the first Prime Minister of India, exercised enormous authority because he had great influence over the public.

Reason (R): Indira Gandhi was also a very powerful leader compared to her colleagues in the Cabinet.

Ans. Option (B) is correct.

This picture is related to which of the following. (A) General meeting (B) Assembly meeting (C) Cabinet meeting (D) None of these Ans. Option (C) is correct. Explanation: This cartoon depicts a cabinet meeting chaired by Prime Minister Indira Gandhi in early 1970s, at the peak of her popularity.

Explanation: In India, we have seen the tendency towards the concentration of powers in the hands of the Prime Minister.

178

Oswaal CBSE Question Bank Chapterwise & Topicwise, SOCIAL SCIENCE, Class-IX

SUBJECTIVE TYPE QUESTIONS Very Short Answer Type Questions Q. 1. What do you know about permanent executive? R

 Ans. The category of people who are appointed on a long-term basis in a democracy is called the permanent executive or civil services. They remain in office even when the ruling party changes. These officers work under political executive and assist them in carrying out the day-to-day administration.  (2) Q. 2. Write any two executive powers of President of R India. Ans. (i) Administration of whole country is carried on in his name. (ii) He appoints the Prime Minister, Central Ministers, Governors of the States and Judges of Supreme Court and High Courts. (iii) He is the Supreme Commander of the Indian Defence Forces. (iv) He receives the credentials of ambassadors from other countries.  (Any Two Points) (1 x 2 = 2)

Q. 3. What do you know about the Cabinet Ministers? 

Ans. Ministers of states with independent charge are usually in charge of smaller ministries. (i) They participate in the cabinet meetings only. (ii) They are attached to and required to assist the respective Cabinet Minister. (iii) They look after the ministry to which attached.  [CBSE Marking Scheme, 2017] (1+2=3)

Commonly Made Error

Student always confuse about the Minister of Union and the Minister of State.

Answering Tip

Ministers of State participate in cabinet

meetings and look after the ministry to which they are attached.

U

Ans. (i) Cabinet Ministers are usually top-level leaders of the ruling party or parties who are in charge of the major ministries. (ii) Usually the Cabinet Ministers meet to take decisions in the name of the Council of Ministers.  (1 x 2 = 2) Q. 4. Within the Cabinet it is the Prime Minister who is R the most powerful. Explain. Ans. The powers of the Prime Minister in all parliamentary democracies of the world have increased so much in recent decades that parliamentary democracies are some times seen as Prime Ministerial form of government. (2) Q. 5. All governmental activities take place in the U name of the President. Discuss.  Ans. All governmental activities take place in the name of the President. (i) All laws and major policy decisions of the government are issued in her name. (ii) All major appointments are made in the name of the President. (1 x 2 = 2)

Short Answer Type Questions Q. 1. Who are the Ministers of State? Write any two R (Board Term II, 2017) function of them.

(2 marks each)

(3 marks each)

Q. 2. Why does the political executive have more powers than the permanent executive? Give any three reasons. U (Board Term II, 2016) Ans. (i) Every minister is the representative of the people and empowered to exercise the will of the people on their behalf. (ii) She/He is finally answerable to the people for all the decision. (iii) The ministers take the advice of the experts on all technical matters and decide according to the overall objective. (1×3=3) [CBSE Marking Scheme, 2016] Q. 3. How are the ministers in the Council of Ministers categorised according to their rank? U (Board Term II, 2013) OR Explain the composition of the Union Council of Ministers. OR Explain the composition of the Council of Ministers.

WORKING OF INSTITUTIONS

two or more political parties usually when no single party enjoys majority support of the members in the legislature.

Ans. The Union Council of Ministers comprises the three categories of ministers. These are:

(i) Cabinet Ministers: They constitute the inner ring of the Council of Ministers. These are the toplevel leaders of the ruling party/parties who are incharge of the important ministries. They usually meet to take decisions in the name of the Council of Ministers.

(ii) Ministers of State with Independent Charge: They are usually incharge of smaller ministries. They participate in the Cabinet meetings only when they are invited. (iii) Ministers of State: They are attached to and are required to assist the Cabinet Ministers. (iv) It is headed by the Prime Minister. (Any three) [CBSE Marking Scheme, 2013] (1×3=3)

Commonly Made Error

Students always get confused about the Council of Minister.



Limitations:

(i) The Prime Minister has to accommodate different groups and sections in his party as well as among his alliance partners. (ii) He also has to heed to the views and positions of coalition partners and other parties, on whose support the survival of the government depends. (1+2=3) Q. 5. What is the tenure of the President in India? Mention the qualifications for President of India.  R Ans. The tenure of the President of India is for five years. (i) The President in India is the head of the State. He has only nominal powers. (ii) The President of India is like the Queen of Britain whose functions are to a large extent ceremonial. (iii) The President supervises the overall functioning of all the political institutions in the country. The President exercises all his powers on the advice of the Council of Ministers. (1×3=3)

Answering Tip

Commonly Made Error

Students must know that the three categories of



ministers includes Cabinet Ministers, Ministers of State with Independent Charge.

Answering Tip

What is a coalition government? Mention any two limitations of a coalition government. Ans. Coalition government is formed by an alliance of

Long Answer Type Questions



A (Board Term II, 2017)

Ans. (i) Prime Minister of the majority party is the Head of the Government. (ii) Prime Minister exercises enormous authority. He is indirectly elected. In fact, he is appointed by the President. (iii) The powers of the Prime Minister in all parliamentary democracies of the world have increased so much in era continued. Decades that

Students always gets confused between President and Vice-President.

Q.4. What is a coalition government? Why the Prime Minister of a coalition government cannot take decisions as he likes? R OR

Q. 1. Compare the position of the Prime Minister of the Majority party (ruling) and the Prime Minister of the Coalition Government. 

179



Students should know that President’s tenure

is of 5 years and he is the head of the country with nominal powers.

(5 marks each) these are sometimes seen as Prime Minister from of government.

(iv) In recent years, the coalition government has changed the position of Prime Minister. Now under coalition government, a Prime Minister cannot take decision as he likes. (v) He has to accommodate views of different groups and factions in his party as well as among the alliance partners on whose support the survival of the government depends. 

[CBSE Marking Scheme, 2017] (1×5=5)

180

Oswaal CBSE Question Bank Chapterwise & Topicwise, SOCIAL SCIENCE, Class-IX

Q. 2. Describe how the Prime Minister and the Council of Ministers are appointed.

A

(Board Term II, 2016)

Ans. (i) The President appoints the leader of the majority party or the coalition of parties that commands a majority in the Lok Sabha, as Prime Minister. (ii) In case no single party or alliance gets a majority, the President appoints the person most likely to secure a majority support. (iii) The President appoints other ministers on the advice of the Prime Minister. (iv) The Ministers are usually from the party or the coalition that has the majority in the Lok Sabha. (v) The Prime Minister is free to choose ministers as they are members of Parliament. A person who is not a member of Parliament can also become a minister, but he has to get elected to one of the Houses of Parliament within six months of appointment as minister.



[CBSE Marking Scheme, 2016] (1×5=5)

Q. 3. “Prime Minister is the most important political institution in the country’’. Justify the statement. A (Board Term II, 2013) OR Explain the position of the Prime Minister with reference to his/her powers and functions. OR Describe any five powers and functions of the Prime Minister of India. Ans. Position: The Prime Minister is the most important political institution of the country. He/she is the head of the government and all important decisions regarding the country are taken by him/her. He/ she enjoys the real executive powers as head of the State. Powers and Functions of Prime Minister: (i) Allocation of departments and formation of Council of Ministers, their distribution of portfolios. (ii) Being the Chairman of the Cabinet, Prime Minister presides the meetings of the Cabinet. (iii) Link between the President and Cabinet. (iv) Leader of the nation. (v) Ex-officio chairman of the Planning Commission. (1×5=5)

Commonly Made Error

Student always get confused about the appointment of ministers.

Commonly Made Error

Answering Tip

Students should know that President appoints

Prime Minister and the Prime Minister further recommends the ministers for his council of ministers who are further appointed by the President of India.

Students are not able to explain the powers and functions of the Prime Minister.

Answering Tip

The Prime Minister of India is the head of the

government and he/she is the responsible for taking important decisions regarding the country.

Topic-3 The Judiciary Revision Notes  The judiciary includes all the courts at different levels in a country and consists of the Supreme Court, High Courts and District Courts. The Supreme Court is the top legal organisation. Scan to know  The Indian courts of law are further divided into two groups : civil courts and criminal courts. more about The civil courts deal with general disputes regarding land, property and rights. The criminal this topic courts deal with cases of murder, riot and looting.  The Supreme Court controls the judicial administration in the country and resolves disputes between citizens and the government, between two or more states and between states and the The Judiciary union governments.  The Supreme Court is the highest court of appeal in civil and criminal cases. It can hear appeals against the decisions of the High Courts.

WORKING OF INSTITUTIONS

181

 The Indian judiciary is known for being independent of the legislature and the executive and is non-partisan. The judges do not act on the direction of the government or the ruling party. The judges for the Supreme Court and High Courts are appointed by the President on the advice of the Prime Minister and in consultation with the Chief Justice of Supreme Court.  The senior judges of the Supreme Court select the new judges of the Supreme Court and the High Courts. There is very little scope for interference by the political executive. Once a person is appointed as judge of the Supreme Court or the High Court, it is nearly impossible to remove him or her from that position. It is as difficult as removing the President of India.  A judge can be removed only when an impeachment motion is passed separately by a two-third majority of members of each of the two houses of the Parliament.

Powers of Judiciary  The Supreme Court and the High Courts have the power to interpret the Constitution. If the courts feel that any law or action of the government is against the Constitution, they can declare it invalid.  The judiciary can review laws and government actions. The Indian judiciary is independent in the appointment and removal of judges, and in the execution of its powers.  Anyone can approach the court if public interest and human rights are affected by the actions of the government. This is called a Public Interest Litigation. The courts can intervene to ensure that the government and its officials are not misusing the powers.

Key Term  Impeachment: A special parliamentary procedure to prosecute or to remove the President and other judges for the violation of the constitution.

OBJECTIVE TYPE QUESTIONS Multiple Choice Questions R, U Q. 1. What is Judiciary?  (A) The branch of government that is endowed with the authority to frame the laws. (B) The branch of government that is endowed with the authority to interpret laws. (C) The branch of government that is endowed with the authority to interpret and apply the laws, adjudicate legal disputes, and otherwise justice. (D) The branch of government that is endowed with the authority to execute the laws. Ans. Option (C) is correct. Q. 2. The senior most judge of the Supreme Court is R appointed by the: (A) President (B) Council of Ministers (C) Chief Justice (D) Prime Minister Ans. Option (C) is correct. Q. 3. Which institution is empowered to administer justice in India? (A) Legislature (B) Executive (C) Judiciary (D) Ruling party Ans. Option (C) is correct.

Assertion and Reason In the questions given below, there are two statements marked as Assertion (A) and Reason (R). Read the statements and choose the correct options:

(1 mark each)



(A) Both (A) and (R) are true and (R) is the correct explanation of (A).



(B) Both (A) and (R) are true but (R) is not the correct explanation of (A).



(C) (A) is correct but (R) is wrong.



(D) (A) is wrong but (R) is correct.

Q. 1. Assertion (A): The Supreme Court controls the E judicial administration in the country.

Reason (R): Its decisions are binding on all other courts of the country.

Ans. Option (A) is correct.

Explanation: Supreme court controls the judicial administration in the country and resolve disputes between citizens and the Government. Supreme Court is the highest court of appeal, it can hear appeal against the decision of high court.

Q. 2. Assertion (A): The judiciary in India is also one of U the most powerful in the world.

Reason (R): The Supreme Court and the High Courts have the power to interpret the Constitution of the country.

Ans. Option (A) is correct. Q. 3. Assertion (A): Anyone can approach the courts if public interest is hurt by the actions of government.

Reason (R): The courts intervene to prevent the misuse of the government’s power to make A decisions.

Ans. Option (A) is correct.

182

Oswaal CBSE Question Bank Chapterwise & Topicwise, SOCIAL SCIENCE, Class-IX

SUBJECTIVE TYPE QUESTIONS Very Short Answer Type Questions

(2 marks each)

Q. 1. India has a single integrated judicial system. R Explain the structure.  Ans. The Indian judiciary consists of a Supreme Court for the entire nation, High Courts in the states, District Courts and the courts at local level. (2) Q. 2. What do you know about the Supreme Court of R India? 

(iii) The Supreme Court controls the judicial administration in the country and resolves disputes between citizens and the government, between two or more states and between states and the union governments. (Any Two Points) (1 x 2 = 2) Q. 3. What do the civil and criminal courts deal with? R 

Ans. (i) The Supreme Court is the highest court of appeal in civil and criminal cases.

Ans. (i) The civil courts deal with general disputes regarding land, property and rights.

(ii) It can hear appeals against the decisions of the High Courts.

(ii) The criminal courts deal with cases of murder, riot and looting. (1 x 2 = 2)

Short Answer Type Questions Q. 1. Why is the Indian judiciary considered one of the most powerful in the world? Give any three reasons. U (Board Term II, 2016) Ans. (i) Once a person is appointed as judge of the Supreme Court or High Court, it is nearly impossible to remove him/her from that position. (ii) A Judge can be moved only by an impeachment motion. (iii) The Supreme Court and the High Court have the power to interpret the constitution of the country. Therefore, it is one of the most powerful in the world. (1×3=3)

Commonly Made Error

(iii) The Constitution of India provides the security to service of the judges. Once appointed by the President, their service cannot be terminated by will or by any authority whatever. (iv) There is a security of pay and allowances of the judges. Their salaries cannot be reduced. (v) The Supreme Court and the High Courts are free to decide their own procedure of work and establishment. (vi) Judge is not allowed to practice after retirement so that they cannot influence judgements of the court. (Any three)



Students are unable to mention the reason for Indian judiciary to be most powerful.



In the Indian Judiciary, Supreme Court and

Ans. Independence of the judiciary: (i) Judiciary is not under the control of the legislature or the executive. (ii) The judges do not act on the direction of the government or according to the wishes of the party in power.

[CBSE Marking Scheme 2020] (1×3=3)

It is thought by the students that the judiciary in India has no connection with the Constitution of India.

Answering Tip

High Court have the power to interpret the Constitution of the country.

Q. 2. What is meant by independence of the judiciary? Explain. U (Board Term II, 2013)



Commonly Made Error

Answering Tip

(3 marks each)



The framework of the current legal system in India has been laid by the Constitution of India and the judiciary system derives its powers from it.

Q. 3. Write about the process of appointment and removal of a judge of the Supreme Court. 

A

Ans. The judges of the Supreme Court and the High Courts are appointed by the President on the advice of the Prime Minister and in consultation with the

WORKING OF INSTITUTIONS

Chief Justice of India. In practice the senior judges of the Supreme Court select the new judges of the Supreme Court.

A judge can be removed only by an impeachment motion passed separately by two-third members of the two houses of the Parliament. 3 Q. 4. After reading the example of the reservation order, three students had different reactions to the role of the judiciary. Which view, according to you, is a correct reading of the role of the judiciary? (a)  Srinivas argues that since the Supreme Court agreed with the government, it is not independent. (b) Anjaiah says that the judiciary is independent because it could have given a verdict against the government order. The Supreme Court did direct the government to modify it. (c)  Vijaya thinks that the judiciary is neither independent nor conformist, but acts as a

mediator between opposing parties. The court struck a good balance between those who supported and those who opposed the order.  U (NCERT) Ans. (a) The argument given by Srinivas is not correct. After hearing arguments of both sides the Supreme Court declared the order of the Government of India on reservations as valid. Agreeing with the valid order of the government does not mean that the judiciary is not independent. Moreover, the Supreme Court directed the government to modify its original order. This also proves that the judiciary is independent. (b) The view of Anjaiah is correct. (c) The view of Vijaya is not correct. The job of the courts is not to act as a mediator between the two parties, but decide the case according to its merits and the law.  (1 x 3 = 3)

Long Answer Type Question Q. 1. Write an article on the functions and the powers of judiciary in India. A (Board Term II, 2016) OR Describe the powers of the Supreme Court.  (Board Term II, 2014) OR What is meant by integrated judiciary? Explain the jurisdiction of the Supreme Court of India. Mention any three points. OR Describe any three jurisdictions of the Supreme Court in India. Ans. (i) All the courts at different levels in a country put together is called a Judiciary. The Indian judiciary consists of the Supreme Court for the entire nation, High courts and the Courts at local level.

(5 marks)

(ii) Supreme Court controls the judicial administration in the country. It’s decisions are binding on all other courts of the country. It can take up any dispute: (a) Between citizens of the country. (b) Between two or more state governments. (c) Between citizens and the government. (d) Between governments at the Union and State level. Powers: (i) The Supreme Court and the High Courts have the power to interpret the constitution of the country. (ii) It can hear appeals against the decisions of the High Courts. (iii) It acts as the guardian of the fundamental rights. (iv) It controls the Judicial administration in the Country. (v) It is the highest court of appeal in civil and criminal cases. (1×5=5)

COMPETENCY BASED QUESTIONS Case based MCQs I. Read the source given below and answer the following questions: (1+1+1+1=4) The Constitution does not say very much about the powers of the Prime Minister or the ministers or their relationship with each other. But as head of the government, the Prime Minister has wide ranging powers. He chairs Cabinet meetings. He coordinates the work of different departments.

183

(4 marks each)

His decisions are final in case disagreements arise between departments. He exercises general supervision of different ministries. All ministers work under his leadership. The Prime Minister distributes and redistributes work to the ministers. He also has the power to dismiss ministers. When the Prime Minister quits, the entire ministry quits. Thus, if the Cabinet is the most powerful institution in India, within the Cabinet it is the Prime Minister who is the most powerful. The powers of the Prime

184

Oswaal CBSE Question Bank Chapterwise & Topicwise, SOCIAL SCIENCE, Class-IX

Minister in all parliamentary democracies of the world have increased so much in recent decades that parliamentary democracies are sometimes seen as Prime Ministerial form of government. As political parties have come to play a major role in politics, the Prime Minister controls the Cabinet and Parliament through the party. The media also contributes to this trend by making politics and elections as a competition between top leaders of parties. In India too we have seen such a tendency towards the concentration of powers in the hands of the Prime Minister. Jawaharlal Nehru, the first Prime Minister of India, exercised enormous authority because he had great influence over the public. Indira Gandhi was also a very powerful leader compared to her colleagues in the Cabinet. Of course, the extent of power wielded by a Prime Minister also depends on the personality of the person holding that position. 1. The political executive includes the ............... and ................ . (A) The President and the Vice President (B) The Governor and the Prime Minister (C) The Prime Minister and Council of Ministers (D) The Prime Minister and the President Ans. Option (C) is correct. 2. When the Prime Minister quits, who quits along with him/her? (A) No Minister quits with him/her (B) The entire Ministry quits (C) Only 50% Minister quits (D) The President quits along with him. Ans. Option (B) is correct. Explanation: When the Prime Minister quits, the entire Council of Ministers quits with him. 3. The ................ also contributes by making politics and election as a competition between top leaders of parties.

(A) Government (B) Public (C) Media (D) Opposition Parties Ans. Option (C) is correct. 4. ................ was also a very powerful leader compared to other colleagues in the cabinet. (A) Indira Gandhi (B) Mahatma Gandhi (C) Jawaharlal Nehru (D) Lal Bahadur Shastri Ans. Option (A) is correct.

Subjective based Questions

I. Read the source given below and answer the following questions: (1+1+2=4) Prime Minister is the most important political institution in the country. Yet there is no direct election to the post of the Prime Minister. The President appoints the Prime Minister. But the President cannot appoint anyone he likes. The President appoints the leader of the majority party or the coalition of parties that commands a majority in the Lok Sabha as Prime Minister. In case no single party or alliance gets a majority, the President appoints the person most likely to secure a majority support. The Prime Minister does not have a fixed tenure. He continues in power so long as he remains the leader of the majority party or coalition. 1. How is the Prime Minister appointed? Ans. There is no direct election to the post of the Prime Minister. The President appoints the Prime Minister. 2. Who is the Head of the Government and exercises all governmental powers? Ans. The Prime Minister is the Head of the government and exercises all governmental powers 3. What is the government formed by an alliance of two or more political parties called? Ans. The government formed by an alliance of two or more political parties is called coalition government. 

Study Time Max Time: 1:45 hr Max Questions: 41

CHAPTER

13

Learning Objectives

Topic-1

DEMOCRATIC RIGHTS l

Evaluate the role of rights in Democracy.

Life Without Rights; Rights in a Democracy

Revision Notes

TOPIC - 1 Life Without Rights; Rights in a Democracy .... P. 185 TOPIC - 2 Rights in the Indian Constitution; Expanding Scope of Rights .... P. 189

 Violation of Citizens’ Rights by the USA:

 About

600 people were secretly picked up by the US forces from all over the world and put in a prison in Guantanamo Bay, an area near Cuba controlled by the American Navy.

 The American Government said that they were enemies of the US and linked to the attack on New York on 11th September 2001.



Families of prisoners, media or even UN representatives were not allowed to meet them. The US Army arrested them, interrogated them and decided to keep them there. There was no trial before any magistrate in the US.





Amnesty International, an International human rights organisation, collected information on the condition of the prisoners in Guantanamo Bay and reported that the prisoners were being tortured in ways that violated the US laws.





Prisoners were not released even after they were officially declared not guilty. An independent inquiry by the UN supported these findings. The UN Secretary General said the prison in Guantanamo Bay should be closed down. The US Government refused to accept these pleas.

 Violation of Citizens’ Rights in Saudi Arabia:



The country is ruled by a hereditary king and the people have no role in electing or changing their rulers.





The king selects the legislature as well as the executive. He appoints the judges and can change any of their decisions.



 Citizens





cannot form political parties or any political organisation. Media cannot report anything that the monarch does not like.

Scan to know more about this topic

Life Without Rights

Scan to know more about this topic

What are Rights?

There is no freedom of religion. Every citizen is required to be Muslim. Non- muslim residents can follow their religion in private, but not in public.

186

Oswaal CBSE Question Bank Chapterwise & Topicwise, SOCIAL SCIENCE, Class-IX

DEMOCRATIC RIGHTS





187

Women are subjected to many public restrictions. The testimony of one man is considered equal to that of two women.

 Violation of Citizens’ Rights in Yugoslavia (Kosovo):



Kosovo was a province of Yugoslavia before its split. In this province, the population was overwhelmingly ethnic Albanian Muslims. But in the entire country, Serbs (Christians) were in majority.





A narrow-minded Serb nationalist Milosevic had won the election. His government was very hostile to the Kosovo Albanians. He wanted the Serbs to dominate the country. Many Serb leaders thought that ethnic minorities like Albanians should either leave the country or accept the dominance of the Serbs.





74 year old Batisha Hoxha was sitting in her kitchen with her 77 year old husband Izet, staying warm by the stove. She knew five or six soldiers had burst through the front door and were demanding her children.





They shot Izet three times in the chest. When her husband was dying, the soldiers pulled the wedding ring off and even before she came out of the house, they burnt her house.





This was typical of what happened to thousands of Albanians in that period. This was one of the worst instances of killings based on ethnic prejudices in the recent times. Finally, Milosevic lost power and was tried by an International Court of Justice for crimes against humanity.

 What are Rights? Why Do We Need Rights in a Democracy?  Rights are reasonable claims of persons recognised by society and sanctioned by law.  Rights are necessary for the very sustenance of democracy.  In a democracy, every citizen has to have the right to vote and the right to be elected to government.  For democratic elections to take place, it is necessary that citizens should have the right to express their opinion, form political parties and take part in political activities.  Rights protect minorities from the oppression of the majority. They ensure that the majority cannot do whatever it likes. Rights are guarantees which can be used, when things go wrong.  The government should protect the citizens’ rights. But sometimes, elected governments may not protect or may even attack the rights of their own citizens.  That is why some rights are needed to be placed higher than the government so that the government cannot violate them. In most democracies, the basic rights of the citizen are written down in the Constitution.

Key Term  Amnesty International: Amnesty International is an international organization of volunteers who campaign for human rights. This organisation brings out independent reports on the violation of human rights all over the world.

OBJECTIVE TYPE QUESTIONS Multiple Choice Questions Q. 1. What was the reason given by America for R imprisoning people at Guantanamo Bay? (A) America considered them as inferior. (B) America considered them as superior and therefore a threat. (C) America considered them as enemies and linked them to the attack on New York on 11th September 2001. (D) America considered them as enemies and linked them to the attack on New York on 11th September, 2004. Ans. Option (C) is correct.

(1 mark each)

R Q. 2. What are rights?  (A) Rights are claims of a person over other fellow beings, over the society and over the government. (B) Rights given only to a particular group. (C) Rights given only to male section of the society. (D) The correct doings of a society.

Ans. Option (A) is correct. Q. 3. Rights are necessary for the very sustenance of U a_________. (A) democracy (B) political party (C) majority Ans. Option (A) is correct.

(D) minority

188

Oswaal CBSE Question Bank Chapterwise & Topicwise, SOCIAL SCIENCE, Class-IX

Explanation: Rights are necessary for the very sustenance of a democracy. In a democracy, every citizen has to have the right to vote and the right to be elected to government.

Assertion and Reason In the questions given below, there are two statements marked as Assertion (A) and Reason (R). Read the statements and choose the correct options:

(A) Both (A) and (R) are true and (R) is the correct explanation of (A).



(B) Both (A) and (R) are true but (R) is not the correct explanation of (A).



(C) (A) is correct but (R) is wrong.



(D) (A) is wrong but (R) is correct.

Q. 1. Assertion (A): The notion of rights changes from U time to time and society to society. 

Reason (R): What is recognised by the society as rightful becomes the basis of rights.

Ans. Option (A) is correct. Q. 2. Assertion (A): Rights ensure that the majority E cannot do whatever it likes. 

Reason (R): Rights do not protect minorities from the oppression of majority.

Ans. Option (C) is correct.

Explanation: Rights protect minorities from the oppression of the Majority. They ensure that the majority cannot do whatever it likes.

SUBJECTIVE TYPE QUESTIONS Short Answer Type Questions Q. 1. Explain in three points why do we need rights in a democracy. R Ans. Rights are essential in a democracy because: (i) Every citizen has the right to vote and the right to be elected to the government. (ii) For any democratic election to take place, it is necessary that the citizens should have the right to express their opinion, form political parties and take part in political activities. (iii) Rights protect the minority from the oppression of the majority. The right ensures that the majority cannot do whatever it likes. (1 × 3 = 3)

Q. 2. Some rights need to the placed higher than the government. Explain. E Ans. (i) The government should protect the citizens’ rights in every situation. (ii) But sometimes elected governments may not protect or may even attack the rights of their own citizens. (iii) That is why some rights need to be placed higher than the government, so that the government cannot violate them. (1 × 3 = 3)

Long Answer Type Questions Q. 1. What is the relationship between the citizens and the Government of Saudi Arabia as far as human rights are concerned? State in five points. R OR What is the position of the citizens in Saudi Arabia with respect to their government? R Ans. (i) Saudi Arabia is ruled by a hereditary king and the people have no role in electing or changing their rulers. (ii) The king selects the legislature as well as the executives. He appoints the judges and can change any of their decisions. (iii) Citizens cannot form political parties or any political organisations. Media cannot report anything that the monarch does not like. (iv) Every citizen is required to be Muslim. Nonmuslim residents can follow their religion in private, but not in public. (1¼×4=5)

(3 marks each)

(5 marks each)

Q. 2. “The claims of the citizens should be such that it can be made available to the others also.” Explain. Also, summarise the important characteristics of rights.  U (Board Term II, 2014) Ans. The above-mentioned sentence means that the claims of the citizens should be reasonable. They should be such that can be made available to others in equal manner. Thus, a right comes with an obligation to respect other’s rights. Characteristics of rights : (i) Rights are necessary for the very sustenance of democracy. (ii) Rights protect minorities from the oppression of the majority. (iii) Rights are guarantees which can be used when things go wrong. (CBSE Marking Scheme 2014) 5

DEMOCRATIC RIGHTS

Commonly Made Error

Students write the meaning of rights & claims.

Topic-2

189

Answering Tip

The claims of citizens for the rights should be reasonable.

Rights in the Indian Constitution; Expanding scope of Rights

Revision Notes Scan to know  Some rights which are fundamental to our life are given a special status. They are called more about Fundamental Rights. this topic  The Fundamental Rights guaranteed by the Indian Constitution are: (i) Right to Equality:  Right to Equality means that the laws apply in the same manner to all, regardless of a person’s status. This is called the Rule of Law. Rule of Law is the foundation of democracy. Fundamental  The government shall not discriminate against any citizen on the grounds of religion, caste, Rights ethnicity, sex or place of birth. guaranteed by the Indian  Every citizen shall have access to public places like shops, restaurants, hotels and cinema halls. Constitution Similarly, there shall be no restriction with regard to the use of wells, tanks, bathing ghats, roads, playgrounds and places of public resorts maintained by government or dedicated to the Scan to know more about use of general public. this topic  All citizens have equality of opportunity in matters relating to employment or appointment to any position in the government. No citizen shall be discriminated against or made ineligible for employment.  The Constitution mentions one extreme form of social discrimination, the practice of untouchability and clearly directs the government to put an end to it. The practice of Expanding Scope untouchability has been forbidden in any form. of Rights (ii) Right to Freedom: Under the Indian Constitution, all citizens have the right to (i) Freedom of speech and expression: (ii) Freedom to assemble in a peaceful manner. (iii) Freedom to form associations and unions. (iv) Freedom to move freely throughout the country. (v) Freedom to reside in any part of the country. (vi) Freedom to practice any profession, or to carry on any occupation, trade or business. (vii) Citizens have the freedom to hold meetings, processions, rallies and demonstrations on any issue. (viii) Your freedom should not cause public nuisance or disorder. You are free to do everything which injures no one else. (iii) Right Against Exploitation:  Right against Exploitation prevents the exploitation of weaker sections of society.  Our Constitution forbids human trafficking.

 The Constitution bans practices of bonded or forced labour.  The Constitution also prohibits child labour. Under this a child who is below 14 years, is restricted to work. (iv) Right to Freedom of Religion:  As India is a secular country, every citizen is free to profess, propagate and practice any religion he/she believes in.  This ensures that no one can force anyone to convert his/her religion, but a person is free to convert religion on his/her own will.  It does not mean that one can do whatever he/she wants in the name of religion.

190

Oswaal CBSE Question Bank Chapterwise & Topicwise, SOCIAL SCIENCE, Class-IX

 Also, the government cannot pressurise any person to pay any tax for promotion of any religion or religious institutions. (v) Cultural and Educational Rights:  This protects the right of every citizen to maintain their culture, language or script.  Admission to any educational institution maintained by government or receiving government aid cannot be refused to anyone on the ground of religion or language.  This ensures right of minorities to establish and administer educational institutions of their choice. (vi) Right to Constitutional Remedies:  The Fundamental Rights in the Constitution are important because they are enforceable. We have a right to seek the enforcement of the above mentioned rights. This is called the Right to Constitutional Remedies.  This Fundamental Right makes other rights effective.  Courts also enforce the Fundamental Rights against private individuals and bodies. The Supreme Court and High Courts have the power to issue directions, orders or writs for the enforcement of the Fundamental Rights.  A person can go to court against the violation of a Fundamental Right. If it is of social or public interest, it is called Public Interest Litigation (PIL). Under the PIL, any citizen or group of citizens can approach the Supreme Court or a High Court for the protection of public interest against a particular law or action of the government.  The flipped coexistence of rights versus duties refers to a philosophical concept that challenges the conventional understanding of the relationship between individual rights and societal duties. Traditionally, it has been widely accepted that individuals possess certain rights that are protected by law, while duties usually are seen as obligations individuals owe to society. However, the flipped coexistence theory proposes a reversal of this perspective, suggesting that duties should be given priority over rights.  Advocates of the flipped coexistence argue that by focusing on duties, individuals become more mindful of the impact of their actions on others and are motivated to actively contribute to the betterment of society. By reframing the relationship between rights and duties, the theory seeks to foster a more holistic approach to societal functioning that takes into account the well-being of the entire community.  However, critics of the flipped coexistence theory raise concerns about the potential for an overbearing state or society that prioritises duties at the expense of individual freedoms. Balancing the rights of individuals with their corresponding duties remains a complex challenge, requiring careful consideration and dialogue to ensure that neither aspect is disproportionately favoured or undermined.  In conclusion, the flipped coexistence theory challenges the traditional understanding of the relationship between rights and duties by proposing that duties should take precedence over rights. By emphasizing the importance of societal responsibilities, the theory seeks to promote a more balanced and harmonious society. While it offers potential benefits such as fostering a sense of civic responsibility, critics caution against potential infringements on individual freedoms. Achieving a proper balance between rights and duties is an ongoing challenge that requires thoughtful deliberation and consideration of societal well-being. Additional Rights Guaranteed by Indian Constitution  Over the years the scope of rights has expanded. From time to time, the courts gave judgments to expand the scope of rights.  Now, school education has become a right for Indian citizens. The governments are responsible for providing free and compulsory education to all children up to the age of 14 years.  Parliament has enacted a law giving the right to information to the citizens. We have a right to seek information from government offices.  Recently, the Supreme Court has expanded the meaning of the right to life to include the right to food.  The right to property and right to vote in elections are important Constitutional Rights. Constitution of South Africa guarantees its citizens several kinds of new rights:  Right to privacy, so that citizens or their homes cannot be searched, their phones cannot be tapped, their communication cannot be opened.  Right to an environment that is not harmful to their health or well being.  Right to have access to adequate housing.  Right to have access to health care services, sufficient food and water; no one must be refused of emergency medical treatment. Human right activists all over the world seek a set of rights as a standard of human rights. These include:  Right to work: opportunity to everyone to earn livelihood by working.

DEMOCRATIC RIGHTS

191

 Right to safe and healthy working conditions, fair wages that can provide decent standard of living for the workers and their families.  Right to adequate standard of living including adequate food, clothing and housing.  Right to social security and insurance.  Right to Health: Medical care during illness, special care for women during childbirth and prevention of epidemics.  Right to education: Free and compulsory primary education, equal access to higher education.

Key Terms  Writ: A formal document containing an order of the court to the government issued only by the High Court or the Supreme Court.  National Human Rights Commission: It is an independent organization established in 1993. Its main work is to focus on human rights and help the victims, whose rights are violated.

Example Explain the Right to Culture and Education for minorities.

Ans.



Step 1: The Constitution specifies the cultural and educational rights of the minorities. Any section of citizens with a distinct language or culture has a right to conserve it. Step 2: Admission to any educational institution maintained by government or receiving government



aid cannot be denied to any citizen on the ground of religion or language. Step 3: All minorities have the right to establish and administer educational institutions of their choice. Here minority does not mean only religious minority at the national level. Step 4: In some places, people speaking a particular language are in majority while, people speaking a different language are in a minority.

OBJECTIVE TYPE QUESTIONS Multiple Choice Questions

(1 mark each)

Q. 2. Study the given picture carefully:

Q. 1. Which of the following is not an instance of an U exercise of a fundamental right?  (A) Workers from Bihar go to the Punjab to work on the farms

(B) Christian missions set up a chain of missionary schools

(C) Men and women government employees get the same salary (D) Parents’ property is inherited by their children Ans. Option (D) is correct. Explanation: (i) Workers from Bihar go to the Punjab to work on the farms comes under Right to Freedom. (ii) Christian missions set up a chain of missionary schools that come under Right to Freedom of Religion. (iii) Men and women government employees get the same salary comes under Right to Work.

This picture is related to which of the following right? (A) Right to Freedom (B) Right to Equality (C) Right against Exploitation (D) Cultural and Educational Rights Ans. Option (A) is correct.

192

Oswaal CBSE Question Bank Chapterwise & Topicwise, SOCIAL SCIENCE, Class-IX

Q. 3. Match the following Column I with Column II: Column I

to own property comes with the duty to pay taxes. The right to vote comes with the duty to serve on a jury. These examples illustrate how rights and duties are interrelated and cannot be separated from one another.

Column II

(i)

Right to Freedom

(a)

Practices infringing women’s freedom.

(ii)

Right against Exploitation

(b)

Reside in any part of the country.

(iii)

Right to Constitutional Remedies

(c)

Prohibits Begar.

(iv)

Right to Freedom of Religion

(d)

‘the heart and soul’ of our Constitution.

(A) i-a, ii-b, iii-c, iv-d (B) i-b, ii-c, iii-d, iv-a (C) i-c, ii-a, ii-d, iv-b (D) None of these Q. 4. What is the flipped coexistence of rights versus duties? (A) A concept that promotes individual rights over individual duties (B) A concept that promotes individual duties over individual rights (C) A concept that promotes the equal coexistence of both rights and duties (D) A concept that rejects both rights and duties Ans. Option (C) is correct. Explanation: Rights and duties are interrelated and cannot be separated from one another. For every right that a citizen enjoys, there is a corresponding duty. The State protects and enforces the rights of the citizens, and it is the duty of all citizens to be loyal to the state. Therefore, citizens enjoy both rights and duties. Q. 5. What are some examples of rights that come with corresponding duties? (A) The right to free speech and the duty to respect the rights of others (B) The right to own property and the duty to pay taxes (C) The right to vote and the duty to serve on a jury (D) All of the above Ans. Option (D) is correct. Explanation: The right to free speech comes with the duty to respect the rights of others. The right

Assertion and Reason In the questions given below, there are two statements marked as Assertion (A) and Reason (R). Read the statements and choose the correct options.

(A) Both (A) and (R) are true and (R) is the correct explanation of (A).



(B) Both (A) and (R) are true but (R) is not the correct explanation of (A).



(C) (A) is correct but (R) is wrong.



(D) (A) is wrong but (R) is correct.

Q. 1. Assertion (A): The fundamental rights in the Constitution are important because they are E enforceable.

Reason (R): If rights are like guarantees, they are of no use if there is no one to honour them.

Ans. Option (A) is correct. Q. 2. Assertion (A): Sometimes the expansion takes U place in what is called human rights.

Reason (R): With the expansion of democracy all over the world, there is greater pressure on governments to accept these claims.

Ans. Option (B) is correct.

Explanation: The scope of rights has been expanding and new rights are evolving over time. They are result of struggle of the people.

Q. 3. Assertion (A): Constitution provides many more rights, which may not be Fundamental Rights. U

Reason (R): Rights are not limited only to Fundamental Rights as enumerated in the Constitution.

Ans. Option (A) is correct.

Explanation: For example, the right to property is not a Fundamental Right but it is a constitutional right.

SUBJECTIVE TYPE QUESTIONS Very Short Answer Type Questions Q. 1. Right to Freedom comes with some limitations. R Justify with giving two suitable examples. Ans. (i) You cannot use your Right to Freedom to incite people to rebel against government or to defame others.

(2 marks each)

(ii) We can hold meetings but peacefully. (iii) We cannot carry weapons while participating in a procession or a meeting. 

(Any Two Points) (1 x 2 = 2)

DEMOCRATIC RIGHTS

Q. 2. Mention any two significances of PIL. Ans. Importance of PIL:

E

(i) The courts intervene to prevent the misuse of the government powers to make decisions. (ii) They check malpractices on part of public officials. 

that will cause damage to his reputation.” Is the action of X protected under Right to Freedom of E Speech? If not, then state the reason. Ans. No, the action is not protected.

(1 x 2 = 2)

Q. 3. Mention any two rights of a detained person. R Ans. (i) To be informed of the reason of arrest and detention. (ii) To be produced before the nearest magistrate within 24 hours of arrest. (iii) Right to consult a lawyer or engage a lawyer for his defence.  (Any Two Points) (1 x 2 = 2) Q. 4. X says. “I will defame Y as I don’t like him. I will even say false and mean things about him

One of the exceptions to Right to Freedom of Speech and Expression, is that this right cannot be used to defame others or to cause damage to reputation.(2)

Q. 5. Why are rights guaranteed by the Indian U Constitution called Fundamental Rights? Ans. (i) These rights are essential for the overall development of the citizens.

(ii) These are enforceable in the courts of law. (iii) These have been given to all the citizens by the Indian Constitution and the government cannot abolish them.  (Any Two Points) (1 x 2 = 2)

Short Answer Type Questions Q. 1. “Secularism is based on the idea that the state is concerned only with relations among human beings”. Explain. U (Board Term II, 2017, 2016) Ans. (i) As a secular nation, every citizen of India has the right to freedom of religion, i.e., “Liberty of thought, belief, faith and worship”. (ii) We have the freedom to run religious affairs. (iii) Indian Constitution firmly states that all religions are equal before law.  (CBSE Marking Scheme, 2017) (1×3=3) Q. 2. “It is the language, culture and religion of minorities that needs special protection”. What provisions have been made by the Indian Constitution for the protection of these for the minorities? U (Board Term II, 2017) Ans. (i) Yes, otherwise, they may get neglected or undermined under the impact of the language, religion and culture of majority. (ii) The Constitution specifies that any section of citizens with a distinct language or culture have a right to conserve it. (iii) Admission to any education institution maintained by government cannot be denied to any citizen on the ground of religion or language. (iv) All minorities have the right to establish and administer educational institution of their choice. 

(Any three points)

 (CBSE Marking Scheme, 2017) (1×3=3) Q. 3. Explain how the scope of the right to information has expanded over time.

U

(Board Term II, 2015)

Ans. Parliament has enacted a law giving the right to information to the citizens.

193



(3 marks each) This Act was made under the Fundamental Right to freedom of thought and expression. Citizens have right to seek information from government offices. (CBSE Marking Scheme, 2015) (3)

Q. 4. Mention the restrictions that can be imposed on various forms of freedoms granted by the Indian Constitution. U Ans. The following restrictions can be imposed on various forms of freedoms granted by the Constitution : (i) Freedom of speech and expression–reasonable restrictions can be imposed on this freedom. (ii) Restriction on freedom to hold meetings, processions, rallies and demonstrations and reasonable restrictions can be put in the interest of public order. (iii) Restriction on freedom to form associations and unions can be imposed in the interest of public order, morality or sovereignty or integrity of India. (1×3=3) Q. 5. “The Right to Constitutional Remedies is the heart and soul of the Indian Constitution”. Justify the statement. R OR

What and why did Dr. B. R. Ambedkar refer to as the heart and soul of our Constitution? OR



Which right is the heart and soul of our Constitution? Explain why is it so called. Ans. The Right to Constitutional Remedy is the ‘heart and soul’ of the Indian Constitution in the following ways:

194

Oswaal CBSE Question Bank Chapterwise & Topicwise, SOCIAL SCIENCE, Class-IX

(i) When any of our rights are violated we can seek a remedy through courts. If it is a Fundamental Right, we can directly approach the Supreme Court or the High Court of a state. (ii) If any act of the legislature or the executive takes away or limits any of the Fundamental Rights it will be invalid. We can challenge such laws of the central and the state government in the court of law. (iii) The Supreme Court and the High Courts have the power to issue directions, orders or writs for the enforcement of the Fundamental Rights.  (1 × 3 = 3)

Q. 6. What is meant by the flipped coexistence of rights versus duties? Ans. The flipped coexistence of rights versus duties means that rights and duties are interrelated and cannot be separated from one another. For every right that a citizen enjoys, there is a corresponding duty. The State protects and enforces the rights of the citizens, and it is the duty of all citizens to be loyal to the state. Therefore, citizens enjoy both rights and duties. (3)

Long Answer Type Questions Q. 1. Describe the Exploitation.

features of Right against A (Board Term II, 2016, KVS) OR Describe the various provisions given under the Right against Exploitation. Ans. Once the Right to Liberty and Equality is granted, it follows that every citizen has a right not to be exploited. (i) The Indian Constitution prohibits trafficking in human beings. Trafficking here means buying and selling of human beings, usually women for immoral purposes. (ii) Our Constitution also prohibits forced labour or ‘begar’ in any form. Begar is a practice where the worker is forced to work without any remuneration. (iii) The Indian Constitution also prohibits child labour. No one can employ children below fourteen years to work in any factory or mine or in any other hazardous work place. (CBSE Marking Scheme, 2016) (2½+2½=5) Q. 2. Explain the right which protects all other rights. U (Board Term II, 2016) OR Explain what is meant by the ‘Right to Constitutional Remedies’? (Board Term II, 2015, KVS) Ans. (i) The right which protects all other rights is called the Right to Constitutional Remedies.

(ii) Dr. B. R. Ambedkar called it ‘the heart and soul’ of the Indian Constitution. (iii) If rights are like guarantees, they are of no use, if there is no one to honour them. (iv) Fundamental rights are important because they are enforceable. (v) If the rights are violated, we can seek protection or remedy through courts. (vi) In case of violation of fundamental rights, a person can go directly to the Supreme Court and High courts. (Any five) (1×5=5)

(5 marks each)

Q. 3. “Rule of law is the foundation of any democracy”. Explain with the help of examples. A (Board 2014, Term II) Ans. (i) Right to Equality means that the laws apply in the same manner to all, regardless of a person’s status. This is called the rule of law. It is the foundation of any democracy. (ii) It means that no person is above the law. There cannot be any distinction between a political leader, government official and an ordinary citizen. (iii) The government shall not discriminate against any citizen on grounds of religion, caste, ethnicity, sex or place of birth. (iv) Every citizen shall have access to public places like shops, restaurants, hotels and cinema halls. There should be no restrictions with regard to the use of wells, tanks, etc. (v) The principle applies to public jobs. All citizens have equality of opportunity in matters relating to employment or appointment to any position in the government. (1×5=5) Q. 4. Can a person change his religion? Support your answer with the concerned Fundamental Right. U (Board Term II, 2016) Ans. Yes, a person can change his religion. He/She is free to change religion on his or her own will. (i) Right to Freedom of Religion states that in India every person has a right to profess, practice and propagate the religion he or she believes in. (ii) The right to propagate ones religion, however it does not mean that a person has right to compel another person to convert into his religion by force, fraud, inducement or allurement. (iii) Every religious group is free to manage its religious affairs. (iv) The government cannot compel any person to pay any taxes for the propagation of any particular religion or religious institution.

DEMOCRATIC RIGHTS

(v) There shall be no religious instruction in the government educational institutions. In educational institutions managed by private bodies no person shall be compelled to take part in any religious instruction or to attend any religious worship. (1×5=5)

Commonly Made Error

The students were unable to mention the

appropriate features related to the Right of Religion.

Answering Tip

Right to Freedom of Religion gives Indians freedom to follow any religion they wish to.

Q. 5. Describe any five provisions of the Right to Freedom. A (Board Term II, 2012) OR “The Right to Freedom is a cluster of six rights.” Explain. OR Why is Right to Freedom called a cluster of Rights? Explain. OR “Right to Freedom is a cluster of different freedoms.” Elaborate. OR Right to Freedom incorporates several freedoms. List any six. Ans. Freedom means the absence of constraints. In practical life, it means — the absence of interference by other individuals or by the government in our internal affairs.

Under the Indian Constitution, all citizens have the following freedoms :

(i) Freedom of speech and expression. (ii) Assembly in a peaceful manner. (iii) Form associations and unions. (iv) Move freely throughout the country. (v) Reside in any part of the country. (vi) Practice any profession, or to carry on any occupation, trade or business. (Any five)

Besides the above-mentioned Constitution says that :



I. Read the source given below and answer the E (1+1+1+1=4) questions that follow: Citizens have the freedom to hold meetings, processions, rallies and demonstrations on any issue. They may want to discuss a problem, exchange ideas, mobilise public support to a cause, or seek votes for a candidate or party in an election. But such meetings have to be peaceful. They should not lead to public disorder or breach of peace in

ones,

the

(i) No person can be deprived of his life or personal liberty except according to procedure established by law.

Even when they do so, they have to follow some procedure.

(ii) A person who is arrested and detained in custody will have to be informed of the reasons for such arrest and detention. (iii) A person who is arrested and detained shall be produced before the nearest magistrate within a period of 24 hours of his arrest. Such a person has the right to consult a lawyer or engage a lawyer for his defence.(1×5=5) Q. 6. How do rights and duties coexist in a society Ans. Rights and duties coexist in a society in an interrelated manner. Every right that a citizen enjoys comes with a corresponding duty. The State protects and enforces the rights of the citizens, and it is the duty of all citizens to be loyal to the state. Duties are not legally enforceable, but they are moral in nature. Citizens owe duties both to the state and to individuals. For instance, there is a legal duty to pay taxes, to refrain from committing crime against fellow-citizens, and to abide by the laws that the Parliament has enacted. Therefore, it can be concluded that rights and duties coexist in a society as two sides of the same coin. (5)

COMPETENCY BASED QUESTIONS Case based MCQs

195

(4 marks each)

society. Those who participate in these activities and meetings should not carry weapons with them. Citizens also can form associations. For example, workers in a factory can form a workers’ union to promote their interests. Some people in a town may come together to form an association to campaign against corruption or pollution. As citizens we have the freedom to travel to any part of the country. We are free to reside and settle in any party of the territory of India. This right allows lakhs of people to migrate from villages to towns and from poorer regions of the countries to prosperous regions and

196

Oswaal CBSE Question Bank Chapterwise & Topicwise, SOCIAL SCIENCE, Class-IX

big cities. The same freedom extends to choice of occupations. No one can force you to do or not to do a certain job. Women cannot be told that some kinds of occupations are not for them. People from deprived castes cannot be kept to their traditional occupations. The Constitution says that no person can be deprived of his life or personal liberty except according to procedure established by law. It means that no person can be killed unless the court has ordered a death sentence. It also means that a government or police officer cannot arrest or detain any citizen unless he has proper legal justification. 1. Citizens have the freedom to: (A) discuss a problem (B) hold meetings (C) form associations (D) All of these Ans. Option (D) is correct. 2. What allows us to settle in any part of the country? (A) Right to Freedom (B) Right to Equality (C) Right against Exploitation (D) None of these Ans. Option (A) is correct. 3. The ............... says that no person can be deprived of his liberty. (A) Women (B) Constitution (C) Lawyer (D) Association Ans. Option (B) is correct. 4. Without ..............., a police officer cannot detain any person. (A) public support (B) weapons (C) legal justification (D) All of these Ans. Option (C) is correct.

Subjective based Questions I. Read the source given below and answer the U (1+1+2=4) questions that follow: Sometimes, it leads to expansion in the legal rights that the citizen can enjoy. From time-to-time, the courts gave judgments to expand the scope of rights. Certain rights like right to freedom of press, right to information and right to education, are derived from the Fundamental Rights. Now school education has become a right for Indian citizens. The governments are responsible for providing free and compulsory education to all children up to the age of 14 years. Parliament has enacted a law giving the right to information to the citizens. This Act was made under the Fundamental Right to freedom of thought and expression. We have a right to seek information from government offices. Recently, the Supreme Court has expanded the meaning of the right to life to include the right to food. Also, rights are not limited only to Fundamental Rights as enumerated in the Constitution. Constitution provides many more rights, which may not be Fundamental Rights. For example, the right to property is not a Fundamental Right but it is a constitutional right. Right to vote in elections is an important constitutional right. 1. Which right is included under right to life? Ans. Right to Food is included under right to life. 2. Name one right which is not a fundamental right but is a constitutional right. Ans. The right to property is not a Fundamental Right but it is a constitutional right. 3. Name any two rights which are derived from the Fundamental Rights. Ans. Certain rights like right to freedom of press, right to information and right to education, are derived from the Fundamental Rights .



UNIT-IV

Economics

CHAPTER

14 Learning Objectives

Study Time Max Time: 3:30 hr Max Questions: 81

PEOPLE AS RESOURCE l Examine the various factors that constitute the quality of population

Analyse the role of government in improving the quality of population. l Examine the factors that contribute to unemployment.

l

Topic-1 Quality of Population

TOPIC - 1 Quality of Population .... P. 197 TOPIC - 2 Unemployment .... P. 202

Revision Notes Introduction  Various activities have been classified into three main sectors: Primary, Secondary and Tertiary. l Primary sector includes agriculture, forestry, animal husbandry, fishing, poultry farming, mining, and quarrying. l Manufacturing is included in the secondary sector. l Trade, transport, communication, banking, education, health, tourism, services, insurance, etc., are included in the tertiary sector. The activities in this sector result in the production of goods and services.  Since these activities add value to the national income, they are called economic activities. l Economic activities have two parts – market activities and non- market activities. l Market activities involve remuneration to anyone who performs, i.e., activity performed for pay or profit. These include production of goods or services including government service. l Non-market activities are the production for self-consumption. These can be consumption and processing of primary products and own account production of fixed assets. l A division of labour exists between men and women in the family because of the historical and cultural reasons. l The household work done by women is not recognized in the national income. l Among the organized sector, teaching and medicine attract the women the most. Some women have entered administrative and other services including job that needs high levels of scientific and technological competence.

Quality of Population  The quality of population depends upon: l The literacy rate. l Health of a person is indicated by life expectancy. l Skill formation acquired by the people of the country. Education  Education is an important input for the growth of a person. l It opens new horizons for the person. l Provides new aspiration.

Scan to know more about this topic

Economic Activities

198

Oswaal CBSE Question Bank Chapterwise & Topicwise, SOCIAL SCIENCE, Class-IX

PEOPLE AS RESOURCE



l Develops values of life. l Contributes towards the growth of the society. l Enhances the national income and cultural richness. l Increases the efficiency of governance.

199

Scan to know more about this topic

The policies that can add to the literate population of India: l Schools like Navodaya Vidyalaya have been established in each district. l Vocational streams have been developed to equip large number of high school students

Quality of Population

with occupations related to knowledge and skills. l “Sarva Siksha Abhiyan” is a significant step towards providing elementary education to all children in the age group of six to fourteen years by 2010. l The bridge courses and back-to-school camps have been initiated to increase the enrolment in elementary education. l Mid-day meal scheme has been implemented to encourage attendance and retention of children and improve their nutritional status.  The eleventh plan endeavoured to increase the enrolment in higher education of the 18 to 23 years age group to 15 percent by 2011–2012 and to 21 percent by twelfth plan. Health  The health of a person helps him to realize his potential and provides the ability to fight illness.  Improvement in the health status of the population has been the priority of the country.  Our national policy aims at improving the accessibility of health care, family welfare and nutritional service, with a special focus on the under-privileged segment of the population.  Increase in longevity of life is an indicator of the good quality of life marked by self-confidence.  Reduction in infant mortality involves the protection of children from infection, ensuring nutrition along with mother and child care.

Key Terms  GNP (Gross National Product): It is the sum total of all the final goods and services produced by the normal residents of a country during an accounting year.  Human capital formation: When the existing human resource is further developed by becoming more educated and healthy, human capital formation takes place. It adds to the productive power of the country, just like physical capital formation.  Literacy rate: Percentage of people above a certain age, who can, along with understanding, both read and write short simple statements in everyday life.  Infant mortality rate: The number of death of infants under one year of age occurring among the live births per thousand of the births in a year.  Death rate: Death rate is the number of deaths per thousand persons in a year.  Birth Rate: It is the number of babies born for every 1,000 people during a particular period of time.  Life expectancy: The average period that a person may expect to live.

Example “Health should be treated as an important asset for human capital”. Analyse the statements with reference to our national health policy. Ans. Step 1: Improvement in the health status of the population has been the priority of the country. Step 2: The country has a well structured three tier public health infrastructure comprising Community Health Centres, Primary Health Centres and SubCentres spread across rural and semi-urban areas and tertiary medical care providing multi- speciality hospitals and medical colleges located almost exclusively in the urban areas.





Step 3: The National Health Policy 2002 aims at achieving an acceptable standard of health for the general population of the country. Step 4: To achieve the objective, a comprehensive approach was advocated, which included improvements in individual health care, public health, sanitation, clean drinking water, access to food and knowledge of hygiene and feeding practices. Step 5: Over the last five decades India has built up a vast health infrastructure and has developed manpower required at primary, secondary and tertiary sector in Government as well as in the private sector.

200

Oswaal CBSE Question Bank Chapterwise & Topicwise, SOCIAL SCIENCE, Class-IX

OBJECTIVE TYPE QUESTIONS Multiple Choice Questions Q. 1. ‘Sarva Shiksha Abhiyan’ is a significant step towards providing elementary education to all children in the age group of ____ (A) 6-12 (B) 6-10 (C) 8-14 (D) 6-14 Ans. Option (D) is correct. Q. 2. What kind of people are a liability for the economy? (A) Poor (B) Healthy (C) Elite (D) Illiterate Ans. Option (D) is correct. Q. 3. What is the purpose of implementation of mid-day meal? (A) To encourage attendance and retention of children (B) To tell the importance of food (C) To add to the task of teachers (D) To teach about food Ans. Option (A) is correct.

Assertion and Reason In the question given below, there are two statements marked as Assertion (A) and Reason (R). Read the statements and choose the correct option.

(1 mark each)



(A) Both (A) and (R) are true and (R) is the correct explanation of (A).



(B) Both (A) and (R) are true and (R) is not the correct explanation of (A).



(C) (A) is correct but (R) is wrong.



(D) (A) is wrong but (R) is correct.

Q. 1. Assertion (A): Educated society has direct and indirect advantages.

Reason (R): Educated and healthier people gain higher incomes.

Ans. Option (A) is correct. Q. 2. Assertion (A): Education helps individual to make better use of the economic opportunities available before him.

Reason (R): Education and skill are the major determinants of the earning of any individual in the market.

Ans. Option (B) is correct. Q. 3. Assertion (A): Education adds to the quality of labour.

Reason (R): Education enhance the total productivity and total productivity adds to the growth of the economy.

Ans. Option (A) is correct.

SUBJECTIVE TYPE QUESTIONS Short Answer Type Questions Q. 1. What are the advantages of having a healthy population?  R (KVS 2019) Ans. Advantages of having a healthy population: (i) Health is an important factor of population as it affects the process of development. (ii) Development depends on the efficiency of manpower and efficiency depends on good health and good environment. (iii) If the health of the people is properly looked after, people can produce more and thus the productivity of the country will get increased. (iv) A well-educated healthy population provides potential power as economic development depends on the quality, capacity and ability of the people. (v) Sick people cannot work efficiently. This in turn reduces the output of the country.  (Any three ) (1 × 3 = 3)

(3 marks each)

Q. 2. Mention the three main sectors of economic activities with examples. R [DDE-2014] OR Name the three sectors of economic activities and give one example of each.  R OR What are the various activities undertaken in the primary, secondary and tertiary sectors ?   R [NCERT] Ans. The three sectors of economic activities are: (i) Primary sector: It produces goods by exploiting natural resources. It includes agriculture, forestry, mining, animal husbandry and fishing. (ii) Secondary sector: It converts all raw material into finished goods such as manufacturing industries and construction activities.

PEOPLE AS RESOURCE

(iii) Tertiary sector: It includes trade, transportation, communication, education, health, tourism, insurance, etc. It helps in the smooth functioning of primary and secondary sectors.  (1 × 3 = 3)

Commonly Made Error

Some Students are not aware about the Midday Meal Scheme.

Answering Tip

Students should learn that Mid-day Meal was

implemented to encourage attendance and provide food to poor.

Q. 3. Describe the steps taken by the government in the field of education. R [Board Term I, 2014] OR Highlight the steps taken by the government to improve the literacy level in India.

R

Ans. Steps: (i) Navodaya Vidyalayas have been set up. (ii) Vocational streams have been developed. (iii) Sarva Shiksha Abhiyan has been launched. (iv) Increase in enrolment at elementary level. (v) Mid-day meal to encourage attendance. (Any three) (1×3=3) [CBSE Marking Scheme, 2014]

Q. 4. Why are women paid lesser as compared to men? Name the fields of organized sector that attract the women most. U OR Why are women employed in low paid work? State any three reasons. [NCERT] U Ans. Women are paid less as compared to men because: (i) A majority of women have meagre education and low skill formation. (ii) Most women work where job security is not present. Employment in this sector is characterized by irregular and low income. (iii) The fields of the organized sector that attract them most are teaching and medical. (2+1=3) Q. 5. What is mid-day meal scheme? Explain any two of its purposes. R Ans. Mid-day meal scheme is the scheme started by the government to provide some nutritional food to students during the day.



(ii) This policy could add to the literate population of India.(1+2=3)

government of India, many children remain out of school education particularly in rural areas. (ii) The girls are made to do domestic chores because spending on them is considered a burden by the rural poor. (iii) Though enrolment of boys in formal education is higher than that of girls, they drop out at secondary and tertiary level of education. (iv) The enrolment of girls in rural areas is much lower than that of their urban counterparts. (v) Almost 20% of children between the age group of 6–14 years all over India still do not go to school.

[CBSE Marking Scheme, 2017] (1 × 5 = 5)

(5 marks each)

Q. 2. Mention any five steps taken in the 10th Five Year Plan to increase enrolment in higher education.

Ans. (i) In spite of provisions of free and compulsory education for children below 14 years by the

Purposes:

(i) Mid-day meal scheme has been implemented to encourage attendance and retention of children and improve their nutritional status.

Long Answer Type Questions Q. 1. Describe the differences between male and female child enrolment. U [Board Term II, 2017]

201



R [Board Term I, 2013] OR State the aim of 10th Five Year Plan with regard to higher education. What strategy is being followed to attain it ?

Ans. Main features of the 10th Five Year Plan: (i) It endeavoured to increase the enrollment of students in the age group of 18 - 23 years in higher education from 6% to 9% by the end of the plan period. (ii) It emphasized on the strategy which focuses on increasing access, quality, and adoption of states specific curriculum modification and vocational and networking on the use of IT. (iii) The strategy also focuses on distant, formal, nonformal and IT Education.

202

Oswaal CBSE Question Bank Chapterwise & Topicwise, SOCIAL SCIENCE, Class-IX

(iv) There has been a significant increase in the number of such universities and institutions, over the last 5 years, which are specialising in different areas. (v) It also focuses on vocational, networking and the use of information technology in distant education. (1×5=5)

Achievements: (i) Life expectancy has increased and infant mortality rate has come down. (ii) Death rate has reduced and birth rate has come down. (3 + 2 = 5)

Commonly Made Error

[CBSE Marking Scheme, 2013]

Q. 3. Highlight the main elements of National Health Policy. Describe any two achievements in this field. U Ans. Elements of National Health Policy: (i) It aimed at improving the accessibility of health care. (ii) It aimed at family welfare and nutritional services with special focus on underprivileged. (iii) Increase in life expectancy and improvement in child care.



Most students write the features of National Health Policy instead of elements.

Answering Tip

Students should know the difference between

elements and features while writing the answer.

Topic-2 Unemployment Revision Notes  Unemployment is said to exist when people who are willing to work at the prevailing wages cannot find jobs.  In case of India, we have unemployment in rural and urban areas. However, the nature of unemployment differs in rural and urban areas. In case of rural areas, there is seasonal and disguised unemployment. Urban areas mostly have educated unemployment.  Seasonal unemployment takes place when people are not able to find jobs during some months of the year. People, dependent upon agriculture, usually face such kind of problem. Scan to know  In case of disguised unemployment, people appear to be employed. They have an agricultural more about this topic plot where they find work. This usually happens among family members engaged in agricultural activity. The work requires the service of five people, but engages eight people. Three people are extra. These three extra people are disguised unemployed.  Unemployment leads to wastage of manpower resource. People who are an asset for the Unemployment economy turn into a liability.  Unemployment has a detrimental impact on the overall growth of an economy.  Unemployment tends to increase economic overload. The dependence of the unemployed on the working population increases. The quality of life of an individual as well as of society is adversely affected.  Increase in unemployment is an indicator of a depressed economy.  In case of India, statistically, the unemployment rate is low. A large number of people represented with low income and productivity are counted as employed.  The employment structure is characterized by self-employment in the primary sector. Agriculture is the most labour absorbing sector of the economy, though the rate has been declining in recent years because of disguised unemployment.  Some of the surplus labour in agriculture has moved to either the secondary or the tertiary sector.  Encourage entrepreneurship: Encouraging people to start their own businesses can help create new jobs. Governments can offer incentives and tax breaks to entrepreneurs and provide access to funding and resources to help them get started.  Invest in education and training: Governments can invest in education and training programs to help workers develop new skills and adapt to changing industries. This can make them more employable and help them find jobs in growing industries.

PEOPLE AS RESOURCE

203

 Create public works programs: Governments can create public works programs to help create jobs in areas such as infrastructure development, construction, and environmental projects.  Increase public sector employment: Governments can increase public sector employment to provide job opportunities for people who are struggling to find work in the private sector.  Provide unemployment benefits and support: Governments can provide unemployment benefits and support to help people who are out of work to meet their basic needs and find new job opportunities.  Promote international trade: International trade can create new job opportunities and stimulate economic growth. Governments can negotiate trade agreements and provide support for businesses that want to export their products and services.  Implement a living wage: Implementing a living wage can help lift people out of poverty and reduce the number of people who are unemployed or underemployed.  Address discrimination and inequality: Discrimination and inequality can prevent some people from accessing job opportunities. Governments can implement policies to address these issues and ensure that everyone has an equal chance to succeed.

Key Terms  Educated Unemployment: This type of unemployment occurs in urban areas when the educated and qualified youth is not able to get employment.

OBJECTIVE TYPE QUESTIONS Multiple Choice Questions Q. 1. Unemployment leads to wastage of what type of U resources?  (A) Natural resources (B) Human resources (C) Artificial resources (D) Renewable resources Ans. Option (B) is correct. Explanation: Unemployment leads to wastage of manpower resource as the educated and skilled youth is not having any employment opportunities. Q. 2. What do you call shifting of labourers from a rural R area to urban area in search of work?  (A) Migration (B) Immigration (C) Transportation (D) Hibernation Ans. Option (A) is correct. Explanation: In rural areas, employment opportunities are less as compared to urban areas due to limited types of industries. Hence, many people shift from villages to cities in search of work. Q. 3. What is the main adverse effect of unemployment? E  (A) Increase in economic overload (B) Hue and cry in the country (C) People stop voting (D) People start moving abroad Ans. Option (A) is correct. Explanation: Unemployment turns people into liability who can be an asset for the economy. Q. 4. How can the education system be improved to better prepare students for the job market?

(1 mark each)

(A) By focusing more on vocational training (B) By increasing funding for STEM programs (C) By promoting apprenticeships and internships (D) All of the above Ans. Option (D) is correct. Q. 5. How can businesses be incentivised to hire more workers and reduce unemployment? (A) By reducing taxes on businesses (B) By providing subsidies for job creation (C) By implementing a minimum wage (D) Both a and b Ans. Option (D) is correct. Explanation: Reducing taxes on business helps to provide the better salary & bonus and providing subsidies for job creation helps to generate more employment.

Assertion and Reason Directions: In the following questions, a statement of Assertion (A) is followed by a statement of Reason (R). Mark the correct choice as:

(A)  Both Assertion (A) and Reason (R) are true, and Reason (R) is the correct explanation of Assertion (A).



(B)  Both Assertion (A) and Reason (R) are true, but Reason (R) is not the correct explanation of Assertion (A).



(C) Assertion (A) is true, but Reason (R) is false.



(D) Assertion (A) is false, but Reason (R) is true.

Q. 1. Assertion (A): People who are an asset for the E economy turn into a liability.

204

Oswaal CBSE Question Bank Chapterwise & Topicwise, SOCIAL SCIENCE, Class-IX



Reason (R): Unemployment has detrimental impact on the overall growth of an economy. Ans. Option (A) is correct. Explanation: The positive aspect of large population is huge human resource because it provides easy availability of cheap labour which averse economic growth of country. Q. 2. Assertion (A): Increase in unemployment is an U indicator of a depressed economy. 

Reason (R): If people cannot be used as a resource they naturally appear as a liability to the economy.

Ans. Option (A) is correct.

Explanation: People become liability to the economy because their skills cannot be used as the asset for the economy.

Q. 3. Assertion (A): Poor people cannot afford to sit U idle. 

Reason (R): They tend to engage in any activity irrespective of its earning potential. Ans. Option (B) is correct. Explanation: Their earning keeps them on a bare subsistence level.

SUBJECTIVE TYPE QUESTIONS Very Short Answer Type Questions

(2 marks each)

Q. 1. What is disguised unemployment? Explain with R the help of an example. Ans. It is a situation in which more workers are working in an activity than required. The people, who are actually engaged in such an activity appear to be employed, but are not fully employed. For example, if for the cultivation of one hectare land, 10 workers are required, but instead of 10 workers, 15 workers are working. In this case, 5 workers are disguised unemployed. In such cases, even if the surplus workers are removed the production does not suffer. (2)

Q. 4. Can you suggest some measures in the Education system to mitigate the problem of the educated unemployed? U [NCERT]

Q. 2. Suggest any two methods to remove rural unemployment problem. R

(iii) Education should be job-oriented.

Ans. (i) By promoting small-scale and cottage industry. (ii) By spreading technical education (iii) By promoting supplement works like animal rearing, horticulture, etc.  (Any Two Points) (1 x 2 = 2) Q. 3. Explain any two consequences of unemployment.

R

Ans. (i) Poverty: The basic cause of poverty is unemployment. People do not have enough money to support their family. Inability of educated people, who are willing to work to find gainful employment implies a great social waste. (ii) Increase in Dependent Population: Unemployment tends to increase the economic overload. The dependence of the unemployed on the working population increases. (iii) Poor Quality of Life: The quality of life of an individual as well as of the society is adversely affected. When a family has to live on a bare subsistence level, there is a general decline in its health status and rising withdrawal from the school system.  (Any Two Points) (1 x 2 = 2)

Ans. Some of the measures in the education system to mitigate the problem of the educated unemployed are:

(i) Vocational education should be encouraged so that people do not have difficulty in getting jobs because they will be better trained for work. (ii) More use of information technology should be made in giving education. (iv) More employment opportunities should be made available to the educated people in the tertiary sector.  (Any Two Points) (1 x 2 = 2) Q. 5. In which field do you think India can build the maximum employment opportunity?

U [NCERT] Ans. Agriculture is the most labour-absorbing sector of the economy. In recent years, there has been a decline in the dependence of population on agriculture partly because of disguised unemployment. Some of the surplus labour in agriculture has moved to either the secondary or the tertiary sector. In the secondary sector, small-scale manufacturing is the most labour - absorbing. In case of the tertiary sector, various new services are now appearing, like biotechnology, information technology and so on. In recent years, maximum employment opportunities have risen in the BPO or call centre. These have been a boon for moderately educated young people. (2)

PEOPLE AS RESOURCE

Short Answer Type Questions Q. 1. ‘Employment structure is characterized by selfemployment in the primary sector.’ Explain the statement. U [Board Term I, 2015)] Ans. Employment structure is characterized by selfemployment in the primary sector. (i) The whole family contributes in a field even though not everybody is really needed. So, there is disguised unemployment in the agricultural sector. (ii) But, the entire family shares what has been produced. However, this does not reduce the poverty of the family. When the surplus labour does not get sufficient employment in the agriculture they migrate to cities in search of jobs. [CBSE marking Scheme, 2015] (1½×2=3) Q. 2. What does unemployment mean? What types of unemployment exist in rural and urban areas ? U

[Board Term I, 2015]

Ans. Unemployment is said to exist when people who are willing to work at the given wages cannot find jobs. In Rural Areas: (i) Seasonal unemployment—When people don’t get jobs during some months of a year. For example, during harvesting time. (ii) Disguised unemployment—People appear to be employed. For example, a job requiring service of five people, but engaging eight people. In Urban Areas: Urban areas have mostly educated and unemployed youth. It has become a common phenomenon. Many youths with matriculation, graduation and post-graduation degrees are not able to find job. Unemployment of graduate and postgraduate has increased. (1+1+1=3) [CBSE Marking Scheme, 2012] Q. 3. Write the difference between disguised unemployment and seasonal unemployment. U [NCERT] [NCT Term I, 2014] Ans. S. No.

Disguised Unemployment

Seasonal Unemployment

(i)

A situation wherein marginal productivity of labour is zero. It is not visible and is found among wage earners, mostly in agriculture.

Getting employment only for the part of the year in certain sectors, like agriculture is called seasonal unemployment.

205

(3 marks each)

(ii)

It is mostly found in It is mostly found agriculture. in agrobased industries. (iii) It is mostly found in rural It is found both in areas. rural as well as urban areas. (1 × 3 = 3) Q. 4. Explain the pattern of seasonal unemployment. R Ans. Seasonal unemployment occurs in a certain season of the year. It is a widespread phenomenon, for example, in agricultural practice. Agricultural practice is based on the natural condition. This means that at a certain period of time, there is heavy work while in the rest period, the work is lean. During sowing of seeds and harvesting of crops, the agriculturist engage themselves the whole day and night. But after harvesting and sowing of seeds, the workers become workless. (3) Q. 5. Which is the most labour-absorbing sector of the Indian economy? What trend has been recently noticed in terms of dependence of population on this sector and what is the reason for that? U Ans. Agriculture is the most labour-absorbing sector of the Indian economy. (i) Disguised unemployment. (ii) Surplus labour in agriculture has moved to either the secondary or tertiary sector. (1 + 2 = 3) Q. 6. What are the most effective ways to stimulate economic growth and create more jobs? Ans. Some effective ways to stimulate economic growth and create more jobs include investing in education and training programs, promoting international trade, encouraging entrepreneurship, creating public works programs, increasing public sector employment, implementing a living wage, and addressing discrimination and inequality. (3) Q. 7. What are some specific examples of incentives and tax breaks that governments can offer to entrepreneurs? Ans. Governments can offer incentives and tax breaks to entrepreneurs such as tax credits for research and development, reduced tax rates for small businesses, grants and loans, exemptions from certain taxes, and accelerated depreciation. (3)

206

Oswaal CBSE Question Bank Chapterwise & Topicwise, SOCIAL SCIENCE, Class-IX

Long Answer Type Questions Q. 1. During 1980s, 80% of Indian population was dependent on agriculture sector but gradually it decreased. Analyse the reasons behind the newly developed trend. U [Board Term I, 2016] Ans. (i) In recent years, there has been a decline in the dependence of population on agriculture partly because of the disadvantages caused by disguised unemployment prevalent there. (ii) Some of the surplus labour in agriculture has moved to either the secondary or the tertiary sector. (iii) In the secondary sector, small-scale manufacturing is the most labour-absorbing. (iv) In case of the tertiary sector, various new services are now appearing, like biotechnology, information technology and so on. (v) With the increase in the number of jobs, dependence on agriculture has reduced to a great extent. (1 × 5 = 5) Q. 2. Which age group is known as work force population? Explain the existing employment structure in the agricultural sector in India. Also explain major problems. 

U [Board Term I, 2015]

Ans. People from 15 to 59 years are included in the work force population. Existing employment structure in the agricultural sector: (i) Employment structure is characterized by selfemployment in the agricultural sector. (ii) Whole family contributes in the field, even though everybody is really not needed. Major problems: (i) Nature of work is just for few hours in the morning as well as in the evening and in the entire day they do not have any job to do. (ii) The disguised unemployment is found in the agricultural sector which is more harmful because people themselves are unaware of their unemployment. (1+2+2=5) [CBSE marking Scheme, 2015] Q. 3. Describe the detrimental effect of increasing unemployment on economic growth. U [Board Term I, 2015] [Board Term I, 2013] OR Explain any five effects of unemployment in India. R [DDE Term 1, 2014] OR Evaluate the impact of unemployment on the overall growth of an economy. A OR “Unemployment leads to a depressed economy.” Justify the statement with five arguments. A OR State any five effects of unemployment. R

(5 marks each) OR “Unemployment leads to wastage of manpower resource.“ Justify the statement with suitable examples. A



Ans. (i) Because of unemployment, people who are an asset turn into a liability. (ii) There is a feeling of hopelessness and despair among youth. (iii) People don’t have money to support their family. (iv) Quality of life of an individual as well as society is adversely affected. (v) When the family has to live in bare subsistence level, there is decline in health. (vi) Wastage of natural resources is another factor which emerges due to unemployment. (Any five) (1×5=5) [CBSE Marking Scheme, 2015]

Commonly Made Error

Few students are not able to explain the effects of unemployment.

Answering Tip

Students should learn the key words while remembering the unemployment.

detrimental

effects

of

Q. 4. How can unemployment benefits and support help individuals who are struggling to find work? Ans. Unemployment benefits and support can help individuals who are struggling to find work in several ways. Firstly, they can provide financial assistance to help cover basic needs such as food, housing, and healthcare while the individual is searching for a new job. This can help alleviate some of the stress and anxiety that comes with being unemployed. Additionally, unemployment benefits can provide a safety net for those who may have lost their job due to circumstances beyond their control, such as a company downsizing or closing. Furthermore, many governments offer job training programs and career counselling services to help individuals develop new skills and find new job opportunities. These programs can be especially helpful for those who may need to transition into a new industry or occupation. Overall, unemployment benefits and support can be an important lifeline for individuals who are struggling to find work and need assistance in meeting their basic needs while they search for new employment opportunities. (5)

PEOPLE AS RESOURCE

Q. 5. How can the government provide more job opportunities for the unemployed? Ans. The government can provide more job opportunities for the unemployed in several ways. One strategy is to increase public sector employment by creating new positions in areas such as healthcare, education, and public works programs. Another approach is to promote international trade by negotiating trade agreements and providing support for businesses that want to export their products and services. This can create new job opportunities and stimulate economic growth. Governments can also invest in education and training programs to help workers



develop new skills and adapt to changing industries. Encouraging entrepreneurship is another way that governments can create new jobs. They can offer incentives and tax breaks to entrepreneurs, as well as provide access to funding and resources to help them get started. Finally, governments can address discrimination and inequality by implementing policies that ensure everyone has an equal chance to succeed in the workforce. By taking these steps, governments can help provide more job opportunities for the unemployed and support their citizens in finding meaningful employment. (5)

COMPETENCY BASED QUESTIONS Case based MCQs



207

(4 marks each)

1. Investment in human resource can give:

(A) high rates of return in future

I. Read the text given below and answer the following questions: (1+1+1+1=4) A

(B) low rates of return in future



There were two friends Ram and Raj living in the village. Raj’s mother looked after domestic chores. His father worked in an agricultural field. He helped his mother in domestic chores. He also looked after his younger brother and sister. His uncle had passed the matriculation examination, but, was sitting idle in the house as he had no job. Raj’s mother and father were eager to teach Raj. They forced him to join the village school which he soon joined. He started studying and completed his higher secondary examination. His father persuaded him to continue his studies. He raised a loan for him to study a vocational course in computers. Raj was interested in studies from the beginning. With great enthusiasm he completed his course. After some time, he got a job in a private firm. He even designed a new kind of software. This software helped him increase the sale of the firm. His boss acknowledged his services and rewarded him with a promotion.

(D) None of the above

Ram was residing in the same village as Raj. Ram’s father was a fisherman. His father was no more and his mother sold fish to earn money to feed the family. She bought fish from the landowner’s pond and sold it in the nearby mandi. She could earn only ` 150 a day by selling fish. Ram became a patient of cancer. His mother could not afford to take him to the doctor. He could not go to school either. He was not interested in studies. He helped his mother in cooking and also looked after his younger brother. After some time, his mother fell sick and there was no one to look after her. There was no one in the family to support them.

(A) Health

(B) Education

(C) Money

(D) None of the above



(C) no rates of return in future Ans. Option (A) is correct.

Explanation: Investment in human capital (through education, training, medical care) yields a return just like investment in physical capital. Higher incomes is earned because of higher productivity of the more educated or the better trained persons.



2. Ram’s mother was engaged in which of the following activity:

(A) Secondary

(B) Primary

(C) Tertiary

(D) All of the above

Ans. Option (B) is correct.

Explanation: Ram’s mother sold fish which comes under primary activity.

3. Raj’s education in the initial years of his life bore him the fruits in the later years in terms of a good job and salary. ______________ was an important input for the growth of Raj.

Ans. Option (B) is correct.

4. Raj’s mother looked after domestic chores. She was involved in which of the following activity?

(A) Economic activity (B) Non-economic activity (C) Market activity (D) None of the above Ans. Option (B) is correct.

Explanation: Activities which are not meant for monetary gains and their purpose is just to serve are known as Non economic activities.

208

Oswaal CBSE Question Bank Chapterwise & Topicwise, SOCIAL SCIENCE, Class-IX

GDP has declined to 2.7% in 2017–18 (B.E.). The

Subjective based Questions

literacy rates have increased from 18% in 1951 to 85% in 2017. Literacy is not only a right, it is also

I. Read the text given below and answer the following questions:

needed if the citizens are to perform their duties

(1+1+2=4) A

and enjoy their rights properly. However, a vast

In case of India, we have unemployment in

difference is noticed across different sections of the

rural and urban areas. However, the nature of

population. Literacy among males is nearly 14.4%

unemployment differs in rural and urban areas. In

higher than females and it is about 14.2% higher in

case of rural areas, there is seasonal and disguised

urban areas as compared to rural areas. In 2017–18,

unemployment. Urban areas have mostly educated

literacy rates varied from 96.2% in Kerala to 70.9%

unemployment. Many youths with matriculation,

in Bihar. The primary school system has expanded

graduation and post-graduation degrees are not able

to over 8.41 lakh in 2015–16. Unfortunately, this

to find jobs. A study showed that unemployment

huge expansion of schools has been diluted by the

of graduate and post-graduate has increased

poor quality of schooling and high dropout rates.

faster than among matriculates. Unemployment

“Sarva Shiksha Abhiyan is a significant step towards

has detrimental impact on the overall growth

providing elementary education to all children in

of an economy. Increase in unemployment is an

the age group of 6–14 years by 2010. It is a time-

indicator of a depressed economy. In case of India,

bound initiative of the Central government, in

statistically, the unemployment rate is low. 1. Urban

areas

have

mostly

which

type

partnership with the States, the local government

of

and the community for achieving the goal of

unemployment?

universalization of elementary education.” Along

Ans. Urban areas mostly have educated unemployment

with it, bridge courses and back to-school camps

2. What is the rate of unemployment in India?

have been initiated to increase the enrolment in

Ans. The rate of unemployment in India is low.

elementary education. Mid-day meal scheme has

3. Which is the most labour absorbing sector of the

been implemented to encourage attendance and

economy?

retention of children and improve their nutritional

Ans. Agriculture, as a primary sector, is the most

status. These policies could add to the literate

labour absorbing sector of the economy because

population of India.

secondary and tertiary sector has less employment

1. What makes people an asset for the economy?

opportunities in country

Ans. The quality of population depends upon the literacy

II. Read the text given below and answer the following questions:

rate, health of a person indicated by life expectancy

E (1+1+2=4)

and skill formation acquired by the people of the

There is also an establishment of pace setting of

country. The quality of the population ultimately

schools like Navodaya Vidyalaya in each district.

decides the growth rate of the country. Literate and

Vocational streams have been developed to

healthy population are an asset.

equip large number of high school students with



occupations related to knowledge and skills. The plan outlay on education has increased from Rs. 151 crore in the first plan to Rs. 99,300 crore in 2020– 21. The expenditure on education as a percentage of GDP rose from 0.64% in 1951–52 to 3.1% in 2019–20 (B.E.) and has remained stagnant around 3% from past few years. The Budgetary Estimate as stated in the Budget Documents of Union State Governments, Reserve Bank of India,

2. What is the investment of the government on education in the year 2020-21?

Ans. Rs 99,300 cr

3. By what percentage literacy rate has increased between 1951 to 2017?

Ans. The literacy rate in 2017 was 85% and in 1951 it was 18%. The difference between the both is 67%. Hence, it increased by 67% between 1951 and 2017.

the expenditure on education as a percentage of

Artificial Intelligence

PEOPLE AS RESOURCE

209

Art Integration: Data Acquisition & Data Exploration integrated with Human Capital Formation. Learning Objective:  To understand the trends in Education and Health in India.  To critically analyse the policies taken in the field of health and education in India.  To classify the economic activities into various sectors.  To define unemployment and its various types in rural and urban areas.  To recognise the effects of unemployment. Material Required: Pen, paper, blackboard, chalk, internet websites for data acquisition & laptops. PARAMETERS

DESCRIPTION

Chapter Covered

Chapter 2: People as Resources

Name of the Book

Economics, Class 9, NCERT

AI CONCEPTS INTEGRATED

Subject and Artificial AI Tools: Data Acquisition & Data Exploration integrated with Intelligence Human Capital Formation Integrated Learning Objectives

To understand the trends in Education and Health in India. To critically analyse the policies taken in the field of Health and Education in India To classify the economic activities into various sectors

Data Acquisition and Data Exploration https:// datavizcatalogue.com

To define unemployment and its various types in rural and urban areas To recognise the effects of unemployment Time Required

2 periods of 40 minutes each

Classroom Arrangement

Flexible

Material Required

Pen, paper, blackboard, chalk, internet websites for data acquisition & laptops

Pre – Preparation Activities

The students are divided into four groups and collect data related to education levels,Health, effectiveness of policies in the field of Education and Health

Previous Knowledge

The students should be aware of the importance of Human capital or entrepreneur as a factor of production

Methodology

The students are asked to discuss the ways in which people can be Data Acquisition and converted to Human capital. Data Exploration The students are asked to collect thedata and interpret the education and health scenario in India The students are asked to present the effectiveness of the education and health policies through the data collected. The students will be explained the various sectors of the economy and will classify the images shown The student will be given case study and define unemployment and its various types in rural and urban areas

210

Oswaal CBSE Question Bank Chapterwise & Topicwise, SOCIAL SCIENCE, Class-IX

Learning Outcomes

The students will understand theconcept of Human Capital Formation The students will understand the education and health trends in India and its growing importance The students will understand how policies in the education and health sector have been effective. The students will be able to classify various economic activities into sectors The students will be able to define unemployment and its various types in rural and urban areas The students will be able to recognise the effects of unemployment The students will be able to learn Data acquisition and exploration.

Follow up Activities

Students are asked to make presentationsto the education and health sector performance and policy effectiveness

Reflections

Students will be asked to analyze thevarious challenges ahead in Human capital Formation





Study Time Max Time: 3:30 hr Max Questions: 81

CHAPTER

15

POVERTY AS A CHALLENGE Comprehend that poverty is a multifaceted concept inherent in the rural and urban conditions. l Examine the measures taken by the government to eradicate poverty. l

Learning Objectives

Poverty–Introduction,

Topic-1 Indicators and Estimates Revision Notes l Poverty can be defined as the lack of common things like food, clothing,

TOPIC - 1 Poverty–Introduction, Indicators and Estimates .... P. 211 TOPIC - 2 Inter-State Disparities and Global Poverty Scenario; Causes of Poverty, Anti-Poverty Measures and the Challenges Ahead..... P. 216

shelter, safe drinking water, medical care and education, which determine the quality of life. Poverty exists in both urban and rural areas of India. l There are certain things that are common in the life of poor people in urban and rural India. These include

hunger and malnutrition, lack of proper housing and shelter, no healthcare in case of illness and no regular job and income, lack of sanitation and safe drinking water, no education for children, hopelessness and illtreatment. l The most commonly-used indicators to understand poverty are related to the levels of Scan to know income of people and their consumption of goods. more about this topic l Poverty is looked through other social indicators like illiteracy level, lack of general resistance due to malnutrition, lack of access to health care, lack of job opportunities, lack of access to safe drinking water, sanitation, etc. l The concept of social exclusion states that poor people have to live in poor surroundings Poverty line excluded from neighbourhoods of people who are better off. l Social exclusion can be both an effect and a cause of poverty. A person may be forced to live in poor surroundings, because he or she is poor. Social exclusion excludes people from equal opportunities of education, healthcare, employment and general quality of life. l Vulnerability is the measure of the probability of certain groups of people becoming poor or remaining poor in future. l Vulnerability is determined by the availability of options for employment, education, healthcare, etc. Poor people are more vulnerable to poverty.  Poverty line and Vulnerable groups (Indicators) l A person is considered poor if his or her income or consumption level falls below a given“minimum level” necessary to fulfill the basic needs.

212

Oswaal CBSE Question Bank Chapterwise & Topicwise, SOCIAL SCIENCE, Class-IX

POVERTY AS A CHALLENGE

213

l Poverty line may vary with time and place. In India, the determination of the poverty line takes into

consideration the minimum requirements of food, clothing, footwear, fuel, power, education and healthcare for the subsistence of an individual. l Minimum requirement of food is done by taking the minimum calorie requirement into consideration. In India, the minimum daily requirement is fixed at 2,400 calories per person in rural areas and 2,100 calories per person in urban areas. l Though the calorie requirement for the people in rural areas is more, their poverty line income is less than the people in urban areas. l The poverty line is revised periodically to accommodate rising prices and the changing requirements of people through nationwide surveys conducted by the National Sample Survey Organization. The percentage of population living under poverty line in India has reduced. Some groups and communities are more vulnerable to poverty than others. l Even within a family, some people are more vulnerable and suffer more than the others due to poverty, like the elderly, women and children, especially the girls.  Estimates of Poverty : The incidence of poverty in India was around 55 per cent in 1973 which declined to 36 per cent in 1993 and further to 26 per cent in 2000.

Key Terms  Poverty line: The concept of poverty line is based on the fact that a person must have a minimum level of income and consumption to satisfy the basic needs of food, clothing, clean water, education and healthcare.

OBJECTIVE TYPE QUESTIONS Multiple Choice Questions R Q. 1. What is meant by social exclusion?  (A) Process by which all individuals are excluded from certain privileges (B) Process by which all individuals are given privileges (C) Process in which individual or groups are excluded from facilities and opportunities that is enjoyed by the general people. (D) None of these Ans. Option (C) is correct. R Q. 2. NSSO stands for……. (A) National Sample Survey Organisation (B) National Sampling Survey Organisation (C) National Sample Survey Organised (D) National Sample Surveyed Organisation Ans. Option (A) is correct. Explanation: The poverty line is estimated periodically (normally every five years) by conducting sample surveys. These surveys are carried out by the National Sample Survey Organization (NSSO). R Q. 3. How frequently is poverty line estimated?  (A) Nearly every four years (B) Nearly every five years (C) Nearly every ten years (D) Nearly every eight years Ans. Option (B) is correct. Explanation: Ref to explanation of Q2. (Same)

(1 mark each)

Assertion and Reason

In the questions given below, there are two statements marked as Assertion (A) and Reason (R). Read the statements and choose the correct option.



(A) Both (A) and (R) are true and (R) is the correct explanation of (A).



(B) Both (A) and (R) are true but (R) is not the correct explanation of (A).



(C) (A) is correct but (R) is wrong.



(D) (A) is wrong but (R) is correct.

Q. 1. Assertion (A): Poverty means hunger and lack of shelter.

U

Reason (R): Poverty also means lack of clean water and sanitation facilities.

Ans. Option (B) is correct.

Explanation: Poverty means living with a sense of helplessness.

Q. 2. Assertion (A): The present formula for food requirement while estimating the poverty line is based on the desired calorie requirement. 

Reason (R): While determining the poverty line in India, a minimum level of food requirement, clothing, footwear, fuel and light, educational and medical requirement, etc., are determined for subsistence.

Ans. Option (A) is correct.

214

Oswaal CBSE Question Bank Chapterwise & Topicwise, SOCIAL SCIENCE, Class-IX

SUBJECTIVE TYPE QUESTIONS Very Short Answer Type Questions Q. 1. Differentiate between ‘minimum subsistence level of living‘ and a ‘reasonable level of living‘. R Ans. (i) A “minimum” subsistence level of living contain food, cloth and shelter (the basic needs). (ii) A ‘reasonable‘ level of living means not only food, cloth and shelter but also health care, education, job security and self-confidence. (1 x 2 = 2) Q. 2. Explain any two social indicators through which poverty is looked upon. R Ans. (i) Poverty is a situation where sick people cannot afford treatment, i.e., they lack access to health care. (ii) Poverty also implies lack of a regular job at a minimum decent level.  (1 x 2 = 2) Q. 3. Describe how the poverty line is estimated in India. U [NCERT] Ans. (i) The calorie requirement depending upon the age, sex, area and type of work is the way of estimating poverty. Average calorie requirement in India is 2,400 per person per day in rural areas and 2,100 per person per day in urban areas. (ii) Monetary expenditure per capita needed is also a way of estimating poverty. In the year 2000, the

(2 marks each)

poverty line for a person was fixed at `328 per month for the rural areas and `454 for the urban areas. (1 x 2 = 2) Q. 4. Mention the statistics related to poverty problem of the vulnerable groups. R Ans. (i) Although the average people below the poverty line for all groups in India is 26 percent, yet 51 out of 100 people belonging to Scheduled Tribes are not able to meet their basic needs. (ii) About 50 percent of casual workers in urban areas are below poverty line. (iii) About 50 percent of landless agricultural workers and 43 percent of SCs are poor. (Any Two Points) (1 x 2 = 2) Q. 5. How is poverty seen by social scientists ? Explain. U Ans. Social scientists use different types of indicators to understand their consumption of goods. Poverty is also observed using other social indicators like lack of education, healthcare, sanitation, job opportunity, illiteracy level and safe drinking water. (2)

Short Answer Type Questions Q. 1. “In poor families, all suffer, but some suffer more than others.” Explain the statement. U (Board Term II, 2017) Ans. Analysis of poverty on the basis of social exclusion and vulnerability shows that there are people in our economy who suffer more than others. Socially excluded people due to caste discrimination are excluded even from facilities, benefits and opportunities that others enjoy. Women, elderly people and female infants are systematically denied equal access to resources available to the family. (3) Q. 2. Which social and economic groups are more vulnerable to poverty ? U OR Identify the social and economic groups which are most vulnerable to poverty in India. [NCERT] (Board Term II, 2017) Ans. (i) Social groups which are most vulnerable to poverty are scheduled caste and scheduled tribe households.

(3 marks each)

(ii) Among the economic groups the most vulnerable group are the rural agricultural labour households. (iii) In urban areas the casual labour households. (CBSE Marking Scheme, 2017) (1×3=3)

Commonly Made Error

Students are not aware about vulnerable groups of society.

Answering Tip

Students should learn about the vulnerable

groups by classifying them into social and economic groups.

Q. 3. List the main problems faced by the poor people. R (Board Term II, 2016)

Ans. (i) Lack of food and shelter.

(ii) Illiteracy.



(iii) Lack of clean water and sanitation facilities.



(iv) Unemployment.

POVERTY AS A CHALLENGE



(v) Malnutrition.



(vi) Helplessness.

indicates a significant reduction in the number of poor to about 26 million. (3)

(½×6=3)

Commonly Made Error

Commonly Made Error



Students are not aware of common problems faced by poor people.



Students should learn the common problems by looking at the real scenario of poor people.

belonging to poverty line or below that.

Students must discuss the decline of people belonging to poverty line or below since 1973.

Q. 5. Explain the two methods to estimate poverty line.  U

Q. 4. Discuss the trend of poverty in India since 1973. U (Board Term II, 2016, KVS)

OR

Ans. The two methods used to estimate poverty line are: (i) Income method. In this method, the poverty line is estimated with the help of minimum income required. In the year 2011-12, the poverty line for a person was fixed at ` 816 per month for the rural areas and ` 1,000 per month for the urban areas. The higher amount for urban areas has been fixed because of high prices of essential products in urban areas. (ii) Consumption method. In this method, the average calorie requirement of a person is multiplied to the current prices. The accepted average calorie requirement in India is 2,400 calories per person per day in rural areas and 2,100 calories per person per day in urban areas. (1.5+1.5=3)



Describe poverty trends in India since 1973? U [NCERT]  Ans. There has been a substantial decline in poverty ratios in India from about 55 per cent in 1973 to 36 per cent in 1993. The proportion of people below the poverty line further came down to about 26 per cent in 2000. If the trend continues, people below the poverty line may come down to less than 20 per cent in the next few years. Although, the percentage of people living under poverty line declined in the earlier two decades (1973 – 1993), yet the number of poor remained stable around 320 million for a fairly long period. The latest estimate

Long Answer Type Questions Q. 1. Social exclusion can be both a cause as well as consequence of poverty. Explain.

Many students mention the rise of people

Answering Tip

Answering Tip

215

U

Ans. Social exclusion can be seen in the terms of the poor who have to live only in a poor surrounding with other poor people. Poor people are excluded from better surroundings with better-off people. For example, in India, people belonging to certain lower castes (i.e., Scheduled Castes) are excluded from equal opportunities. Poor people of certain castes have to live in a separate locality and are excluded from mixing with better-off people. Due to such discrimination, there comes a prevalent scene of inequality which leads to poverty and can cause more damage than having a very low income. (5)

(5 marks each)

Q. 2. Do you think that present methodology of poverty estimation is appropriate? U (NCERT) Ans. No, the present methodology of poverty estimation does not look appropriate. It only takes one factor in view and that is the economic factor. Moreover, it considers about a “minimum” subsistence level of living rather than a “reasonable” level of living. Poverty has many dimensions. It is no longer confined to economic factors. With development, the definitions of what constitutes poverty also changes. Its concept has broadened to human poverty. A few persons may have been able to feed themselves but if they are without education, without shelter, without health-care, without job security, without self-confidence, without social equality, they are considered poor. If poverty is to be removed in real sense and the people are to be brought above the poverty line, not only that we need to increase their income but also, we have to provide the people with education, shelter, healthcare, job-security, respect and dignity. (5)

216

Oswaal CBSE Question Bank Chapterwise & Topicwise, SOCIAL SCIENCE, Class-IX

Inter-State Disparities and Global Poverty Topic-2 Scenario; Causes of Poverty, Anti-Poverty Measures and the Challenges Ahead Revision Notes Scan to know  Poverty in India also has another aspect or dimension. The proportion of poor people is not the more about same in every state. The success rate of reducing poverty varies from state to state. this topic  Odisha with a poverty ratio of 47 percent and Bihar with a poverty ratio of 43 percent are the two poorest states in India. The poverty ratios in Madhya Pradesh, Assam, Tripura and Uttar Pradesh are also much higher. Global Poverty  The states that have shown the most significant decline in poverty are Jammu and Kashmir, Scenario Punjab, Haryana, Gujarat, West Bengal, Andhra Pradesh, Tamil Nadu and Kerala. The decline in poverty in Punjab and Haryana was driven by high agricultural growth rates after the Green Revolution in India.  In West Bengal, the decline in poverty is associated with land reforms that aimed to improve the condition of small farmers and agricultural workers. In Andhra Pradesh and Tamil Nadu, a well-implemented Public Distribution System of food grains is the cause of the decline in poverty.  Kerala has the highest literacy rate in India for both its male and female population. The focus on education and training and development of human resources has led to the decline of poverty of Kerala.  Global Scenario:  The proportion of people in different countries living in extreme economic poverty— defined by the World Bank as living on less than $1.90 per day—has fallen from 36 per cent in 1990 to 10 per cent in 2015.  The rapid decline in poverty in China, and Southeast Asian countries is associated with rapid economic growth and huge investments in human resource development in these regions.  In Sub-Saharan Africa, poverty in fact declined from 51 per cent in 2005 to 40.2 per cent in 2018 .  In Latin America, the ratio of poverty has also declined from 10 per cent in 2005 to 4 per cent in 2018.  Poverty has also resurfaced in some of the former socialist countries like Russia, where officially it was nonexistent earlier.  The proportion of people living under poverty in different countries as defined by the international poverty line. Causes of Poverty:

 The main causes of poverty in India include low economic growth during the colonial rule, population explosion, lack of job opportunities, irregular low-paying employment, inequitable distribution of resources and indebtedness. Anti-Poverty Measures and the Challenges Ahead  Anti-poverty measures taken by the Indian Government are based on two main objectives: Increasing economic growth in the country and launching anti-poverty programmes for specific groups of people.  The Government of India has launched several anti-poverty programmes like the Prime Minister Rozgar Yojna, Rural Employment Guarantee Programme and Swarnajayanti Gram Swarozgar Yojna which are aimed at generating self-employment opportunities in rural areas.  Human poverty extends beyond the traditional definition of poverty to include lack of housing, education, healthcare, job security, and lack of equal opportunities or dignity, due to discrimination based on caste, colour or gender.  India’s future in combating poverty appears bright. Rising economic growth, falling population growth rate, radical schemes for free elementary education for all and empowerment of women and the weaker sections of society, should result in an appreciable reduction of poverty in the years to come.

Key Terms  Inter-state disparities: It is the proportion of poor people in different states of the country and varies from state to state.

POVERTY AS A CHALLENGE

217

Example NREGA 2005 is an programme. Explain.

important

anti-poverty

Answer Step 1: NREGA stands for National Rural Employment Guarantee Act, 2005. Step 2: It aims at providing 100 days assured employment every year to every rural household. If an applicant is not provided employment within 15 days he/she will be entitled to a daily unemployment allowance.



Step 3: It initially started for 625 districts, but later on it was extended to all districts of India. One-third of the proposed jobs were reserved for women.



Step 4: The Central Government established National Employment Guarantee Funds and State Government established State Employment Guarantee Funds for implementation of the scheme.

OBJECTIVE TYPE QUESTIONS

in the region; in 2022, an estimated 62% of people living in extreme poverty lived in Sub-Saharan Africa.

Multiple Choice Questions R Q. 1. Which of the following is the poorest state? (A) Punjab (B) Bihar (C) Haryana (D) Himachal Pradesh Ans. Option (B) is correct. Explanation: Bihar is the poorest states with poverty ratios of 33.7 per cent. Q. 2. Which industry suffered the most during the R colonial period?  (A) Agriculture (B) Textiles (C) Raw Materials (D) Manufacturing Ans. Option (B) is correct. Explanation: The policies of the Colonial Government ruined traditional handicrafts and discouraged development of industries like textiles. Q. 3. Who advocated that India would be truly independent only when the poorest of its people R become free of human suffering?  (A) Mahatma Gandhi (B) Indira Gandhi (C) Rajeev Gandhi (D) B.R. Ambedkar Ans. Option (A) is correct. Q. 4. For how many days NREGA provides wage R employment in India? (A) 120 days (B) 100 days (C) 90 days (D) 60 days Ans. Option (B) is correct. Explanation: The mandate of the MGNREGA is to provide at least 100 days of guaranteed wage employment in a financial year to every rural household whose adult members volunteer to do unskilled manual work. Q. 5. Which part of the world shows an increasing trend R in poverty?  (A) African region (B) Asian region (C) Sub-Saharan Africa (D) European region Ans. Option (C) is correct. Explanation: Extreme poverty is increasingly centered in Sub-Saharan Africa. In 1990, 13% of people who were living in extreme poverty lived

(1 mark each)

Assertion and Reason In the questions given below, there are two statements marked as Assertion (A) and Reason (R). Read the statements and choose the correct option:

(A) Both (A) and (R) are true and (R) is the correct explanation of (A).



(B) Both (A) and (R) are true but (R) is not the correct explanation of (A).



(C) (A) is correct but (R) is wrong.



(D) (A) is wrong but (R) is correct.

Q. 1. Assertion (A): Another feature of high poverty rates has been the huge income inequalities. 

U



Reason (R): One of the major reasons for this is the unequal distribution of land and other resources. Ans. Option (A) is correct. Q. 2. Assertion (A): It is becoming clear that there is a strong link between economic growth and poverty E reduction.

Reason (R): Over a period of thirty years lasting up to the early eighties, there were little per capita income growth and not much reduction in poverty. Ans. Option (B) is correct. Explanation: Economic growth widens opportunities and provides the resources needed to invest in human development. Q. 3. Assertion (A): The major emphasis in recent years is on proper monitoring of all the poverty U alleviation programmes.

Reason (R): Despite good intentions, the benefits of these schemes are not fully reached to the deserving poor. Ans. Option (A) is correct.

218

Oswaal CBSE Question Bank Chapterwise & Topicwise, SOCIAL SCIENCE, Class-IX

SUBJECTIVE TYPE QUESTIONS Very Short Answer Type Questions Q. 1. Explain the principal measures taken in Punjab and Kerala to reduce poverty. E Ans. (i) The principal measures taken in Punjab to reduce poverty is increasing the agricultural growth rates. (ii) Kerala focussed more on human resource development to reduce poverty.  (1 x 2 = 2) Q. 2. Give one positive and one negative side of poverty conditions in India. U Ans. Positive aspect: India’s economic growth is one of the fastest in the world which helps in reducing poverty. Negative aspect: Large number of poor live in villages and are dependent upon agriculture where growth is much below expectation.  (1 + 1 = 2) Q. 3. Discuss any two major reasons for poverty in India? U [NCERT] Ans. The major reasons for poverty in India can be grouped as follows: (i) Inequalities in income distribution

(ii) British Colonial Administration (iii) Population explosion (iv) Slow growth in agriculture and industry  (Any Two Points) (1 x 2 = 2) Q. 4. What are the major objectives of the Prime Minister Rozgar Yojana? R Ans. (i) The aim of the programme is to create self-employment opportunities for educated unemployed youth in rural areas and small towns. (ii) Under this unemployed are helped in setting up small businesses and industries.  ( 1+1=2) Q. 5. Write a short note on Pradhan Mantri Gramodaya Yojana. R Ans. (i) PMGY was introduced in 2000. (ii) Its objective is to focus on village level development in five critical areas, that is, primary health, primary education, rural shelter, drinking water and roads. As a result of this, the quality of life of rural people will improve.  ( 1+1=2)

Short Answer Type Questions Q. 1. How is the regular growth of population a major cause of poverty in India ? Explain. U

(Board Term II, 2016)

Ans. (i) Pressure on resources has increased. (ii) Unemployment has increased. (iii) Led to increase in the rate of depletion of resources. (iv) The people belonging to below poverty line has increased. (v) Growth rate of per capita income has become low. (Any three) (CBSE Marking Scheme, 2016) (1×3=3) Q. 2. “Poverty reduction is expected to mark better progress in the next ten to fifteen years.” Justify the statement. A (Board Term II, 2016) Ans. (i) This is sure to happen. (ii) This would be possible mainly due to higher economic growth, increasing stress on universal free elementary education. (iii) Declining population growth, increasing empowerment of the women and the economically weaker section of the society.  (CBSE Marking Scheme, 2016) (1×3=3)

(2 marks each)

(3 marks each)

Q. 3. Explain the causes of poverty. 

U (Board Term II, 2016, KVS) OR What are the main causes of poverty in India? U Ans. Main causes of poverty in India are : (i) Huge income inequalities make it difficult for the government policies to implement properly for poverty elimination. Therefore, income inequality is a major cause of poverty in India. (ii) Exploitation of traditional Indian handicrafts and textile industries by British colonial administration is another major cause of poverty. (iii) In order to fulfil social obligations and observe religious ceremonies, the poor spend a lot of money. Poor people borrow money for different reasons and become the victims of indebtedness.  (1×3=3)

Q. 4. Write any three effects of economic growth on the people of India. U (Board Term II, 2016) Ans. (i) Over a period of 30 years lasting up to the early 1980s, there was little per capita income growth and not much reduction in poverty.

POVERTY AS A CHALLENGE

219

(ii) Official poverty estimates which were about 45 per cent in the early 1950s remained the same even in the early eighties. (iii) Since the 1980s, India’s economic growth has been one of the fastest in the world.

Q. 5. List any three objectives of Swarnajayanti Gram Swarozgar Yojana. A (Board Term II, 2016)

(iv) The growth rates jumped from the average of about 3.5 per cent a year in the 1970s to about 6 per cent during the 1980s and 1990s.

(iii) It helps the rural people to organize themselves into self-help groups through a mix of bank credit and government subsidy. (1 × 3 = 3)

(v) The higher growth rates have helped significantly in the reduction of poverty. (Any three) (1×3=3)

Ans. (i) It was started in 1999. (ii) The programme aims at bringing the assisted poor families above the poverty line.

Long Answer Type Questions Q. 1. Describe the global poverty trends. A (Board Term II, 2013 NCERT)

Commonly Made Error

Ans. (i) The proportion of people living in extreme poverty has fallen from 25 percent in year the 1990



to 21 percent in the year 2001. (ii) Poverty has declined substantially in China and South East Asian countries as a result of rapid

1981 to 212 million in 2001. (iv) However, in some countries of South Asia (India, Pakistan, Sri Lanka, Nepal, Bangladesh and Bhutan), the fall has been marginal. (v) In Sub-Saharan Africa, poverty actually rose from

41 percent in 1981 to 46 percent in 2001. (CBSE Marking Scheme, 2013) (1×5=5)

Q. 2. Give a brief account of inter-state disparities of poverty in India. A (NCERT) Ans. (i) Proportion of poor people is not the same in every state.

(ii) In 20 states and union territories, the poverty ratio is less than national average.

(iii) In the states of Odisha, Bihar, Assam, Tripura and Uttar Pradesh, poverty is a serious problem. Along with rural poverty, urban poverty is also high in these states. (iv) While in the states like Kerala, Jammu and Kashmir, Tamil Nadu, Andhra Pradesh, there is a significant decline in poverty.

(v) States like Punjab and Haryana have succeeded in reducing poverty with the help of high agricultural growth rates.

(vi) In West Bengal, land reform measures have helped in reducing poverty. (Any five) (1×5=5)

Some students have an overall idea of poverty in India. They are not aware about state wise statistics of poverty.

Answering Tip

economic growth. (iii) The number of poor has fallen from 606 million in

(5 marks each)



Students should discuss about the status of poverty in the different states of India.

Q. 3. Describe current government strategy of poverty alleviation.  U (KVS 2018-19) OR Explain any five anti-poverty measures taken by the government of India. U (Board Term II, 2017) OR Describe in brief the important poverty alleviation programmes undertaken by the Government of India. A (Board Term II, 2015 KVS) Ans. The current anti-poverty strategy of the government is based broadly on two planks: (i) Promotion of economic growth. (ii) Targeted anti-poverty programmes. Important anti-poverty programmes which are in operation in rural and urban areas are : (i) Prime Minister Rojgar Yojana (PMRY): Ref to Very Short Type Q4 (ii) Swarnajayanti Gram Swarojgar Yojana (SGSY): SGSY was launched on 1st April, 1999. It aims at promoting enterprises at the village level. it helps the rural people to organise themselves into self-help groups. The objective of SGSY is to bring the existing poor families above the poverty line by providing them income generating assets through bank credit and government subsidy. (iii) Pradhan Mantri Gramodaya Yojana (PMGY) : Ref to Very Short Q5. (iv) National Rural Employment Guarantee Act (NREGA): NREGA was passed in September 2005. The Act provides for 100 days assured employment

220

Oswaal CBSE Question Bank Chapterwise & Topicwise, SOCIAL SCIENCE, Class-IX

to every rural household in 200 districts. Later, the scheme will be extended to 600 districts. (v) National Food for Work Programme (NFWP) : This was launched in 2004 in 150 most backward districts of the country. The programme is open to all rural poor who are in need of wage employment and desire to do manual unskilled work. Once the NREGA is in force, the NWFP will be subsumed within this programme.  (1 × 5 = 5)

Commonly Made Error

Generally, students write wrong full forms and names of programmes.

Answering Tip

Learn the full forms by heart and discuss

the major functions of PMRY, SGSY, PMGY, NREGA, NFWP programmes.

COMPETENCY BASED QUESTIONS Case based MCQs I. Read the text given below and answer the following questions:  A (1+1+1+1=4)



In our daily life, we come across many people who we think are poor. They could be landless labourers in villages or people living in over-crowded jhuggis in cities. They could be daily wage workers at construction sites or child workers in dhabas. They could also be beggars with children in tatters. We see poverty all around us. In fact, every fourth person in India is poor. This means, roughly 270 million (or 27 crore) people in India live in poverty in 2011-12. This also means that India has the largest single concentration of the poor in the world. Since poverty has many facets, social scientists look at it through a variety of indicators. Usually the indicators used relate to the levels of income and consumption. A person is considered poor if his or her income or consumption level falls below a given “minimum level” necessary to fulfill basic needs. What is necessary to satisfy basic needs is different at different times and in different countries. Therefore, poverty line may vary with time and place. Each country uses an imaginary line that is considered appropriate for its existing level of development and its accepted minimum social norms. The proportion of people below poverty line is also not same for all social groups and economic categories in India. In poor families all suffer, but some suffer more than others. Women, elderly people and female infants are systematically denied equal access to resources available to the family. Therefore women, children (especially the girl child) and old people are poorest of the poor. 1. What does poverty means?

(A) Poverty means hunger and lack of shelter (B) Poverty means where sick people cannot afford treatment

(4 marks each)

(C) Poverty means lack of clean water and sanitation facilities (D) All of the above Ans. Option (D) is correct.

2. Poverty is looked through other social indicators like:

(A) Lack of job opportunities (B) Lack of access to safe drinking water, sanitation. (C) Both (a) and (b) (D) None of the above Ans. Option (C) is correct.

3. Roughly 270 million (or 27 crore) people in India live in poverty in:

(A) 2011-12

(B) 2012-13

(C) 2013-14

(D) None of the Above

Ans. Option (A) is correct.

4. Every ______ person in India is poor.

(A) third

(B) fourth

(C) fifth

(D) second

Ans. Option (B) is correct.

Subjective based Questions

I. Read the text given below and answer the following questions: A (1+1+2=4) Poverty in India also has another aspect or dimension. The proportion of poor people is not the same in every state. States like Madhya Pradesh, Assam, Uttar Pradesh, Bihar and Orissa had above all India poverty level. Bihar and Orissa continue to be the two poorest states with poverty ratios. There has been a substantial reduction in global poverty. Poverty declined substantially in China and Southeast Asian countries as a result of rapid economic growth and massive investments in human resource development. There were a number of causes for the widespread poverty in India. One historical reason is the low level of economic development under the British colonial

POVERTY AS A CHALLENGE

administration. The policies of the colonial government ruined traditional handicrafts and discouraged development of industries like textiles. Removal of poverty has been one of the major objectives of Indian developmental strategy. In these circumstances, there is a clear need for targeted anti-poverty programmes. Although there are so many schemes which are formulated to affect poverty directly or indirectly. These anti-poverty programmes are Mahatma Gandhi National Rural Employment Guarantee Act, Prime Minister Rozgar Yozana, Rural Employment Generation Programme, Swarnajayanti Gram



221

Swarozgar Yojana, Pradhan Mantri Gramodaya Yozana, etc. 1. In which year was the Swarnajayanti Gram Swarozgar Yojana (SGSY) launched?

Ans. The SGSY Scheme is operative from 1st April 1999 in rural areas of the country. 2. Which states have traditionally succeeded in reducing poverty with the help of high agricultural growth rates? Ans. Punjab and Haryana. 3. What has been the major objective of the Indian developmental strategy? Ans. Removal of poverty has been one of the major objectives of Indian developmental strategy.

Artificial Intelligence PARAMETERS

DESCRIPTION

Chapter Covered

Chapter 3: Poverty

Name of the Book

Economics, Class 9, NCERT

AI CONCEPTS INTEGRATED

Subject and Artificial AI integrated with Poverty Trends in India: Causes of Poverty and Intelligence Anti-Poverty Measures. Integrated Objectives

l

 o understand the concept and Dimensions of Poverty with the T help of problem Scoping concept of AI project cycle.

l

 o understand the poverty trends in India and Global Poverty T Trends.

l

To understand the Poverty Line and Poverty Estimation.

l

To study two typical cases of Poverty: Urban case & Rural case.

l

To understand Interstate disparities.

l

To understand the various causes of Poverty

l

 o understand the Anti-Poverty Measures taken by the T Government for poverty eradication.

Data Acquisition and Data Exploration https:// datavizcatalogue.com

Time Required

2 periods of 40 minutes each

Classroom Arrangement

Flexible

Material Required

Pen, paper, Black Board chalk, Laptops and Internet connections.

Pre – Preparation Activities

The students are divided into two different groups to deliberate about the Poverty Trends, Causes and Measures.

Previous Knowledge

The students are asked to research about poverty and different issues related to poverty: Landlessness, Unemployment, Illiteracy, Malnutrition, Child Labour etc.

Introduction

The teacher will initiate a discussion about two typical cases of Problem scoping: Poverty in the Urban and Rural Context. She/He assigns a case Who, What, Where study of different vulnerable groups. and Why

Methodology

l l l

 he students are asked to collect data on different vulnerable Who? T groups. The students are asked to discuss the concept of Poverty? How is What? poverty line estimated in India? The students are asked to discuss Inter State Disparities and Global Poverty Scenario.

222

Oswaal CBSE Question Bank Chapterwise & Topicwise, SOCIAL SCIENCE, Class-IX l l l

The students are asked to discuss the causes of Poverty. The students are asked to collect the data and interpret the state wise trends of poverty in India. The students are asked to discuss Anti-Poverty Measures taken by Government.

Discussion on the Text

There is an open discussion and group wise presentation on: l How poverty line is estimated in India l Whether the present methodology of Poverty Estimation is appropriate l Poverty Trends in India. l Global trends of Poverty. l Interstate Disparities of Poverty in India. l Causes of Poverty. l Government strategy of Poverty alleviation.

Learning Outcomes

l

l l l

l

Where? Why? Data Acquisition and Data Exploration

 he students will understand the concept and Dimensions of T Poverty with the help of problem Scoping concept of the AI project cycle. The students will understand the poverty trends in India and Global Poverty Trends. The students will understand Poverty Line and Poverty Estimation. The students will understand the various causes of Poverty and Anti-Poverty Measures taken by the Government for poverty eradication. The students will be able to learn Data acquisition and exploration.

Self-Evaluation and Students are asked to make presentations to compare the situation Follow up Activities in different groups and states in India Follow-up Activity

Students will be asked to analyze the various challenges ahead in poverty reduction and alleviation. 

Study Time: Reading Time: 1 hr Max Questions: 35

CHAPTER

16 Learning Objectives

FOOD SECURITY IN INDIA l Examine the critical role of food security for its masses. l Justify the rationale for the system of food security in India. l Appraise the contributory role of Public Distribution system to address FSI. l Substantiate the role of green revolution in strengthening the PDS.

TOPIC - 1 Introduction to Food Security .... P. 223

Topic-1 Introduction to Food Security

TOPIC - 2 Role of Cooperatives in Food Security .... P. 229

Revision Notes  What is Food Security? l Food security means availability, accessibility and affordability of food to all people at all times.

 Why Food Security? l People living in poverty suffer from lack of food security most of the time as they cannot afford food. In situations of natural disasters like flood, drought and earthquake, the problem of food security assumes larger proportions and affects a larger number of people. l Natural calamities like flood and drought destroy crops and existing food stocks triggering a chain reaction. Destruction and decrease in the production of food causes decrease in the availability of food. A decrease in the availability of food causes food prices to rise. The rise in food prices decreases the afford ability, and more people are unable to buy food. l A natural calamity affecting a large area for a long duration of time leads to starvation and conditions of famine. l A famine is characterized by wide spread deaths due to starvation and epidemics caused by forced use of contaminated water or decaying food and loss of body resistance due to weakening from starvation.  Who are Food-Insecure?

Scan to know more about this topic

Food Security Scan to know more about this topic

Food Security in India

l A large section of people suffers from food and nutrition insecurity in India, the worst

affected groups are landless people with little or no land to depend upon, traditional artisans, providers of traditional services, petty self-employed workers and destitute including beggars. l In the urban areas, the food insecure families are those whose working members are generally employed in

ill-paid occupations and casual labour market.

224

Oswaal CBSE Question Bank Chapterwise & Topicwise, SOCIAL SCIENCE, Class-IX

FOOD SECURITY IN INDIA

225

l The people affected by natural disasters, who have to migrate to other areas in search of work, are also

among the most food insecure people. l A large proportion of pregnant and nursing mothers and children under the age of 5 years constitute an

important segment of the food insecure population. l Hunger is another aspect indicating food insecurity. Hunger has chronic and seasonal dimensions. Chronic

hunger is a consequence of diets persistently inadequate in terms of quantity and/or quality. Seasonal hunger is related to cycles of food growing and harvesting.  Food Security in India l India has become self-sufficient in food grains during the last thirty years because of a variety of crops

grown all over the country. l The introduction of modern farming methods brought about the Green Revolution in India and was marked

by a dramatic increase in the production of food grains. l The availability of food grains at the country level has further been ensured with a carefully designed food

security system by the government. This system has two components: buffer stock and public distribution system. l Buffer Stock is the stock of food grains, namely wheat and rice procured by the government through Food

Corporation of India (FCI). l The food procured by the FCI is distributed through government regulated ration shops among the poorer

section of the society. This is called the public distribution system (PDS). l Public Distribution System (PDS) is the most important step taken by the Government of India (GoI) towards

ensuring food security.

Key Terms  Food Corporation of India: Since the Green Revolution, food grain production in India has increased manifold. The food security system in India consists of the creation of buffer stocks of food grains and their distribution through the public distribution system. Every year, after the harvest of food grains like wheat and rice, the government buys food grains from farmers through the Food Corporation of India.  Wheat Revolution: It was a special stamp released in July 1968 by Indira Gandhi, the Prime Minister of India, to officially record the impressive achievement of the Green Revolution.

Example What are the problems of the functioning of ration shops? Answer: Step 1: The public distribution system (PDS) is the most important step taken by the Indian government towards ensuring food security. However, there have been several problems related to the functioning of ration shops. Step 2: The food grains supplied by the ration shops are not enough to meet the consumption needs of the poor. As a result, they have to depend on markets instead. Step 3: The average all-India level of consumption of PDS grains is only 1 kg per person per month. Step 4: Most public-distribution-system dealers’ resort to malpractices like diverting food grains to



open market to make profits, selling poor quality grains at ration shops, irregular opening of the shops, etc. Such actions make safe and nutritious food inaccessible and unaffordable for many of the poor. Step 5: Under the targeted public distribution system, there are three kinds of ration cards: Antyodaya cards (for the poorest of the poor), BPL cards (for those below poverty line) and APL cards (for all others). Prices of the food materials are fixed accordingly. Under this system, any family above the poverty line gets very little discount at the ration shop. The price of food items for an APL family is almost as high as in the open market, so there is little incentive for them to buy the items from the ration shop.

226

Oswaal CBSE Question Bank Chapterwise & Topicwise, SOCIAL SCIENCE, Class-IX

OBJECTIVE TYPE QUESTIONS Multiple Choice Questions U Q. 1. What does food security mean? (A) Food being secured from rodents (B) Availability, accessibility and affordability of food to all at all times. (C) Food being secured from pests (D) Food being secured from insects. Ans. Option (B) is correct. Q. 2. Which of the following group in India does not U suffer from food insecurity?  (A) Farmers (B) Landlords (C) Illiterates (D) All of the above Ans. Option (B) is correct. Explanation: Landlords does not suffers from food insecurity because they have high income. Q. 3. Which organisation maintains buffer stock in R India? (A) Indian Food Corporation (B) Food Corporation (C) Food Corporation of India (FCI) (D) None of these Ans. Option (C) is correct. Explanation: Buffer Stock is the stock of food grains, namely Wheat and Rice procured by the Government through Food Corporation of India (FCI). The FCI purchases Wheat and Rice from the farmers in states where there is surplus production. R Q. 4. Match the correct Columns:

Column I (a) Minimum Price. (b) Public system

Column II Support (i) Pre-announced price

distribution (ii) Ration shops

(c) The National Food (iii) 2013 Security Act (d) Revamped Public (iv) 1992 Distribution System (A) a-i, b-ii, c-iii, d-iv (B) a-ii, b-i, c-iii, d-iv (C) a-iv, b-i, c-ii, d-iii (D) a-iii, b-iv, c-ii, d-i Ans. Option (A) is correct.

(1 mark each)

Q. 5. Who released a special stamp entitled ‘Wheat Revolution’ in July, 1968?  R (A) Jawaharlal Nehru (B) Indira Gandhi (C) Sardar Vallabhbhai Patel (D) Bhimrao Ambedkar Ans. Option (B) is correct.

Assertion and Reason In the questions given below, there are two statements marked as Assertion (A) and Reason (R). Read the statements and choose the correct option:

(A) Both (A) and (R) are true and (R) is the correct explanation of (A).



(B) Both (A) and (R) are true but (R) is not the correct explanation of (A).



(C) (A) is correct but (R) is wrong.



(D) (A) is wrong but (R) is correct.

Q. 1. Assertion (A): Drought creates a shortage of food E in the affected areas. 

Reason (R): Due to a natural calamity total production of food grains decreases. Ans. Option (A) is correct. Q. 2. Assertion (A): The agricultural labourers, fishermen, transport workers and other casual labourers were the ones who died in the Bengal R famine. 

Reason (R): The famine killed thirty lakh people in the province of Bengal. Ans. Option (A) is correct. Q. 3. Assertion (A): India has become self-sufficient in food grains during the last thirty years because of a variety of crops grown all over the country. 

U



Reason (R): Since the advent of the Green Revolution in the early-’70s, the country has avoided famine even during adverse weather conditions. Ans. Option (A) is correct.

SUBJECTIVE TYPE QUESTIONS Very Short Answer Type Questions Q. 1. Which are the people more prone to food insecurity? E

(2 marks each)

Ans. Categories of people who are prone to food insecurity are:

FOOD SECURITY IN INDIA

(i) Landless people with little or no land to depend upon. (ii) Traditional artisans and providers of traditional services. (iii) Self-employed workers and destitute including beggars. (iv) In urban areas (a)  Working members employed in ill-paid occupations. (b) Casual labour in the market. (Any Two Points) (1 x 2 = 2) Q. 2. Explain the different dimensions of food security. 

E

Ans. (i) Availability of food: This means food production within the country, food imports and the previous year’s stock stored in government granaries. (ii) Accessibility of food: This means that food is within the reach of every person. (iii) Affordability of food: This implies that a person has enough money to buy sufficient, safe and nutritious food to meet one’s dietary needs.  (Any Two) (1 x 2 = 2) Q. 3. What are famines? Who were the most affected R group of the devastating famine of Bengal?  Ans. Meaning of famine: Famine can be defined as widespread deaths due to starvation and epidemics

caused by the forced use of contaminated water or decayed food and loss of resistance due to weakening from starvation. The most affected group: Agricultural labourers, fishermen, transport workers and casual labourers.  (2)



Q. 4. Why buffer stock is created by the government? U [NCERT]  Ans. A buffer stock of food grains is created by the government so as to distribute the procured food grains in the food-deficit areas and among the poorer strata of society at a price lower than the market price. A buffer stock helps resolve the problem of shortage of food during adverse weather conditions or during periods of calamity.  (2) Q. 5. How can food security be ensured in a country? 

Ans. (i) NFWP was launched in 2004 in 150 most backward districts of the country. (ii) The programme is open to all the rural poor who are in need of wage employment and desire to do manual unskilled work. (iii) Implemented on a 100% centrally sponsored scheme and food grains are provided free of cost to the states.  (1 × 3 = 3)  (CBSE Marking Scheme 2017) Q. 2. How does social composition play an important role in food security of the country?  U (Board Term II, 2017) Ans. (i) The SCs, STs and sub sections of the OBCs (lower caste among them) who have either poor land base or very low land productivity are prone to food insecurity. (ii) The people affected by natural disasters, who have to migrate to other areas in search of work, are also among the most food insecure people.

E [NCERT]

Ans. The food security is ensured in a country only if: (i) Enough food is available for all the people, i.e., no person should die because of hunger. Everyone should get enough food for their livelihood. (ii) Everyone has the capacity to buy food. Food should be available to all, i.e., it should not be so expensive that it becomes difficult for a person to buy it. (iii) Food should be accessible to all ignoring all the barriers of rich and poor classes, etc. (2)

Short Answer Type Questions Q. 1. What were the main features of National Food for Work Programme? A (Board Term II, 2017)

227

(3 marks each)

(iii) High incidence of malnutrition prevails among women. 

(CBSE Marking Scheme 2017) (1 × 3 = 3)

Q. 3. Compare the average consumption levels of PDS grain in different states of India.  U (Board Term II, 2014) Ans. The average consumption of PDS grain at the allIndia level is only 1 kg per person per month. (i) The average consumption figure is as low as less than 300 gms per person per month in the states of Bihar, Odisha and Uttar Pradesh. (ii) In contrast, the average consumption in most of the southern states like Kerala, Karnataka, Tamil Nadu and Himachal Pradesh is in the range of 3–4 kgs per person per month. (iii) In Madhya Pradesh, only 5% of wheat and rice consumption of the poor are met through ration shop. (1 × 3 = 3)

228

Oswaal CBSE Question Bank Chapterwise & Topicwise, SOCIAL SCIENCE, Class-IX

Q. 4. Who are said to be unsecured people in rural and urban areas of India? Name the two states where the largest number of food unsecured people are found. R (Board Term II, 2013) Ans. (i) In the rural areas, food unsecured people comprise of landless people, traditional artisans, self-employed workers and beggars. (ii) In urban areas, they include people employed in ill-paid occupations and casual labour, and workers engaged in seasonal activities. (iii) The largest number of food unsecured people are found in the state of UP, Bihar, Jharkhand, Odisha, West Bengal, Chhattisgarh, parts of Madhya Pradesh and Maharashtra.  (1 × 3 = 3)

Q. 5. How is food security affected by a calamity? Explain.  A (Board Term II 2013 ) OR What happens to food security when there is a disaster or a calamity? U (NCERT) Ans. Food security is affected by a calamity as: (i) Total production of food grains decreases. (ii) It creates a shortage of food in the affected areas. (iii) Due to shortage of food, the price goes up. (iv) At the high prices, some people cannot afford to buy food. (Any three) (1 × 3 = 3)

Long Answer Type Questions Q. 1. What is FCI? Write any four functions of FCI  U (Board Term II, 2017) Ans. FCI refers to Food Corporation of India. The Food Corporation of India is a government agency which purchases food grains from the farmers wherever they are in surplus. It has helped the poor farmers of India to rise and earn. The main functions of Food Corporation of India are as under: (i) Food Corporation of India helps in transportation of seeds and other kinds of food to major parts of the country. (ii) Food Corporation of India helps in storing and packaging of food items. The main function is to maintain the temperature of the food so that it does not get spoiled. (iii) Another function of Food Corporation of India is to maintain the quantity of food required. (iv) One of the major functions of Food Corporation of India is to help in providing a proper price support to farmers. (1+4=5) Q. 2. What has our government done to provide food security to the poor? Discuss two schemes launched by the government. U (NCERT) Ans. The government has ensured the availability of food grains with the help of a carefully-designed food-security system. This system involves the maintenance of a buffer stock of food grains, and the distribution of this food among the poorer sections of the society with the help of a public distribution system. The government has also come up with several poverty-alleviation and food-intervention programmes that enhance food security; for example, the Antyodaya Anna Yojana and the National Food for Work Programme. The two schemes launched by the government in this direction are: Antyodaya Anna Yojana (i) Launched in December 2000, it caters to the families below poverty level.

(5 marks each)

(ii) Under this scheme, one crore of the poorest among the BPL families covered under the targeted public distribution system were identified. National Food for Work Programme (i) Launched in November 2004, it caters to 150 most backward districts of the country. Its objective is to intensify the generation of supplementary wage employment. (ii) Any rural poor in need of wage employment and having the desire to do manual unskilled work can avail of this programme. (1+2+2=5) Q. 3. Explain the role of Public Distribution System in food security. U OR How does PDS ensures food security in India? U OR Mention major features of Public Distribution System in India. U Ans. Following points indicate the role of Public Distribution System in food security: (i) Government regulated shops: The food procured by the FCI is distributed through government regulated ration shops among the poorer section of the society. This is called the public distribution system (PDS). Ration shops are now present in most localities, villages, towns and cities. There are about 5.5 lakh ration shops all over the country. (ii) Food at lower cost: Ration shops also known as Fair Price Shops keep stock of food grains, sugar, kerosene oil for cooking. These items are sold to people at a price lower than the market price. (iii) Stabilising prices: The PDS has proved to be the most effective instrument of government policy over the years in stabilising prices

FOOD SECURITY IN INDIA

229

(iv) Increase in food production: The system, including the minimum support price and procurement has contributed to an increase in food grain production and provided income security to farmers in certain regions.(1+4=5)

and making food available to consumers at affordable prices. It has been instrumental in averting widespread hunger and famine by supplying food from surplus regions of the country to the deficit ones.

Topic-2 Role of Cooperatives in Food Security Revision Notes  Cooperative societies in India are playing a significant role in ensuring food security and are more active in western and southern regions of the country. In Tamil Nadu, around 94% of all fair price shops run under the Public Distribution System are managed by cooperatives. Scan to know  Mother Dairy is a cooperative that sells milk and vegetables at fixed rates decided by the more about this topic Delhi Government.  Anand Milk Union Limited or Amul, Gujarat is one of the most successful cooperatives in India. Amul gave a tremendous boost to milk production leading to the White Revolution in India. Amul not only encourages dairy farming, but also provides a variety of milk products to consumers all over the country. Role of  The Academy of Development Sciences in Maharashtra is a society devoted to welfare of Cooperatives in tribal and rural communities. ADS has set up grain banks in tribal and rural areas with the Food Security help of local NGOs to food security.  A grain bank functions like a normal bank. The members can make deposits of surplus grain after the harvest and make withdrawals during lean periods. Grain bank members can also take loans in the form of grain and repay the loans with interest at the time of the next harvest.

OBJECTIVE TYPE QUESTIONS Multiple Choice Questions U Q. 1. Why is Mother Dairy popular?  (A) It is present everywhere. (B) It has the best quality of milk. (C) It has efficient and courteous staff. (D) It offers milk and vegetables to the consumers at a controlled rate which is decided by the Government of Delhi. Ans. Option (D) is correct.

(1 mark each)

Q. 2. Mother Dairy Cooperative is from which of the R following state? (A) Haryana (B) Rajasthan (C) Delhi (D) Kerala Ans. Option (C) is correct. Q. 3. In which state of India, the famous cooperative R Amul is situated?  (A) Delhi (B) Gujarat (C) Karnataka (D) Punjab Ans. Option (B) is correct.

SUBJECTIVE TYPE QUESTIONS Short Answer Type Questions Q. 1. What are cooperative societies?

R

Ans. The cooperative societies are the societies which are run by the local people, who are democratically elected by the people. These cooperative societies provide people the basic necessities of life like food grains, milk, vegetables, etc. at reasonable rates. 

(3)

(3 marks each)

Q. 2. Mention the names of any three cooperative societies working in different states of India. R Ans. (i) Mother Dairy-Delhi (ii) Amul-Gujarat (iii) Verka-Punjab (iv) Academy of Development Science- Maharashtra (Any three)(1 × 3 = 3)

230

Oswaal CBSE Question Bank Chapterwise & Topicwise, SOCIAL SCIENCE, Class-IX

Long Answer Type Questions

(ii) The co-operative societies set up shops to sell low-priced goods to poor people. (iii) Out of all fair price shops running in Tamil Nadu, around 94 percent are being run by the co-operatives. (iv) In Delhi, Mother Dairy is making strides in the provision of milk and vegetables. (v) In Gujarat, Amul is the success story of cooperatives in milk and milk products. It has resulted in White Revolution in the country. (vi) In Maharashtra, Academy of Development Science (ADS) has facilitated a network of NGOs for setting up grain banks in different regions. (Any five)(1 × 5 = 5)

Q. 1. Explain with examples the role of co-operatives in food security in India. U OR

Evaluate the role of co-operatives in food security.  A OR



(5 marks each)

Describe the role of co-operatives in providing food and related items.  U (Board 2012, Term II) (NCERT)

Ans. (i) The co-operatives are playing an important role in food security in India especially in the southern and western parts of the country.

COMPETENCY BASED QUESTIONS Case based MCQs

Read the text given below and answer the following questions: (1+1+1+1=4) A Food security means availability, accessibility and affordability of food to all people at all times. The poorest section of the society might be food insecure most of the times while persons above the poverty line might also be food insecure when the country faces a national disaster/calamity like earthquake, drought, flood, tsunami, widespread failure of crops causing famine, etc. A large section of people suffers from food and nutrition insecurity in India. The social composition along with the inability to buy food also plays a role in food insecurity. India is aiming at self-sufficiency in food grains since Independence. India has become self-sufficient in food grains during the last thirty years because of a variety of crops grown all over the country. The National Food Security Act, 2013 provides for food and nutritional security life at affordable prices and enable people to live a life with dignity.

1. Accessibility of food means: (A) Within the reach of every person (B) Food production within the country (C) Food can be easily purchased (D) All of the above Ans. Option (A) is correct. 2. In the urban areas, the food insecure families are: (A) Traditional artisans (B) Those whose working members are generally

(4 marks each)

employed in ill-paid occupations (C) Petty self-employed workers (D) All of the above Ans. Option (B) is correct. 3. Children under the age of how many years constitute an important segment of the food insecure population? (A) 4 Years (B) 10 Years (C) 5 Years (D) 6 Years Ans. Option (C) is correct.

4. Under National Food Security Act, how many percentages of rural population have been categorised as eligible households for food security? (A) 50% (B) 75% (C) 25% (D) 40% Ans. Option (B) is correct.

Subjective based Questions

Read the text given below and answer the following questions. (1+1+2=4) A Buffer Stock is the stock of food grains, namely wheat and rice procured by the government through Food Corporation of India (FCI). The food procured by the FCI is distributed through government regulated ration shops among the poorer section of the society. In the beginning the coverage of PDS was universal with no discrimination between the poor and non-poor. Over the years, the policy related to PDS has been revised to make it more efficient and targeted. The PDS has proved to be the most effective instrument of government policy over the years in stabilising prices and making food available to consumers at affordable prices. Ration shops keep stock of food

FOOD SECURITY IN INDIA

grains, sugar, kerosene oil for cooking. These items are sold to people at a price lower than the market price. Any family with a ration card can buy a stipulated amount of these items. The cooperatives are also playing an important role in food security in India especially in the southern and western parts of the country. 1. Which revolution was brought by the Amul cooperative in the country? (A) White Revolution (B) Green Revolution (C) Yellow Revolution (D) None of the above Ans. Option (A) is correct.

(A) Buffer stock (B) Public Distribution System (PDS) (C) Food Corporation of India (D) All of the above Ans. Option (D) is correct.

3. Which System was introduced to adopt the principle of targeting the ‘poor in all areas’? (A) Targeted Public Distribution System (B) Revamped Public Distribution System (C) Both (a) and (b) (D) None of these Ans. Option (C) is correct.



2. Which of the following is the most important step taken by the Government of India towards ensuring food security?

231

4. Which of the following ration card is for the poorest of the poor?

(A) BPL

(B) APL

(C) Antyodaya cards

(D) None of the above

Ans. Option (C) is correct. 

232

Oswaal CBSE Question Bank Chapterwise & Topicwise, SOCIAL SCIENCE, Class-IX